Complete Math

You might also like

Download as pdf or txt
Download as pdf or txt
You are on page 1of 144

ऐसी और PDFs के लिए

“e1 coaching center” APP


Download करें

BHUTESH SIR
CAT 98.74 % iler
CGL 193/200 SCORER
3 TIMES CGL SELECTED
Number System & HCF LCM

Instruction: - For Video Solution scan QR code 𝒙, 𝒚 तथा 𝒛 ववशशष्ठ अभाज्य संख्याएं है , िहााँ
given below or you can also CLICK on Click here
button to watch the Video Solution. 𝒙 < 𝒚 < 𝒛 है| यदद 𝒙 + 𝒚 + 𝒛 = 𝟕𝟎 है , तो 𝒛 का
मान क्या है ?
a) 𝟐𝟗 b) 𝟒𝟑 c) 𝟑𝟏 d) 𝟑𝟕

5. If the digit of a two digit number is reversed,


then the number is decreased by 𝟑𝟔. Which of
1. Find the average of all the prime numbers the following is correct regarding the number?
between 𝟏 to 𝟓𝟎. [Give your answer correct to I. The difference of the digits is 𝟒.
one decimal place.] II. The value of number can be 𝟖𝟒
𝟏 से 𝟓𝟎 के बीच की सभी अभाज्य संख्याओं का III. Number is always a composite number.
औसत ज्ञात कीजिए| [दशमलव के बाद एक स्थान दो अंको वाली एक संख्या के अंको को िोड़ 10
तक पूर्ाांककत] है | यदद दो अंको वाली एक संख्या के अंकों को
a) 𝟐𝟒. 𝟗 b) 𝟑𝟒. 𝟗 व्युत्क्रशमत कर ददया िाये, तो संख्या 𝟑𝟔 कम हो
c) 𝟓𝟐. 𝟗 d) 𝟐𝟏. 𝟗
िाती है | संख्या के सन्दभभ में ननम्नशलखित में से
2. How many composite numbers are there from कौन सा कथन सही है ?
𝟓𝟑 to 𝟗𝟕? I. अंको का अंतर 4 है|
𝟓𝟑 से 𝟗𝟕 तक ककतनी संयुक्त संख्याएं है ? II. संख्या 𝟖𝟒 हो सकती है |
a) 𝟑𝟔 b) 𝟑𝟖 c) 𝟑𝟕 d) 𝟑𝟓
III. संख्या सदे व एक भाज्य संख्या है|
3. 𝑷𝟏 is the average of all prime numbers below 𝟏𝟐 a) I, II and III b) II and III
𝑷𝟐 is the average of all prime numbers c) I and III d) I and II
between 𝟏𝟐 and 𝟑𝟎. what is the value of 𝟓𝑷𝟏 −
𝑷𝟐 ?
𝟏𝟐 के पहले की सभी अभाज्य संख्याओं का
औसत 𝑷𝟏 है और 𝟏𝟐 और 𝟑𝟎 के बीच की सभी 6. Find the greatest possible value of (𝒂 + 𝒃) for
अभाज्य संख्याओं का औसत 𝑷𝟐 है| 𝟓𝑷𝟏 − 𝑷𝟐 which the 𝟖-digit number 𝟏𝟒𝟑𝒃𝟐𝟎𝟑𝒂 is
divisible by 𝟏𝟓.
का मान ज्ञात कीजिए|
(𝒂 + 𝒃) का अधिकतम संभव मान ज्ञात करें ,
a) 𝟕. 𝟖 b) 𝟖. 𝟔
c) 𝟗. 𝟖 d) 𝟕. 𝟓 जिसके शलए 𝟖–अंकीय संख्या 𝟏𝟒𝟑𝒃𝟐𝟎𝟑𝒂, से
𝟏𝟓 से ववभाज्य है |
4. 𝒙, 𝒚 and 𝒛 are distinct prime numbers where a) 𝟏𝟔 b) 𝟏𝟓 c) 𝟏𝟒 d) 𝟏𝟕
𝒙 < 𝒚 < 𝒛. If 𝒙 + 𝒚 + 𝒛 = 𝟕𝟎, then what is the
value of 𝒛? 7. From the following numbers, find the numbers
which exactly divisible by 𝟒𝟐.
ननम्नशलखित संख्याओं में से वे संख्याएाँ ज्ञात 12. If the number 𝟒𝟖𝒌𝟐𝟎𝟒𝟖𝒑𝟔 is divisible by 𝟗𝟗,
then (𝒌 × 𝒑) is equal to:
कीजिए िो 𝟒𝟐 से पूर्त
भ ः ववभाज्य हैं।
यदद 𝟒𝟖𝒌𝟐𝟎𝟒𝟖𝒑𝟔 एक ऐसी संख्या है िो 𝟗𝟗 से
a) 𝟐𝟓𝟐𝟑𝟐 b) 𝟐𝟓𝟐𝟒𝟐
c) 𝟐𝟓𝟐𝟒𝟒 d) 𝟐𝟓𝟐𝟏𝟐 ववभाज्य है , तो (𝒌 × 𝒑) का मान क्या होगा?
a) 𝟐 b) 𝟔 c) 𝟒 d) 𝟎
8. If the 𝟕-digit number 𝒙𝟖𝟗𝟒𝟐𝒚𝟒 is divisible by
𝟓𝟔, what is the value of (𝒙𝟐 + 𝒚) for the largest 13. Find the value of 𝒌 in the number 𝟑𝟒𝟐𝟔𝒌 if the
value of 𝒚, where 𝒙 and 𝒚 are natural numbers? number is divisible by 𝟔 but NOT divisible by 𝟓.
यदद 𝒙𝟖𝟗𝟒𝟐𝒚𝟒 एक ऐसी 𝟕 अंको की संख्या है संख्या 𝟑𝟒𝟐𝟔𝒌 में 𝒌 का मान ज्ञात कीजिए, यदद
िो 𝟓𝟔 से ववभाज्य है , तो 𝒚 के सबसे बड़े मान संख्या 𝟔 से ववभाज्य है , लेककन 𝟓 से ववभाज्य नहीं
के शलए (𝒙𝟐 + 𝒚) का मान क्या है ? िहााँ 𝒙 और है |
𝒚 प्राकृत संख्याएाँ है? a) 𝟒 b) 𝟔 c) 𝟑 d) 𝟗
a) 𝟑𝟑 b) 𝟒𝟒 c) 𝟓𝟓 d) 𝟕𝟎
14. If the 9-digit number 5p42978n6 is divisible by
72, what is the value of (𝟐𝒑 − 𝟏), where n is
9. Find the greatest number 𝟐𝟑𝟒𝒂𝟓𝒃, which is
the second largest of all the possible value of n?
divisible by 𝟐𝟐, but NOT divisible by 𝟓.
Given that p and n are natural numbers.
सबसे बड़ी संख्या 𝟐𝟑𝟒𝒂𝟓𝒃 ज्ञात कीजिए, िो 𝟐𝟐
यदद 𝟗 अंको की एक सख्या 5p42978n6 , 72 से
से ववभाज्य है , लेककन 𝟓 से ववभाज्य नहीं है |
ववभाज्य है , तो (𝟐𝒑 − 𝟏), का मान क्या होगा,
a) 𝟐𝟑𝟒𝟎𝟓𝟖 b) 𝟐𝟑𝟒𝟖𝟓𝟎
c) 𝟐𝟑𝟒𝟔𝟓𝟐 d) 𝟐𝟑𝟒𝟕𝟓𝟏 िहााँ
n, n के सभी संभाववत मानो मे से दस
ू रा सबसे
10. Find the greatest number 𝟐𝟑𝒂𝟔𝟖𝒃, which is
बड़ा है ? ददया गया है कक p और n प्राकृनतक
divisible by 𝟑 but NOT divisible by 𝟗.
वह सबसे बड़ी संख्या 𝟐𝟑𝒂𝟔𝟖𝒃 ज्ञात कीजिए, िो संख्याएाँ हैं।
a) 𝟏𝟓 b) 𝟐𝟏 c) 𝟏𝟏 d) 𝟏𝟕
𝟑 से ववभाज्य है लेककन 𝟗 से ववभाज्य नहीं है |
a) 𝟐𝟑𝟖𝟔𝟖𝟗 b) 𝟐𝟑𝟗𝟔𝟖𝟓 15. If the number 𝟕𝟑𝟐𝒙𝒚 is divisible by 𝟕𝟎, then
c) 𝟐𝟑𝟗𝟔𝟖𝟖 d) 𝟐𝟑𝟕𝟔𝟖𝟕
.
𝒙+𝒚
find the minimum value of 𝟐
यदद 𝟕𝟑𝟐𝒙𝒚 एक ऐसी संख्या है िो 𝟕𝟎 से
ववभाज्य है , तो का न्यूनतम मान ज्ञात
𝒙+𝒚
𝟐
11. If a number 𝟓𝟒𝒌𝟑𝟏𝒎𝟖𝟐 is divisible by 11, what कीजिए|
will be the maximum value of (𝒌 + 𝒎)? a) 𝟐 b) 𝟏 c) 𝟎 d) 3
यदद संख्या 𝟓𝟒𝒌𝟑𝟏𝒎𝟖𝟐, 11 से ववभाज्य है , तो
(𝒌 + 𝒎) का अधिकतम मान ज्ञात करें ?
a) 23 b) 13 c) 12 d) 11
16. The number 𝟐𝟏𝟒𝟑𝟐𝟓𝟏 is divisible by:
संख्या 𝟐𝟏𝟒𝟑𝟐𝟓𝟏 ननम्न मे से ककससे ववभाज्य छह अंक वाली संख्या 𝟓𝟑𝟕𝒙𝒚𝟓, 𝟏𝟐𝟓 से ववभाज्य
है? है | ऐसे छह अंको वाली ककतनी संख्याएं है ?
a) 𝟕 b) 𝟏𝟕 c) 𝟏𝟑 d) 𝟑 a) 4 b) 2
c) 3 d) 5
17. If the 8-digit number 𝟖𝟖𝟖𝒙𝟓𝟑𝒚𝟒 is divisible by
72, then what is the value of (𝟕𝒙 + 𝟐𝒚), for the 22. What is the value of k such that number
maximum value of 𝒚? 𝟕𝟐𝒌𝟒𝟔𝟎𝒌 is divisible by 6?
यदद 8-अंक वाली संख्या 𝟖𝟖𝟖𝒙𝟓𝟑𝒚𝟒, 72 से k का वह मान ज्ञात करें कक संख्या 𝟕𝟐𝒌𝟒𝟔𝟎𝒌 ,
ववभाज्य है , तो 𝒚 के अधिकतम मान के शलए 6 से ववभाज्य हो िाए?
(𝟕𝒙 + 𝟐𝒚) का मान ज्ञात करें | a) 4 b) 7 c) 9 d) 8
a) 19 b) 23 c) 27 d) 15
23. If the number 87m6203m is divisible by 6 then
18. If the 5-digit number 𝟔𝟖𝟖𝒙𝒚 is divisible by 3, 7 find the sum of all possible values of 'm’.
and 11, then what is the value of (𝟓𝒙 + 𝟑𝒚)? यदद संख्या 87m6203m 6 से ववभाज्य हो, तो 'm'
यदद 5-अंक वाली संख्या 𝟔𝟖𝟖𝒙𝒚, 3, 7 और 11 से के सभी संभाववत मानों का योगफल ज्ञात करें |
ववभाज्य है , तो (𝟓𝒙 + 𝟑𝒚) का मान ज्ञात करें | a) 10 b) 20 c) 16 d) 15
a) 39 b) 36 c) 43 d) 38 ̅+𝟎.𝟓𝟔𝟕
̅̅̅̅−𝟎.𝟓𝟗
̅
𝟎.𝟒
24. The value of ̅)×𝟑.𝟒÷𝟐.𝟓𝟒
̅̅̅̅÷𝟎.𝟕𝟓
(𝟎.𝟐𝟐𝟔 ̅̅̅̅×𝟑.𝟑
̅
lies
19. If the 5-digit number 𝟔𝟕𝟔𝒙𝒚 is divisible by 3, 7 between :
and 11, then what is the value of (𝟑𝒙 − 𝟓𝒚)? ̅+𝟎.𝟓𝟔𝟕
̅̅̅̅−𝟎.𝟓𝟗
̅
का मान ____ के बीच
𝟎.𝟒
यदद 5-अंक वाली संख्या 𝟔𝟕𝟔𝒙𝒚, 3, 7 और 11 से ̅)×𝟑.𝟒÷𝟐.𝟓𝟒
̅̅̅̅÷𝟎.𝟕𝟓
(𝟎.𝟐𝟐𝟔 ̅̅̅̅×𝟑.𝟑
̅

ववभाज्य है , तो (𝟑𝒙 − 𝟓𝒚) का मान ज्ञात करे | होगा|


a) 10 b) 7 a) 𝟎. 𝟏 and 𝟎. 𝟐 b) 𝟎. 𝟑 and 𝟎. 𝟒
c) 9 d) 11 c) 𝟎. 𝟒 and 𝟎. 𝟓 d) 𝟎. 𝟐 and 𝟎. 𝟑

25. Let 𝒙 = 𝟏. 𝟎𝟓 ̅ ÷ 𝟎. ̅̅̅̅ ̅̅̅̅ and 𝒚 =


𝟗𝟓 × 𝟎. 𝟒𝟎𝟗
20. The sum of 3-digit numbers abc, cab and bca is
̅ ÷ 𝟎. 𝟐𝟐𝟔
(𝟎. 𝟕𝟓 ̅̅̅̅) × (𝟓. ̅̅̅̅
𝟎𝟗 ÷ 𝟏. 𝟕), then the
not divisible by :
value of 𝒙𝒚 is:
3-अंक वाली संख्याओं abc, cab और bca का
माना कक 𝒙 = 𝟏. 𝟎𝟓 ̅ ÷ 𝟎.̅̅̅̅̅ ̅̅̅̅ और 𝒚 =
𝟗𝟓 × 𝟎. 𝟒𝟎𝟗
योगफल______से ववभाज्य नहीं है |
a) 37 b) 3
̅ ÷ 𝟎. 𝟐𝟐𝟔
(𝟎. 𝟕𝟓 𝟎𝟗 ÷ 𝟏. 𝟕) हैं, तो 𝒙𝒚 का
̅̅̅̅) × (𝟓. ̅̅̅̅
c) 31 d) 𝒂 + 𝒃 + 𝒄 मान ककतना होगा?
a) 𝟎. 𝟒𝟓 b) 𝟒. 𝟓 c) 𝟎. 𝟓𝟒 d) 𝟓. 𝟒

21. The six-digit number 𝟓𝟑𝟕𝒙𝒚 𝟓 is divisible by


𝟏𝟐𝟓. How many such six-digit numbers are ̅̅̅̅÷𝟏.𝟎𝟓
(𝟎.𝟗𝟓 ̅)÷𝟎.𝟒𝟎𝟗̅̅̅̅
26. The value of ̅̅̅̅÷𝟎.𝟗𝟑 ̅̅̅̅)
̅÷𝟎.𝟓𝟒
is:
there? (𝟎.𝟖𝟒
̅̅̅̅÷𝟏.𝟎𝟓̅)÷𝟎.𝟒𝟎𝟗̅̅̅̅
̅̅̅̅) का मान ज्ञात कीजिए|
(𝟎.𝟗𝟓 respectively. What will be the remainder when
̅̅̅̅÷𝟎.𝟗𝟑
(𝟎.𝟖𝟒 ̅÷𝟎.𝟓𝟒
42 divides the same number?
𝟐 𝟏 𝟏 𝟏
a) 𝟏 𝟑 b) 𝟐 𝟔 c) 𝟏 𝟑 d) 𝟐 𝟑 िब ककसी संख्या को रमशः 3, 4 और 7 से

̅̅̅̅ ÷ 𝟎. 𝟓𝟐
̅) ÷ 𝟐. 𝟐𝟕𝟐
̅̅̅̅ and ववभाजित ककया िाता है , तो शेषफल रमशः 2, 3
27. Let 𝒙 = (𝟎. 𝟒𝟕 𝒚=
𝟑 𝟐 𝟑 𝟏 𝟕
𝟏 𝟓 − 𝟏 𝟗 𝒐𝒇 𝟑 𝟏𝟏 ÷ 𝟓 𝟕 𝒐𝒇 𝟗. What is the value और 5 प्राप्त होते है। िब उसी संख्या को 42 से
𝒙
of 𝒚 ? ववभाजित ककया िाए तो प्राप्त शेषफल ज्ञात करें |
a) 41 b) 30 c) 29 d) 31
माना कक ̅̅̅̅ ÷ 𝟎. 𝟓𝟐
𝒙 = (𝟎. 𝟒𝟕 ̅̅̅̅ और
̅) ÷ 𝟐. 𝟐𝟕𝟐
है, तो का
𝟑 𝟐 𝟑 𝟏 𝟕 𝒙
𝒚 = 𝟏 𝟓 − 𝟏 𝟗 𝒐𝒇 𝟑 𝟏𝟏 ÷ 𝟓 𝟕 𝒐𝒇 𝟗 𝒚
मान ककतना होगा?
𝟏 𝟐 𝟓 𝟑
a) 𝟓 b) 𝟑 c) 𝟑 d) 𝟐 31. How many number are there from 𝟓𝟎𝟎 to 𝟔𝟓𝟎
(including both) which are neither divisible by
𝟑 nor by 𝟕 ?
28. If 𝑨 = 𝟎. 𝟑𝟏𝟐, 𝑩 = 𝟎. 𝟒𝟏𝟓 and 𝑪 = 𝟎. 𝟑𝟎𝟗,
then what is the value of 𝑨 + 𝑩 + 𝑪? 𝟓𝟎𝟎 से 𝟔𝟓𝟎 तक ( दोनों को सजम्मशलत करते
यदद 𝑨 = 𝟎. 𝟑𝟏𝟐, 𝑩 = 𝟎. 𝟒𝟏𝟓 तथा 𝑪 = 𝟎. 𝟑𝟎𝟗 हुए) ऐसी ककतनी संिाएं है िो 3 और 7 दोनों
है, तो 𝑨 + 𝑩 + 𝑪 का मान ककतना है ? से ववभाज्य नही है ?
a) 𝟏𝟏𝟒𝟏/𝟏𝟏𝟎𝟎 b) 𝟏𝟎𝟗𝟕/𝟏𝟏𝟎𝟎 a) 𝟐𝟏 b) 𝟏𝟐𝟏 c) 𝟖𝟕 d) 𝟗𝟗
c) 𝟏𝟐𝟏𝟏/𝟏𝟏𝟎𝟎 d) 𝟏𝟎𝟒𝟑/𝟏𝟏𝟎𝟎
32. On dividing 𝟖𝟔𝟕𝟓𝟏𝟐𝟑 by a certain number, the
29. Let 𝒑, 𝒒 𝒓 and 𝒔 be positive natural numbers quotient is 𝟑𝟑𝟔𝟏𝟏 and the remainder is 𝟑𝟒𝟖𝟓.
having three exact factors including 1 and the The divisor is _________.
number itself. If 𝒒 > 𝒑 and both are two-digit 𝟖𝟔𝟕𝟓𝟏𝟐𝟑 को एक ननजचचत संख्या से भाग दे ने
numbers, and 𝒓 > 𝒔 and both are one-digit
पर भागफल 𝟑𝟑𝟔𝟏𝟏और शेषफल 𝟑𝟑𝟔𝟏𝟏आता है।
numbers, then the value of the expression
𝒑−𝒒−𝟏
is: भािक ज्ञात कीजिए।
𝒓−𝒔
a) 𝟐𝟒𝟖 b) 𝟓𝟑𝟖 c) 𝟑𝟓𝟔 d) 𝟐𝟓𝟖
माना कक 𝒑, 𝒒 𝒓 और 𝒔 िनात्क्मक प्राकृनतक
संख्याएाँ हैं, जिनके तीन यथातथ्य गर्
ु निंडो 𝟑
33. If √𝑵 lies between 𝟔 and 𝟕, where 𝑵 is an
(exact factors) में 1 और स्वयं संख्या भी है। integer then how many values 𝑵 can take?
यदद √𝑵 िो कक 𝟔 तथा 𝟕 के मध्य जस्थत है , और
𝟑
यदद 𝒒 > 𝒑 और दोनों दो-अंकीय संख्याएाँ हैं, तथा
𝒓 > 𝒔और दोनों एक-अंकीय संख्याएाँ हैं, तो व्यंिक िहााँ N एक पूर्ाभक है , तो N के ककतने मान हो
𝒑−𝒒−𝟏
का मान ककतना होगा? सकते है ?
𝒓−𝒔
a) 𝟏𝟐𝟔 b) 𝟏𝟐𝟕 c) 𝟏𝟐𝟖 d) 𝟏𝟐𝟓
a) −𝒔 − 𝟏 b) 𝒔 − 𝟏
c) 𝟏 − 𝒔 d) 𝒔 + 𝟏
HCF & LCM
30. When a number is successively divided by 3, 4
and 7, the remainder obtained is 2, 3 and 5,
34. When 𝟓𝟑𝟗𝟔, 𝟔𝟐𝟒𝟖 and 𝟕𝟕𝟑𝟗 are divided by the respective quotients are 𝟏, 𝟏, 𝟓 and 𝟐. What
the greatest number 𝒙, the remainder is each is the LCM of 𝟐𝟕𝟗 and the greater of the given
case is 𝒚. Which of the following is true? two numbers?
𝟓𝟑𝟗𝟔, 𝟔𝟐𝟒𝟖 और 𝟕𝟕𝟑𝟗 को सबसे बड़ी संख्या भाग ववधि से दो िन पूर्ाांकों का महत्तम
𝒙 से ववभाजित करने पर प्रत्क्येक जस्थनत में समापवतभक ज्ञात करने पर अंनतम भािक 𝟑𝟏
शेषफल 𝒚 प्राप्त होता है | इनमे से कौन-सा सही और संबंधित भागफल 𝟏, 𝟏, 𝟓 और 𝟐 प्राप्त होते
है? हैं| 𝟐𝟕𝟗, तथा दी गई दो संख्याओं में से बड़ी
a) 𝟑𝒙 − 𝟐𝒚 = 𝟐𝟕𝟒 b) 𝟑𝒙 − 𝟕𝒚 = 𝟏𝟒𝟎 संख्या का लघुत्तम समापवत्क्यभ ककतन होगा?
d) 𝟐𝒙 = 𝟑𝒚 d) 𝒙 = 𝟑𝒚 a) 𝟏𝟗𝟓𝟑 b) 𝟏𝟒𝟒𝟎
c) 𝟐𝟐𝟑𝟐 d) 𝟏𝟔𝟕𝟒
35. Let 𝒙 be the greatest number which when
divides 𝟐𝟖𝟕𝟏, 𝟒𝟔𝟐𝟕 and 𝟕𝟕𝟎𝟎, the remainder 38. The LCM of two number is 𝟐𝟖 times their HCF
in each case is 𝒚. The value of (𝒙 − 𝒚) lies and the sum of their LCM and HCF is 𝟒𝟗𝟑. The
between: difference of the two number is 𝟓𝟏. If 𝒔 is the
मान लीजिए कक 𝒙 वह बड़ी से बड़ी संख्या है , sum of the two numbers, then what is the sum
जिससे 𝟐𝟖𝟕𝟏, 𝟒𝟔𝟐𝟕 और 𝟕𝟕𝟎𝟎 को ववभाजित of digits of 𝒔?
करने पर प्रत्क्येक मामले में 𝒚 शेष बचता है | (𝒙 − दो संख्याओं का लघत्त
ु म समापवत्क्यभ (LCM), उनके

𝒚) का मान, ननम्नशलखित में से ककसके बीच महत्तम समापवतभक (HCF) का 𝟐𝟖 गन


ु ा है और

होगा? उनके लघुत्तम समापवत्क्यभ (LCM) और महत्तम


a) 𝟏𝟗𝟓 and 𝟐𝟎𝟎 b) 𝟐𝟎𝟓 and 𝟐𝟎𝟏𝟎 समापवतभक (HCF) का योग 𝟒𝟗𝟑 है | दोनों
c) 𝟐𝟎𝟎 and 𝟐𝟎𝟓 d) 𝟐𝟏𝟎 and 𝟐𝟏𝟓 संख्याओं का अंतर 𝟓𝟏 है| यदद उन दोनों संख्याओं
का योग 𝒔 है, तो 𝒔 के अंकों का योग ज्ञात कीजिए|
a) 𝟏𝟔 b) 𝟏𝟕 c) 𝟏𝟑 d)𝟏𝟗

36. In finding the 𝑯𝑪𝑭 of two numbers by the 39. 𝟏𝟑, 𝒂, 𝒃, 𝒄 are four distinct numbers and the
division method, the last divisor is 𝟐𝟖 and the 𝑯𝑪𝑭 of each pair of numbers
quotients are 𝟏, 𝟑𝟎, 𝟏 and 𝟑, respectively. What (𝟏𝟑, 𝒂); (𝟏𝟑, 𝒃); (𝟏𝟑, 𝒄) is 𝟏𝟑, where 𝒂, 𝒃, 𝒄
is the sum of the two numbers? are each less than 𝟔𝟎 and 𝒂 < 𝒃 < 𝒄. What is
𝒂+𝒄
ववभािन ववधि द्वारा दो संख्याओं का महत्तम the value of 𝒃 ?
समापवतभक (𝑯𝑪𝑭) ज्ञात करने में. अंनतम भािक 𝟏𝟑, 𝒂, 𝒃, 𝒄 चार अलग-अलग संख्याएं है और
𝟐𝟖 है और भागफल रमश: 𝟏, 𝟑𝟎, 𝟏 और 𝟑 हैं| संख्याओं के प्रत्क्येक िोड़े
उन दो संख्याओं का योग ककतना है ? (𝟏𝟑, 𝒂); (𝟏𝟑, 𝒃); (𝟏𝟑, 𝒄) का म.स 𝟏𝟑 है , िहााँ
a) 𝟕𝟏𝟒𝟎 b) 𝟔𝟗𝟒𝟒 𝒂, 𝒃, 𝒄 प्रत्क्येक 𝟔𝟎 से कम है और 𝒂 < 𝒃 < 𝒄 है |
c) 𝟕𝟎𝟎𝟎 d) 𝟔𝟖𝟔𝟎
का मान क्या है ?
𝒂+𝒄
𝒃
37. In finding the HCF of two positive integers by a) 𝟑. 𝟓 b) 𝟐
the division method, the last divisor is 𝟑𝟏, and c) 𝟓 d) 𝟒. 𝟓
𝟗, 𝟏𝟎 और 𝟏𝟓 से ववभाजित करने पर प्रत्क्येक
40. The LCM of two positive integers is twice the
larger number and the HCF of the two number जस्थनत में शेषफल 𝟓 हो?
is 𝟑. The smaller number is: a) 41 b) 78 c) 82 d) 39
दो िनात्क्मक पूर्ाभक संख्याओं का लघुत्तम
44. Let 𝒙 be the least number between 56,000 and
समापवत्क्यभ (LCM) बड़ी संख्या का दोगुना है और 60,000 which when divided by 40, 45, 50 and 55
उन दोनों संख्याओं का महत्तम समापवतभक (HCF) leaves a remainder of 23 in each case. What is
the sum of the digits of 𝒙?
𝟑 है| छोटी संख्या ज्ञात कीजिए|
माना 𝒙, 56,000 और 60,000 के बीच वह छोटी से
a) 𝟔 b) 𝟏𝟎 c) 𝟗 d) 𝟖
छोटी संख्या है , जिसे िब 40, 45, 50 और 55 से
ववभाजित ककया िाता है , तो प्रत्क्येक जस्थनत में
शेषफल 23 प्राप्त होता है | 𝒙 के अंकों का योगफल
41. The sum of and difference between the LCM ज्ञात करें |
and HCF of two numbers are 512 and 496, a) 23 b) 21 c) 26 d) 19
respectively. If one number is 72, then the
other number is: 45. Let 𝒙 be the smallest 4-digit number such that
दो संख्याओं के लघत्त
ु म समापवत्क्यभ (LCM) और when it is divided by 5, 6 and 7, it leaves the
समापवतभक (HCF) का योग तथा उनके बीच का remainders 2, 3 and 4, respectively. The sum of
the digits of 𝒙 is:
अंतर रमशः 512 और 496 है। यदद उनमे से एक
माना 4-अंकों की छोटी से छोटी संख्या 𝒙 इस
संख्या 72 है , तो दस
ू री संख्या कौन-सी है ?
प्रकार है कक इसे 5, 6 और 7 से ववभाजित ककया
a) 64 b) 80 c) 40 d) 56
िाता है , तो रमश: 2, 3 और 4 शेषफल प्राप्त होता
42. The HCF of two 2-digit numbers is 19 and their है | 𝒙 के अंकों का योगफल ज्ञात करें |
sum is 152. What is their difference? a) 9 b) 11 c) 12 d) 14
2-अंकों वाली दो संख्याओं का म.स. (HCF) 19 है
और उनका योगफल 152 है। उनके बीच के
अंतर की गर्ना करें ?
a) 19 b) 38 c) 57 d) 76 46. Let 𝒙 be the least number which when divided
by 𝟏𝟔, 𝟐𝟒, 𝟑𝟎, 𝟑𝟔 and 𝟒𝟓, the remainder in
43. What least number must be subtracted from each case is 𝟒 and 𝒙 is divisible by 𝟐𝟖. If the
2963 so that the resulting number when 𝑯𝑪𝑭 of 𝒙 and 𝟑, 𝟏𝟗𝟑 is 𝒚, then what is the sum
divided by 9, 10 and 15, the remainder in each of the digits of 𝒚 ?
case is 5? मान लीजिए 𝒙 वो सबसे छोटी संख्या है जिसे
𝟐𝟗𝟔𝟑 में से कौन सी छोटी से छोटी संख्या को 𝟏𝟔, 𝟐𝟒, 𝟑𝟎, 𝟑𝟔 और 𝟒𝟓 से ववभाजित करने पर
घटाया िाना चादहए ताकक पररर्ामी संख्या को प्रत्क्येक जस्थत में शेषफल 𝟒 प्राप्त होता है , और 𝒙
𝟐𝟖 से ववभाज्य है | यदद 𝒙 और 𝟑, 𝟏𝟗𝟑 का महत्तम
समापवतभक (𝑯𝑪𝑭) 𝒚 है, तो 𝒚 के अंकों का योग 𝟏𝟐𝟎 सेब, 𝟐𝟒𝟎 संतरे और 𝟏𝟓𝟎 नाशपाती काटभ न्स
क्या है ? में इस तरह पैक ककए िाते हैं कक प्रत्क्येक काटभ न
a) 𝟓 b) 𝟏𝟎 c) 𝟒 d) 𝟗 में फलों की संख्या बराबर होती है | प्रत्क्येक काटभ न

47. The HCF and the LCM of two numbers are 𝟓 and में केवल एक ही प्रकार के फल हैं और कोई भी
𝟏𝟐𝟎 respectively. If the sum of the two number फल बबना पैक ककए नही छोड़ा िाता है| इसके
is 𝟓𝟓, then the sum of the reciprocals of these शलए आवचयक काटभ न्स की न्यूनतम संख्या ज्ञात
two numbers is equal to:
कीजिए|
दो संख्याओं का महत्तम समापवतभक (HCF) 𝟓 और
a) 𝟓𝟎 b) 𝟒𝟎 c) 𝟏𝟕 d) 𝟑𝟎
लघुतम समापवत्क्यभ (LCM) 𝟏𝟐𝟎 है| यदद इन दोनों
संख्याओं का योग 𝟓𝟓 है , तो इनके व्युत्क्रमों का 51. What is the LCM of 𝟎. 𝟏𝟐𝟔, 𝟎. 𝟑𝟔 and 𝟎. 𝟗𝟔?
योग ज्ञात कीजिए| 𝟎. 𝟏𝟐𝟔, 𝟎. 𝟑𝟔 और 𝟎. 𝟗𝟔 का ल.स.प. (LCM)
𝟓𝟓
a) 𝟔𝟎𝟏
𝟏𝟏
b) 𝟏𝟐𝟎 c)
𝟏𝟐𝟎
d)
𝟔𝟎𝟏 ज्ञात करें |
𝟏𝟏 𝟓𝟓
a) 20160 b) 𝟐𝟎. 𝟏𝟔
48. The product of the LCM and the HCF of two c) 𝟐𝟎𝟏. 𝟔𝟎 d) 𝟐. 𝟎𝟏𝟔
numbers is 𝟐𝟏𝟔. The difference between the
two numbers is 𝟔. What are the numbers? Answer Key
दो संख्याओं के लघुत्तम समापवत्क्यभ (LCM) और 1. D 2. D 3. A 4. D 5. D
6. C 7. B 8. C 9. C 10. B
महत्तम समापवतभक (HCF) का गुर्नफल 𝟐𝟏𝟔 है| 11. B 12. D 13. B 14. A 15. B
दोनों संख्याओं का अंतर 𝟔 है| वे संख्याएं कौन 16. D 17. B 18. C 19. C 20. A
सी है ? 21. C 22. A 23. A 24. A 25. B
26. D 27. B 28. A 29. A 30. C
a) 𝟗, 𝟐𝟒 b) 𝟔, 𝟑𝟔
c) 𝟏𝟐, 𝟏𝟖 d) 𝟖, 𝟐𝟕 31. C 32. D 33. A 34. D 35. C
36. C 37. C 38. A 39. B 40. A
49. Find the HCF of (𝟒𝟑𝟏𝟓 − 𝟏) and (𝟒𝟐𝟓 − 𝟏). 41. D 42. B 43. B 44. A 45. C
46. C 47. B 48. C 49. D 50. C
(𝟒𝟑𝟏𝟓 − 𝟏) और (𝟒𝟐𝟓 − 𝟏) का महत्तम 51. B
समापवतभक (HCF) ज्ञात कीजिए|
a) 𝟏 b) (𝟒𝟐𝟓 − 𝟏)
c) 𝟏𝟎𝟐𝟒 d) 𝟏𝟎𝟐𝟑

50. 𝟏𝟐𝟎 apples, 𝟐𝟒𝟎 oranges and 𝟏𝟓𝟎 pears are


packed in cartons in such a way that each
carton has the same number of fruits, each
carton contains only one type of fruit, and no
fruit is left unpacked. What is the smallest
possible number of cartons needed for the
purpose?
Time and Distance

Instruction: - For Video Solution scan QR code 3. A person goes from P to Q at a speed of 𝟐𝟎 km/h.
given below or you can also CLICK on Click here Then he goes from Q to R at a speed of q km/h.
button to watch the Video Solution. Finally the person goes from R to S at a speed
of r km/h. The distances from P TO Q, Q to R
and R to S are equal. If the average speed from
𝟐𝟖𝟎
P to R is 𝟏𝟏 km/h, and the average speed from
𝟏𝟏𝟐
Q to S is 𝐤𝐦/𝐡, then what is the value of r?
𝟑
1. A car travelled distances of 𝟏𝟎𝒌𝒎, 𝟐𝟎𝒌𝒎 and एक व्यजतत 𝟐𝟎 km/h की चाल से P से Q तक
𝟑𝟎𝒌𝒎 at speeds of 𝟑𝟎𝒌𝒎/h, 𝟒𝟎𝒌𝒎/h and
िाता है | किर वह q km/h की चाल से Q से R
𝟔𝟎𝒌𝒎/h, respectively. The average speed of
the car is: तक िाता है| अंत में व्यजतत r km/h की चाल से
एक कार 𝟑𝟎𝒌𝒎/h, 𝟒𝟎𝒌𝒎/h और 𝟔𝟎𝒌𝒎/h की R से S तक िाता है | P से Q, Q से R और R से
चाल से क्रमशः 𝟏𝟎𝒌𝒎, 𝟐𝟎𝒌𝒎 और 𝟑𝟎𝒌𝒎 की S के बीच की दरू ी बराबर है | यदद P से R के
दरु ी तय करती है | कार की औसत चाल ज्ञात बीच उसकी औसत चाल km/h है और Q से
𝟐𝟖𝟎
𝟏𝟏
कीजिए| S के बीच उसकी औसत चाल
𝟏𝟏𝟐
𝐤𝐦/𝐡 है, तो
𝟑
a) 𝟒𝟎𝒌𝒎/h b) 𝟒𝟓𝒌𝒎/h r का मान तया होगा?
c) 𝟐𝟎𝒌𝒎/h d) 𝟑𝟎𝒌𝒎/h
a) 𝟒𝟎 b) 𝟑𝟕. 𝟓 c) 𝟒𝟐. 𝟓 d) 𝟒𝟓
2. Three cities , 𝑨, 𝑩, and 𝑪 are located such that
4. One-fifth of a journey is covered at a speed of 30
they form the vertices of an equilateral triangle
km/h, one-fourth of the journey at a speed of
if Joined by straight lines. Rashid travels from
25 km/h and the rest at 60 km/h. What is the
𝑨 to 𝑩 at the speed of 𝟒𝟎 km/h, from B to C at
average speed (in km/h, correct to one decimal
the speed of 𝟔𝟎 km/h and from 𝑪 to 𝑨 at the
place) for the whole journey?
speed of 𝟕𝟐 Km/h. Find the average speed of
Rashid for the entire journey. एक यािा का पांचवां दहथसा 30 km/h की चाल
तीन शहर, 𝑨, 𝑩 और 𝑪, इस तरह जथित हैं कक से तय ककया गया, एक चौिाई यािा 25 km/h की
यदद वे सीधी रे खाओं से िुड़ते हैं तो वे एक चाल से और शेर् 60 km/h की चाल से तय की
समबाहु त्रिभुि के शीर्ष बनाते हैं।राशशद 𝑨 से 𝑩 गई है | परू ी यािा में औसत चाल (km/h में , एक
तक 40 km/h की चाल से, B से C तक 𝟔𝟎 km/h दशमलव थिान तक सही) तया होगी?
की चाल से और 𝑪 से 𝑨 तक 𝟕𝟐 की km/h की a) 𝟑𝟖. 𝟕 b) 𝟑𝟎. 𝟔
c) 𝟒𝟎. 𝟓 d) 𝟐𝟓. 𝟒
चाल से चलता है। पूरी यािा के शलए राशशद की
औसत गतत ज्ञात कीजिए। 5. One-third of a journey is covered at a rate of 40
𝟐 km/h, one-fourth at the rate of 35 km/h, and
a) 𝟓𝟒 Km/h b) 𝟓𝟔 𝟑 km/h
𝟏
the rest at the rate of 45 km/h. The average
c) 𝟓𝟓 km/h d) 𝟓𝟔 𝟑 km/h speed for the whole journey (in km/h) is:
ककसी यािा का एक ततहाई भाग, 40 km/h की एक व्यजतत त्रबंद ु 𝑨 से 𝑩 तक 𝟒𝟎𝒌𝒎/𝒉 की चाल
चाल से तय ककया िाता है , यािा का एक-चौिाई से, तत्पश्चात 𝑩 से 𝑪 तक 𝟓𝟎𝒌𝒎/𝒉 की चाल
भाग, 35 km/h की चाल से तय ककया िाता है से, और तत्पपश्चात 𝑪 से 𝑫 तक 𝟔𝟎𝒌𝒎/𝒉 की
और यािा का शेर् भाग, 45 km/h की चाल से चाल से यािा करता है | 𝑨𝑩, 𝑩𝑪 और 𝑪𝑫 के
तय ककया िाता है। पूरी यािा के शलए औसत मध्य दरू रयों का अनुपात 𝟏: 𝟐: 𝟑 है | वह 𝑫 से A
चाल (km/h में) ज्ञात करें | तक 𝒙 𝒌𝒎/𝒉 की चाल से वापस आता है | यदद
परू ी यािा में उसकी औसत चाल 𝒌𝒎/𝒉 रही
𝟓𝟔𝟕𝟎 𝟔𝟕𝟓𝟎 𝟒𝟖𝟎
a) b)
𝟏𝟖𝟕 𝟏𝟖𝟕 𝟏𝟑
हो, तो 𝒙 का मान ककतना होगा?
𝟔𝟓𝟕𝟎 𝟕𝟓𝟔𝟎
c) d)
𝟏𝟖𝟕 𝟏𝟖𝟕
𝟐 𝟔
a) 𝟑𝟒 𝟕 b) 𝟑𝟐 𝟕
𝟒 𝟓
c) 𝟐𝟖 𝟕 d) 𝟐𝟓 𝟕

6. A man travel from a point 𝑨 to 𝑩 at the speed of 8. A man had to travel 49 km, which he divided into
𝟓𝟎𝒌𝒎/𝒉 and then further from 𝑩 to 𝑪 at six stretches which were exact multiples of
𝟕𝟓𝒌𝒎/𝒉. Then he returns from 𝑪 to 𝑨 at either 4 km or 5 km. The first four stretches
𝒙 𝒌𝒎/𝒉. If the ratio of the distance between 𝑨 were 4 km, 5 km, 8 km and 10 km. Multiples of
to 𝑩 and 𝑩 to 𝑪 is 𝟐 : 𝟑, respectively, and the 4 km and 5 km were travelled at the rates of 40
average speed for the entire journey is and 60 km/h, respectively, and between two
𝟏𝟏 consecutive stretches, he rested for six
𝟔𝟏 𝟒𝟗 𝒌𝒎/𝒉, then what is the value of 𝒙?
minutes. What was his average speed (in km/h)
एक व्यजतत 𝟓𝟎𝒌𝒎/𝒉 की चाल से त्रबंद ु 𝑨 से 𝑩 for the whole journey?
तक की यािा करता है और इसके पश्चात एक व्यजतत को 49 km की यािा करनी है जिसे
𝟕𝟓𝒌𝒎/𝒉 की चाल से 𝑩 से 𝑪 तक की यािा उसने छः भागों में ववभाजित ककया प्रत्येक दरू ी
करता है | किर, वह 𝒙 𝒌𝒎/𝒉 की चाल से 𝑪 से 𝑨 या तो 4 km अिवा 5 km की गुणि है| पहली चार
पर वापस आता है | यदद 𝑨 से 𝑩 की दरू ी और 𝑩 दरू रयां 4 km, 5 km, 8 km और 10 km िी| 4 km
से 𝑪 की दरू ी का क्रमश: अनप
ु ात 𝟐 : 𝟑 हो, और और 5 km के गुणिों वाली दरू रयां क्रमश: 40 और
परू ी यािा के दौरान उसकी औसत चाल 60 km/h की चाल से तय की गई और दो क्रशमक
𝟔𝟏 𝟒𝟗 𝒌𝒎/𝒉 रही हो, तो 𝒙 का मान ककतना होगा?
𝟏𝟏
दरू रयों के बीच उसने 6 शमनट का ववश्राम शलया|
a) 𝟔𝟓 b) 𝟔𝟐. 𝟓 c) 𝟔𝟒. 𝟓 d) 𝟔𝟎 पूरी यािा के दौरान उसकी औसत चाल (km/h

7. A man travels from point 𝑨 to 𝑩 at 𝟒𝟎𝒌𝒎/𝒉, में) ककतनी िी?


𝟕 𝟒
further from 𝑩 to 𝑪 at 𝟓𝟎𝒌𝒎/𝒉, then further a) 𝟑𝟏 𝟏𝟏 b) 𝟑𝟐 𝟏𝟑
from 𝑪 to 𝑫 at 𝟔𝟎𝒌𝒎/𝒉. The ratio between the 𝟑 𝟒
c) 𝟑𝟑 𝟖 d) 𝟑𝟒 𝟏𝟓
distances 𝑨𝑩, 𝑩𝑪 and 𝑪𝑫 is 𝟏: 𝟐: 𝟑. He returns
from 𝑫 to 𝑨 at 𝒙 𝒌𝒎/𝒉. If his average speed for
𝟒𝟖𝟎
the entire journey is 𝟏𝟑 𝒌𝒎/𝒉. Then what is
the value of 𝒙?
9. An car moves through 𝟐𝟎 m in 𝟑 minutes and he से पूरी की| बाइक और कार द्वारा तय की गई
next 𝟓𝟎 km at a speed of 𝟏𝟎𝟎 km/h. What is
the approximate average speed of the car? दरू रयो का अनुपात _____ है |
a) 𝟓: 𝟑 b) 𝟑: 𝟓
एक कार पहले 𝟑 min में 𝟐𝟎 m चलती है और
c) 𝟐: 𝟑 d) 𝟏: 𝟏
अगले 𝟓𝟎 km, 𝟏𝟎𝟎 km/h की चाल से तय करती
है| कार की औसत चाल लगभग ककतनी है ? 12. For the first part of her Journey, Sunita
a) 𝟐𝟐. 𝟑𝟖 m/s b) 𝟐𝟎 m/s travelled at a speed of 𝟒𝟓𝟎 𝒎/𝒎𝒊𝒏 and, for
c) 𝟐𝟓. 𝟐𝟔 m/s d) 𝟐𝟔. 𝟐𝟖 m/s the rest of the journey, at a speed 𝟏. 𝟒 times of
her initial speed. If Sunita travelled a total
10. A man travelled a distance of 47 km in 6 hours. distance of 𝟑𝟑. 𝟑 𝒌𝒎 in 𝟏 hour, what was the
He travelled partly on foot at the rate of distance that Sunita travelled at a lower speed?
𝟏
𝟔 𝒌𝒎/𝒉, and partly on bicycle at the rate of अपनी यािा के पहले भाग के शलया सुनीता ने
𝟐
𝟏
𝟖 𝟐 𝒌𝒎/𝒉. The distance travelled on foot is: 𝟒𝟓𝟎 𝒎/𝒎𝒊𝒏 की चाल से यािा की और शेर् यािा
के शलए, अपनी प्रारं शभक चाल की 𝟏. 𝟒 गुना चाल
कोई व्यजतत, 47 km की दरू ी 6 घंटे में तय करता
से यािा कक| यदद सुनीता ने 1 घंटे में 𝟑𝟑. 𝟑 𝒌𝒎
है| वह पैदल 𝟔 𝟐 𝒌𝒎/𝒉 की चाल से कुछ दरू ी
𝟏

की दरू ी तय की, तो सुनीता ने कम चाल से ककतनी


तय करता है और साइककल द्वारा 𝟖 𝟐 𝒌𝒎/𝒉 की
𝟏

दरू ी तय की?
चाल से बाकी दरू ी तय करता है | पैदल चलकर
a) 𝟏𝟏. 𝟐𝟎𝒌𝒎 b) 𝟏𝟏. 𝟐𝟓𝒌𝒎
तय की गई दरू ी ज्ञात करें | c) 𝟏𝟏. 𝟓𝟎𝒌𝒎 c) 𝟏𝟏. 𝟏𝟓𝒌𝒎
a) 13 km b) 15 km
c) 12 km d) 16 km 13. Ravi drove for 𝟒 hours at a speed of 𝟕𝟎 miles
per hour and for 𝟐 hours at 𝟒𝟎 miles per hour.
What was his average speed ( in miles per hour)
for the whole journey?
रवव ने 𝟕𝟎 मील प्रतत घंटे की चाल 𝟒 घंटे और
11. Ram covered 𝟐𝟎𝟎 km in 10 hours during a trip.
He started his Journey on his bike at the of 𝟏𝟓 𝟒𝟎 मील प्रतत घंटे की चाल से 𝟐 घंटे गाड़ी चलाई|
km/h after that, his bike broke down and he पूरी यिा में उसकी औसत चाल (मील प्रतत घंटे
completed the remaining journey in his friend’s
में) तया िी ?
car at the speed of 𝟐𝟓 km/h . The ratio of the
distances covered on the bike and in the car is: a) 𝟒𝟓 b) 𝟓𝟎 c) 𝟓𝟓 d) 𝟔𝟎
एक दिप में राम 𝟏𝟎 घंटे में 𝟐𝟎𝟎 𝒌𝒎 की दरू ी
14. A car starts at a speed of 𝟑𝟎 km/h and
तय करता है| सबसे पहले उसकी बाइक से increases its speed by 𝟓 km/h after every hour.
𝟏𝟓 𝒌𝒎/𝒉 चाल से अपनी यािा शुरू की उसके Another car has a constant speed of 𝟒𝟎 km/h.
In how much time will both the cars cover an
बाद उसकी बाइक खराब हो गई और उसने शेर् equal distance?
यािा अपने दोथत की कार से 𝟐𝟓 𝒌𝒎/𝒉 की चाल एक कार 𝟑𝟎 km/h की चाल से चलना शुरू करती
है और प्रत्येक घंटे के बाद अपनी चाल में 𝟓
km/h की वद्
ृ धध करती है| एक अन्य कार 17. A person travels a distance of 300 km and then
returns to the starting point. The time taken by
𝟒𝟎 km/h की तनयत (जथिर) चाल से चल रही है | him for the outward journey is 5 hours more
ककतने समय में दोनों कारें समान दरू ी तय करे गी? than the time taken for the return journey. If he
a) 𝟔 hours b) 𝟑hours returns at a speed of 10 km/h more than the
c) 𝟓 hours d) 𝟒 hours speed of going, what is the average speed (in
km/h) for the entire journey?
15. A person runs at an average speed of 𝟐𝟎 𝒌𝒎/𝒉 कोई व्यजतत 𝟑𝟎𝟎𝒌𝒎 की दरु ी तय करता है और
for 𝟑𝟎 minutes and cycles at a speed of
किर आरं शभक त्रबंद ु पर वापस आता है| उसके
𝟑𝟐 𝒌𝒎/𝒉 for 𝟏 hour 𝟏𝟓 minutes. Calculate the
average speed ( in km/h, off to 𝟏 decimal द्वारा िाने में लगा समय, वापस आने में लगे
place). समय से 𝟓 घंटे अधधत है | यदद वह िाने की चल
एक व्यजतत 30 शमनट के शलए 𝟐𝟎 𝒌𝒎/𝒉 की से 𝟏𝟎𝒌𝒎/𝒉 की अधधक चाल से वापस लौटता
औसत गतत से दौड़ता है और 1 घंटे 15 शमनट के है , तो पूरी यािा के दौरान औसत चाल (𝒌𝒎/𝒉
शलए 𝟑𝟐 𝒌𝒎/𝒉 की गतत से साइककल चलाता है। में) ज्ञात करें |
औसत गतत की गणना करें (ककमी/घंटा में , 1 a) 12 b) 15 c) 24 d) 30
दशमलव थिान तक)।
a) 𝟐𝟓. 𝟒 b) 𝟐𝟔. 𝟑 18. A train is to cover 𝟑𝟕𝟎 km at a uniform speed.
c) 𝟐𝟖. 𝟔 d) 𝟑𝟎. 𝟓 After running 𝟏𝟎𝟎 km. the train could run at a
speed 𝟓 km/h less than is normal speed due to
some technical fault. The train got delayed by
𝟑𝟔 minutes. What is the normal speed of the
train, in km/h?
16. Akhil takes 30 minutes extra to cover a distance एकसमान चाल से चलते हुए, ककसी रे लगाड़ी को
of 150 km if he drives 10 km/h slower than his 𝟑𝟕𝟎 km की दरू ी तय करती िी| 𝟏𝟎𝟎 km चलने
usual speed. How much time will he take to
drive 90 km if he drives 15 km per hour slower के बाद, कुछ तकनीकी खराबी के कारण रे लगाड़ी
than his usual speed? अपनी सामान्य चाल से 𝟓 km/h की कम चाल
अखखल यदद अपनी सामान्य चाल की तुलना में से चलती है | रे लगाड़ी 𝟑𝟔 शमनट दे री से पहुंची|
10 km/h धीमी चाल से गाड़ी चलाता है तो उसे रे लगाड़ी की सामान्य चाल (km/h में) ककनती िी?
150 km की दरू ी तय करने में 30 शमनट अधधक a) 40 b) 48 c) 50 d) 45
समय लगता है | यदद वह अपनी सामान्य चाल
19. A boat sails downstream from a point P to point
की तुलना में 15 km/h धीमी चाल से गाड़ी चलाता Q, which is 24 km away from P, and then
है, तो उसे 90 km की दरू ी तय करने में ककतना returns to P. If the actual speed of the boat (in
still water) is 6 km/h, the entire trip from P to
समय लगेगा?
Q takes 3 hours less than that from Q to P.
a) 2 h 45 m b) 2 h 30 m What should be the actual speed (in km/h) of
c) 2 h d) 2 h 15 m the boat to sail from P to Q in 2 hours?
कोई नाव, धारा की ददशा में त्रबंद ु P से त्रबंद ु Q तक तो उस त्रबंद,ु िहााँ वे दोनों शमलेंगी, से प्रारं शभक
िाती है , िो P से 24 km की दरू ी पर है और किर त्रबंद ु की दरु ी ज्ञात कीजिए|
त्रबंद ु P पर वापस आती है | यदद नाव की वाथतववक a) 𝟐𝟒𝟎 𝒎 b) 𝟑𝟎𝟎 𝒎
c) 𝟑𝟏𝟓 𝒎 d) 𝟐𝟔𝟓 𝒎
चाल (शांत िल में) 6 km/h है, तो P से Q तक
की पूरी यािा से 3 घंटा कम समय लगता है | P 22. In a 𝟏𝟐𝟎𝟎 m race ratio of the speeds of two
से Q तक 2 घंटे में िाने के शलए नाव की contestants Meenal and Nitu is 𝟓: 𝟕 . If Meenal
has a start of 𝟓𝟎𝟎 m , then Meenal wins by:
वाथतववक चाल (km/h में) ज्ञात करें |
𝟏𝟐𝟎𝟎 m दौड़ में , दो प्रततयोधगयों मीनल और नीतू
a) 12 b) 15 c) 10 d) 22
की चालों का अनुपात 𝟓: 𝟕 है | यदद मीनल 𝟓𝟎𝟎
20. A motorboat whose speed is 𝟐𝟎 km/h in still m आगे से दौड़ना शरू
ु करती है , तो मीनल
water takes 30 minutes more to go 𝟐𝟒 km
ककतने m से िीतेगी?
upstream than to cover the same distance
downstream. If the speed the boat in still water a) 𝟐𝟓𝟎 m b) 𝟐𝟐𝟓 m
is increased by 𝟐 𝒌𝒎/𝒉, then how much time c) 𝟐𝟐𝟎 m c ) 𝟐𝟒𝟎 m
will it take to go 𝟑𝟗 𝒌𝒎 downstream and 30 km
upstream? 23. Rakesh runs 𝟒/𝟑 times as fast as Mukesh. In a
race, if Rakesh gives a lead of 𝟔𝟎 𝒎 to Mukesh,
एक मोटरबोट जिसकी शांत िल में चाल find the distance from the starting point where
𝟐𝟎𝒌𝒎/𝒉, धारा की ववपरीत ददशा में 𝟐𝟒𝒌𝒎 both of them will meet.
िाने में , धारा की ददशा में उतनी ही दरू िाने में राकेश मुकेश से 𝟒/𝟑 गुना तेि दौड़ता है।एक दौड़
शलए गए समय से, 30 शमनट अधधक का समय में , यदद राकेश मुकेश को 𝟔𝟎 𝒎 की बढ़त दे ता है ,
लेती है | यदद शांत िल में नाव की चाल में तो प्रारं शभक त्रबंद ु से वह दरू ी ज्ञात कीजिए िहां
𝟐𝒌𝒎/𝒉 की वद्
ृ धध हो िाती है , तो उसे धारा की वे दोनों शमलेंगे।
ददशा में 𝟑𝟗𝒌𝒎 और धारा की ववपरीत ददशा a) 𝟐𝟔𝟒 m b) 𝟐𝟒𝟎 m c) 𝟐𝟓𝟓 m d) 𝟐𝟑𝟐 m

𝟑𝟎𝒌𝒎 िाने में ककतना समय लगेगा? 24. A can beat B in a 𝟏𝟎𝟎-metre race by 𝟏𝟎 metres,
a) 𝟐 𝒉 𝟒𝟎 𝒎 b) 𝟑 𝒉 𝟏𝟎 𝒎 B can beat C in a 𝟏𝟎𝟎-metre race by 𝟏𝟎-metres.
c) 𝟑 𝒉 𝟒𝟎 𝒎 d) 𝟐 𝒉 𝟓𝟎 𝒎 What is the ratio (𝒕𝑨 : 𝒕𝑩 : 𝒕𝒄 ), which are the
respective times taken by 𝑨, 𝑩 and 𝑪 to
complete the race?
A 𝟏𝟎𝟎 m की दौड़ में B को 𝟏𝟎 m से हरा सकता
21. Riya runs 𝟑/𝟐 times as fast as Prerna. In a race, है | B 𝟏𝟎𝟎 m की दौड़ में C को 𝟏𝟎 m से हरा
If Riya gives a lead of 𝟏𝟎𝟎 𝒎 to Prerna, find the सकता है | अनुपात (𝒕𝑨 : 𝒕𝑩 : 𝒕𝒄 ) तया है , िो दौड़
distance Riya has to run before both of them
को पूरा करने के शलए 𝑨, 𝑩 और 𝑪 द्वारा शलया
meet.
ररया प्रेरणा से 𝟑/𝟐 गुना तेि दौड़ती है। एक दौड़ गया संबंधधत समय है ?
a) 𝟏𝟎𝟎 : 𝟗𝟎 : 𝟖𝟏 b) 𝟗𝟎 : 𝟖𝟏 : 𝟏𝟎𝟎
में , यदद ररया प्रेरणा को 𝟏𝟎𝟎 𝒎 की बढ़त दे ती है ,
c) 𝟏𝟎𝟎 : 𝟖𝟏 : 𝟗𝟎 d) 𝟖𝟏 : 𝟗𝟎 : 𝟏𝟎𝟎
प्रतत घंटे की दर से लगाती है।वे दोनों 𝟎𝟐: 𝟒𝟓
25. In a 200 m walk race, Zakia walks at an average
speed of 5 km/h. She gives a start of 20 m to p.m. पर एक ही त्रबंद ु से समान ददशा में मे
Veena and still beats her by 18 seconds. What चलना शुरू करते हैं। वे पहली बार ककस समय
is the average speed of Veen (in km/h) ? एक दस ू रे को पार करें गे?
200 m की दौड़ में , िककया 5 km/h की औसत a) 𝟎𝟑: 𝟎𝟎 a.m. b) 𝟎𝟑: 𝟎𝟓 p.m.
चाल से दौड़ती है। वह वीना से 20 m पीछे से c) 𝟎𝟑: 𝟎𝟎 p.m. c) 𝟎𝟐: 𝟓𝟎 p.m.
दौड़ती है और किर भी उसे 18 सेकंड से हराती
28. Anup can row 𝟑𝟑 𝒌𝒎 downstream and 𝟑𝟓 𝒌𝒎
है। वीन की औसत चाल (km/h में) ज्ञात करें | upstream in 𝟖 hours. He can also row 𝟒𝟒 𝒌𝒎
a) 𝟒. 𝟐 b) 𝟒. 𝟓 downstream and 𝟐𝟖 𝒌𝒎 upstream in the same
c) 4 d) 𝟑. 𝟓 time. How much time (in hours) will he take to
row 𝟓𝟓 𝒌𝒎 downstream and 𝟏𝟒 𝒌𝒎 upstream
?
अनूप धारा की ददशा में 𝟑𝟑 𝒌𝒎 की दरू ी और धारा

26. Anjali and Babita are running on a circular track की ववपरीत ददशा में 𝟑𝟓 𝒌𝒎 की दरू ी 𝟖 घंटे में तैर
in opposite directions from same time at same कर तय कर सकता है। वह इतने ही समय में
point with speeds of 𝟖 m/sec and 𝟔 m/sec,
धारा की ददशा में 𝟒𝟒 𝒌𝒎 और धारा की ववपरीत
respectively. If the length of the circular track
is 𝟗𝟔𝟎 m, how many times they will meet at ददशा में 𝟐𝟖 𝒌𝒎 की दरू ी तैर कर तय कर सकता
distinct points ? है। धारा की ददशा 𝟓𝟓 𝒌𝒎और धारा की ववपरीत
अंिशल और बबीता एक वत्त
ृ ाकार िै क पर एक ही ददशा में 𝟏𝟒 𝒌𝒎 की दरू ी तय करने में वह ककतना
समय और एक ही त्रबंद ु से ववपरीत ददशाओं में समय (घंटों में) लेगा?
क्रमश 𝟖 m/sec और 𝟔 m/sec की चला से दौड़ a) 𝟗 b) 𝟔 c) 𝟖 d) 𝟕
रही है | यदद वत
ृ ाकार िै क की लंबाई 𝟗𝟔𝟎 m है,
29. A motorboat can go to 𝟖. 𝟒𝒌𝒎 downstream
तो वे अलग-अलग शमलन त्रबंदओ
ु ं पर ककतनी बार and 𝟒. 𝟖𝒌𝒎 upstream in 𝟏 hour. It can go
शमलेगी ? 𝟏𝟕. 𝟓𝒌𝒎 downstream and 𝟗𝒌𝒎 upstream in 𝟐
a) 𝟏𝟐 times b) 𝟕 times hours. How much time (in hours) will it take to
c) 𝟔 times d) 𝟏𝟒 times go 𝟑𝟏. 𝟐𝒌𝒎 in still water?
एक मोटरबोट 𝟏 घंटे में धारा की ददशा में 𝟖. 𝟒𝒌𝒎
27. Malay walks around a circular field at the rate
और धारा के ववपरीत ददशा में 𝟒. 𝟖𝒌𝒎 िा सकती
of 𝟕 rounds per hour, while Neha runs around
it at the rate of 𝟏𝟏 rounds per hour. They start है | यह 𝟐 घंटे में धारा की ददशा में 𝟏𝟕. 𝟓𝒌𝒎
in the same directions from the same point at और धारा के ववपरीत ददशा में 𝟗𝒌𝒎 िा सकती
𝟎𝟐: 𝟒𝟓 p.m. They shall first cross each other at:
है | शांत िल में 𝟑𝟏. 𝟐𝒌𝒎 की दरू ी तय करने में
मलय एक वत्त
ृ ाकार मैदान के चतकर
इसे ककतना समय (घंटे में) लगेगा?
𝟕 चतकर प्रतत घंटे की दर से लगाता है , िबकक
a) 𝟐. 𝟒 b) 𝟏. 𝟐 c) 𝟑. 𝟗 d) 𝟏. 𝟑
नेहा उसी वत्त
ृ ाकार मैदान के चतकर 𝟏𝟏 चतकर
30. The speed of a boat downstream is to the time taken by him to row 𝟒 km
𝟏𝟓𝟎% more than its speed upstream. If the upstream. What is the speed of the steam?
time taken by the boat for going 80 km एक आदमी 𝟏𝟐 h में 𝑨 से 𝑩 (धारा की ववपरीत
downstream and 50 km upstream is 8.2 hours,
then what is the speed (in km/h) of the boat ददशा में ) और 𝑩 से 𝑨 (धारा की ददशा में ) तक
downstream? नाव खेता है | A और 𝑩 के बीच की दरू ी 𝟐𝟒𝟎
धारा की ददशा में , ककसी नाव की चाल धारा की km है | धारा की ददशा में 𝟔 km की दरू ी तय करने
ववपरीत ददशा की इसकी चाल से 𝟏𝟓𝟎% अधधक में व्यजतत द्वारा शलया गया समय, धारा की
है| यदद धारा की ददशा में 80 km और धारा की ववपरीत ददशा में 𝟒 km की दरू ी तय करने में उसके
ववपरीत ददशा में 50 km की दरू ी तय करने में द्वारा शलए गए समय के समान है | धारा की चाल
लगा समय 8.2 घंटा है , तो धारा की ददशा में नाव ककतनी है ?
की चाल (ककमी. में) ज्ञात करें | a)
𝟑𝟓
𝒌𝒎/𝒉 b)
𝟐𝟓
𝒌𝒎/𝒉
𝟑 𝟑
a) 16 b) 30 c)
𝟒𝟔
𝒌𝒎/𝒉 d)
𝟓𝟎
𝒌𝒎/𝒉
c) 24 d) 25 𝟑 𝟑

33. The speed of a boat is 𝟏𝟐 𝒌𝒎/𝒉 in still water.


It goes 𝟏𝟖 𝒌𝒎 downstream and comes back in
𝟔
a total time 𝟔 𝟕 hours. Find the speed of the
31. A man can row a distance of 8 km downstream stream (in Km/h).
in a certain time and can row 6 km upstream in जथिर िल मे एक नाव की चाल 𝟏𝟐 𝒌𝒎/𝒉 है |
the same time. If he rows 24 km upstream and
𝟑 धारा की ददशा मे 𝟏𝟖 𝒌𝒎 िाने और वापस आने
the same distance downstream in 𝟏 𝟒 hours,
मे इसे कुल 𝟔 𝟕 घंटे का समय लगता है | धारा की
𝟔
then the speed (in km/h) of the current is:
एक आदमी, धारा की ददशा में 8 km की दरू ी एक चाल ( Km/h मे) ज्ञात कीजिए|
तनजश्चत समय तय कर सकता है और उतने ही a) 𝟖 b) 𝟕 c) 𝟔 d) 𝟗

समय में धारा की ववपरीत ददशा में 6 km की दरू ी 34. A boat covers a round trip journey between
तय कर सकता है | यदद वह धारा की ववपरीत two points A and B in a river in T hours. If its
speed in still water becomes 2 times, it would
ददशा में 24 km की दरू ी और धारा की ददशा में 𝟖𝟎
take 𝟏𝟔𝟏 𝑻 hours for the same journey. Find the
उतनी ही दरू ी 𝟏 𝟒 घंटा में तय करता है , तो धारा
𝟑
ratio of its speed in still water to the speed of
की चाल (km/h में) ज्ञात करें | the river.
कोई नाव, ककसी नदी में त्रबन्दओ
ु ं A और B के बीच
𝟏 𝟏
a) 𝟒 𝟐 b) 𝟐 𝟐 c) 𝟒 d) 𝟑
राउं ड-दिप यािा वाली दरू ी को T घंटे में तय करती
32. A man rows from 𝑨 to 𝑩 (upstream) and 𝑩 to 𝑨
है | यदद जथिर िल में इसकी चाल 2 गुनी हो
(downstream) in 𝟏𝟐 hours. The distance
िाती है , तो इसे उसी दरू ी तय करने में 𝑻 घंटे
𝟖𝟎
between 𝑨 to 𝑩 is 𝟐𝟒𝟎 km. The time taken by 𝟏𝟔𝟏
the man to row 𝟔 km downstream is identical
लगें गे| जथिर िल में इसकी चाल का, नदी की 37. A train can completely cross a 𝟏𝟎𝟎𝟎 −metre
long bridge in 𝟏𝟓 seconds. It can completely
चाल से अनुपात ज्ञात करें | cross another 𝟏𝟑𝟎𝟎 −metre long bridge in 𝟏𝟗
a) 𝟏𝟏 : 𝟏 b) 𝟏𝟔𝟏 : 𝟒𝟎 seconds. What is the length of the train?
c) 𝟏 : 𝟏𝟏 d) 𝟐 : 𝟏
एक िे न 𝟏𝟎𝟎𝟎 m लंबे पुल को 𝟏𝟓 सेकंड में पूरी
35. X, Y are two points in river. Points P and Q तरह से पार कर सकती है| यह 𝟏𝟑𝟎𝟎 m लंबे एक
divide the straight line XY into three equal अन्य पुल को 𝟏𝟗 सेकेंड में पूरी तरह से पार कर
parts. The river flows along XY and the time
taken by a boat to row from X to Q and from Y सकती है | िे न की लंबाई ककतनी है ?
and Q are in the ratio 𝟒 : 𝟓. The ratio of the a) 𝟏𝟐𝟓 metres b) 𝟐𝟎𝟎 metres
speed of the boat downstream to that of the C) 𝟏𝟓𝟎 metres d) 𝟏𝟕𝟓 metres
river current is equal to:
38. A train travelling at the speed of 𝒙 km/h
X और Y ककसी नदी पर दो त्रबंद ु हैं| त्रबंद ु P और Q
crossed a 300 m long platform in 30 seconds,
सीधी रे खा XY को तीन बराबर भागों में ववभाजित and overtook a man walking in the same
करते है | नदी, XY के समानांतर बहती है और direction at 6 km/h in 20 seconds. What is the
value of 𝒙?
ककसी नाव द्वारा X से Q तक और Y से Q तक
𝒙 𝒌𝒎/𝒉 की चाल से चलने वाली रे लगाड़ी, 𝟑𝟎𝟎𝒎
की यािा में लगने वाले समय का अनुपात 𝟒 : 𝟓
लम्बे प्लेटफ़ॉमष को 30 सेकंड में पार करती है
है| बहाव की ददशा में नाव की चाल और नदी की
और उसी ददशा में 𝟔𝒌𝒎/𝒉 की चाल से चल रहे
धारा की चाल का अनुपात ज्ञात करें |
व्यजतत को 20 सेकंड में ओवरटे क करती है | 𝒙
a) 𝟏𝟎 : 𝟑 b) 𝟑 : 𝟏𝟎
c) 𝟑 : 𝟒 d) 𝟕 : 𝟑 का मान ज्ञात करें |
a) 60 b) 96 c) 48 d) 102

39. A and B leave from point M at the same time


towards point N. B reaches N and stats towards
36. Two trains having lengths of 𝟐𝟑𝟎𝒎 and 𝟐𝟒𝟎𝒎 M instantly. He meets A at point O (between M
are 𝟏𝟑𝟎𝒎 apart. They start moving toward and N). The distance between M and N is
each other on parallel tracks, at speeds of 𝟐𝟒𝟓 Km. If the speeds of A and B are 𝟐𝟔 km/h
𝟏𝟔𝟎𝒌𝒎/h and 𝟐𝟎𝟎𝒌𝒎/h, respectively. In how and 𝟔𝟓 km/h, respectively, then what is the
much time will the trains cross each other? distance between O and N ?
𝟐𝟑𝟎𝒎 और 𝟐𝟒𝟎𝒎 लम्बाई वाली दो िे नें 𝟏𝟑𝟎𝒎 A और B एक ही समय पर त्रबंद ु M से त्रबंद ु N
की दरु ी पर है | वे सामानांतर पटररयों पर क्रमशः की और चलना शरू ु करते हैं| B, N पर पहुाँचता
𝟏𝟔𝟎𝒌𝒎/h और 𝟐𝟎𝟎𝒌𝒎/h की चाल से एक दस
ु रे है और तरु ं त वापस M की और चलना शरू ु करता
की ओर चलना सुरु करती है| िे नें एक दस
ु रे को है | वह त्रबंद ु O (M और N के बीच) पर A से
ककतने समय में पार करें गी? शमलता है | M और N के बीच की दरू ी 𝟐𝟒𝟓 Km
a) 𝟖 sec b) 𝟕 sec c) 𝟔 sec d) 𝟓 sec है | यदद A और B की चाल क्रमश: 𝟐𝟔 km/h और
𝟔𝟓 km/h है, तो O और N के बीच की दरू ी ककतनी 42. A takes 8 hours more than the time taken by B
to cover a distance of 160 km. If A doubles his
है? speed, he takes 3 hours more than B to cover
a) 𝟏𝟏𝟓 km b) 𝟏𝟎𝟓 km c) 𝟗𝟓 Km d) 𝟏𝟒𝟎 km the same distance. The speed (in km/h) of B is:
A को 160 km की दरू ी तय करने में , B की तुलना
40. 𝑷 starts from A at 𝟔 a.m and reaches 𝑩 after 𝟖
hours. 𝑸 starts from B at 𝟖 a.m and reaches 𝑨 में 8 घंटे अधधक समय लगता है | यदद A अपनी
after 𝟗 hours. When they meet, what is the चाल को दोगुना करता है , तो उसी दरू ी को तय
ratio of the distance travelled by 𝑸 to the
distance travelled by 𝑷? करने में , उसे B की तुलना में 3 घंटा अधधक समय
𝑷, थिान A से 𝟔 a.m पर चलना शुरू करता है लगता है | B की चाल (km/h में) ज्ञात करें )
a) 72 b) 70 c) 75 d) 80
और 𝟖 h बाद थिान B पर पहुाँच िाता है| 𝑸,
थिान B से 𝟖 a.m पर चलना शुरू करता है और 43. Mohan takes 2 hours more than Kishore to
𝟗 h बाद थिान 𝑨 पर पहुाँच िाता है | जिस समय walk 63 km. If Mohan increases his speed by
𝟓𝟎%, then he can make it in 1 hour less than
वे शमलते है , उस समय तक 𝑸 द्वारा तय की गई Kishore. How much time does Kishore take to
दरू ी और 𝑷 द्वारा तय की गई दरू ी का अनुपात walk 63 km?
तया है ? मोहन को 63 km की दरू ी तय करने में , ककशोर से
a) 𝟑 : 𝟖 b) 𝟔 : 𝟕 2 घंटा अधधक समय लगता है | यदद मोहन अपनी
c) 𝟔 : 𝟏𝟏 d) 𝟖 : 𝟑
चाल में 𝟓𝟎% की वद्
ृ धध करता है , तो वह इस दरू ी
को ककशोर से 1 घंटे कम समय में तय कर सकता
है | 63 km की दरू ी तय करने में , ककशोर को

41. A bus starts from a bus station after every 𝟑𝟎 ककतना समय लगेगा?
minutes in the same direction. The speed of a) 7 hours b) 9 hours
each bus is the same. Z is riding a bike in the c) 6 hours d) 5 hours
opposite direction at a speed of 𝟐𝟓 km/h. Z
meets each bus in 𝟐𝟎 minutes. What is the 44. The distance covered by 𝑨 in 𝟏𝟎 hours is 𝟐𝟕𝒌𝒎
speed of each bus ? less than the distance covered by 𝑩 in 𝟏𝟐
एक बस एक बस-थटे शन से प्रत्येक 𝟑𝟎 min के hours. The distance covered by 𝑩 in 𝟖 hours is
𝟏
𝟕 𝟐 𝒌𝒎 more than the distance covered by 𝑨 in
बाद उसी ददशा में चलना शुरू करती है| प्रत्येक
𝟗 hours. If they start moving towards each
बस की चाल समान है| Z ववपरीत ददशा में 𝟐𝟓 other from the two ends of a road, 𝟒. 𝟐𝒌𝒎 in
km/h की चाल से बाइक चला रहा है | Z प्रत्येक length at the same time, they will cross each
other after.
बस से 𝟐𝟎 min में शमलता है | प्रत्येक बस की
𝑨 द्वारा 𝟏𝟎 घंटे में तय की गई दरू ी, 𝑩 द्वारा
चाल ककतनी है ?
a) 𝟓𝟎 km/h b) 𝟕𝟓 km/h 𝟏𝟐 घंटे में तय की गई दरू ी से 𝟐𝟕 𝒌𝒎 कम है|
c) 𝟑𝟎 km/h d) 6𝟎 km/h 𝑩 द्वारा 𝟖 घंटे में तय की गई दरू ी, 𝑨 द्वारा 𝟗
घंटे में तय की गई दरू ी से 𝟕 𝟐 𝒌𝒎 अधधक है |
𝟏
यदद वे 𝟒. 𝟐 𝒌𝒎 लंबी एक सड़क के दोनों शसरों से सामान्य चाल से रे लगाड़ी को 300 km की दरू ी
एक-दस
ू रे की और एक साि चलना प्रांरभ करें तो तय करने में ककतना समय लगेगा?
वे ककतने समय के पश्चात एक-दस
ू रे को पार a) 6 hours b) 5 hours 15 minutes
c) 5 hours d) 5 hours 25 minutes
करें गे?
𝟏
a) 𝟐𝟑 𝟑 𝒎𝒊𝒏𝒖𝒕𝒆𝒔 b) 𝟐𝟒 𝒎𝒊𝒏𝒖𝒕𝒆𝒔 47. A takes 2 hours more than B to cover a distance
𝟐 𝟏
c) 𝟐𝟒 𝟑 𝒎𝒊𝒏𝒖𝒕𝒆𝒔 d) 𝟐𝟏 𝒎𝒊𝒏𝒖𝒕𝒆𝒔 of 40 km. If A doubles his speed, he takes 𝟏 𝟐
hour more than B to cover 80 km. To cover a
𝟑
45. The average speed of a car 𝟏 𝟓 times the distance of 120 km, how much time (in hours)
will B take travelling at his same speed?
average speed of a bus. A tractor covers 𝟏𝟏𝟓𝟎
km in 𝟐𝟑 hours. How much distance will the car 40 km की दरू ी तय करने में A, B से 2 h अधधक
cover in 𝟒 hours if the speed of the bus is twice लेता है | अगर A अपनी चाल को दोगुनी करता है ,
the speed of the tractor?
तो 80 km तय करने में वह B से 𝟏 𝟐 h अधधक
𝟏

एक कार की औसत चाल एक बस की औसत


लेता है | B को अपनी चाल से 120 km की दरू ी
चाल की 𝟏 𝟓 गुनी है| एक िे तटर 𝟐𝟑 घंटे में
𝟑
तय करने में ककतना समय (h में) लगेगा?
𝟏𝟏𝟓𝟎km की दरू ी तय करता है | यदद बस की चाल 𝟐 𝟏 𝟏 𝟏
a) 𝟏 𝟑 b) 𝟏 𝟒 c) 𝟏 𝟑 d) 𝟏 𝟐
िे तटर की चाल की दोगन
ु ी है तो कार 𝟒 घंटे में
ककतनी दरू ी तय करे गी? 48. Suresh and Dinesh travel a distance of 120 km
a) 𝟒𝟒𝟎 Km b) 𝟕𝟒𝟎 km such that speed of Suresh is more than that of
c) 𝟔𝟒𝟎 km d) 𝟓𝟒𝟎 km Dinesh. The sum of their speeds is 75 km/h, and
the total time taken by both is 6 hours and 40
minutes. The speed of Suresh is:
सरु े श और ददनेश 120 km की यािा इस तरह करते
हैं कक सुरेश की चाल, ददनेश की चाल से अधधक
46. A train runs first 75 km at a certain uniform
speed and next 90 km at an average speed of होती है | उनकी चालों का योग 75 km/h है , और
10 km/h more than the normal speed. If it takes दोनों के द्वारा शलया गया समय 6 घंटे और 40
3 hours to complete the journey, then how
much time will the train take to cover 300 km शमनट है | सुरेश की चाल ज्ञात कीजिए|
with normal speed? a) 45 km/h b) 42 km/h
c) 40 km/h d) 30 km/h
एक रे लगाड़ी, पहले 75 km की दरू ी तक तनजश्चत
एकसमान चाल से तय करती है और अगले 90 49. Suman travels from place X to Y and Rekha
km की दरू ी सामान्य चाल से 10 km/h अधधक travels from Y to X, simultaneously. After
meeting on the way, Suman and Rekha reach Y
की औसत चाल से तय करती है | यदद यािा को
and X, in 3 hours 12 minutes and one hour 48
पूरा करने में 3 घंटे का समय लगता है , तो minutes, respectively. If the speed of Rekha is 9
km/h, then the speed (in km/h) of Suman is:
एक ही समय पर, सुमन थिान X से Y के शलए त्रबना रूकावट के ककसी बस की चाल 40 km/h है
चलती है और रे खा थिान Y से X के शलए चलती और रूकावट के साि 32 km/h है | प्राजप्त घंटे बस
है| राथते में शमलने के बाद, सुमन और रे खा थिान ककतने शमनट के शलए रूकती है ?
Y और X पर क्रमश: 3 घंटा 12 शमनट और एक a) 12 b) 18 c) 15 d) 16
घंटा 48 शमनट में पहुाँचती हैं| यदद रे खा की चाल 52. Aman travels a certain distance at 𝒙 𝒌𝒎/𝒉. In
9 km/h है, तो सुमन की चाल (km/h में) ज्ञात the return journey. He covers 𝟐𝟎% of the same
करें | distance in 𝟑𝟓% of the time taken to cover the
𝟏 𝟑 earlier distance. For the remaining part of the
a) 𝟕 𝟐 b) 𝟔 𝟒 c) 6 d) 8 return journey, if his speed is 𝒚 𝒌𝒎/𝒉. and he
can cover the entire return journey in the same
50. Two cyclists start simultaneously from two time as for the onward journey, then which of
points A and B, their destinations being B and the following is true?
A, respectively. After crossing each other, they अमन एक तनजश्चत दरू ी को 𝒙 𝒌𝒎/𝒉 की चाल
precisely take 2 hours 33 minutes 36 seconds
and 1 hour 26 minutes 24 seconds respectively से तय करता है | वापसी की यािा में , वह उसी दरू ी
to reach their destinations. What is the ratio of का 𝟐𝟎% भाग, पूवव
ष ती दरू ी तय करने में लगे
the speed of the speed of the first to that of the
समय के 𝟑𝟓% भाग में तय करता है| यदद वापसी
second cyclist?
दो साइककल चालक दो त्रबंदओ की यािा के शेर् भाग में उसकी चाल 𝒚 𝒌𝒎/𝒉
ु ं क्रमशः A और B
से एक साि चलना शुरू करते हैं, उनके गंतव्य रही हो, और वह वापसी की पूरी यािा को, िाते

क्रमशः B और A हैं। एक दस समय की यािा में लगे समय के समान समय में
ू रे को पार करने के
बाद, वे अपने गंतव्य तक पहुंचने के शलए क्रमशः पूरा कर सकता हो, तो इनमें से कौन-सा सही है ?
a) 𝟏𝟑𝒚 = 𝟏𝟔𝒙 b) 𝟖𝒚 = 𝟏𝟓𝒙
2 घंटे 33 शमनट 36 सेकंड और 1 घंटा 26 c) 𝟏𝟏𝒚 = 𝟏𝟓𝒙 d) 𝟏𝟓𝒚 = 𝟏𝟗𝒙
शमनट 24 सेकंड का समय लेते हैं। पहले साइककल
चालक दस 53. In a race, Anil beats Bhushan by 𝟑𝟎 seconds. In
ू रे साइककल चालक की चाल के अनुपात
the same race, Bhushan beats Raghav by 𝟔𝟎
की गणना करें ? seconds. Bhushan’s speed is the average speed
a) 𝟐 : 𝟑 b) 𝟑 : 𝟐 of Anil and Raghav. The time (in seconds) taken
c) 𝟒 : 𝟑 d) 𝟑 : 𝟒 by Bhushan to run the race is:
एक दौड़ में , अतनल ने भर्
ू ण को 𝟑𝟎 सेकेंड से
पराजित ककया| इसी दौड़ में , भर्
ू ण ने राघव को
𝟔𝟎 सेकेंड से पराजित ककया| भर्
ू ण की चाल,
51. The speed of a bus without stoppages is 40
अतनल और राघव की औसत चाल के बराबर है |
km/h and with stoppages is 32 km/h. How
many minutes per hour does the bus stop? भर्
ू ण द्वारा इस दौड़ को परू ा करने में लगा समय
(सेकेंड में) ककतना है ?
a) 𝟗𝟎 b) 𝟏𝟐𝟎
c) 𝟕𝟓 d) 𝟔𝟎

Answer Key
1. B 2. A 3. A 4. A 5. D
6. D 7. C 8. B 9. C 10. A
11. B 12. B 13. D 14. C 15. C
16. C 17. C 18. C 19. C 20. B
21. B 22. C 23. B 24. D 25. C
26. B 27. C 28. D 29. A 30. D
31. C 32. B 33. D 34. A 35. A
36. C 37. A 38. B 39. B 40. C
41. A 42. D 43. A 44. B 45. C
46. A 47. D 48. A 49. B 50. D
51. A 52. A 53. B
Profit & Loss

Instruction: - For Video Solution scan QR code 4. 𝑨 varies jointly with 𝑩 and 𝑪. 𝑨 = 𝟔 when 𝑩 =
given below or you can also CLICK on Click here 𝟑 and 𝑪 = 𝟐. Find 𝑨 and 𝑩 = 𝟓 and 𝑪 = 𝟕.
button to watch the Video Solution. A सयक्
ु त रूप से 𝑩 और 𝑪 का अनक्र्
ु मानप
ु ती है
| 𝑩 = 𝟑 और 𝑪 = 𝟐 होने पर 𝑨 = 𝟔 होता है |
𝑩 = 𝟓 और 𝑪 = 𝟕 होने पर, 𝑨 का मान ज्ञात
ककजजए |
Variation a) 𝟏𝟕. 𝟓 b) 𝟏𝟎𝟓 c) 𝟑𝟓 d) 𝟕𝟎
1. 𝑨 varies directly as 𝑩 and inversely as C. When
𝟒 𝟗 𝟑
𝑨 = 𝟑 and 𝑪 = 𝟏𝟒, 𝑩 = 𝟕, When 𝑨 = 𝟒√𝟑 and 5. 𝟏𝟓 men can do a work in 𝟏𝟓 days working 𝟖
hours a day. 𝑵 men can do twice the work in
𝑩 = 𝟑𝟎, then 𝑪 =?
𝟐𝟎 days working 𝟏𝟎 hours a day. What is the
𝑨, 𝑩 के अनुक्रमानुपाती और 𝑪 के व्युत्क्क्रमानुपाती
value of 𝑵 ?
है| जब 𝑨 = 𝟑 और 𝑪 = 𝟏𝟒, है , तो 𝑩 = 𝟕 है , जब
𝟒 𝟗 𝟑
एक काम को 𝟏𝟓 आदमी प्रततददन 𝟖 घंटे काम
𝑨 = 𝟒√𝟑 और 𝑩 = 𝟑𝟎 है, तो 𝑪 का मान ज्ञात
करते हुए 𝟏𝟓 ददन में पूरा कर सकते हैं| 𝑵
कीजजए|
आदमी प्रततददन 𝟏𝟎 घंटे काम करते हुए 𝟐𝟎
a) 𝟓√𝟑 b) 𝟔√𝟓 c) 𝟔√𝟑 d) 𝟑√𝟓
ददनों में उसी काम का दोगुना काम कर सकते
2. 𝑹 varies directly as 𝑺 and inversely as 𝑻. When हैं| 𝑵 का मान ज्ञात कीजजए|
𝟐 𝟓 𝟒 𝟑
𝑹 = 𝟑 and 𝑻 = 𝟕, 𝑺 = 𝟐𝟏. Find 𝑺 when 𝑹 = 𝟒 a) 𝟏𝟒 b) 𝟏𝟐 c) 𝟏𝟔 d) 𝟏𝟖
𝟏
and 𝑻 = 𝟗.
𝑹 सीधे 𝑺 के रूप में बदलता है और 𝑻 के
विपरीत। जब 𝑹 = 𝟑 और 𝑻 = 𝟕, 𝑺 = 𝟐𝟏. 𝑺 का
𝟐 𝟓 𝟒

पता लगाएं जब 𝑹 = 𝟒 और 𝑻 = 𝟗.
𝟑 𝟏
6. During his morning walk. Atul, walks for 45
𝟏 𝟕 𝟑 𝟏 minutes at a speed of 8 km/h and takes 15
a) 𝟏𝟓 b) 𝟏𝟎 c) 𝟓 d) 𝟑𝟎
rounds of a park. Shekhar takes 10 rounds of
the same park in 40 minutes. What is the speed
3. 𝑿 varies directly as √𝒀. When 𝒀 = 𝟐𝟎. 𝟐𝟓, 𝑿 = of Shekhar in km/h?
𝟏𝟖. When 𝑿 = 𝟕. 𝟐, then 𝒀 =?
सब
ु ह की सैर के दौरान, अतल
ु 8 km/h की चाल
𝑿, √𝒀 के अनुक्रमानुपाती है| जब 𝒀 = 𝟐𝟎. 𝟐𝟓 है ,
से 45 ममनट चल चलता है और एक पाकक के 15
तो 𝑿 = 𝟏𝟖 जब 𝑿 = 𝟕. 𝟐 हो, तो 𝒀 का मान
चक्कर लगाता है। शेखर उसी पाकक में 10 चक्कर
ज्ञात कीजजए|
40 ममनट में लगाता है। km/h में शेखर की चाल
a) 𝟒. 𝟖𝟒 b) 𝟐. 𝟓𝟔 c) 𝟏. 𝟗𝟔 d) 𝟑. 𝟐𝟒
ककतनी है ?
a) 7.2 b) 6 c) 𝟔. 𝟕𝟓 d) 𝟕. 𝟓
Profit, loss and Discount 11. The profit triples if the selling price is doubled.
7. By selling 𝟏𝟐𝟎 mangoes, a shopkeeper gains the The profit percentage is:
selling price of 𝟐𝟎 mangoes. Find his profit यदद विक्रय मूल्य को दोगुना कर ददया जाए तो
percentage.
लाभ तीन गन
ु ा हो जाता है। लाभ प्रततशत है ?
𝟏𝟐𝟎 आमों को बेचने पर, एक दक
ु ानदार को 𝟐𝟎
a) 𝟏𝟏𝟎 b)𝟓𝟎 c) 𝟏𝟎𝟎 d) 𝟐𝟖
आमों के विक्रय मूल्य के बराबर लाभ प्राप्त होता
है| उसका लाभ प्रततशत ज्ञात कीजजए| 12. A shopkeeper claims that he is selling sugar at
a) 𝟐𝟎% b) 𝟐𝟓% c) 𝟏𝟎% d) 𝟏𝟐% Rs. 𝟐𝟕/kg which cost him Rs. 𝟑𝟎/kg, but he
gives 𝟕𝟓𝟎 grams instead of 𝟏𝟎𝟎𝟎𝒈𝒓𝒂𝒎𝒔.
8. On selling 𝟏𝟖 pens at ₹𝟔𝟔𝟎, there is a profit What is his profit or loss percentage?
equal to the cost price of 𝟒 Pens. What is the एक दक
ु ानदार का दािा है कक िह Rs. 𝟐𝟕/kg पर
cost price of one pen (in ₹)? चीनी बेच रहा है जो उसे Rs. 𝟑𝟎/kg के मूल्य पर
𝟏𝟖 पेनों को ₹𝟔𝟔𝟎 में बेचने पर, 𝟒 पेनों के क्रय
ममलती है , लेककन िह 𝟏𝟎𝟎𝟎 ग्राम के बजाय 𝟕𝟓𝟎
मल्
ू य के बराबर लाभ प्राप्त होता है| एक पेन का ग्राम दे ता है | उसका लाभ या हातन प्रततशत ज्ञात
क्रय मल्
ू य (₹ में) ज्ञात कीजजए| कीजजए|
a) 𝟑𝟑 b) 𝟒𝟓 c) 𝟑𝟎 d) 𝟐𝟐
a) 𝟐𝟎% profit b) 𝟐. 𝟓% loss
c) 𝟕. 𝟓% profit d) 𝟏𝟎% profit
9. Kamal purchases 𝟖𝟎 articles for ₹𝟏𝟏, 𝟏𝟖𝟎 and
sells them at a loss equal to the selling price of
13. Sudhir claimed to sell his item at only 𝟖%
6 articles. What will be the selling price of 1
above the cost of production, but used a
article ?
weight that had 𝟕𝟓𝟎 grams written on it,
कमल ₹𝟏𝟏, 𝟏𝟖𝟎 में 𝟖𝟎 िस्तए
ु ँ खरीदता है और though it actually weighted 𝟕𝟐𝟎 grams. What
उन्हें 𝟔 िस्तुओं के विक्रय मूल्य के बराबर हातन was the actual profit percentage earned by
पर बेचता है। 𝟏 िस्तु का विक्रय मूल्य क्या होगा? Sudhir?
a) ₹𝟏𝟑𝟎 b) ₹𝟏𝟐𝟓 c) ₹𝟏𝟐𝟎 d) ₹𝟏𝟑𝟓 सुधीर ने अपनी िस्तुओं को उत्क्पादन लागत से
केिल 𝟖% अधधक पर बेचने का दािा ककया,
10. On selling 𝟏𝟓 balls for ₹𝟒𝟓𝟎, there is a loss
equal to the cost price of 𝟓 balls. The cost लेककन एक ऐसे बाट का उपयोग ककया, जजस पर
price of a ball is: 𝟕𝟓𝟎𝒈 मलखा हुआ था, हालाँकक िास्ति में इसका
𝟏𝟓 गें दों को ₹𝟒𝟓𝟎 में बेचने पर, 𝟓 गें दों के क्रय
िजन 𝟕𝟐𝟎𝒈 था| सध
ु ीर द्िारा अजजकत ककया
मूल्य के बराबर हातन होती है | एक गें द का क्रय
गया िास्तविक लाभ प्रततशत ककतना था?
मूल्य क्या होगा? a) 𝟏𝟐. 𝟕𝟓 b) 𝟏𝟐. 𝟓 c) 𝟏𝟐. 𝟔 d) 𝟏𝟐. 𝟐𝟓
a) ₹𝟐𝟑 b) ₹𝟓𝟒 c) ₹𝟒𝟓 d) ₹𝟑𝟐
14. A man sells goods at 𝟒% loss on cost price but
he gives 𝟐𝟎 g instead of 𝟒𝟎 g. What is his profit
or loss percentage?
एक व्यजक्त क्रय मूल्य पर 𝟒% हातन पर सामान गलत िजन का उपयोग करके एक दक
ू ानदार
बेचता है लेककन िह 𝟒𝟎𝒈 के बजाय 𝟐𝟎𝒈 तौलता चािल खरीदते और बेचते समय 𝟔% तक की
है। उसके लाभ या हातन प्रततशत क्या है ? बेईमानी करता है | उसका लाभ प्रततशत (दशमलि
a) 𝟗𝟐% profit b) 𝟖𝟒% profit के दो स्थनों तक पूर्ाकककत) ज्ञात कीजजए|
c) 𝟗𝟐% loss d) 𝟖𝟐% loss a) 𝟏𝟑. 𝟔𝟓% b) 𝟏𝟒. 𝟔%
c) 𝟏𝟏. 𝟐𝟓% d) 𝟏𝟐. 𝟕𝟕%
15. Subir claimed to sell his items at only 5% above
the cost of production, but used a weight that 18. What is the profit percentage of a dishonest
had 1 kg written on it, though it actually cloth merchant who uses a scale which
weighed 960 grams. What was the actual profit 𝟐
measure 𝟏𝟔 % less than what is marked on it,
percentage earned by Subir? 𝟑
and also sells at a price 𝟏𝟎% higher than the
सब
ु ीर ने अपनी िस्तओ
ु ं को उत्क्पादन लागत से
cost price?
केिल 𝟓% अधधक पर बेचने का दािा ककया, लेककन एक बेईमान कपड़ा व्यापारी कपड़ा मापने के मलए
एक ऐसे भार का उपयोग ककया जजस पर ऐसे पैमाने का उपयोग करता है , जो कपड़े को
𝟏 𝒌𝒈 मलखा था, हालांकक िास्ति में इसका िजन इसके अंककत मान से 𝟏𝟔 𝟑 % कम मापता है साथ
𝟐

𝟗𝟔𝟎 ग्राम था। सुबीर द्िारा अजजकत िास्तविक


ही िह कपड़े को क्रय मूल्य से 𝟏𝟎% अधधक
लाभ प्रततशत ककतना था?
मल्
ू य पर भी बेचता है| उसके द्िारा अजजकत कुल
a) 𝟗. 𝟓 % b) 𝟗. 𝟑𝟕𝟓% c) 𝟗. 𝟐𝟓% d) 𝟗. 𝟏𝟐𝟓
प्रततशत लाभ ज्ञात कीजजए|
a) 𝟐𝟎% b) 𝟑𝟐% c) 𝟐𝟔% d) 𝟑𝟎%

19. A dealer allows 𝟏𝟗% discount on the marked


16. A milkman purchase milk at ₹’𝒂’ per liter and price of an article and gains 𝟏𝟕%. If the cost
sells it at ₹𝟏. 𝟓a per liter. Still he mixes 𝟐 liters 𝟏
price of the article increases by 𝟏𝟏 𝟗 %, how
water with every 𝟗 liters of pure milk. What is
much discount percentage should he allow on
his profit percentage?
the marked price so as to earn the same
एक दध
ू िाला ₹ 𝒂 प्रतत लीटर की दर से दध

percentage of profit as before?
खरीदता है और ₹𝟏. 𝟓a प्रतत लीटर की दर से
एक विक्रेता एक िस्तु के अंककत मूल्य पर
बेचता है | इसके अततररक्त िह प्रतत 𝟗 लीटर शद्
ु ध
𝟏𝟗% की छूट दे ता है और 𝟏𝟕% लाभ कमाता
दध
ू में 𝟐 लीटर पानी ममलता है | उसका लाभ
है | यदद िस्तु के लागत मूल्य में 𝟏𝟏 % की
𝟏

प्रततशत ककतना है ? 𝟗

𝟐𝟓𝟎 𝟑𝟐𝟖 𝟕𝟖𝟗 𝟏𝟎𝟎𝟎 िद्


ृ धध हो जाती है , तो उसी अंककत मल्
ू य पर
a) % b) % c) % d) %
ककतने प्रततशत की छूट दी जानी चादहए ताकक
𝟑 𝟗 𝟓 𝟑

17. By using faulty weight, a shopkeeper cheats to लाभ के प्रततशत में कोई पररितकन न हो?
the extent of 𝟔% while buying and selling rice.
a) 𝟏𝟎% b) 𝟏𝟐. 𝟓% c) 𝟏𝟎. 𝟓% d) 𝟗. 𝟎𝟗%
Find his gain percentage (rounded to two
decimal places).
20. A trader gains 𝟐𝟓% by selling an article with be give now on the same marked price to earn
𝟐𝟎% discount on its marked price. If the cost the same percentage of profit as earlier?
price of the article increases by 𝟑𝟎%, then how एक दक
ु ानदार ककसी िस्तु को उसके अंककत मल्
ू य
much discount (in %) should he offer on the
same marked price to gain 𝟏𝟓% of profit? पर 𝟐𝟓% छूट दे कर बेचने पर उस पर 𝟐𝟎% का
एक व्यापारी, एक िस्तु को उसके अंककत मल्
ू य लाभ अजजकत करता है | यदद उस िस्तु के क्रय
पर 𝟐𝟎% की छूट दे कर बेचने पर 𝟐𝟓% का लाभ मूल्य में 𝟏𝟎% की िद्
ृ धध होती है , तो उसे पूिक के
प्राप्त करता है। यदद िस्तु के क्रय मल्
ू य में 𝟑𝟎% समान प्रततशत लाभ अजजकत करने के मलए, अब
की िद्
ृ धध होती है , तो उसे 𝟏𝟓% लाभ प्राप्त करने उस िस्तु के उसी अंककत मूल्य पर ककतने प्रततशत
के मलए उसी अंककत मूल्य पर ककतनी छूट (% की छूट दे नी चादहए?
a) 𝟏𝟔. 𝟓% b) 𝟏𝟕. 𝟓% c) 𝟏𝟖% d) 𝟏𝟓%
में) दे नी चादहए?
a) 𝟓. 𝟏𝟐% b) 𝟓. 𝟎𝟖% c) 𝟒. 𝟖𝟕% d) 𝟒. 𝟑𝟐% 23. The marked price of an article is 𝟒𝟎% more
𝟐
than the cost price and a discount of 𝟏𝟒 𝟕 % is
given on the market price. In this kind of sale
the seller gains:
एक िस्तु की अंककत मल्
ू य, लागत मल्
ू य से
21. A dealer gain 𝟐𝟎% by selling an article at 𝟐𝟓%
𝟒𝟎% अधधक है और बाजार मूल्य पर 𝟏𝟒 𝟕 %
𝟐
discount on its marked price. If the cost price
of the article is decreased by 𝟏𝟓%, how much की छूट दी गई है। इस तरह की बबक्री में
discount percentage should he now give on
the same marked price so as to earn the same विक्रेता को ककतना फायदा होता है ?
𝟏 𝟐
percentage of profit as before? a) 𝟔𝟑 𝟑 % b) 𝟐𝟎% c) 𝟐𝟓% d) 𝟐𝟖 𝟕 %
एक विक्रेता ककसी िस्तु को उसके अंककत मूल्य
24. A shopkeeper offers two successive discounts
पर 𝟐𝟓% की छूट दे कर 𝟐𝟎%का लाभ प्राप्त
of 𝟏𝟓% and 𝟐𝟎% on an article and is still able
करता है। यदद िस्तु का क्रय मूल्य 𝟏𝟓% कम to earn a 𝟕𝟎% profit. If he offers a single
हो जाता है , तो अब उसे उसी अंककत मूल्य पर discount of 𝟑𝟎% what will his profit percentage
be?
ककतने प्रततशत की छूट दे नी चादहए ताकक ककसी िस्तु पर 𝟏𝟓% और 𝟐𝟎% की दो क्रमागत
उसका लाभ प्रततशत पूिि
क त बना रहे ? छूटें दे ने के बाद भी दक
ु ानदार को उस पर 𝟕𝟎%
a) 𝟑𝟐. 𝟓𝟎% b) 𝟑𝟓% का लाभ प्राप्त होता है | यदद िह उस िस्तु पर
c) 𝟑𝟔. 𝟐𝟓% d) 𝟑𝟕. 𝟕𝟓%
केिल 𝟑𝟎% की एकल छूट ही दे ता, तो उसे प्राप्त
22. A shopkeeper earns a profit of 𝟐𝟎% by selling होने िाला लाभ प्रततशत क्या होता?
an article by giving 𝟐𝟓% discount on its marked a) 𝟕𝟎% b) 𝟕𝟓% c) 𝟔𝟓% d) 𝟓𝟎%
price. If the cost price of the article increases by
𝟏𝟎% then how much discount percent should 25. A trader marks his goods at 𝟒𝟓% above the
cost price. He offers a discount of 𝟐𝟏% to his
customers. What is his percentage profit एक दक
ु ानदार एक िस्तु के अंककत मूल्य पर
(rounded off to the nearest integer)?
𝟏𝟎% की छूट दे ने के बाद भी 𝟏𝟐. 𝟓% का लाभ
एक व्यापारी अपने माल पर क्रय मूल्य से 𝟒𝟓%
प्राप्त करता है। यदद िस्तु को बबना ककसी छूट
अधधक अंककत करता है। िह अपने ग्राहकों को
के अंककत मूल्य पर बेचा जाता है तो उसका लाभ
𝟐𝟏% की छूट प्रदान करता है। उसका लाभ प्रततशत ककतना होता?
प्रततशत ज्ञात करे | (तनकटतम पूर्ाांक में)? a) 𝟐𝟕% b) 𝟐𝟐. 𝟓% c) 𝟑𝟎% d) 𝟐𝟓%
a) 𝟏𝟒% b) 𝟏𝟐% c) 𝟏𝟓% d) 𝟏𝟖%
29. A shopkeeper marks up his goods by 𝟑𝟎% and
then gives a discount of 𝟏𝟎% on the marked
price. Apart from this, he uses a faulty balance
which reads 𝟏 kg for 𝟖𝟕𝟎 g. What is his net
profit percentage?
26. To gain 𝟖% after allowing a discount of 𝟏𝟎%, एक दक
ु ानदार अपने माल पर 𝟑𝟎% अधधक करता
by what per cent cost price should be hiked in
है और कफर अंककत मूल्य पर 𝟏𝟎% की छूट दे ता
the list price?
ककसी िस्तु पर लागत मूल्य से ककतना % मूल्य है |

बढ़ाकर अंककत ककया जाये कक 𝟏𝟎% की छूट दे ने इसके अलािा, िह एक दोषपूर्क तराजू (बैलन्स) का

पर भी 𝟖% लाभ हो? उपयोग करता है जजसमे 𝟖𝟕𝟎 g के िजन मे 𝟏 kg


a) 𝟗% b) 𝟏𝟏% c) 𝟏𝟖% d) 𝟐𝟎% ददखाई दे ता है । उसका शुद्ध लाभ प्रततशत क्या
है ?
27. The cost of manufacture of a tape recorder is a) 𝟑𝟒. 𝟒𝟖 % b) 𝟑𝟒. 𝟖𝟒%
Rs.1500. The manufacturer fixes the marked c)𝟑𝟖. 𝟒𝟐% d) 𝟑𝟖. 𝟐𝟒%
price 𝟐𝟎% above the cost of manufacture and
allows a discount in such a way as to get a profit 30. One single discount which is equivalent to
of 𝟖%. The rate of discount is 𝟐𝟎%, 𝟏𝟎% and 𝟓% is given by:
एक टे प ररकॉर्कर के उत्क्पादन की लागत 𝟏𝟓𝟎𝟎 रु 𝟐𝟎%, 𝟏𝟎% और 5% की तीन क्रमागत छूटे ,
है। उत्क्पादक टे प ररकॉर्कर का मूल्य लागत मूल्य तनम्न मे से ककस एकल छूट के बराबर होगी?
से 𝟐𝟎% बढाकर मूल्य अंककत करता है ,यदद 𝟖% a) 𝟑𝟐. 𝟔𝟎% b) 𝟑𝟐. 𝟖𝟎%
का लाभ होता है। तो छूट का दर ज्ञात करे : c) 𝟑𝟎. 𝟔𝟎% d) 𝟑𝟏. 𝟔𝟎%
a) 𝟏𝟐 b) 𝟖 c) 𝟐𝟎 d) 𝟏𝟎

28. A shopkeeper makes a profit of 𝟏𝟐. 𝟓% after


allowing a discount of 𝟏𝟎% on the marked
31. The price of an article is raised by 𝟒𝟓% and
price of an article. Find his profit percentage if
the article is sold at the marked price, allowing then two successive discounts of 𝟏𝟓% each
no discount. are allowed. Ultimately the price of the article
is _________.
एक िस्तु की कीमत में 𝟒𝟓% की िद्
ृ धध की एक थोक व्यापारी द्िारा बेची जा रही प्रत्क्येक
जाती है और इसके बाद 𝟏𝟓% प्रत्क्येक की दो िस्तु का अंककत मूल्य ₹𝟑𝟎𝟎 था। थोक व्यापारी
क्रममक छूट दी जाती है | अंत मे िस्तु के मूल्य एक स्टॉक-तनकासी बबक्री की पेशकश कर रहा था,

मे ककतने प्रततशत की िद् जजसके तहत भुगतान की गई प्रत्क्येक तीन िस्तओ


ु ं
ृ धध/कमी होगी?
a) Increased by 𝟒. 𝟕𝟔𝟐𝟓% के मलए, एक िस्तु मुफ्त दी जा रही थी। इसके
b) Decreased by 𝟕. 𝟕𝟔𝟐𝟓% अलािा, 𝟏𝟎, 𝟎𝟎𝟎 रुपये से अधधक की खरीदारी
c) Increased by 𝟕. 𝟕𝟔𝟐𝟓% करने िाले ककसी भी व्यजक्त को 𝟑 खरीदें , 𝟏
d) Decreased by 𝟒. 𝟕𝟔𝟐𝟓%
मफ्
ु त पाएं' योजना मल्
ू य पर दे य रामश पर 𝟏𝟎%
32. A shopkeeper offers the following four की छूट की पेशकश की जा रही थी। रमेश ने
Schemes. Which scheme has the maximum
खरीदारी की जजसके मलए दे य यह रामश ₹𝟏𝟖𝟎𝟎𝟎
discount percentage?
a) Two Successive discounts of 𝟐𝟐% and 𝟐𝟓% थी। इस लेन-दे न के दौरान रमेश को ककतनी
b) Buy 𝟑, get 𝟐 free प्रभािी छूट की पेशकश की गई थी?
c) Buy 5, get 𝟒 free a) 𝟑𝟐% b) 𝟑𝟐. 𝟕𝟓% c) 𝟑𝟐. 𝟓% c) 𝟑𝟏. 𝟓%
d) Buy 4, get 𝟑 free
एक दक
ु ानदार तनम्नमलखखत चार योजनाएं प्रदान 34. A retailer bought 𝟓𝟎 kg of Arhar Dal at a
करता है । ककस योजना में अधधकतम छूट प्रततशत discount of 𝟐𝟎%. Another 𝟐 kg of Arhar dal was
offered to him for free by the wholesaler on the
है? purchase of 𝟓𝟎𝒌𝒈 of the dal. If the retailer sells
a) 𝟐𝟐% और 𝟐𝟓% की दो क्रममक छूट the entire quantity of the dal at the marked
b) 3 खरीदें , 2 मुफ़्त पाएं price to a customer. What is the profit
percentage gained by the retailer?
c) 5 खरीदें , 4 मुफ़्त पाएं
एक फुटकर विक्रेता ने 𝟓𝟎 kg अरहर की दाल
d) 4 खरीदें , 3 मुफ़्त पाएं
𝟐𝟎% की छूट पर खरीदी| 𝟓𝟎𝒈 दाल की खरीद
a) c b) b c) a d) d
पर थोक व्यापरी द्ििारा उसे 𝟐 kg अरहर की
33. The marked price of every item being sold by a दाल मुफ्त में दी गई| यदद फुटकर विक्रेता दाल
wholesaler was ₹𝟑𝟎𝟎. The wholesaler was
की पूरी मात्रा एक ग्राहक को अंककत मूल्य पर
offering a stock-clearance sale under which, for
every three items paid for, one item was being बेचता है , तो फुटकर विक्रेता को प्रततशत ज्ञात
given free. In addition to this, a further 𝟏𝟎% कीजजए|
discount on the amount payable on the `Buy a) 𝟑𝟎% b) 𝟐𝟐% c) 𝟏𝟐% d) 𝟏𝟓%
𝟑,Get 𝟏 Free’ Scheme price was being offered
to anyone making purchases worth more than 35. A trader marks his goods at 𝟒𝟎% above the
₹ 10,000. Ramesh made purchases for which cost price. He sells 𝟕𝟎% of the goods at the
this amount payable was ₹𝟏𝟖, 𝟎𝟎𝟎. What was marked price and the rest, he sells by allowing
the effective percentage discount that was a 𝟒𝟎% discount on the marked price. His
offered to Ramesh during this transaction? percentage profit is:
एक व्यापारी अपने माल को लागत से 𝟒𝟎% c) Loss/ हातन𝟑. 𝟏𝟐𝟓 %
ऊपर अंककत करता है | िह अंककत मूल्य पर d) Gain/लाभ 𝟒. 𝟑𝟕𝟓%
𝟕𝟎% सामान बेचता है और शेष माल अंककत
38. A dealer bought some toys for ₹1800. He sold
मूल्य पर 𝟒𝟎% छुट दे कर बेच दे ता है | उसका 𝟏
𝟒𝟎% of these at a loss of 𝟏𝟓% and 𝟑𝟑 𝟑 % of
लाभ प्रततशत ककतना है ?
the remaining toys at 𝟐𝟎% profit. At what
a) 𝟐𝟒. 𝟒 b) 𝟐𝟑. 𝟐 c) 𝟐𝟑. 𝟒 d) 𝟐𝟒. 𝟐 percent profit should he sell the remaining toys
to earn an overall profit of 𝟏𝟎%?
एक विक्रेता ने ₹1800 में कुछ खखलौने खरीदे ।
उसने इनमें से 𝟒𝟎% खखलौनों को 𝟏𝟓% हातन पर
बेचा और शेष में से 𝟑𝟑 𝟑 % खखलौनों को 𝟐𝟎%
𝟏
36. Radha marks her goods 𝟐𝟓% above the cost
price. She sells 𝟑𝟓% of goods at the marked लाभ पर बेचा| कुल ममलाकर 𝟏𝟎% लाभ प्राप्त
price, 𝟒𝟎% at 𝟏𝟓% discount and the remaining करने के मलए, उसे शेष खखलौनों ककतने लाभ
at 𝟐𝟎% discount. What is her overall
प्रततशत पर बेचना चादहए?
percentage gain?
a) 𝟐𝟎 % b) 𝟑𝟎% c) 𝟐𝟒% d) 𝟐𝟓%
राधा अपनी िस्तुओ को उनके लागत मूल्य से
𝟐𝟓% अधधक अंककत करती है | िह 𝟑𝟓% िस्तुओ 39. Raghuveer purchased some perishable items
को अंककत मूल्य पर,𝟒𝟎% को 𝟏𝟓% की छूट पर, for sale but 𝟑𝟔% of those items could not be
sold and went bad. However, Raghuveer
और शेष को 𝟐𝟎% की छूट पर बेचती है | उसका managed to sell the rest of the items at a price
स्मगक प्रततशत लाभ क्या है | that helped him earn an overall profit of 𝟐𝟖%.
a) 𝟏𝟏. 𝟐𝟓 b) 𝟏𝟎 c) 𝟏𝟏. 𝟕𝟓 d) 𝟏𝟐. 𝟕𝟓 At What percentage above the cost price of
each item did Raghuveer sell each item that did
37. A marks his goods 𝟐𝟓% above the cost price. not go bad?
She sells 𝟐𝟓% of the goods at the market price. रघुिीर ने कुछ नाशिान िस्तुएँ बबक्री के मलए
𝟔𝟎% at 𝟐𝟓% discount and the remaining at
खरीदीं, लेककन उसने से 𝟑𝟔% िस्तुएँ बेची नहीं जा
𝟏𝟎% discount. What is her overall gain or loss
per cent? सकी और खराब हो गई| िैस,े रघुिीर शेष िस्तुओं
A अपनी िस्तुओं पर क्रय मूल्य से 𝟐𝟓% अधधक का उस मुल्य पर बेचने मे सफल रहा, जजससे उसे
कीमत अंककत करती है | िह 𝟐𝟓% िस्तुओं की 𝟐𝟖% का कुल लाभ अजजकत करने मे मदद ममली|
कीमत अंककत मूल्य पर 𝟔𝟎% िस्तुओं को रघुिीर ने प्रत्क्येक िस्तु के क्रय मुल्य से ककतने
𝟐𝟓% की छूट पर तथा बाकी िस्तुओं को 𝟏𝟎% की प्रततशत अधधक पर उन िस्तुओं को बेचा था, जो
छूट पर बेचता है | उसका कुल लाभ या हातन खराब नहीं हुई थी|
प्रततशत ज्ञात करें | a) 𝟔𝟑% b) 𝟗𝟐% c) 𝟏𝟎𝟎% d) 𝟏𝟐𝟎%

a) Gain/लाभ 𝟏𝟓. 𝟏𝟐𝟒% 40. A shopkeeper sold 𝟐𝟒 shirts, each at a profit of


b) Loss/ हातन𝟏𝟖. 𝟏𝟕𝟓 % 𝟐𝟎% and 𝟏𝟔 shirts each at a profit of 𝟏𝟎%. If
he had sold all the 𝟒𝟎 shirts each at a profit of 𝟐𝟎% and 𝟑𝟎%. Find his profit in whole
𝟏𝟓%, then his overall profit would have been transaction.
reduced by ₹𝟑𝟔. What is the cost price of each एक आदमी ने एक र्ेस्कटॉप और एक लैपटॉप
shirt ?
को 𝟑𝟖, 𝟎𝟎𝟎 रुपये में खरीदा, उसने दोनों को 𝟐𝟎%
एक दक
ु ानदार ने 𝟐𝟒 शटक , प्रत्क्येक 𝟐𝟎% के लाभ
और 𝟑𝟎% के मुनाफे पर बेचा। पूरे लेनदे न में
पर और 𝟏𝟔 शटक , प्रत्क्येक 𝟏𝟎% के लाभ पर बेचे|
उसका लाभ ज्ञात करें ।
यदद िह सभी 𝟒𝟎 शर्टकस, प्रत्क्येक 𝟏𝟓% के लाभ
a) 𝟗𝟓𝟎𝟎 b) 𝟖𝟓𝟎𝟎
पर बेचता, तो उसका कुल लाभ ₹𝟑𝟔 कम होता| c) 𝟏𝟗𝟎𝟎𝟎 d) 𝟗𝟎𝟎𝟎
प्रत्क्येक शटक का क्रय मूल्य ज्ञात कीजजए|
a) ₹𝟏𝟎𝟎 b) ₹𝟐𝟐𝟎 c) ₹𝟗𝟎 d) ₹𝟏𝟓𝟎 43. A dealer sold two types of goods for Rs.𝟏𝟎𝟎𝟎𝟎
each. On one of them, he lost 𝟐𝟎% and on the
other he gained 𝟐𝟎%. His gain or loss per cent
in the entire transaction was
एक र्ीलर ने दो प्रकार के सामान को प्रत्क्येक को
𝟏𝟎𝟎𝟎𝟎 रुपये में बेचा। उनमें से एक पर, उसे
41. Under a sale offer, Tanvir was offered a 𝟑𝟐%
discount on the part of the marked price that 𝟐𝟎% की हातन हुई और दस ू रे पर उसे 𝟐𝟎% का
was paid in cash, but had to add 𝟏. 𝟐% on the लाभ हुआ। परू े लेन-दे न में उसका लाभ या हातन
part of the marked price paid through a credit
प्रततशत ज्ञात कीजजये:
card. If Tanvir paid 𝟕𝟓% of the marked price in
cash and the rest through a credit card, what a) 𝟐% loss b) 𝟐% gain
percentage of the marked price was his total c) 𝟒% gain d) 𝟒% loss
final payment?
44. Amit sold two pens each at the same price, one
बबक्री प्रस्ताि के तहत, नकद भुगतान ककए गए at a profit or 𝟐𝟎% and the other at a loss of
अंककत मूल्य के दहस्से पर तनिीर 𝟑𝟐% छूट का 𝟑𝟎%. What is the net profit or loss percentage?
प्रस्ताि ददया गया था, लेककन क्रेडर्ट कार्क के अममत ने दो कलम बराबर मल्
ू य पर बेचे| एक
माध्यम से भुगतान ककए गए अंककत मूल्य के पर उसे 𝟐𝟎% लाभ हुआ तथा दसु रे पर 30%
दहस्से पर 𝟏. 𝟐% जोड़ना पड़ा। यदद तनिीर ने हातन तो उसका कुल ककतने % लाभ या हातन
अंककत मूल्य का 𝟕𝟓% नकद में और शेष क्रेडर्ट हुआ?
𝟏𝟏
कार्क के माध्यम से भुगतान ककया हो , तो उसका a) 𝟏𝟏 𝟏𝟗 % b) 𝟐𝟐% c) 𝟏𝟎% d) 𝟐𝟓%
कुल अंततम भुगतान अंककत मूल्य का ककतना
45. Two articles are sold for ₹ 𝟓𝟏𝟎𝟒 each. On one,
प्रततशत था? the seller gains 𝟏𝟔% and on the other, he loses
a) 𝟕𝟓. 𝟗% b) 𝟕𝟔. 𝟑% c)𝟕𝟔. 𝟏% d)𝟕𝟔. 𝟔% 𝟏𝟐%. What is his overall gain percent, nearest
to two decimal places?
42. A man bought a desktop and a laptop for Rs. दो िस्तुओ में से प्रत्क्येक को ₹ 𝟓, 𝟏𝟎𝟒 के मूल्य
𝟑𝟖, 𝟎𝟎𝟎 each, he sold both of them at profits of
पर बेचा जाता है | उनमें से एक िस्तु पर विक्रेता
को 𝟏𝟔% लाभ होता है ओर दस
ू री िस्तु पर 𝟏𝟐% नही होता| उसके द्िारा 𝑨 को 𝟐𝟓% के लाभ पर
की हातन होती है| विक्रेता को कुल ममलकर ककतने और 𝑩 को 𝟏𝟎% की हातन पर बेचे जाने पर उसे
प्रततशत लाभ हुआ : ₹𝟏𝟏𝟒 का लाभ होता है | प्रथम जस्थतत में , 𝑨 और
a) 𝟎. 𝟎𝟖% b) 𝟎. 𝟏𝟐% c) 𝟎. 𝟏𝟒% d) 𝟎. 𝟏𝟎% 𝑩 के विक्रय मूल्यों का अंतर ज्ञात कीजजए|
a) ₹𝟖𝟎 b) ₹𝟕𝟓 c) ₹𝟗𝟎 d) ₹𝟔𝟎

49. Kisna buys a cow for ₹𝟏𝟓, 𝟎𝟎𝟎. After one year,
he sells it for ₹𝟏𝟖, 𝟓𝟎𝟎. After one year, again
he buys the same cow at ₹𝟏𝟗, 𝟎𝟎𝟎 and sells it
46. An article is sold at 𝟐𝟓 percent loss. If its cost
for ₹𝟐𝟐, 𝟑𝟎𝟎. What is the overall profit
price is doubled and selling price is increased by
percentage of Kisna?
₹𝟔𝟔𝟎, then there is a profit of 𝟐𝟎 percent.
What is the original cost price of the article? ककसना ₹𝟏𝟓, 𝟎𝟎𝟎 में एक गाय खरीदता है | एक
एक िस्तु को 𝟐𝟓 प्रततशत हातन पर बेचा जाता िषक के बाद, िह इसे ₹𝟏𝟖, 𝟓𝟎𝟎 में बेच दे ता है |
है। यदद इसके क्रय मल्
ू य को दग
ु न
ु ा कर ददया एक िषक बाद, िह कफर से उसी गाय को
जाता है और विक्रय मल्
ू य में ₹𝟔𝟔𝟎 की िद्
ृ धध की ₹𝟏𝟗, 𝟎𝟎𝟎 में खरीदता है और ₹𝟐𝟐, 𝟑𝟎𝟎 में बेच
जाती है , तो 𝟐𝟎 प्रततशत का लाभ होता है। िस्तु दे ता है | ककसना का कुल लाभ प्रततशत ककतना है ?
का मलू लागत मल्
ू य क्या है ?
a) 𝟏𝟖% b) 𝟏𝟎% c) 𝟐𝟎% d) 𝟏𝟓%
a) ₹𝟒𝟖𝟎 b) ₹𝟓𝟎𝟎 c) ₹𝟒𝟎𝟎 d) ₹𝟑𝟔𝟎
50. Kapil bought 𝟏𝟎𝟎 frocks at the rate of ₹𝟑𝟐𝟎 per
frock. Transport expenditure was ₹𝟏, 𝟎𝟎𝟎 . He
47. A woman sold her earphone for ₹𝟐, 𝟎𝟎𝟎 and
paid a tax of ₹𝟏. 𝟓𝟎 per frocks and the labor fee
got a percentage profit equal to the numerical
was ₹𝟓𝟎𝟎 . If she wants to make a profit of
value of cost price. The cost price of the
𝟏𝟎%, then what should be the selling price per
earphone is:
frock?
एक मदहला ने अपना ईयरफोन ₹𝟐, 𝟎𝟎𝟎 में बेचा
कवपल ने ₹𝟑𝟐𝟎 प्रतत फ्राक की दर से 𝟏𝟎𝟎 फ्राक्स
और क्रय मूल्य के बराबर प्रततशत लाभ प्राप्त
खरीदी| पररिहन व्यय ₹𝟏, 𝟎𝟎𝟎 था| उसने प्रतत
ककया। ईयरफोन का क्रय मूल्य ककतना है ?
फ्राक ₹𝟏. 𝟓𝟎 का कर ददया और श्रम शल्
ु क ₹𝟓𝟎𝟎
a) ₹𝟔𝟎𝟎 b) ₹𝟒𝟎𝟎 c) ₹𝟓𝟎𝟎 d) ₹𝟐𝟎𝟎
था| यदद िह 𝟏𝟎% का लाभ कमाना चाहती है , तो
48. Abhi has two items 𝑨 and 𝑩. When he sells 𝑨 at प्रतत फ्राक विक्रय मल्
ू य ककतना होना चादहए?
a loss of 𝟏𝟓% and 𝑩 at a profit of 𝟐𝟓%, he has a) ₹𝟑𝟗𝟎. 𝟖𝟎 b) ₹𝟑𝟖𝟓. 𝟒𝟓
no loss or profit. When he sells A at a profit of c) ₹𝟑𝟕𝟎. 𝟏𝟓 d) ₹𝟑𝟑𝟔. 𝟓𝟎
𝟐𝟓% and 𝑩 at a loss of 𝟏𝟎%, then he gains
₹𝟏𝟏𝟒. What is the difference between the
selling prices of 𝑨 and 𝑩 in the first case?
अमभ के पास दो िस्तुएं 𝑨 और 𝑩 हैं| उसके द्िारा
𝑨 को 𝟏𝟓% की हातन पर और 𝑩 को 𝟐𝟓% के
51. 𝑨 shopkeeper announces a discount of 𝟒𝟖%
लाभ पर बेचे जाने पर उसे कोई हातन या लाभ and then by a further discount of 𝟏𝟓%. What is
the final sale price (in Rs, to the nearest rupee) 51. B 52. A
of a sofa costing Rs 𝟐𝟗𝟔𝟎𝟎 and what is the
discount (in Rs) ?
कोई दक
ु ानदार 𝟒𝟖% और कफर 𝟏𝟓% की एक और
छूट की घोषर्ा करता है। ₹𝟐𝟗, 𝟔𝟎𝟎 मूल्य िाले
सोफे का अंततम विक्रय मूल्य (₹ में , लगभग) और
छूट (₹ में) ज्ञात करें |
a) 𝟏𝟑𝟐𝟖𝟎, 𝟏𝟔𝟓𝟏𝟕 b) 𝟏𝟑𝟎𝟖𝟑, 𝟏𝟔𝟓𝟏𝟕
c) 𝟏𝟔𝟓𝟏𝟕, 𝟏𝟑𝟎𝟖𝟑 d) 𝟏𝟔𝟓𝟏𝟕, 𝟏𝟑𝟐𝟖𝟎

52. A manufacturer sells an item to a wholesale


dealer at a profit of 𝟏𝟏%. The wholesaler sells
the same to a retailer at a profit of 𝟏𝟓%. The
retailer in turn sells it to a customer for
₹17,045, thereby earning a profit of 𝟏𝟗%. The
cost price of the manufacturer is (consider
integral part only):
कोई तनमाकता एक िस्तु को 𝟏𝟏% के लाभ पर
थोक व्यापारी को बेचता है। थोक व्यापारी उसे को
𝟏𝟓% के लाभ पर एक खुदरा विक्रेता को बेचता
है। खुदरा विक्रेता उस पर 𝟏𝟗% का लाभ अजजकत
करते हुए ग्राहक को ₹𝟏𝟕, 𝟎𝟒𝟓 में बेचता है |
तनम्रता के मलए उस िस्तु का लागत मूल्य
______ है | (तनकटतम पूर्ाांक मे)
a) ₹𝟏𝟏, 𝟐𝟐𝟎 b) ₹𝟏𝟓, 𝟎𝟐𝟎
c) ₹𝟏𝟑, 𝟓𝟐𝟎 d) ₹𝟏𝟎, 𝟐𝟎𝟎

Answer Key
1. A 2. D 3. D 4. C 5. D
6. B 7. A 8. C 9. A 10. C
11. C 12. A 13. B 14. A 15. B
16. A 17. D 18. B 19. A 20. D
21. C 22. B 23. B 24. B 25. C
26. D 27. D 28. D 29. A 30. D
31. A 32. A 33. C 34. A 35. B
36. A 37. D 38. B 39. C 40. C
41. B 42. C 43. D 44. A 45. A
46. C 47. B 48. D 49. C 50. C
SI and CI

Instruction: - For Video Solution scan QR code c) ₹𝟔𝟎𝟗. 𝟎𝟓 d) ₹𝟔𝟎𝟔. 𝟎𝟐


given below or you can also CLICK on Click here
button to watch the Video Solution. 4. Find the compound interest on ₹𝟒𝟎, 𝟎𝟎𝟎 after
𝟐 years 𝟔 months at 𝟐𝟎% per annum, if the
interests is compounded half-yearly.
यदि ब्याज छमाही आर्ार पर चक्रवद्
ृ धर् होता
है , तो ₹𝟒𝟎, 𝟎𝟎𝟎 की रालि पर 𝟐𝟎% की वार्षधक
1. Find the compound interest on ₹𝟏, 𝟔𝟎, 𝟎𝟎𝟎 at
िर से 𝟐 वषध 𝟔 महीिे का चक्रवद्
ृ धर् ब्याज
𝟏𝟎% per annum for 𝟐 years if the interest is
compounded half-yearly. ज्ञात कीजजए।
यदि ब्याज अर्धवार्षधक रूप से चक्रवद्
ृ धर् ककया a) ₹24,420.40 b) ₹24,820.40
जाता है , तो ₹𝟏, 𝟔𝟎, 𝟎𝟎𝟎 पर 𝟏𝟎% वार्षधक की c) ₹24,920.40 d) ₹25,520
िर से 𝟐 वषो के लिए चक्रर्िधर् ब्याज ज्ञात
5. A sum of ₹𝟓, 𝟎𝟎𝟎 was deposited for 𝟑 Years at
कीजजए| 𝟏𝟎% per annum, compounded annually. The
a) ₹𝟑𝟒, 𝟒𝟖𝟏 b) ₹𝟑𝟐, 𝟐𝟖𝟐 difference between the interest for 𝟐 Years and
c) ₹𝟑𝟓, 𝟓𝟓𝟎 d) ₹𝟑𝟑, 𝟒𝟓𝟖 that for 𝟑 Years is :
₹𝟓, 𝟎𝟎𝟎 की रालि 𝟏𝟎% की वार्षधक ब्याज िर
2. Find the compound interest on ₹𝟑𝟓, 𝟎𝟎𝟎 in
पर 𝟑 वषों के लिए जमा की गई है , ब्याज वार्षधक
𝟐 years at 𝟔% per annum, the interest being
compounded half-yearly (nearest to a ₹): रूप से चक्रवद्
ृ धर् होता है | 𝟐 वषों और 3 वषों के
यदि ब्याज अद्धर्-वार्षधक रूप से सयोजजत ककया ब्याज का अंतर ककतिा होगा?
जा रहा हो, तो ₹𝟑𝟓, 𝟎𝟎𝟎 की रालि पर 𝟔% की a) ₹𝟓𝟔𝟎 b) ₹506
c) ₹𝟔𝟓𝟎 d) ₹𝟔𝟎𝟓
वार्षधक िर पर 𝟐 वषध का चक्रवद्
ृ धर् ब्याज ज्ञात
कीजजए|(₹ के निकटम)
a) ₹𝟒, 𝟏𝟗𝟑 b) ₹𝟒, 𝟎𝟎𝟎
c) ₹𝟒, 𝟑𝟗𝟑 d) ₹4,388
6. A certain sum is deposited for 4 years at a rate
3. What is the compound interest on ₹𝟐𝟎, 𝟎𝟎𝟎 for of 𝟏𝟎% per annum on compound interest
𝟗 months at the rate of 𝟒% per annum, when compounded annually. The difference between
interest is compounded quarterly? the interest at the end of 𝟐 years and that at
the end of 𝟒 years is ₹𝟓, 𝟎𝟖𝟐. Find the sum
यदि ब्याज त्रैमालसक आर्ार पर चक्रवद्
ृ धर् होता
(𝒊𝒏 ₹).
है, तो ₹𝟐𝟎, 𝟎𝟎𝟎 की रालि पर 𝟒% की वार्षधक कोई रालि वार्षधक रूप से चक्रर्िधर् होिे वािी
िर से 𝟗 महीिे मे प्राप्त चक्रवद्
ृ धर् ब्याज ककतिा 𝟏𝟎% वार्षधक ब्याज िर पर 𝟒 वषध के लिए जमा
होगा?
की जाती है | 𝟐 वषध के अंत में और 𝟒 वषध के
a) ₹𝟔𝟏𝟎 b) ₹𝟔𝟎𝟓
अंत में लमििे वािे ब्याज के बीच का अंतर होता है । यह रालि ककतिे वषों में िोगुिा हो
₹𝟓, 𝟎𝟖𝟐 है| वह रालि (₹ में) ज्ञात कीजजए| जाएगी?
a) 𝟐𝟎, 𝟎𝟎𝟎 b) 𝟐𝟓, 𝟓𝟎𝟎 a) 𝟓 b) 𝟑 c) 𝟐 d) 𝟒
c) 𝟓𝟎, 𝟖𝟐𝟎 d) 𝟏𝟎, 𝟏𝟔𝟒
10. The effective annual rate of interest,
7. A certain sum amounts to ₹𝟓, 𝟗𝟖𝟗. 𝟓𝟎 in 𝟐 years corresponding to a nominal rate of 𝟔% per
at 𝟏𝟓% p.a., interest compounded annum, compound half-yearly, is:
𝟖 −monthly. What is the simple interest on the ब्याज की प्रभावी वार्षधक िर, 𝟔% प्रनत वषध की
𝟐
same sum for 𝟒 𝟑 years at 𝟏𝟗% p.a.? िर के अिुरूप, अर्ध-वार्षधक चक्रवद्
ृ धर् है :
एक निजचचत रालि पर 𝟏𝟓% वार्षधक िर से 𝟐 a) 𝟎. 𝟎𝟔𝟎𝟕 b) 𝟎. 𝟎𝟔𝟎𝟗
वषध में प्राप्त लमश्रर्ि 𝟓, 𝟗𝟖𝟗. 𝟓𝟎 है, जहां ब्याज c) 𝟎. 𝟎𝟔𝟎𝟔 d) 𝟎. 𝟎𝟔𝟎𝟖

की गणिा 𝟖-मालसक चक्रवद्


ृ धर् आर्ार पर की
जाती है | उसी रालि पर 𝟏𝟗% वार्षधक िर से
𝟒 𝟑 वषध में प्राप्त होिे वािा सार्ारण ब्याज
𝟐
11. A sum of ₹𝒙 was invested for 𝟏𝟐 years in
ककतिा होगा? scheme A which offers simple interest at 𝟖%
a) ₹𝟑, 𝟕𝟐𝟒 b) ₹𝟑, 𝟗𝟗𝟎 p.a. The amount received after 𝟏𝟐 years was
𝟏
c) ₹𝟒, 𝟓𝟐𝟐 d) ₹𝟒, 𝟐𝟓𝟔 invested in scheme B for 𝟐 𝟐 years at
compound interest at 𝟏𝟐% p.a., compounded
8. If the rate of interest is 𝟒% for the first year, 𝟓% 10-monthly. If the interest received from B was
for the second year and 𝟔% for the third year, ₹𝟑𝟐, 𝟒𝟑𝟖, then what is the value of 𝒙 ?
then find the compound interest on ₹𝟏𝟎, 𝟎𝟎𝟎
योजिा A में 𝟏𝟐 वषों के लिए ₹𝒙 की रालि का
for 𝟑 years, if interest is compounded annually?
यदि ब्याज की िर पहिे वषध के लिए 𝟒% िस
ू रे निवेि ककया गया िा जो 𝟖% प्रनत वषध की िर

वषध के लिए 𝟓% और तीसरे वषध के लिए 𝟔% है, से सार्ारण ब्याज प्रिाि करता है। 𝟏𝟐 वषों के
बाि प्राप्त रालि योजिा B में 𝟐 𝟐 वषों के लिए
𝟏
तो ब्याज वार्षधक रूप से चक्रवद्
ृ धर् होिे की जथिनत
में ₹𝟏𝟎, 𝟎𝟎𝟎 की रालि पर इि 𝟑 वषीं का 𝟏𝟐% प्रनत वषध की िर से चक्रवद्
ृ धर् ब्याज पर
चक्रवद्
ृ धर् ब्याज ज्ञात कीजजए| निवेि कर दिया जो कक प्रनत 10 मास में
a) ₹𝟏, 𝟓𝟖𝟎. 𝟐𝟓 b) ₹𝟏, 𝟓𝟕𝟓. 𝟐𝟎 संयोजजत ककया गया। यदि B से प्राप्त ब्याज ₹
c) ₹𝟏, 𝟓𝟎𝟎. 𝟎𝟎 d) ₹𝟏, 𝟓𝟕𝟎. 𝟓𝟎 ₹𝟑𝟐, 𝟒𝟑𝟖 िा, तो 𝒙 का माि क्या है ?
a) 𝟒𝟓, 𝟎𝟎𝟎 b) 𝟒𝟎, 𝟎𝟎𝟎
9. A Certain sum in invested at 𝟐𝟔% p.a, interest
c) 𝟑𝟐, 𝟎𝟎𝟎 d) 𝟓𝟎, 𝟎𝟎𝟎
compounded annually. In how many years
(approx) will it double?
12. Vipul and Manish invested the sum of ₹𝟏𝟓𝟎𝟎𝟎
एक निजचचत 𝟐𝟔%की वार्षधक ब्याज िर से निवेि and ₹𝟐𝟎𝟎𝟎𝟎 at the rate of 𝟐𝟎 percent p.a. and
ककया जाता है , जजसपर वार्षधक रूप मे चक्रवद्
ृ धर् 𝟑𝟎 percent p.a. respectively on compound
interest (compounding annually). If time period
is 𝟑 years for both, then what will be the total
compound interest earned by Vipul and a) 𝟔 b) 𝟗 c) 𝟖 d) 𝟕
Manish?
र्वपि
ु तिा मिीष िे ₹𝟏𝟓𝟎𝟎𝟎 तिा ₹𝟐𝟎𝟎𝟎𝟎
की रालियों को क्रमिः 𝟐𝟎 प्रनतित तिा 𝟑𝟎
प्रनतित प्रनतवषध की िर से चक्रर्िधर् ब्याज 16. A Sum of money amounts to ₹𝟒, 𝟔𝟐𝟒 in 𝟐 years
and to ₹𝟒, 𝟗𝟏𝟑 in 𝟑 years at a certain rate of
(वार्षधक रूप से चक्रर्िधर् होिे वािे) पर निवेि
interest, when interest is compounded per
ककया| यदि िोिों के लिए समयावधर् 3 वषध हो, annum. The sum is_______.
तो र्वपि
ु तिा मिीष द्वारा अजजधत कुि चक्रर्िधर् ककसी र्िरालि पर वार्षधक चक्रविी ब्याज की िर
ब्याज ककतिा होगा? से 𝟐 वषध मे प्राप्त लमश्रर्ि ₹𝟒, 𝟔𝟐𝟒 और 3 वषध
a) ₹𝟑𝟐𝟒𝟖𝟎 b) ₹𝟑𝟏𝟔𝟖𝟖 मे प्राप्त लमश्रर्ि ₹𝟒, 𝟗𝟏𝟑 है| वह रालि ज्ञात
c) ₹𝟐𝟗𝟒𝟔𝟎 d) ₹𝟑𝟒𝟖𝟔𝟎
कीजजए|
13. What will be the present value of ₹𝟒𝟖𝟒 due in a) ₹𝟒, 𝟑𝟑𝟓 b) ₹𝟒, 𝟒𝟏𝟐
𝟐 years at 𝟏𝟎% per annum compound interest? c) ₹𝟒, 𝟎𝟗𝟔 d) ₹𝟒, 𝟐𝟔𝟎
𝟏𝟎% की वार्षधक चक्रवद्
ृ धर् ब्याज पर 𝟐 वषध मे
17. A man borrows a sum of ₹ 𝟏, 𝟎𝟐𝟓 and pays back
₹𝟒𝟖𝟒 का वतधमाि मूल्य क्या होगा? in two equal yearly instalments. If the rate of
a) ₹𝟔𝟎𝟎 b) ₹𝟖𝟎𝟎 interest is 𝟓% p.a, compounded yearly, then
c) ₹𝟒𝟎𝟎 d) ₹𝟐𝟎𝟎 how much is each instalments?
एक व्यजक्त ₹ 𝟏, 𝟎𝟐𝟓 उर्ार िेता है , जजसका
14. In what time will ₹𝟑, 𝟗𝟎, 𝟔𝟐𝟓 amount to ₹
𝟒, 𝟓𝟔, 𝟗𝟕𝟔 at 𝟖% per annum, interest being भग
ु ताि 2 समाि वार्षधक ककथतों में ककया जािा
compounded half-yearly? है | यदि उस पर 𝟓% की वार्षधक िर से ब्याज
₹𝟑, 𝟗𝟎, 𝟔𝟐𝟓 की रालि 𝟖% वार्षधक िर पर ककतिे लिया जाता है , जो वार्षधक रूप में चक्रवदृ ि होता
समय में ₹ 𝟒, 𝟓𝟔, 𝟗𝟕𝟔 हो जाएगी, यदि ब्याज अर्ध- है , तो प्रत्येक ककथत की रालि क्या होगी?
वार्षधक रूप से चक्रर्िधर् ककया जाता हो? a) ₹𝟓𝟐𝟓, 𝟑𝟐 b) ₹𝟒𝟐𝟓, 𝟐𝟓
𝟏 c) ₹𝟒𝟓𝟏. 𝟑𝟐 d) ₹ 𝟓𝟓𝟏. 𝟐𝟓
a) 𝟐 𝟐 𝒚𝒆𝒂𝒓𝒔 b) 𝟐 𝒚𝒆𝒂𝒓𝒔
𝟏
c) 𝟏 𝟐 𝒚𝒆𝒂𝒓𝒔 d) 𝟏 𝒚𝒆𝒂𝒓𝒔 18. A loan is to be repaid in two equal yearly
instalments. If the rate of interest is 𝟏𝟎% per
15. A sum of ₹𝟏𝟒, 𝟑𝟕𝟓 When invested at annum, compounded annually, and each
𝒓% interest per year compounded annually, instalment is ₹𝟔, 𝟖𝟗𝟕, then find the total
amounts to ₹𝟏𝟔, 𝟕𝟔𝟕 after two years. What is interest charged
the value of 𝒓? एक ऋण को िो समाि वार्षधक ककचतों में चुकाया
₹𝟏𝟒, 𝟑𝟕𝟓 की रालि जब वार्षधक आर्ार पर जािा है। यदि ब्याज की िर 𝟏𝟎% वार्षधक है, जो
चक्रवदृ िर्त 𝒓% ब्याज पर निवेि की जाती है ,तो वार्षधक रूप से चक्रवद्
ृ धर् होती है और प्रत्येक
वह िो वषध के बाि ₹𝟏𝟔, 𝟕𝟔𝟕 हो जाती है | 𝒓 का ककथत ₹𝟔, 𝟖𝟗𝟕 की, तो लिया जािे वािा कुि
माि प्रात कीजजए| ब्याज ज्ञात कीजजए|
a) ₹𝟏, 𝟗𝟏𝟒 b) ₹𝟏, 𝟕𝟑𝟒 three equal monthly instalments. If the rate of
c) ₹𝟏𝟔, 𝟒𝟒 d) ₹𝟏𝟖, 𝟐𝟒 interest charged under instalment plan is 𝟏𝟔%
p.a., then what is the monthly payment
19. A TV is available for ₹𝟏, 𝟓𝟎, 𝟎𝟎𝟎 cash or for interest being simple?
₹𝟏𝟕, 𝟔𝟎𝟎 as cash down payment and the एक सोफा सेट ₹𝟒𝟖, 𝟓𝟎𝟎 िकि में या ₹𝟏𝟎, 𝟓𝟎𝟎
balance in three equal half yearly installments.
If the interest is charged at 𝟐𝟎% p.a., की िकि डाउि पेमेंट और उसके बाि तीि बराबर
compounded half-yearly, then the total मालसक ककचतों पर उपिब्र् है | यदि ककचत योजिा
interest (in ₹) paid by a customer under this के तहत िगाए जािे वािे ब्याज की िर 𝟏𝟔%
instalment scheme is:
वार्षधक है , तो ब्याज को सार्ारण ब्याज मािते
एक टीवी ₹𝟏, 𝟓𝟎, 𝟎𝟎𝟎 िकि या ₹𝟏𝟕, 𝟔𝟎𝟎
हुए, मालसक भुगताि की जािे वािी रालि ज्ञात
िकि डाउि पेमेंट और िेष तीि समाि
कीजजए|
अद्धर्वार्षधक ककचतों में उपिब्र् है | यदि ब्यास की
a) ₹𝟏𝟑, 𝟓𝟎𝟎 b) 𝟏𝟑, 𝟒𝟎𝟎
िर 𝟐𝟎% वार्षधक है और ब्यास की गणिा c) ₹𝟏𝟑, 𝟎𝟎𝟎 c) ₹𝟏𝟑, 𝟐𝟎𝟎
अद्धर्वार्षधक चक्रवद्
ृ धर् आर्ार पर की जाती है , तो
22. Calculate the difference between the
इस ककचत- योजिा के अंतगधत ग्राहक द्वारा
compound interest and the simple interest on
भुगताि ककया गया कुि ब्यास (₹ में) ककतिा है ? a sum of ₹𝟓, 𝟎𝟎𝟎 the rate of 𝟖% for 𝟑 Years.
a) 𝟐𝟔, 𝟗𝟖𝟎 b) 𝟐𝟕, 𝟒𝟒𝟎 (to nearest ₹)
c) 𝟐𝟕, 𝟑𝟐𝟎 d) 𝟐𝟔, 𝟖𝟐𝟎 ₹𝟓, 𝟎𝟎𝟎 की रालि के लिए 𝟖% की िर से 3 वषों

20. A moneylender gave Rs.𝟐𝟖𝟎𝟎 to one of his के चक्रवद्


ृ धर् ब्याज और सार्ारण ब्याज के
client. His client decided to pay the money back बीच अंतर की गणिा कीजजये । (निकटतम ₹)
in five equal annual installments. What amount a) ₹𝟏𝟎𝟖 b) ₹𝟏𝟎𝟐
will the moneylender receive from his client if c) ₹𝟗𝟗 d) ₹𝟏𝟎𝟓
the rate of interest is 𝟏𝟎%p.a.
एक साहूकार िे अपिे ग्राहको में से एक को 𝟐𝟖𝟎𝟎 23. The difference between the compound and the
simple interests for 𝟑 years at the rate of
रुपये दिए। उिके ग्राहक िे पांच बराबर वार्षधक
𝟐𝟎% per annum is ₹𝟒𝟑𝟐. What is the principal
ककचतों में पैसे वापस िे िे का फैसिा ककया। यदि lent?
ब्याज की वार्षधक िर 𝟏𝟎% है तो साहूकार को ककसी उर्ार िी गई रालि पर 𝟐𝟎% वार्षधक िर से
अपिे ग्राहक से सािािा ककतिी रकम लमिेगी? 𝟑 वषों के चक्रवद्
ृ धर् और सार्ारण ब्याज का
a) 𝟕𝟎𝟎 b) 𝟕𝟓𝟎 अंतर ₹𝟒𝟑𝟐 है । मि ू रालि ज्ञात कीजजए|
c) 𝟖𝟎𝟎 d) none a) ₹𝟑, 𝟑𝟖𝟓 b) ₹𝟑, 𝟑𝟕𝟓 c) ₹𝟑, 𝟑𝟕𝟖 d) ₹𝟑, 𝟑𝟖𝟎

24. The difference between the compound


interest, when compounded annually, and the
simple interest on a certain sum of money for 𝟐
21. A sofa set is available for ₹𝟒𝟖, 𝟓𝟎𝟎 cash or for
years at 𝟒. 𝟐% is ₹𝟖. 𝟖𝟐. What is the sum?
₹𝟏𝟎, 𝟓𝟎𝟎 cash down payment followed by
एक निजचचत र्िरालि पर 𝟒. 𝟐% की वार्षधक िर II. Rate of interest 𝟐𝟒% per annum
compounded half yearly
से 𝟐 वषीं में प्राप्त चक्रवद्
ृ धर् ब्याज (ब्याज III. Rate of interest 𝟐𝟒% per annum
वार्षधक रूप में संयोजजत होिे पर) और सार्ारण compounded annually
ब्याज का अंतर ₹𝟖. 𝟖𝟐 है| र्िरालि ज्ञात कीजजए| IV. Rate of interest 𝟐𝟒% per annum with simple
interest
a) ₹𝟒, 𝟓𝟎𝟎 b) ₹𝟐, 𝟏𝟎𝟎
c) 𝟓, 𝟎𝟎𝟎 d) ₹𝟑, 𝟎𝟎𝟎 एक बैंक निम्िलिखित 𝟒 निवेि योजिाओं की
पेिकि करता है | यदि कोई ग्राहक ₹𝑷 की रालि
25. Rachit invests ₹𝟏𝟐, 𝟎𝟎𝟎 for a 𝟐 −year period at
का निवेि करिा चाहता है , तो 𝟏 वषध की अवधर्
a certain rate of simple interest per annum.
Prasad invests ₹𝟏𝟐, 𝟎𝟎𝟎 for a 𝟐 −year period at के बाि कौि सी योजिा ग्राहक के लिए अधर्क
the same rate of interest per annum as Rachit, िाभिायक होगी?
but in Prasad’s case the interest in
compounded annually. Find the rate of interest I. ब्याज िर 𝟐𝟒% वार्षधक, नतमाही चक्रवद्
ृ धर्
per annum if Prasad receives ₹𝟏𝟕𝟐. 𝟖𝟎 more as आर्ार पर गणिीय
interest than Rachit at the end of the 𝟐-year II. ब्याज िर 𝟐𝟒% वार्षधक, अर्ध-वार्षधक चक्रवद्
ृ धर्
period.
आर्ार पर गणिीय
रधचत ₹𝟏𝟐, 𝟎𝟎𝟎 की रालि का निवेि वार्षधक
III. ब्याज िर 𝟐𝟒% वार्षधक, अर्ध-वार्षधक चक्रवद्
ृ धर्
सार्ारण ब्याज की एक निचचत िर पर 𝟐 वषध
आर्ार पर गणिीय
की अवधर् के लिए करता है | प्रसाि ₹𝟏𝟐, 𝟎𝟎𝟎 की
IV. 𝟐𝟒% वार्षधक सार्ारण ब्याज की िर
रालि का निवेि रधचत के समाि वार्षधक ब्याज
a) 𝑰𝑰𝑰 b) 𝑰𝑽 c) 𝑰𝑰 d) 𝑰
िर पर 𝟐 वषध की अवधर् के लिए करता है | िेककि
प्रसाि के ब्याज की गणिा वार्षधक चक्रवद्
ृ धर् 27. The sum of the simple interest and the
compound interest on ₹𝟓, 𝟔𝟎𝟎 at the same rate
आर्ार पर होती है | यदि 𝟐 वषध की अवधर् के अंत
of interest per annum of two years is
में प्रसाि को ब्याज के रूप में रधचत से ₹𝟏𝟕𝟐. 𝟖𝟎 ₹𝟐, 𝟎𝟔𝟏. 𝟑𝟔 when in the case of compound
अधर्क लमिते है , तो वार्षधक ब्याज की िर ज्ञात interest, it is compounded annually. What is
the rate of interest per annum?
कीजजए|
₹𝟓, 𝟔𝟎𝟎 की रालि पर समाि वार्षधक ब्याज िर
a) 𝟏𝟎% b) 𝟖% c) 𝟏𝟐% d) 𝟓%
से िो वषीं में प्राप्त सार्ारण ब्याज और चक्रवद्
ृ धर्
ब्याज का योग ₹𝟐, 𝟎𝟔𝟏. 𝟑𝟔 है | यदि चक्रवद्
ृ धर्
ब्याज वार्षधक रूप से चक्रवद्
ृ धर् ककया जाता है , तो
26. A bank offers following 𝟒 schemes for ब्याज की वार्षधक िर ज्ञात कीजजए|
investment. If a customer wants to invest an a) 𝟗% b) 𝟔%
amount of ₹𝑷. then which scheme is more c) 𝟏𝟓% d) 𝟏𝟐%
beneficial for a customer after a period of 𝟏
year? 28. A sum of ₹𝟐, 𝟒𝟎𝟎 is invested for 𝟒 years at 𝑹%
I. Rate of interest 𝟐𝟒% per annum p.a, simple interest and the simple interest
compounded quarterly
received is ₹𝟏, 𝟏𝟓𝟐 . Find the amount received c) ₹𝟐, 𝟏𝟎𝟎 d) ₹𝟐, 𝟑𝟓𝟎
after 𝟑 years when the same sum is invested at
(𝑹 + 𝟖)% p.a, simple interest.
₹𝟐, 𝟒𝟎𝟎 की रालि को 𝟒 वषों के लिए 𝑹% प्रनत
वषध की िर से सार्ारण ब्याज पर निवेि ककया 31. A sum of ₹𝟏𝟖, 𝟓𝟎𝟎 is invested at 𝒙% p.a. at
जाता है , और प्राप्त सार्ारण ब्याज ₹𝟏, 𝟏𝟓𝟐 है। simple interest and another sum, which is twice
the former, at (𝒙 + 𝟐)% p.a. at simple interest.
𝟑 वषों के बाि लमश्रर्ि का पता िगाएं जब समाि
If the interest received on these investments
रालि (𝑹 + 𝟖)% प्रनत वषध, सार्ारण ब्याज पर 𝟏
after 𝟑 𝟐 years is ₹𝟐𝟓, 𝟗𝟎𝟎. the rate of interest
निवेि की जाती है। p.a. on the second investment is:
a) ₹𝟑, 𝟐𝟔𝟎 b) ₹𝟑𝟏𝟖𝟎 ₹𝟏𝟖, 𝟓𝟎𝟎 की एक रालि 𝒙% वार्षधक सार्ारण
c) ₹𝟐, 𝟖𝟖𝟎 d) ₹𝟑𝟖𝟒𝟎
ब्याज की िर पर निवेि की जाती है , और िस
ू री
29. At simple interest, an amount of रालि, जो पहिी रालि की िोगुिी है , (𝒙 + 𝟐)%
₹𝟕𝟎𝟖 becomes ₹𝟗𝟒𝟒 in 𝟒 years. If the rate of वार्षधक सार्ारण ब्याज िर पर निवेि की जाती
interest is increased by 𝟓%, then the amount
है | यदि 𝟑 𝟐 वषध बाि इि निवेिों पर प्राप्त ब्याज
𝟏
of ₹𝟔𝟕𝟓 will increase to how much in 𝟒 years?
सार्ारण ब्याज पर, ₹𝟕𝟎𝟖 की रालि 𝟒 वषों में ₹𝟐𝟓, 𝟗𝟎𝟎 है , तो िस
ु रे निवेि पर वार्षधक ब्याज
₹𝟗𝟒𝟒 हो जाती है | यदि ब्याज की िर में 𝟓% की िर ककतिी है ?
ृ धर् की जाती है , तो ₹𝟔𝟕𝟓 की रालि 𝟒 वषीं में
वद् a) 𝟏𝟐% b) 𝟏𝟐. 𝟓%
c) 𝟏𝟒% d) 𝟏𝟓. 𝟓%
बढ़कर ककतिी हो जाएगी?
a) ₹𝟗𝟔𝟓 b) ₹𝟏, 𝟎𝟕𝟓 32. A person invested a sum of ₹𝟒, 𝟓𝟎𝟎 at 𝒓% per
c) ₹𝟏, 𝟎𝟑𝟓 d) ₹𝟏, 𝟏𝟒𝟓 annum at simple interest and a sum of ₹𝟔, 𝟕𝟓𝟎
at (𝒓 − 𝟐)% per annum at simple interest. If
30. Karun borrowed ₹𝟏𝟐, 𝟎𝟎𝟎 from Varun and the total interest earned on both the
₹𝟏𝟓, 𝟎𝟎𝟎 from Tarun, each for two years at the investments for 𝟓 years is ₹𝟒, 𝟔𝟔𝟖. 𝟕𝟓, then
same rate of simple interest per annum. If he average of the rate of interest on both the
paid an interest of ₹𝟏, 𝟔𝟖𝟎 to Varun at the end investments is:
of 𝟐 years, then what is the interest that Karun
एक व्यककत िे ₹𝟒, 𝟓𝟎𝟎 की र्िरालि को 𝒓%
paid to Tarun at the end of the 𝟐 −year period?
करुण िे वरुण से ₹𝟏𝟐, 𝟎𝟎𝟎 और तरुण से वार्षधक सार्ारण ब्याज की िर पर और ₹𝟔, 𝟕𝟓𝟎

₹𝟏𝟓, 𝟎𝟎𝟎 समाि वार्षधक सार्ारण ब्याज िर पर की रालि को (𝒓 − 𝟐)% वार्षधक सार्ारण ब्याज

िो वषध के लिए उर्ार लिए| यदि करुण िे 𝟐 वषध की िर पर निवेलित ककया| यदि िोिों निवेिों पर

की अवधर् के अंत में वरुण, को ₹𝟏, 𝟔𝟖𝟎 का ब्याज 𝟓 वषध में अजजधत कुि ब्याज ₹𝟒, 𝟔𝟔𝟖. 𝟕𝟓 है , तो

दिया, तो 𝟐 वषध की अवधर् के अंत में वह तरुण िोिों निवेिों पर िगिे वािी ब्याज िरों का औसत

को ककतिा ब्याज िे गा? ककतिा है ?


a) ₹𝟐, 𝟎𝟓𝟎 b) ₹𝟏, 𝟖𝟎𝟎 a) 𝟕. 𝟓% b) 𝟗%
c) 𝟗. 𝟓% d) 𝟖. 𝟓%
Answer Key
1. A 2. C 3. D 4. A 5. D
6. A 7. B 8. B 9. B 10. B
11. D 12. D 13. C 14. B 15. C
16. C 17. D 18. D 19. C 20. A
21. C 22. C 23. B 24. C 25. C
26. D 27. A 28. D 29. C 30. C
31. C 32. D
Average & Ration and Proportion

क्रमागत 𝟏𝟎 पूणााकों का औसत है | इन पूणााकों


Instruction: - For Video Solution scan QR code 𝟏𝟏
𝟐
given below or you can also CLICK on Click here
button to watch the Video Solution. में से “सबसे छोटी चार” संख्याओं का औसत
ककतना है ?
a) 𝟐. 𝟓 b) 𝟑. 𝟓
c) 𝟑 d) 𝟐

5. The average of nine consecutive numbers is 24.


Average What is the average of the smallest three of
1. The average of six consecutive odd numbers is these 9 given numbers?
𝟐𝟔. Find the difference between the largest and नौ क्रमागत संख्याओं का औसत 24 है । इन 9
the smallest of these numbers.
दी गई संख्याओं में से सबसे छोटी तीन संख्याओं
छ: क्रमागत विषम संख्याओं का औसत 𝟐𝟔 है|
का औसत क्या है ?
इन संख्याओं में से सबसे बड़ी और सबसे छोटी
a) 𝟐𝟏 b) 𝟐𝟎 c) 𝟏𝟖 d) 𝟏𝟗
संख्या का अंतर ज्ञात कीजिए|
a) 𝟖 b) 𝟏𝟔 c) 𝟏𝟎 d) 𝟏𝟐

2. The average of 𝟓 consecutive odd number is 𝟑𝟑.


What is the product of the smallest and the 𝟐𝟕
6. The average of 𝟖 consecutive integers is .
𝟐
largest of these five numbers?
What is the average of the largest four of these
𝟓 क्रमागत विषम संख्याओं का औसत 𝟑𝟑 है| इन integers?
पांचों संख्याओं में से सबसे छोटी और सबसे बड़ी 𝟖 क्रमागत पण
ू ाांकों का औसत है | इन पण
ू ाांकों
𝟐𝟕
𝟐
संख्याओं का गण
ु नफल क्या होगा? में से सबसे बड़े चार पूणाांकों का औसत ककतना
a) 𝟏𝟏𝟒𝟕 b) 𝟏𝟐𝟑𝟕 c) 𝟗𝟓𝟕 d) 𝟏𝟎𝟕𝟑 है ?
a) 𝟏𝟔. 𝟓 b) 𝟏𝟓
3. There are 𝟑𝟐 consecutive natural numbers. The
c) 𝟏𝟒. 𝟓 d) 𝟏𝟓. 𝟓
average of the first 𝟏𝟑 numbers is x. What is the
average of the next 𝟏𝟗 numbers?
7. The average of eight consecutive odd numbers is
𝟑𝟐 क्रमागत प्राकृत संख्याएँ है | पहली 𝟏𝟑 𝟏𝟔. If the odd number which is immediately
संख्याओं का औसत 𝒙 है| अगली 𝟏𝟗 संख्याओं का preceding the smallest of the given eight odd
numbers is also included, then what will be the
औसत क्या होगा?
new average?
a) 𝒙 + 𝟏𝟓 b) 𝒙 + 𝟏𝟔
आठ क्रमागत विषम संख्याओं का औसत 𝟏𝟔 है |
c) 𝒙 + 𝟏𝟕 d) 𝒙 + 𝟏𝟖
यदद दी गई आठ विषम संख्याओं में से सबसे
4. The average of 𝟏𝟎 consecutive integers is
𝟏𝟏
छोटी से ठीक पहले िाली विषम संख्या को भी
. What is the average of the “smallest four’’
𝟐 शममल कर मलया िाए, तो नया औसत ककतना
of these integers?
होगा?
a) 𝟏𝟓 b) 𝟏𝟒 c) 𝟗 d) 𝟏𝟑
𝟗𝟗 𝟗𝟗 + 𝟗𝟗 𝟗𝟗 + 𝟗𝟗 𝟗𝟗 +. . .. . +𝟗𝟗 𝟗𝟗 का मान
𝟏𝟏 𝟏𝟑 𝟏𝟓 𝟔𝟕

8. The average of 10 consecutive even numbers is


25, If the even number just preceding the क्या है ?
smallest of the 10 given numbers is also a) 𝟗𝟒𝟐𝟐𝟎/𝟑𝟑 b) 𝟗𝟓𝟏𝟐𝟎/𝟑𝟑
included, then what will be the new average? c) 𝟗𝟕𝟏𝟐𝟎/𝟑𝟑 d) 𝟗𝟔𝟐𝟐𝟎/𝟑𝟑
𝟏𝟎 क्रमागत सम संख्याओं का औसत 𝟐𝟓 है| यदद
12. What is the difference between the average of
इन 𝟏𝟎 संख्याओं में , इनमें में सबसे छोटी संख्या first 𝟏𝟒𝟖 even positive numbers and the
से ठीक पहले आने िाली सम संख्या को में शममल average of first 𝟏𝟐𝟗 odd positive numbers?
कर मलया िाए, तो नया औसत क्या होगा? प्रथम𝟏𝟒𝟖 सम धनात्मक संख्याओं के औसत तथा
a) 𝟏𝟏 b) 𝟗 c) 𝟐𝟑 d) 𝟐𝟒 प्रथम 𝟏𝟐𝟗 विषम धनात्मक संख्याओं के औसत
के बीच का अंतर क्या है ?
9. The average of seven consecutive odd numbers
a) 𝟐𝟏 b) 𝟐𝟎 c) 𝟐𝟑 d) 𝟏𝟗
is 27. If the last number is removed from the
list, then the average of the remaining numbers
13. The average of squares of five consecutive odd
is:
natural numbers is 233. What is the average of
सात क्रमागत विषम संख्याओं का औसत 27 है। the largest number and the smallest number?
यदद अंततम संख्या सूची से हटा दी िाए, तो शेष पांच क्रमागत विषम प्राकृततक संख्याओं के िगों
संख्याओं का औसत ज्ञात कीजिए| का औसत 233 है| सबसे बड़ी संख्या और सबसे
a) 26 b) 25 c) 24 d) 22 छोटी संख्या का औसत ज्ञात करें |
a) 15 b) 17 c) 11 d) 13
10. The average of 𝟑𝟓 consecutive natural
numbers is 𝑵. Dropping the first 𝟏𝟎 numbers
14. The average of the squares of four consecutive
and including the next 𝟏𝟎 numbers, the
odd natural numbers is 201. The average of 7
average is changed to 𝑴. If the value of 𝑴𝟐 − times of the largest number and 3 times of the
𝑵𝟐 = 𝟔𝟎𝟎, then the average of 𝟑𝑴 and 𝟓𝑵 is: smallest number is:
क्रमागत 𝟑𝟓 प्राकृततक संख्याओं का औसत 𝑵 है| चार क्रमागत विषम प्राकृततक संख्याओं के िगों
अगर पहली 𝟏𝟎 संख्याओं को तनकाल ददया िाए का औसत 201 है | इनमें से सबसे बड़ी संख्या के
और आगे की 𝟏𝟎 संख्याओं को शाममल कर मलया 7 गुने और सबसे छोटी संख्या के 3 गुने का
िाए, तो यह औसत 𝑴 हो िाता है , यदद 𝑴 − 𝟐
औसत ज्ञात करें |
𝑵 = 𝟔𝟎𝟎 है तो 𝟑 𝑴 और 𝟓 𝑵 का औसत ____
𝟐
a) 72 b) 78 c) 76 d) 66
होगा|
a) 90 b) 120 c) 100 d) 115 15. The average of the squares of four consecutive
even natural numbers is 126. The average of 8
times of the greatest number and 5 times of the
smallest number is:
चार क्रमागत सम प्राकृततक संख्याओं के िगों का
𝟏𝟏 𝟏𝟑 𝟏𝟓
11. What is the value of 𝟗𝟗 𝟗𝟗 + 𝟗𝟗 𝟗𝟗 + 𝟗𝟗 𝟗𝟗 +. . .. औसत 126 है | सबसे बड़ी संख्या के 8 गुने और
𝟔𝟕
. +𝟗𝟗 𝟗𝟗 ? सबसे छोटी संख्या के 5 गुने का औसत ज्ञात
करें |
a) 66 b) 76 c) 68 d) 74
19. The average weight of some student in a group
is 𝟓𝟖 kg. If 𝟖 student of average weight 𝟓𝟒 kg
leave the group, and 𝟑 students weighing 𝟓𝟑. 𝟔
kg, 𝟓𝟒 kg and 𝟓𝟕. 𝟒 kg join the group, then the
16. Consider ten consecutive odd numbers starting average weight of the remaining students in
from 5. Multiply each of them, except the first the group will increase by 𝟓𝟕𝟓 g. The number
and the second, with three. What will be the of students, initially, in the group is:
average of the ten numbers so formed? ककसी समह
ू में कुछ छात्रों का औसत ििन 𝟓𝟖
𝟓 से शुरू करके दस क्रममक विषम संख्याए लें|
ककग्रा है | यदद 𝟓𝟒 ककग्रा औसत ििन िाले 8
पहली और दस
ू रे को छोड़कर, शेष प्रत्येक को
छात्र समह
ू छोड़ दे ते है और 𝟓𝟑. 𝟔 ककग्रा, 𝟓𝟒
तीन से गुणा करें । तीन के साथ। इस प्रकार से
ककग्रा और 𝟓𝟕. 𝟒 ककग्रा ििन िाले 3 छात्र समह

बनने िाली दस संख्याओं का औसत क्या होगा?
मे शाममल होते है , तो समह
ू मे शेष छात्रों के
a) 𝟑𝟗. 𝟔 b) 𝟑𝟗 c) 𝟒𝟐 d) 𝟑𝟖. 𝟒
औसत ििन मे 𝟓𝟕𝟓 ग्राम की िद्
ृ धध होगी |
17. The average of ten numbers is A. If c is समूह में छात्रों की आरं मभक संख्या ककतनी थी?
subtracted from each number, except the tenth a) 40 b) 45 c) 35 d) 50
and (𝒄 − 𝟏) is subtracted from the tenth
number, then what will be the new average? 20. The average weight of a group of 3 people A, B
दस संख्याओं का औसत A है। यदद दसिीं संख्या and C is 70 kg. When D joins this group, the
को छोड़कर प्रत्येक संख्या से c घटाया िाता है average becomes 60 kg. A man E, whose weight
is 5 kg more than that of D, replaces A and the
और दसिीं संख्या से (𝒄 − 𝟏) घटाया िाता है , तो average weight of B, C, D, and E now become
नया औसत क्या होगा? 59 kg. What is the average weight (in kg) of A,
a) 𝑨 − 𝒄 + 𝟎. 𝟏 b) 𝑨 − 𝒄 D and E? (correct to the nearest integer)
c) 𝑨 − 𝒄 + 𝟏 d) 𝑨 − (𝟎. 𝟏)𝒄 + 𝟎. 𝟏 𝟑 व्यजक्तयों 𝑨, 𝑩 और 𝑪 िाले समह
ू का औसत
ििन 𝟕𝟎𝒌𝒈 है| िब इस समूह में 𝑫 शाममल
18. The average weight of some students in a class
was 69.5 kg. When 10 students of average होता है , तो औसत 𝟔𝟎𝒌𝒈 हो िाता है | एक व्यजक्त
weight 68 kg joined the class, and 6 students of 𝑬 जिसका ििन 𝑫 के ििन से 𝟓𝒌𝒈 अधधक है,
average weight 60 kg left the class, it was noted
𝑨 का स्थान ले लेता है और अब 𝑩, 𝑪, 𝑫 और
that the average weight of the new group of
students increased by 2 kg. How many students 𝑬 का औसत ििन 𝟓𝟗𝒌𝒈 हो िाता है | 𝑨, 𝑫 और
are there in the class now? 𝑬 का औसत ििन (𝑲𝒈 में) ज्ञात करें (तनकतम
ककसी कक्षा में कुछ छात्रों का औसत ििन 69.5 पूणााक तक शुद्ध)
kg था। िब 68 kg औसत ििन िाले 10 छात्र a) 40 b) 35 c) 30 d) 39
कक्षा में शाममल होते हैं और 60 kg औसत ििन
िाले 6 छात्र कक्षा छोड़ते है , तो यह दे खा गया कक
छात्रों के नए समूह के औसत ििन में 2 kg की
21. The average weight of some girls in a group is
िद्
ृ धध हुई है । अब कक्षा में ककतने छात्र हैं?
𝟓𝟖. 𝟒𝒌𝒈. When 𝟏𝟓 girls of average weight 𝟔𝟐. 𝟐
a) 19 b) 21 c) 29 d) 26 kg leave the group or 𝟏𝟎 girls of average weight
𝟒𝟗. 𝟖𝟓𝒌𝒈 join the group, the average weight 24. The average weight of all the 𝟐𝟎𝟎 employees
(in kg) of girls in both the cases is equal. What 𝟐
in an office is 𝟔𝟖. 𝟐 kg, where 𝟓 of the
is the number of girls in the group initially? employees are ladies and the ratio of the
एक समूह में कुछ बामलकाओं का औसत ििन average weight of a man to that of a woman is
𝟓𝟖. 𝟒𝒌𝒈 है| िब 𝟔𝟐. 𝟐 kg औसत ििन िाली 𝟏𝟓 𝟕 : 𝟓. Find the average weight of the female
employees.
बामलकाएं समूह छोड़ दे ती हैं, या 𝟒𝟗. 𝟖𝟓𝒌𝒈 औसत
एक कायाालय के सभी 𝟐𝟎𝟎 कमाचाररयों का औसत
ििन िाली 𝟏𝟎 बामलकाएं उस समूह में शाममल
ििन 𝟔𝟖. 𝟐 kg है, िहां कमाचारी मदहलाएं हैं|
𝟐

हो िाती हैं, तो दोनों ही मामलों में बामलकाओं का 𝟓


एक पुरुष और एक मदहला के औसत ििनों का
औसत ििन (kg में) समान प्राप्त होता है | प्रारं भ
अनुपात 𝟕 : 𝟓 है| सभी मदहला कमाचाररयों का
में समह
ू में बामलकाओं की संख्या ककतनी थी?
औसत ििन ज्ञात कीजिए|
a) 𝟔𝟓 b) 𝟕𝟓 c) 𝟒𝟓 d) 𝟓𝟓
a) 𝟔𝟎 kg b) 𝟓𝟒 kg c) 𝟓𝟓 kg d) 𝟓𝟗 kg
22. There are some children in a camp and their
average weight is 40 kg. If 5 children with 25. The number of students in a class is 45. out of
𝟏
average weight 36 kg join the camp or if 5 which 𝟑𝟑 𝟑 % are boys and the rest are girls.
children with average weight 𝟒𝟑. 𝟐 kg leave the 𝟐
The average score of girls in science is 𝟔𝟔 𝟑 %
camp, the average weight of children in both
more than that of boys. If the average score of
cases is equal. How many children are there in
all the students is 78, then the average score of
the camp, initially?
girls is:
एक मशविर में कुछ बच्चों का औसत ििन 40
एक कक्षा में विद्याधथायों की संख्या 45 है , जिसमें
kg है। यदद 36 kg औसत ििन िाले 5 बच्चे
से 𝟑𝟑 𝟑 % लड़के हैं और शेष लड़ककयां हैं। विज्ञान
𝟏

मशविर में शाममल हो िाएं या यदद 𝟒𝟑. 𝟐 kg


में लड़ककयों के औसत प्राप्तांक लडकों के औसत
औसत ििन िाले 5 बच्चे मशविर से चले िाएं,
प्राप्तांक से 𝟔𝟔 𝟑 % अधधक हैं। यदद सभी
𝟐

तो दोनों जस्थततयों में बच्चों का औसत ििन


विद्याधथायों के औसत प्राप्तांक 78 है, तो लड़ककयों
समान रहे गा| आरं भ मशविर में ककतने बच्चे थे?
के औसत प्राप्तांक ककतने है ?
a) 50 b) 45 c) 35 d) 40
a) 𝟕𝟖 b) 𝟓𝟒 c) 𝟗𝟎 d) 𝟔𝟓
23. In a test, the average score of 𝟓𝟎 students is 66.
Out of the total students, 𝟒𝟎% are girls and the
rest are boys. The average score of the boys is
𝟐𝟎% less than that of the girls. What is the
26. The total number of students in class A and B is
average score of the girls?
92. The number of students in A is 𝟑𝟎% more
एक टे स्ट मे, 𝟓𝟎 विद्याधथायों का औसत प्राप्तांक than that in B. The average weight (in kg) of
𝟔𝟔 है। कुल विद्याधथायों में से, 𝟒𝟎% लड़ककयां हैं students in B is 𝟓𝟎% more than that of
students in A. If the average weight of all the
और शेष लड़के हैं। लड़कों का औसत प्राप्तांक
students in A and B is 56 kg, then what is the
लड़ककयों का औसत प्राप्तांक क्या है ? average weight (in kg) of students in B?
a) 𝟖𝟎 b) 𝟔𝟓 c) 𝟕𝟎 d) 𝟕𝟓 कक्षा A और B में कुल छात्रों की संख्या 92 है।
कक्षा A छात्रों की संख्या, कक्षा B में छात्रों की
संख्या की तुलना में 𝟑𝟎% अधधक है। कक्षा B में
29. The average of 40 numbers is 𝟒𝟖. 𝟐. The
छात्रों का औसत ििन (kg में), कक्षा A में छात्रों average of the first 15 numbers is 45 and that
के औसत ििन से 𝟓𝟎% अधधक है। यदद कक्षा of the next 22 numbers is 𝟓𝟎. 𝟓 The 38th
A और B में सभी छात्रों का औसत ििन 56 kg है, number is 1 more than the 39th number and the
39th number is 3 less than the 40th number .
तो कक्षा B में छात्रों का औसत ििन (kg में) What is the average of the 39th and 40th
ककतना है ? numbers?
a) 46 b) 40 c) 69 d) 52 40 संख्याओं का औसत 𝟒𝟖. 𝟐 है। प्रथम 15
संख्याओं का औसत 45 है और अगली 22
27. The average of eighteen numbers is 42. The
average of the last ten numbers is 40 and that संख्याओं का औसत 𝟓𝟎. 𝟓 है 38 िीं संख्या, 39
of the first five numbers is 44. The seventh िीं संख्या, से 1 अधधक है और 39 िीं संख्या, 40
number is 6 less than the sixth and 7 less than
िीं संख्या से 3 कम है। 39िीं और 40 िीं संख्या
the eighth number. The average of the sixth
and the eighth number is: का औसत ज्ञात करें |
अठारह संख्याओं का औसत 42 है। अंततम दस a) 48 b) 49 c) 47.5 d) 48.5
संख्याओं का औसत 40 है और पहली पाँच 30. There are three positive numbers. If the
संख्याओं का औसत 44 है। सातिीं संख्या, छठी average of any two of them is added to the
संख्या से 6 कम और आठिीं संख्या से 7 कम third number, the resulting sums are 154, 148
and 132. The sum of the original three numbers
है। छठी और आठिीं संख्याओं का औसत-------- is:
--- है| तीन घनात्मक संख्याओं में से ककनहीं दो संख्याओं
a) 𝟒𝟔. 𝟓 b) 𝟒𝟕. 𝟓 c) 𝟒𝟓 d) 𝟒𝟖 का औसत, तीसरी संख्या में िोड़ा िाता है , तो

28. Average age of 𝟕 students of a class is 𝟐𝟖 years. पररणामी योगफल 154, 148 और 132 प्राप्त होते
Average age of first three students is 𝟑𝟎 years. हैं। तीन आंरमभक संख्याओं का योगफल ज्ञात
Age of fourth student is 𝟒 years less than the
करें |
age of fifth student. Ages of last two students is
same and is 𝟓 more than the average age of a) 222 b) 231 c) 246 d) 217
first three students. What is the average age of
fourth fifth student?
एक कक्षा के 𝟕 छात्रों की औसत आयु 𝟐𝟖 िषा
है| प्रथम तीन छात्रों की औसत आयु 𝟑𝟎 िषा है | 31. There are three positive integers. If the average
of any two of them is added to the third, the
चौथे छात्रों की आयु, पाँचिे छात्र की आयु से 𝟒
resulting sums are 181, 172 and 160. The
िषा कम है | अंततम दो छात्रों की आयु समान है average of the given integers is:
तथा प्रथम तीन छात्रों की औसत आयु से 𝟓 िषा तीन धनात्मक पूणाांक हैं। यदद उनमें से ककनहीं
अधधक है | चौथे तथा पाँचिे छात्र की औसत आयु दो पूणाांकों का औसत, तीसरे पूणाांक में िोड़ा िाता
ककतनी है ? है , तो पररणामी योगफल 181, 172 और 160
a) 𝟐𝟎 𝒚𝒆𝒂𝒓𝒔 b) 𝟑𝟔 𝒚𝒆𝒂𝒓𝒔 होता है । ददए गए पूणाांकों का औसत ज्ञात करें |
c) 𝟏𝟔 𝒚𝒆𝒂𝒓𝒔 d) 𝟏𝟖 𝒚𝒆𝒂𝒓𝒔 a) 𝟕𝟖. 𝟓 b) 𝟖𝟓. 𝟓
c) 86 d) 84 एक डडनर पाटी के मलए 30 लोग ककसी रे स्टोरे नट

32. If the average of the 3-digit numbers में गए| उनमें से 20 ने प्रत्येक रु 880 का भुगतान
𝟑𝟑𝟓, 𝟐𝒙𝟓, 𝒙𝟑𝟓, 𝟔𝟑𝒙 and 406 is 411, then ककया और शेष लोगों में से प्रत्येक ने कुल खचों
what will be the average of 𝒙– 𝟏, 𝒙– 𝟑, 𝒙 + 𝟑 के औसत से 110 अधधक भुगतान ककया| डडनर के
and 𝒙 + 𝟓?
मलए कुल खचा (रु में ) ककतना था?
3 अंक िाली संख्याओं 𝟑𝟑𝟓, 𝟐𝒙𝟓, 𝒙𝟑𝟓, 𝟔𝟑𝒙 और
a) 27,840 b) 29,360
406 का औसत 411 है , तो 𝒙– 𝟏, 𝒙– 𝟑, 𝒙 + 𝟑 c) 24,580 d) 28,050
और 𝒙 + 𝟓 का औसत ज्ञात करें |
a) 6 b) 3 c) 5 d) 4

33. The average score of A. B and C in a test is 78


and that of C,D and E is 52. The average score 36. There were 1200 students in a hostel. If the
of E and F is 48 and that of E and C is 60. What number of students is increased by 300, then
is the average score of A, B, C, D, E and F ? the expenses of the mess increase by ₹5,000
per day, while the average expenditure per
एक परीक्षण में A, B और C का औसत स्कोर 𝟕𝟖 head diminishes by ₹5. The original
है और C,D और E का 𝟓𝟐 है। E और F का औसत expenditure of the mess, per day, is:
स्कोर 𝟒𝟖 है और E और C का औसत स्कोर 60 एक होस्टल में 1200 छात्र थे| यदद 300 छात्र और
है। A, B, C, D, E और F का औसत स्कोर क्या है ? बढ़ िाते हैं, तो मेस का खचा ₹5,000 प्रतत ददन
a) 62 b) 67 c) 63 d) 61 बढ़ िाता है , िबकक प्रतत व्यजक्त औसत खचा में
₹5 की कमी हो िाती है| मेस का प्रतत ददन का
34. 𝒙, 𝒚 and 𝒛 are three positive numbers such that
𝟒 𝟓
𝒚 is 𝟓 times of 𝒙 and 𝒛 is 𝟖 times of 𝒚. If the मूल्य खचा ककतना है ?
average of reciprocals of the numbers 𝒙, 𝒚 and a) ₹ 52,000 b) ₹ 45,000
𝟏𝟕 c) ₹ 50, 000 d) ₹ 55,000
𝒛 is 𝟐𝟒𝟎, then the average of 3 times of 𝒙 and 5
times of 𝒚 will be: 37. The average age of 7 members of a family is 35
𝒙, 𝒚 और 𝒛 तीन धनात्मक संख्याएं इस प्रकार हैं years. If the youngest members are twins and
they are 15 years old, then what was the
कक 𝒚, 𝒙 का गुना है और 𝒛,𝒚 का गुना है|
𝟒 𝟓
𝟓 𝟖 average age (in years) of the family members at
यदद संख्या 𝒙, 𝒚 और 𝒛 के व्यत्ु क्रमों का औसत the time of the birth of the twins?
है, तो 𝒙 के तीन गन
ु े और 𝒚 के 5 गन
ु े का एक पररिार के 7 सदस्यों की औसत आयु 35
𝟏𝟕
𝟐𝟒𝟎
औसत ज्ञात करें | िषा है । यदद सबसे कम उम्र के सदस्य िड़
ु िाँ हैं
a) 70 b) 60 c) 40 d) 45 और िे 15 िषा के है , तो िड़
ु िाँ बच्चों के िनम
के समय पररिार के सदस्यों की औसत आयु (िषों
35. 30 people went to a restaurant for a dinner
party. 20 of them paid Rs. 880 each of the rest में) ककतनी थी?
of them paid Rs. 110 more than the average of a) 28 b) 27 c) 30 d) 25
the total expenses. What was the total expense
(in Rs.) for the dinner? 38. 𝟖 years ago, the average age of Rahul’s family
consisting of 𝟔 members was 𝟑𝟓 years. One
तीन संख्याओं ,𝒃 और 𝒄 का माध्य िही है , िो
year ago a new baby was born in this family. 𝒂
𝟑
What will be the average age of the family four
संख्याओं 𝟑, 𝟑 और 𝒙 का माध्य है। 𝟑𝒙 का मान
𝒃 𝒄
years hence?
𝟖 िषा पहले, राहुल के 𝟔 सदस्यों िाले पररिार क्या है (िहाँ a, b, c धनात्मक संख्याएँ हैं)?
की औसत आयु 𝟑𝟓 िषा थी| एक िषा पहले इस a) 𝒃 + 𝒄 b) 𝒂 + 𝟐𝒃 + 𝟐𝒄
c) 𝒂 d) 𝒂 + 𝟑𝒃 + 𝟑𝒄
पररिार में एक नए बच्चे का िनम हुआ था| चार
िषा बाद इस पररिार की औसत आयु क्या होगी? 42. The average of the numbers a, b, c and d is
a) 𝟒𝟏 years b) 𝟑𝟕 years 𝟐𝒅 + 𝟒. Also, the averages of the numbers a
c) 𝟒𝟎 years d) 𝟒𝟑 years and b; b and c; c and d; are 8, 5 and 4,
respectively. If 𝒆 = 𝒂 + 𝒅 + 𝟏, then what is
39. The average of the four numbers, given in a the average of the numbers d and e?
particular order, is 𝟒𝟖. The average of the first संख्याओं a, b, c और d का औसत 𝟐𝒅 + 𝟒 है।
three numbers is 𝟒𝟐 while the average of the साथ ही, संख्या a और b; b और c; c और d; का
last three numbers is 𝟓𝟎. what is the average of
the first and last numbers? औसत क्रमशः 8, 5 और 4 है। यदद 𝒆 = 𝒂 +
एक विशेष क्रम में दी गई चार संख्याओं का 𝒅 + 𝟏 है, तो संख्या d और e का औसत ज्ञात
औसत 𝟒𝟖 है | पहली तीन संख्याओं का औसत करें |
𝟒𝟐 है, िबकक अंततम तीन संख्याओं का औसत a) 8 b) 𝟖. 𝟓 c) 3 d) 7

𝟓𝟎 है पहली और अंततम संख्याओं का औसत 43. The average of 𝐱, 𝒚 and 𝒛 is 6 more than 𝒛. The
ज्ञात कीजिए| average of 𝒙 and 𝒚 is 50. If u is 6 less than 𝒛,
a) 𝟓𝟑 b) 𝟓𝟒 c) 𝟓𝟔 d) 𝟓𝟓 then what is the average of u and 𝒛 ?
𝒙, 𝒚 और 𝒛 का औसत 𝒛 से 6 अधधक है। 𝒙 और
40. The average of five numbers, given in a 𝒚 का औसत 50 है। यदद u, 𝒛 से 6 कम है , तो u
particular order, is 𝟑𝟐. The average of the first
three numbers is 𝟐𝟖, while that of the last और 𝒛 का औसत ज्ञात करें |
three numbers is 𝟑𝟒. What is the average of the a) 43 b) 38 c) 40 d) 39
first two numbers?
एक तनजचचत क्रम में दी गई पांच संख्याओं का 44. The average monthly expenditure of a family
for the first four months is ₹𝟏𝟑, 𝟕𝟓𝟎, for the
औसत 𝟑𝟐 है| पहली तीन संख्याओं का औसत next three months is ₹𝟏𝟏, 𝟕𝟓𝟎and for the last
28 है , तथा अंततम तीन संख्याओं का औसत 𝟑𝟒 five months is ₹𝟑𝟏, 𝟕𝟓𝟎. If the family saves
₹𝟏𝟓, 𝟓𝟓𝟎 during the whole year, find the
है| पहली दों संख्याओं का औसत क्या है ?
average monthly income of the family during
a) 𝟑𝟎 b) 𝟐𝟕 c) 𝟐𝟖 d) 𝟐𝟗 the year. ( consider integral part only)
एक पररिार का औसत मामसक खचा पहले चार
महीनों के मलए ₹𝟏𝟑, 𝟕𝟓𝟎 है , अगले तीन महीनों

𝒂 के मलए ₹𝟏𝟏, 𝟕𝟓𝟎 है और अंततम पांच महीनों के


41. The mean of three number numbers ,𝒃 and 𝒄
𝟑
𝒃 𝒄
मलए ₹𝟑𝟏, 𝟕𝟓𝟎 है ।यदद पररिार पूरे िषा के दौरान
is the same as the mean of the numbers 𝟑, 𝟑 and
₹𝟏𝟓, 𝟓𝟓𝟎 बचत करता है , तो िषा के दौरान पररिार
𝒙. The value of 𝟑𝒙 is ( where a, b, c are positive
numbers):
की औसत मामसक आय ज्ञात करे । (तनकटतम 47. In an examination, the ratio of the number of
candidates who passed, to those who failed
पूणाांक में) was 𝟓 : 𝟐. If the number of failed candidates
a) ₹𝟐𝟏, 𝟎𝟓𝟓 b) ₹𝟐𝟐, 𝟎𝟒𝟓 had been 14 more, then the ratio of the number
c) ₹𝟐𝟑, 𝟎𝟒𝟎 d) ₹𝟐𝟐, 𝟎𝟎𝟎 of passed candidates to those who failed would
have been 𝟒 : 𝟑. How many candidates passed
Ratio & Proportion the examination?
45. The ratio of two numbers is 𝟒 : 𝟓. If one is एक परीक्षा में पास हुए अभ्यधथायों की संख्या का,
subtracted from the first number, and two is
added to the second number, then the ratio फेल हुए अभ्यधथायों की संख्या से अनुपात 𝟓 : 𝟐
becomes 𝟑 : 𝟒. What will be the ratio when है | यदद फेल होने िाले अभ्यधथायों की संख्या 14
eight and four are, respectively, added to the
अधधक होती तो पास हुए अभ्यधथायों की संख्या
first and the second number?
का, फेल हुए अभ्यधथायों की संख्या से अनुपात
दो संख्याओं का अनप
ु ात 𝟒 : 𝟓 है। यदद पहली
𝟒 : 𝟑 हो िाता| परीक्षा में ककतने अभ्यथी पास
संख्या में से एक घटाया िाता है , और दस
ू री
हुए?
संख्या में दो िोड़ा िाता है , तो अनप
ु ात 𝟑 : 𝟒 हो
a) 126 b) 70 c) 100 d) 98
िाता है। यदद पहली और दस
ू री संख्या में क्रमशः
आठ और चार िोड़ ददया िाए, तो यह अनुपात 48. In an examination, the number of students who
क्या हो िाएगा? passed and the number of students who failed
were in the ratio 𝟐𝟓 : 𝟒. If one more student
a) 𝟐 : 𝟑 b) 𝟖 : 𝟗
had appeared and passed and the number of
c) 𝟔 : 𝟕 d) 𝟏 : 𝟓
failed students was 3 less than earlier, the ratio
of passed students to failed students would
have become 𝟐𝟐 : 𝟑. What is the difference
between the number of students who, initially,
passed the examination and the number of
46. On a tree, there are some parrots and some
students who failed the examination?
pigeons in the ratio of 𝟕 : 𝟗, respectively. After
an hour, 𝟖 parrots fly away, and 𝟔 pigeons and एक परीक्षा में , उत्तीणा और अनुत्तीणा विद्याधथायों
𝟏𝟎 sparrows come and sit on the tree. The ratio की संख्या का अनुपात 𝟐𝟓 : 𝟒 था। यदद एक और
of the parrots and the pigeons on the tree now
विद्याधथायों परीक्षा में भाग लेता और उत्तीणा होता,
is 𝟏 : 𝟒. What is the ratio of the parrots and the
sparrows that are now on the tree? साथ ही अनुत्तीणा विद्याधथायों की संख्या वपछली
एक पेड़ पर कुछ कबूतर क्रमश: 𝟕 : 𝟗 के अनुपात संख्या से 3 कम होती, तो उत्तीणा विद्याधथायों की
में बैठे हैं| एक घंटे बाद 𝟖 तोते उड़ िाते हैं और संख्या का, अनुत्तीणा विद्याधथायों की संख्या से
𝟔 कबूतर और 𝟏𝟎 गौरै यों आकर पेड़ पर बैठे िाते अनुपात 𝟐𝟐 : 𝟑 होता। आरं भ में, परीक्षा में उत्तीणा
हैं| अब पेड़ पर तोते और कबूतरों का अनुपात हुए और अनतु ीणा हुए विद्याधथायों की संख्या का
𝟏 : 𝟒 है| अब पेड़ पर तोते और गौरै यों का अनुपात अंतर ककतना है ?
ज्ञात कीजिए| a) 132 b) 126 c) 174 d) 150
a) 𝟏 : 𝟏 b) 𝟏 : 𝟐 c) 𝟑 : 𝟓 d) 𝟒 : 𝟓
49. 𝟒𝟎% of Ranita’s income is equal to 𝟓𝟔% of
Bhaskar’s Income. If Ranita’s Income was ₹ 𝟖𝟎𝟎
more than what it is and Bhaskar’s Income was its numerator and denominator, the fraction
₹ 𝟐, 𝟎𝟎𝟎 more than what it is , than the ratio of 𝟏
becomes 𝟖. Find the original fraction.
the incomes of Ranita and Bhaskar would have
been 𝟒: 𝟑. what is the actual combined income िब ककसी मभनन के अंश में 𝟖 िोड़ा िाता है और
उसके हर में 𝟏𝟐 िोड़ा िाता है , तो मभनन हो
𝟏
of Ranita and Bhaskar?
𝟐
रतनता की आय का 𝟒𝟎% , भास्कर की आय के िाता है | िब उसके अंश और हर में से 𝟐 घटाया
𝟓𝟔% के बराबर है| यदद रतनता की आय, उसकी िाता है , तो मभनन
𝟏
हो िाता है | मूल मभनन ज्ञात
𝟖
ितामान आय से ₹ 𝟖𝟎𝟎 अधधक होती और भास्कर करें |
की आय उसकी ितामान आय से ₹ 𝟐, 𝟎𝟎𝟎 अधधक 𝟗
a) 𝟏𝟗
𝟑
b) 𝟏𝟎
𝟒
c) 𝟕
𝟓
d) 𝟏𝟏
होती, तो रतनता और भास्कर की आय का अनप
ु ात
𝟒: 𝟑 होता| रतनता और भास्कर की िास्तविक 52. If 𝟏 is subtracted from the numerator and 𝟑 is
added to the denominator of a fraction, the
संयुक्त आय क्या है ? 𝟏
fraction becomes 𝟐. If 1 is added to the
a) ₹ 𝟔𝟖, 𝟐𝟎𝟎 b) ₹ 𝟔𝟕, 𝟖𝟎𝟎
c) ₹ 𝟔𝟕, 𝟐𝟎𝟎 d) ₹ 𝟔𝟔, 𝟖𝟎𝟎 numerator and 𝟏 is subtracted from the
denominator of the original fraction, it
𝟓
50. In 𝟐𝟎𝟐𝟎, the ratio of the incomes of A and B is becomes 𝟔. What is the sum of the numerator
𝟓: 𝟕. A and B save ₹𝟒, 𝟎𝟎𝟎 and ₹𝟓, 𝟎𝟎𝟎, and the and the denominator of the original fraction?
यदद ककसी मभनन के अंश में से 𝟏 घटाया िाए
𝟐
expenditure of A is equal to 𝟔𝟔 𝟑 % of the
और हर में 𝟑 िोड़ ददया िाए, तो प्राप्त मभनन
expenditure of B. If in 𝟐𝟎𝟐𝟏, the income of A 𝟏
𝟐
increases by 𝟐𝟕% and the income of B
decreases by 𝟏𝟏%, then the total income of A के बराबर होती है | यदद उस मूल मभनन के अंश
and B (in ₹) in 𝟐𝟎𝟐𝟏 is: में 𝟏 िोड़ा िाए और हर में 𝟏 घटा ददया िाए,
𝟐𝟎𝟐𝟎 में, A और B की आय का अनुपात 𝟓: 𝟕 तो प्राप्त मभनन
𝟓
के बराबर होती है उस मल

𝟔
था| A और B क्रमश: ₹𝟒, 𝟎𝟎𝟎 और ₹𝟓, 𝟎𝟎𝟎 की मभनन के अंश और हर का योगफल ककतना है ?
बचत करते हैं, तथा A का व्यय, B के 𝟔𝟔 𝟑 % के
𝟐
a) 𝟐𝟑 b) 𝟐𝟐 c) 𝟐𝟏 d) 𝟏𝟗
बराबर है | यदद 𝟐𝟎𝟐𝟏 में , A की आय में 𝟐𝟕%
53. Lest year, the ratio of the incomes of A and B
की िद्
ृ धध होती है और B की आय में 𝟏𝟏% की was 𝟒 : 𝟑. The ratios of their individual incomes
कमी होती है , तो 𝟐𝟎𝟐𝟏 में A और B की कुल आय of last year and present year are 𝟑 : 𝟒 and 𝟓 : 𝟔
respectively. If their total income for the
(₹ में) ककतनी है ?
present year is ₹𝟏𝟔. 𝟎𝟖 lakhs, then the income
a) 𝟐𝟏, 𝟓𝟔𝟎 b) 𝟐𝟔, 𝟏𝟓𝟎 of A (in ₹ lakhs) for the present year is:
c) 𝟐𝟓, 𝟏𝟔𝟎 d) 𝟐𝟏, 𝟔𝟓𝟎
वपछले िषा, A और B की आय का अनुपात 𝟒 : 𝟑
था| वपछले िषा और ितामान िषा के दौरान उनकी
व्यजक्तगत आय के अनुपात क्रमश: 𝟑 : 𝟒 और
51. When 𝟖 is added to the numerator of a fraction 𝟓 : 𝟔 है | यदद ितामान िषा में उनकी कुल आय
and 𝟏𝟐 is added to its denominator, the ₹𝟏𝟔. 𝟎𝟖 लाख है , तो ितामान िषा में A की आय
𝟏
fraction becomes 𝟐 . When 𝟐 is subtracted from ₹ _______ लाख है |
a) 𝟗. 𝟔 b) 𝟖. 𝟒 c) 𝟕. 𝟐 d) 𝟔. 𝟔
ratio of the quantities imported of iron in 𝟐𝟎𝟏𝟖
54. The ratio of the incomes of A and B in 2020 was and 𝟐𝟎𝟐𝟎?
𝟓 : 𝟒. The ratios of their individual incomes in 2018 में आयत ककए गए स्टील और लोहे की
2020 and 2021 were 𝟒 : 𝟓 and 𝟐 : 𝟑,
respectively. If the total income of A and B in मात्राओं का अनप
ु ात 𝟑: 𝟐 था और 𝟐𝟎𝟐𝟎 में आयत
2021 was ₹7,05,600, then what was the income ककए गए स्टील और लोहे की मात्राओं का अनुपात
(in ₹) of B in 2021? 1:1 था| 𝟐𝟎𝟏𝟖 और 𝟐𝟎𝟐𝟎 में स्टील की आयत
िषा 2020 में, A और B की आय का अनुपात
की गई मात्राओं का अनुपात 𝟒: 𝟑 था| 𝟐𝟎𝟏𝟖 और
𝟓 : 𝟒 था। िषा 2020 और 2021 में, उनकी
𝟐𝟎𝟐𝟎 में लोहे की आयत की गई मात्राओं का
व्यजक्तगत आय के अनुपात क्रमशः 𝟒 : 𝟓 और
अनुपात ज्ञात कीजिए|
𝟐 : 𝟑 थे। यदद 2021 में A और B की कुल आय a) 𝟏𝟏: 𝟏𝟐 b)𝟕: 𝟓 c) 𝟖: 𝟗 d)𝟕: 𝟗
₹7,05,600 थी, तो 2021 में B की आय (₹ में)
57. A sum of ₹𝟏𝟐𝟓𝟎 has to distributed among
ककतनी थी?
𝑨, 𝑩, 𝑪 and 𝑫. Total share of 𝑩 and 𝑫 is equal
a) 𝟑, 𝟒𝟓, 𝟔𝟎𝟎 b) 𝟐, 𝟕𝟗, 𝟕𝟎𝟎 to (𝟏𝟒/𝟏𝟏) of total share of 𝑨 and 𝑪. Share of
c) 𝟑, 𝟔𝟎, 𝟎𝟎𝟎 d) 𝟒, 𝟐𝟓, 𝟗𝟎𝟎 𝑫 is half of share of 𝑨. Share of 𝑪 is 𝟏. 𝟐 of share
of 𝑨. What are the shares of 𝑨, 𝑩, 𝑪 and 𝑫
55. The ratio of the incomes of A and B in the last respectively?
year was 𝟒 : 𝟑. The ratios of their individual
incomes in the last year and the present year ₹𝟏𝟐𝟓𝟎 को 𝑨, 𝑩, 𝑪 तथा 𝑫 में विभाजित ककया
are 𝟑 : 𝟒 and 𝟓 : 𝟔, respectively. If their total िाना है | 𝑩 तथा 𝑫 का कुल दहस्सा, 𝑨 तथा 𝑪
income in the present year is ₹24.12 lakhs, then की कुल दहस्से का (𝟏𝟒/𝟏𝟏) है | 𝑫 का दहस्सा, 𝑨
the sum of the income (in ₹ lakh) of A in the last
year and that of B in the present year is: के दहस्से का आधा है | 𝑪 का दहस्सा, 𝑨 के दहस्से
वपछले िषा में A और B की आय का अनुपात का 𝟏. 𝟐 गुना है| 𝑨, 𝑩, 𝑪 तथा 𝑫 के दहस्से
𝟒 : 𝟑 था। वपछले िषा और ितामान िषा में उनकी क्रमशः ककतने ककतने है ?
व्यजक्तगत आय का अनुपात क्रमशः 𝟑 : 𝟒 और a) ₹𝟐𝟓𝟎, ₹𝟓𝟕𝟓, ₹𝟑𝟎𝟎, ₹𝟏𝟐𝟓
b) ₹𝟑𝟓𝟎, ₹𝟓𝟐𝟓, ₹𝟑𝟎𝟎, ₹𝟏𝟐𝟓
𝟓 : 𝟔 है। यदद ितामान िषा में उनकी कुल आय c) ₹𝟐𝟓𝟎, ₹𝟓𝟐𝟓, ₹𝟑𝟎𝟎, ₹𝟏𝟐𝟓
₹24.12 लाख है, तो वपछले िषा में A की आय और d) ₹𝟐𝟓𝟎, ₹𝟓𝟕𝟓, ₹𝟑𝟎𝟎, ₹𝟏𝟕𝟓
ितामान िषा में B की आय (₹ लाख में) का योग
58. A sum of ₹46,800 is divided among A, B, C and
ज्ञात करें | D in such a way that the ratio of the combined
a) 𝟐𝟏. 𝟐𝟖 b) 𝟐𝟐. 𝟏𝟕 c) 𝟐𝟎. 𝟓𝟐 d) 𝟏𝟎. 𝟗𝟖 share of A and D to the combined share of B and
C is 𝟖 : 𝟓. The ratio of the share of B to that of C
is 𝟓 : 𝟒. A receives ₹18,400. If 𝒙 is the difference
between the shares of A and B and 𝒚 is the
difference between the shares of C and D, then
56. The ratio of the quantities imported of steel what is the value of (𝒙 − 𝒚) (𝒊𝒏 ₹) ?
and iron in 2018 was 𝟑: 𝟐 and in 𝟐𝟎𝟐𝟎 it was ₹46,800 की रामश को A, B, C और D में इस प्रकार
𝟏: 𝟏. The ratio of the quantities imported of
steel in 2018 and 2020 was 𝟒: 𝟑. What was विभाजित ककया िाता है कक A और D के संयुक्त
दहस्से का B और C के संयुक्त दहस्से से अनप
ु ात
𝟖 : 𝟓 है। B के दहस्से का, C के दहस्से से अनुपात B के व्यय के 𝟗𝟎% के बराबर है , तो A की बचत,
𝟓 : 𝟒 है। A को ₹18,400 ममलते हैं। यदद A और B B की बचत से ककतने प्रततशत अधधक है ?
के दहस्सों के बीच का अंतर 𝒙 है और C और D a) 𝟏𝟓𝟎% b) 𝟏𝟒𝟎% c) 𝟏𝟐𝟓% d) 𝟏𝟎𝟎%
के दहस्सों के बीच का अंतर 𝒚 है, तो (𝒙 − 𝒚) (में 62. If 𝒙 is subtracted from each of 24, 40, 33 and
₹) ककतना है ? 57, the numbers, so obtained are in proportion.
a) 6500 b) 6000 c) 5000 d) 7000 The ratio of (𝟓𝒙 + 𝟏𝟐) to (𝟒𝒙 + 𝟏𝟓) is:
24, 40, 33 और 57 में से प्रत्येक से 𝒙 घटाने पर,
59. The selling price of an article is four times of its
प्राप्त संख्याएं समानप
ु ात में है | (𝟓𝒙 + 𝟏𝟐) और
cost price. If the selling price is reduced by
₹𝟑𝟎𝟎, then the profit becomes 𝟐𝟓𝟎%. What is (𝟒𝒙 + 𝟏𝟓) का अनप ु ात ज्ञात करें |
the cost price of the article? a) 𝟏𝟒 : 𝟏𝟑 b) 𝟒 : 𝟑 c) 𝟕 : 𝟓 d) 𝟕 : 𝟒
एक िस्तु का विक्रय मूल्य उसके क्रय मूल्य का
63. When 𝒙 subtracted from each of the numbers
चार गुना है | यदद विक्रय मूल्य में ₹𝟑𝟎𝟎 की कमी 54, 49, 22 and 21, the numbers so obtained are
की िाती है , तो लाभ 𝟐𝟓𝟎% हो िाता है | उस in proportion. The ratio of (𝟖𝒙 − 𝟐𝟓) to
िस्तु का क्रय मूल्य ककतना है ? (𝟕𝒙 − 𝟐𝟔) is:
a) ₹𝟒𝟎𝟎 b) ₹𝟒𝟓𝟎 संख्याओं 54, 49, 22 और 21 में से प्रत्येक से 𝒙
c) ₹𝟖𝟎𝟎 d) ₹𝟔𝟎𝟎 को घटाने पर प्राप्त संख्याएं समानप
ु ात में हैं|
(𝟖𝒙 − 𝟐𝟓) और (𝟕𝒙 − 𝟐𝟔) का अनुपात ज्ञात
60. The ratio of the incomes of A and B is 𝟏 : 𝟐 and
that of their expenditure is 𝟐 : 𝟑. If 𝟖𝟎% of B’s करें |
expenditure is equal to the income of A, then a) 𝟓 : 𝟒 b) 𝟏𝟓 : 𝟏𝟑 c) 𝟐𝟗 : 𝟐𝟒 d) 𝟐𝟕 : 𝟐𝟔
what is the ratio of the savings of B to the
savings of A? 64. Seven years ago, the ratio of the ages of A and
A और B की आय का अनुपात 𝟏 : 𝟐 और उनके B was 𝟒 : 𝟓. Eight years hence, the ratio of the
ages of A and B will be 𝟗 : 𝟏𝟎. What is the sum
व्यय का अनुपात 𝟐 : 𝟑 है| यदद B के व्यय का of their present ages in years?
𝟖𝟎% A की आय के बराबर है , तो B और A की सात िषा पहले, A और B की आयु का अनुपात
बचतों का अनुपात क्या होगा? 𝟒: 𝟓 था| आठ िषा बाद, A और B की आयु का
a) 𝟐 : 𝟗 b) 𝟗 : 𝟐
अनुपात 𝟗: 𝟏𝟎 होगा| उनकी ितामान आयु का
c) 𝟕 : 𝟑 d) 𝟑 : 𝟕
योगफल (िषा में ) ज्ञात करें |
a) 32 b) 82 c) 41 d) 56

65. The ratio of the ages of A and B, four years ago,


61. The income of A is 𝟖𝟎% of B’s income and the was 𝟒 : 𝟓. Eight years hence, the ratio of the
expenditure of A is 𝟔𝟎% of B’s expenditure. If ages of A and B will be 𝟏𝟏 : 𝟏𝟑. What is the ratio
the income of A is equal to 𝟗𝟎% of B’s of their present ages?
expenditure, then by what percentage are the चार िषा पहले, A और B की आयु का अनुपात
savings of A more than B’s savings?
𝟒 : 𝟓 था। आठ िषा बाद, A और B की आयु का
A की आय, B की आय का 𝟖𝟎% है और A का
व्यय, B के व्यय का 𝟔𝟎% है। यदद A की आय,
अनुपात 𝟏𝟏 : 𝟏𝟑 होगा। उनकी ितामान आयु का and 𝟑 times the denominator of the original
fraction is:
अनुपात ज्ञात करें |
यदद ककसी मभनन के अंश और हर में से प्रत्येक
a) 𝟗 : 𝟏𝟏 b) 𝟏𝟏 : 𝟗
में एक घटा ददया िाए, तो प्राप्त मभनन के
𝟏
c) 𝟖 : 𝟕 d) 𝟕 : 𝟖 𝟒
बराबर होती है | यदद अंश और हर में से प्रत्येक
में 𝟓 िोड़ ददया िाए, तो प्राप्त मभनन के बराबर
𝟏
𝟐
होती है | मूल मभनन के अंश के 𝟏𝟏 गुने और हर
66. The number of students in three sections of
के 𝟑 गुने का अंतर ज्ञात कीजिए|
Grade 10 in a school are in the proportion
𝟑 : 𝟓 : 𝟖. If 15, 30 and 15 more students are a) 𝟒 b) 𝟓 c) 𝟕 d) 𝟔
admitted in the three section, respectively, the
new proportion becomes 𝟒 : 𝟕 : 𝟗. The total 69. Three fractions 𝒙, 𝒚 and 𝒛 are such that 𝒙 >
number of students before the new admission 𝒚 > 𝒛. When the smallest of them is divided by
𝟗
is: the greatest, the result is 𝟏𝟔, which exceeds y by
ककसी विद्यालय में कक्षा 10 के तीन सेक्शन में 𝟑
0.0625. If 𝒙 + 𝒚 + 𝒛 = 𝟐 𝟏𝟐, then what is the
छात्रों की संख्या का अनप
ु ात 𝟑 : 𝟓 : 𝟖 है | यदद value of 𝒙 + 𝒛?
तीनों सेक्शन में क्रमश: 15, 30 और 15 अधधक 𝒙, 𝒚और 𝒛 तीन ऐसे हैं कक 𝒙 > 𝒚 > 𝒛 है | िब
छात्र दाखखल होते है , तो नया अनुपात 𝟒 : 𝟕 : 𝟗 हो उनमें से सबसे छोटे को सबसे बड़े से विभाजित
ककया िाता है , तो पररणाम प्राप्त होता है , िो
𝟗
िाता है | नए दाखखले से पहले छात्रों की कुल 𝟏𝟔

संख्या ज्ञात करें | कक y से 0.0625 अधधक है। यदद 𝒙 + 𝒚 + 𝒛 =


𝟐 𝟏𝟐 है, तो 𝒙 + 𝒛 का मान ज्ञात करें |
𝟑
a) 320 b) 160 c) 240 d) 400
𝟕 𝟏 𝟓 𝟑
a) 𝟒 b) 𝟒 c) 𝟒 d) 𝟒
67. A and B currently have an age ratio of 𝟕 : 𝟑
After 𝟕 years this age ratio will become 𝟐 : 𝟏.
After how many years from now will the age 70. Five years from now, the ratio of the ages of A,
ratio between the ages of A and B become B and C will be 𝟑 : 𝟓 : 𝟐 . The sum of the squares
𝟑 : 𝟐? of their present ages is 𝟓𝟐𝟓. What is the
present age of B?
ितामान में A और B की आयु का अनुपात 𝟕 : 𝟑
अब से पांच िषा बाद, A, B और C की आयु का
है| 𝟕 िषा बाद उनकी आयु का अनुपात 𝟐 : 𝟏 हो
अनुपात 𝟑 : 𝟓 : 𝟐 होगा| उनकी ितामान आयु के
िाएगा| अब से ककतने िषा बाद A और B की
िगी का योग 𝟓𝟐𝟓 है| तो B की ितामान आयु
आयु का अनुपात 𝟑 : 𝟐 होगा?
क्या होगी?
a) 𝟒𝟗 years b) 𝟒𝟐 years
c) 𝟑𝟓 years d) 𝟐𝟖 years a) 𝟏𝟎 years b) 𝟏𝟓 years
c) 𝟐𝟓 years d) 𝟐𝟎 years
68. If one is subtracted form each of the numerator
𝟏
and denominator of a fraction, it becomes 𝟒. If
𝟓 is added to each of the numerator and
𝟏
denominator, the fraction becomes 𝟐. The 71. A person has same coins of ₹𝟏𝟎, ₹𝟓, and ₹𝟐
difference between 𝟏𝟏 times the numerator denominations. The ratio of the products of the
numbers of ₹𝟏𝟎 and ₹𝟓 coins, the numbers of 75. If 𝑲𝟐 : 𝑴 ∷ 𝑴: 𝟏𝟔, then what is the value of
₹𝟓 and ₹𝟐 coins, and the numbers of ₹ 𝟐 and (𝒌𝟐 + 𝟐𝑴𝟐 ): (𝑲𝟐 + 𝑴𝟐 )?
₹ 𝟏𝟎 coins is 𝟑 : 𝟒 : 𝟐 respectively. What could यदद 𝑲𝟐 : 𝑴 ∷ 𝑴: 𝟏𝟔 हो, तो (𝒌𝟐 + 𝟐𝑴𝟐 ): (𝑲𝟐 +
be the minimum amount of money this person
has? 𝑴𝟐 ) का मान ककतना है ?
a) 𝟏 : 𝟏 b) 𝟑𝟑 : 𝟏𝟕
एक व्यजक्त के पास of ₹𝟏𝟎, ₹𝟓 और ₹𝟐 मल्
ू यिगा
c) 𝟏𝟕 : 𝟏𝟔 d) 𝟑𝟐 : 𝟏𝟕
के कुल मसक्के हैं| ₹𝟏𝟎 और ₹𝟓 के मसक्कों की
संख्या के गण
ु नफल, ₹𝟓 और ₹𝟐 के मसक्कों की Answer Key
संख्या के गण
ु नफल और ₹ 𝟐 और ₹ 𝟏𝟎 के 1. C 2. D 3. B 4. A 5. A
6. D 7. A 8. D 9. A 10. D
मसक्कों की संख्या के गण
ु नफल का अनप
ु ात 11. B 12. B 13. A 14. C 15. B
क्रमश: 𝟑 : 𝟒 : 𝟐 है| उस व्यजक्त के पास नयन
ू तम 16. A 17. A 18. B 19. B 20. B
ककतनी धनरामश हो सकती है ? 21. A 22. B 23. D 24. C 25. C
26. C 27. B 28. D 29. C 30. D
a) ₹𝟓𝟐 b) ₹𝟖𝟖
31. B 32. C 33. D 34. A 35. D
c) ₹𝟔𝟖 d) 𝟕𝟒
36. C 37. A 38. A 39. B 40. D
72. The value of a stone is proportional to the 41. B 42. A 43. B 44. B 45. B
square of its weight. A stone worth ₹𝟏, 𝟐𝟎, 𝟎𝟎𝟎 46. C 47. B 48. B 49. C 50. C
is broken into two pieces in the ratio of 𝟐 : 𝟑 51. B 52. B 53. A 54. A 55. C
what is the total price of the two small stones? 56. C 57. A 58. B 59. D 60. B
61. B 62. A 63. C 64. C 65. A
ककसी पत्थर का मान उसके भार के िगा के
66. C 67. C 68. B 69. A 70. D
समानुपाती होता है | ₹𝟏, 𝟐𝟎, 𝟎𝟎𝟎 मूल्य के एक 71. C 72. B 73. C 74. B 75. B
पत्थर को 𝟐 : 𝟑 अनप
ु ात में दो टुकड़ों में तोड़ा
िाता है | दो छोटे पत्थरों का कुल मल्
ू य ककतना
है?
a) ₹𝟔𝟖, 𝟒𝟎𝟎 b) ₹𝟔𝟐, 𝟒𝟎𝟎
c) ₹𝟔𝟔, 𝟓𝟎𝟎 d) ₹𝟔𝟓, 𝟒𝟎𝟎

73. If 𝒂 : 𝒃 = 𝒃 : 𝒄, then what is the value of 𝒂𝟔 : 𝒃𝟔 ?


यदद 𝒂 : 𝒃 = 𝒃 : 𝒄 है, तो 𝒂𝟔 : 𝒃𝟔 का मान क्या
होगा?
a) 𝒂𝟑 : 𝒃𝟑 b) 𝒃𝟑 : 𝒂𝒄
c) 𝒂𝟑 : 𝒄𝟑 d) 𝒂𝟐 : 𝒃𝟐

74. If 𝟑𝒂 : 𝟐𝒃 = (𝟔𝒂 − 𝟑): (𝟑𝒃 − 𝟐) and 𝒂 = 𝒃 −


𝟏, then what will be the value of 𝟏𝟐ab?
यदद 𝟑𝒂 : 𝟐𝒃 = (𝟔𝒂 − 𝟑): (𝟑𝒃 − 𝟐) और 𝒂 =
𝒃 − 𝟏,
है, तो 𝟏𝟐ab का मान ज्ञात कीजिए|
a) 𝟐𝟖 b) 𝟐𝟒 c) 𝟑𝟎 d) 32
Percentage and Mixture & Alligation Latest pattern Difficult
Questions From latest SSC CGL, CHSL, MTS, CPO, NTPC 2020 Exams

FOR VIDEO SOLUTIONS he maintained the percentage level of E, but T


FROM QUESTIONS 1 TO 10 CLICK HERE becomes 𝟑𝟎% of E and C becomes 𝟐𝟎% of T.
OR SCAN QR CODE GIVEN BELOW The difference between the two savings (in ₹)
is:
ककि व्यक्ति का मासिक वेिन ₹50,000 है| क्जिे
वह पाररवाररक खर्च (E), कर (T), दान (C) में व्यय
करिा है और िेष रािी बर्ािा है | E और का
1. The monthly salary of a person was ₹1,60,000. 60%, T, E का 20% और C, T का 15% है | जब
He used to spend on three heads-Personal and उिकी आय में 40 की वद् ृ धध हुई, उिने E का
family expenses (P). Taxes (T) and Education
प्रतििि पुवरच ि रहने ददए लेकीन T, E का 30%
loan (E). The rest were his savings. P was 𝟓𝟎%
of the income. E was 𝟐𝟎% of P. and T was 𝟏𝟓% और C,T का 20% कर ददया| दोनों बर्ि के बीर्
of E. When his salary got raised by 𝟑𝟎%, he अंिर (₹ में) ज्ञाि करें |
maintained the percentage level of P. but E
a) 128 b) 220 c) 130 d) 250
became 𝟑𝟎% of P and T became 𝟐𝟎% of E. The
sum of the two savings (in ₹) is:
3. The monthly salary of a person was ₹75,000. He
एक व्यक्ति का मासिक वेिन ₹1,60,000 था। वह used to spend on Family Expenses (E), Taxes (T),
िीन मदों-व्यक्तिगि और पाररवाररक खर्च (P), Charity (C) and rest were his savings. E was
𝟔𝟎% of the income, T was 𝟐𝟎% of E, and C was
कर (T) और सिक्षा ऋण (E) पर खर्च करिा है|
𝟏𝟓% of T. When his salary got raised by 𝟒𝟎%,
िेष धन उिकी बर्ि है । P आय का 𝟓𝟎% था he maintained the percentage level of E, but T
िथा E, P का 𝟐𝟎% था और T, E का 𝟏𝟓% था| become 𝟑𝟎% of E and C become 𝟐𝟎% of T. The
ratio of the savings of his earlier salary to that
जब उिके वेिन में 𝟑𝟎% की वद् ृ धध हुई िो उिने of his present salary is:
P का प्रतििि स्िर पहले क्जिना ही रखा, लेककन एक व्यक्ति का मासिक वेिन ₹75,000 था, क्जिे
E, P का 𝟑𝟎% हो गया और T, E का 𝟐𝟎% हो वह पाररवाररक खर्ो (E), करो (T), दान (C) पर
गया और T, E का 𝟐𝟎% हो गया| दोनों बर्िों व्यय ककया करिा था और िेष रािी बर्ािा था|
का योग (₹ में) ककिना है ? E आय का 60% था, T, E का 20% था और C, T
a) 𝟐, 𝟏𝟏, 𝟔𝟖𝟎 b) 𝟏, 𝟐𝟖, 𝟏𝟔𝟎
का 15% था| जब उिके वेिन में 40% की
c) 𝟏, 𝟏𝟖, 𝟔𝟐𝟎 d) 𝟏, 𝟔𝟐, 𝟖𝟏𝟎
वद्
ृ धध हुई, िो उिने E का प्रतििि स्िर िो बनाए
2. The monthly salary of a person was ₹50,000. He रखा, लेककन T, E का 30% और C, T का 20% हो
used to spend on Family expenses (E), Taxes (T),
गया| उिके पूवच वेिन की बर्ि और उिके
Charity (C), and the rest were his savings. E was
𝟔𝟎% of the income. T was 𝟐𝟎% of E, and C was विचमान वेिन की बर्ि का अनप ु ाि ज्ञाि करें |
𝟏𝟓% of T. When his salary got raised by 𝟒𝟎%, a) 𝟔𝟓𝟓 : 𝟔𝟒𝟒 b) 𝟑𝟐𝟓 : 𝟑𝟑𝟕
c) 𝟔𝟒𝟒 : 𝟔𝟓𝟓 d) 𝟑𝟑𝟕 : 𝟑𝟐𝟓 7. Savita spends 𝟐𝟎% of her monthly income on
groceries, 𝟏𝟓% of the remaining on rent and
4. The difference between a positive number and then 𝟔𝟎% of the left over on children's
𝟏𝟕𝟓 education and others. If she saves ₹ 9,792 a
its reciprocal increases by a factor of 𝟏𝟒𝟒 when
month, then how much (in ₹ ) does she spend
the number is made to increase by 𝟐𝟎%. What
on rent?
is the number?
िवविा, अपनी मासिक आय का 𝟐𝟎% ककराने के
ककिी धनात्मक िंख्या और उिके व्युत्रम का
िामान पर, िेष आय का 𝟏𝟓% ककराये पर और
अंिर के गुणाक में बढ़ जािा है , यदद उि
𝟏𝟕𝟓

बाकी बर्े हुए का 𝟔𝟎% बच्र्ो की सिक्षा और


𝟏𝟒𝟒
िंख्या में 𝟐𝟎% की वद्
ृ धध की जािी है | वह िंख्या
अन्य पर खर्च करिी है| यदद वह महीने में ₹
ज्ञाि कीक्जए|
a) 𝟕. 𝟓 b) 𝟔 c) 𝟐. 𝟓 d) 𝟓 9,792 बर्ािी है , वह ककराये पर, ककिना (₹ में )
खर्च करिी है ?
5. An employee gets two successive increments in a) 𝟑, 𝟗𝟔𝟎 b) 𝟒, 𝟒𝟓𝟎
his salary, as a result of which his salary c) 𝟒, 𝟐𝟎𝟎 d) 𝟒, 𝟑𝟐𝟎
becomes 𝟐. 𝟓 times the initial salary. What was
his first percentage increment, if the second 8. In a class, 𝟔𝟎% of the students are girls and the
percentage increment was 4 times as high as rest are boys. 𝟒𝟓% of the girls pass an
the first? examination and 𝟒𝟎% of the boys failed. If the
ककिी कमचर्ारी को उिके वेिन में दो रमागि number of girls who failed is 66, the number of
वेिन-वद्
ृ धध (increments) समलिी है , क्जिके
boys who passed the examination is:
ककिी कक्षा में , 𝟔𝟎% छात्र लडककयााँ हैं और िेष
पररणामस्वरूप उिका वेिन प्रारं सिक वेिन का
लडकें हैं| ककिी परीक्षा में 𝟒𝟓% लडककयााँ उत्तीणच
𝟐. 𝟓 गुना हो जािा है| उिकी पहली प्रतििि वेिन-
होिी हैं और 𝟒𝟎% लडकें अनुत्तीणच होिे हैं| यदद
वद्
ृ धध (increments) ज्ञाि करें , यदद दि
ू री प्रतििि
अनुत्तीणच होने वाली लडककयााँ की िंख्या 66 हैं, िो
वेिन-वद्
ृ धध पहले की िल
ु ना में 4 गुना अधधक
परीक्षा में उत्तीणच होने वाले लड़कों की िंख्या ज्ञाि
है ?
a) 𝟏𝟎% b) 𝟐𝟓% c) 𝟓𝟎% d) 𝟏𝟐% करें |
a) 36 b) 48 c) 54 d) 60
6. By what percentage is the percentage of the
number of vowels in the word CONSONANT 9. In a constituency, 𝟓𝟓% of the total number of
less than the percentage of consonants in it? voters are males and the rest are females. If
CONSONANT िब्द में स्वरों की िंख्या का 𝟒𝟎% of the males are illiterate and 𝟒𝟎% of the
females are literate, then by what percentage
प्रतििि, इिमें व्यंजनों की िंख्या के प्रतििि िे is the number of illiterate females more than
ककिने प्रतििि कम है ? that of the illiterate males (correct to one
a) 𝟑𝟑. 𝟑𝟑 b) 𝟐𝟎𝟎 decimal place)?
c) 𝟔𝟔. 𝟔𝟕 d) 𝟓𝟎 एक तनवाचधर्ि क्षेत्र में कुल मिदािाओं में िे 𝟓𝟓%
पुरुष और िेष मदहलाएं हैं| यदद 𝟒𝟎% पुरुष
असिक्षक्षि िथा 𝟒𝟎% मदहलाएं सिक्षक्षि हैं िो 12. Due to some crisis. Some people from state 𝑨
migrated to state 𝑩, and thereby the
असिक्षक्षि मदहलाओं की िंख्या असिक्षक्षि पुरुषों population of the second state increased by
की िंख्या िे ककिने प्रतििि (दिमलव के एक 𝟏𝟗. 𝟗𝟔%. However, at a later stage, all of them
स्थान िक िही) अधधक है returned to state 𝑨. Then by what percentage
(correct up to two decimal places) did the
a) 𝟐𝟎. 𝟖% b) 𝟏𝟔. 𝟒%
population of state 𝑩 decrease?
c) 𝟐𝟐. 𝟕% d) 𝟏𝟏. 𝟓
कुछ िंकट के कारण। राज्य 𝑨 िे कुछ लोग राज्य
10. If decreasing 180 by 𝒙% gives the same result B में र्ले गए, और इि िरह दि
ू रे राज्य की
as increasing 60 by 𝒙%, then 𝒙% of 410 will be
जनिंख्या में 𝟏𝟗. 𝟗𝟔% की वद्
ृ धध हुई। हालांकक,
more than (𝒙 + 𝟐𝟎)% of 210 (correct to two
decimal places): बाद में वे ििी लोग राज्य A में वापि लौट
यदद 180 को 𝒙% की कमी करने पर और 60 में आए। िो राज्य 𝑩 की जनिंख्या में ककिने
𝒙% की वद्
ृ धध करने पर िमान पररणाम प्राप्ि प्रतििि की कमी हुई (दिमलव के बाद दो स्थानों
होिा है , िो 410 का 𝒙%, 210 के (𝒙 + 𝟐𝟎)% िे िक गड़ना करे )?
ककिना अधधक होगा (दिमलव के दो स्थान िक a) 𝟏𝟗. 𝟗𝟔 b) 𝟏𝟔. 𝟔𝟒
c) 𝟏𝟖. 𝟐𝟒 d) 𝟏𝟕. 𝟓𝟒
िही)?
a) 𝟑𝟔. 𝟓𝟕% b) 𝟑𝟕. 𝟓𝟕% 13. The present population of a village (males and
c) 𝟑𝟏. 𝟔𝟕% d) 𝟑𝟗. 𝟒𝟔% females) is 7640. If the number of males
increases by 𝟐𝟓% and the number of females
FOR VIDEO SOLUTIONS increases by 𝟏𝟓%, then the population will
FROM QUESTIONS 11 TO 20 CLICK HERE become 9214. The difference between the
OR SCAN QR CODE GIVEN BELOW present population of males and females in the
village is:
ककिी गााँव की विचमान जनिंख्या (पुरुषों और
मदहलाओं) 7640 है| यदद पुरुषों की िंख्या में
𝟐𝟓% की वद्
ृ धध होिी है और मदहलाओं की िंख्या
11. If decreasing 110 by 𝒙% gives the same result में 𝟏𝟓% की वद्
ृ धध होिी है , िो जनिंख्या 9214
as increasing 50 by 𝒙%, then 𝒙% of 650 is what
हो जाएगी| गााँव में पुरुषों और मदहलाओं की
percentage more than (𝒙 + 𝟐𝟎)% of 180?
(correct nearest integer) विचमान जनिंख्या के बीर् का अंिर ज्ञाि करें |
यदद 𝟏𝟏𝟎 को 𝑿% कम करने पर प्राप्ि पररणाम, a) 820 b) 960 c) 940 d) 920
𝟓𝟎 को 𝑿% बढ़ाने पर प्राप्ि पररणाम के िमान
14. Ramesh spends 𝟒𝟎% of his monthly salary on
है , िो 𝟔𝟓𝟎 का 𝑿% 𝟏𝟖𝟎 के (𝒙 + 𝟐𝟎)% िे food, 𝟏𝟖% on house rent, 𝟏𝟐% on
ककिने प्रतििि अधधक होगा ? entertainment, and 𝟓% on conveyance. But
due to a family function, he has to borrow
(तनकटिम पूणाचक िक िुद्ध) ₹16,000 from a money lender to meet the
a) 𝟏𝟑𝟔% b) 𝟗𝟎% c) 𝟏𝟓𝟒% d) 𝟖𝟎% expenses of ₹20,000. His monthly salary is:
रमेि अपनी मासिक आय का 𝟒𝟎% खाद्य पर,
𝟕
17. A number is mistakenly multiplied by 𝟓 instead
𝟏𝟖% घर के ककराए पर, 𝟏𝟐% मनोरं जन पर और
𝟑
𝟓% वाहन पर खर्च करिा है। लेककन एक of being multiplied by 𝟐. What is the percentage
change in the result due to this mistake?
पाररवाररक िमारोह के कारण, उिे ₹20,000 के
ककिी िंख्या को िे गुना करने के बजाय गलिी
𝟑

खर्च को पूरा करने के सलए ककिी ऋणदािा िे 𝟐


िे िे गण
ु ा ककया जािा है | इि गलिी के कारण
𝟕
₹16,000 उधार लेना पड़िा हैं। उिका मासिक 𝟓
पररणाम में प्रतििि पररविचन ज्ञाि करें |
वेिन ज्ञाि करें | 𝟐 𝟐 𝟐 𝟐
a) ₹16,500 b) ₹15,000 a) 𝟔 𝟑 % b) 𝟕 𝟑 % c) 𝟑 𝟑 % d) 𝟓 𝟑 %
c) ₹16,000 d) ₹18,000
18. By mistake, the reciprocal of a positive fraction
15. The price of an article increases by 𝟐𝟎% every got typed in place of itself, and thereby its value
𝟏𝟕𝟓
year. If the difference between the price of the got reduced by 𝟒 %. What was the value of
article at the end of the third year and at the the fraction?
end of the fourth year is ₹324, what is the price
गलिी िे, ककिी धनात्मक सिन्न के स्थान पर
of the article at the end of the first year?
प्रत्येक वषच ककिी वस्िु के मूल्य में 𝟐𝟎% की इिका व्युत्रम टाइप हो जािा है और इि िरह
इिके मान में % की कमी होिी है। सिन्न
𝟏𝟕𝟓
वद्
ृ धध होिी है | यदद िीिरे और र्ौथे वषच के अंि 𝟒

में , वस्िु के मूल्य के बीर् अंिर ₹324 है, िो पहले का मान ज्ञाि करें |
𝟏 𝟏 𝟒 𝟑
वषच के अंि में उि वस्िु मूल्य ज्ञाि करें | a) 𝟒 b) 𝟐 c) 𝟑 d) 𝟒
a) ₹1,350 b) ₹1,180
c) ₹1,125 d) ₹1,250 19. The income of A is 𝟑𝟎% less than the income of
B and the income of B is 𝟏𝟑𝟕. 𝟓% more than
𝟐 that of C. If the income of A is ₹28500 less than
16. The monthly expenses of a person are 𝟔𝟔 𝟑 %
that of B, then the income (in ₹) of C is:
more than her monthly savings. If her monthly
A की आय, B की आय िे 𝟑𝟎% कम है और B
income increases by 𝟒𝟒% and her monthly
expenses increase by 𝟔𝟎%, then there is an की आय C की आय िे 𝟏𝟑𝟕. 𝟓% अधधक है | यदद
increase of ₹1,040 in her monthly savings. A की आय B की आय िे ₹28500 कम है , िो C
What is the initial expenditure ( in ₹) ?
की आय (₹ में) ज्ञाि करें |
एक व्यक्ति का मासिक व्यय उिकी मासिक
a) 36000 b) 48000
बर्ि िे 𝟔𝟔 % अधधक है। यदद उिकी मासिक
𝟐
𝟑 c) 40000 d) 50000
आय में 𝟒𝟒% की वद्
ृ धध होिी है और उिके
20. A is 𝟐𝟓% less than B, B is 𝟑𝟎% less than C, and
मासिक व्यय में 𝟔𝟎% की वद्
ृ धध होिी है , िो C is 𝟓𝟎% more than D. If the difference
उिकी मासिक बर्ि में ₹1,040 की वद्
ृ धध होिी 𝟏
between A and C is 285, then 𝟑𝟑 𝟑 % of B is
है। आरं सिक व्यय (₹ में) ककिना है ? equal to:
a) 9,000 b) 10,000
c) 12,000 d) 13,000
A, B िे 𝟐𝟓% कम है , B, C िे 𝟑𝟎% कम है और होिी है | इिकी खपि में होने वाली प्रतििि वद्
ृ धध
C, D िे 𝟓𝟎% अधधक है। यदद A और C के बीर् या कमी ज्ञाि करें |
का अंिर 285 है , िो B के 𝟑𝟑 𝟑 % का मान ज्ञाि a) 𝟏𝟔% Increase/वद्
ृ धध
𝟏

करें | b) 𝟏𝟐. 𝟓% Decrease/कमी


a) 140 b) 150 c) 𝟏𝟐. 𝟓% Increase/वद्
ृ धध
c) 120 d) 105
d) 𝟏𝟔% Decrease/कमी
FOR VIDEO SOLUTIONS
FROM QUESTIONS 21 TO 30 CLICK HERE 23. The price of diesel increased by 𝟏𝟔%. A person
OR SCAN QR CODE GIVEN BELOW wants to increase his expenditure on diesel by
𝟏𝟎% only. By what percentage, correct to one
decimal place, should he reduce his
consumption?
डीजल की कीमल में 16% की वद्
ृ धध होिी है |
एक व्यक्ति अपने डीजल के व्यय में केवल 10%
21. In an examination. B obtained 𝟐𝟎% more की वद्
ृ धध करना र्ाहिा है | उिे अपनी खपि को
marks than those obtained by A, and A ककिने प्रतििि (दिमलव के एक स्थान िक
obtained 𝟏𝟎% less marks than those obtained
by C . D obtained 𝟐𝟎% more marks than those िुद्ध) कम करना होगा?
obtained by C. By what percentage are the a) 𝟔. 𝟓% b) 𝟓. 𝟐% c) 𝟑. 𝟕% d) 𝟒. 𝟓%
marks obtained by D more than those obtained
by A? 24. When the price of an item was reduced by
एक परीक्षा में , B ने A के प्राप्िांकों िे 𝟐𝟎% अधधक 𝟐𝟎%, its sale increased by 𝒙%. If there is an
increase of 𝟐𝟓% in receipt of the revenue, then
अंक प्राप्ि ककए, और A ने C के प्राप्िांकों िे 𝟏𝟎% the value of 𝒙 is:
कम अंक प्राप्ि ककए। D ने C के प्राप्िांकों िे 𝟐𝟎% जब ककिी वस्िु के मूल्य में 𝟐𝟎% की कमी की
अधधक अंक प्राप्ि ककए। D के प्राप्िांकों A के गई, िो उिकी बबरी में 𝒙% की वद् ृ धध हुई है।
प्राप्िांकों िे ककिने प्रतििि अधधक हैं? यदद राजस्व प्राक्प्ि में 𝟐𝟓% की वद्
ृ धध होिी है ,
𝟏 𝟏
a) 𝟑𝟑 𝟑 % b) 𝟏𝟑 𝟑 % िो 𝒙 का मान ज्ञाि करें |
𝟏 𝟏
c) 𝟒𝟑 𝟑 % d) 𝟐𝟑 𝟑 % a) 𝟓𝟕. 𝟕𝟓 b) 𝟓𝟔. 𝟐𝟓
c) 𝟓𝟓. 𝟑𝟓 d) 𝟓𝟒. 𝟑𝟓
22. The price of a commodity increases by 𝟐𝟖%.
However, the expenditure on it increases by 25. A reduction of 𝟐𝟎% in the rate of sugar enables
𝟏𝟐%. What is the percentage increase or Sudhir to get 6 kg more sugar for Rs 960. What
decrease in its consumption? is the reduced rate of sugar per kg?
ककिी वस्िु की कीमि में 𝟐𝟖% की वद् र्ीनी के मूल्य में 𝟐𝟎% की कमी होने पर, िुधीर
ृ धध होिी
है | हालांकक, इि पर हुए खर्च में 𝟏𝟐% की वद् को 960 रु. 6 ककलोग्राम अधधक र्ीनी समलिी है |
ृ धध
र्ीनी का प्रति ककलोग्राम घटा हुआ मल्
ू य ककिना यदद 𝟐𝟕𝟎 का (𝒙 + 𝟐𝟓)% 𝟐𝟒𝟎 के 𝒙% िे 𝟐𝟎%
है ? अधधक है, िो (𝒙 + 𝟏𝟐𝟓) का 𝟏𝟐%, 𝒙 के 𝟏𝟎% िे
a) Rs. 35 b) Rs. 36 c) Rs. 32 d) Rs. 40 ककिना प्रतििि अधधक है ?
a) 55 b) 45 c) 42 d) 60
26. Last year, 300 students appeared for an
𝟏
examination from school A, and 𝟑𝟑 𝟑 % of the 29. One cup has juice and water in the ratio 𝟓 : 𝟐,
students failed. In the same year, 480 students while another cup of the same capacity has
appeared for the examination from school B. them in the ratio 𝟕 : 𝟒, respectively. If contents
𝟔𝟎% of the total students from schools A and B of both the cups (when full) are poured in a
passed. What is the fail percentage of students vessel, then what will be the final ratio of water
from school B? to juice in the vessel?
वपछले वषच, ककिी परीक्षा में ववद्यालय A िे 300 एक कप में , रि और पानी का अनुपाि 𝟓 : 𝟐 है ,
छात्र िासमल हुए और 𝟑𝟑 𝟑 % छात्र अनुत्तीणच हुए| जबकक उिी धाररिा रमि: 𝟕 : 𝟒 है | यदद दोनों
𝟏

उिी वषच, परीक्षा में ववद्यालय B िे 480 छात्र कपों की िामग्री (जब कप पूणि
च : िरी हों) एक
िासमल हुए| ववद्यालय A और B िे कुल 𝟔𝟎% बिचन में डाल दी जािी है , िो बिचन में पानी का
छात्र उत्तीणच हुए| ववद्यालय B िे अनत्त
ु ीणच छात्रों रि िे अंतिम अनुपाि तया होगा?
का प्रतििि ज्ञाि करें | a) 𝟐𝟔 : 𝟐𝟓 b) 𝟐𝟓 : 𝟓𝟐
𝟑 𝟐 𝟏 𝟏 c) 𝟐𝟓 : 𝟐𝟔 d) 𝟓𝟐 : 𝟐𝟓
a) 𝟒𝟑 𝟒 % b) 𝟒𝟎 𝟑 % c) 𝟒𝟐 𝟑 % d) 𝟒𝟒 𝟔 %
30. Alloy A contains metals 𝒙 and 𝒚 only in the ratio
27. In a company, 𝟕𝟓% of the workers are skilled 𝟓 : 𝟐, while alloy B contains them in the ratio
and the remaining are unskilled, 𝟖𝟒% of skilled 𝟑 : 𝟒. Alloy C is prepared by mixing alloys A and
workers and 𝟐𝟖% of unskilled workers are B in the ratio 𝟒 : 𝟓. The percentage of 𝒙 in alloy
permanent. If the number of temporary C is:
workers is 180, then the total number of समश्र धािु A में , धािए
ु ाँ 𝒙 और 𝒚 केवल 𝟓 : 𝟐 के
workers in the company is:
अनप
ु ाि में हैं, जबकक समश्र धािु B में , उनका
ककिी कंपनी में , 𝟕𝟓% श्रसमक कुिल हैं और िेष
𝟑 : 𝟒 है | समश्र धािु A और B को 𝟒 : 𝟓 के अनप
ु ाि
अकुिल हैं| 𝟖𝟒% कुिल श्रसमक और 𝟐𝟖%
में समलाकर समश्र धािु C िैयार की जािी है। समश्र
अकुिल श्रसमक स्थायी हैं| यदद अस्थायी श्रसमकों
धािु C में 𝒙 का प्रतििि ज्ञाि करें |
की िंख्या 180 है, िो कंपनी में श्रसमकों की कुल 𝟒 𝟏 𝟐 𝟓
a) 𝟓𝟓 b) 𝟓𝟓 c) 𝟓𝟓 d) 𝟓𝟓
िंख्या ज्ञाि करें | 𝟗 𝟗 𝟗 𝟗

a) 800 b) 700 c) 650 d) 600


FOR VIDEO SOLUTIONS
FROM QUESTIONS 31 TO 38 CLICK HERE
28. If (𝒙 + 𝟐𝟓) % of 270 is 𝟐𝟎% more than 𝒙% of
OR SCAN QR CODE GIVEN BELOW
240, then 𝟏𝟐% of (𝒙 + 𝟏𝟐𝟓) is what per cent
more than 𝟏𝟎% of 𝒙 ?
31. Mixture A contains chocolate and milk in the 34. A man sells apples, bananas and oranges at
ratio 𝟒 : 𝟑 and mixture B contains chocolate 𝟐𝟎%, 𝟐𝟓% and 𝟑𝟎% profit, respectively. If the
and milk in the ratio 𝟓 : 𝟐. A and B are taken in ratio of the cost of the fruits is 𝟐 : 𝟑 : 𝟓, and the
the ratio 𝟓 : 𝟔 and mixed to form a new mixed. fruits are sold in the ratio 𝟓 : 𝟒 : 𝟐, then his
The percentage of chocolate in the new profit percentage is:
mixture is closest to: एक व्यक्ति िेब, केला और िंिरे रमि: 𝟐𝟎%,
समश्रण A में र्ॉकलेट और दध
ू का अनप
ु ाि 𝟒 : 𝟑 𝟐𝟓% और 𝟑𝟎% के लाि पर बेर्िा है | यदद फलों
है और समश्रण B में र्ॉकलेट और दध
ू का अनप
ु ाि के रय मूल्यों का अनुपाि 𝟐 : 𝟑 : 𝟓 है , और फल
𝟓 : 𝟐 है। A और B को 𝟓 : 𝟔 के अनुपाि में समलाकर 𝟓 : 𝟒 : 𝟐 का अनूपट में बेर्े जािे है , िो लाि
एक नया समश्रण बनाया गया| नए समश्रण में प्रतििि ज्ञाि कीक्जए|
र्ॉकलेट का प्रतििि लगिग______ होगा| a) 𝟏𝟖% b) 𝟑𝟎%
a) 𝟑𝟓% b) 𝟔𝟗% c) 𝟑𝟏% d) 𝟔𝟓% c) 𝟐𝟎% d) 𝟐𝟓%

32. The ratio of acid to water in solutions A and B 35. A container has 30 litres of milk, from which 3
is 𝟐 : 𝟕 and 𝟒 : 𝟓, respectively, these solutions litres of milk is taken out and replaced with
are mixed in the ratio of 𝟒 : 𝟑. What is the ratio water. The process is done three times. What
of acid to water in the resulting solution? is the final ratio of the water and the milk in the
ववलयन A और B में अम्ल और जल का अनुपाि container
रमिः 𝟐 : 𝟕 िथा 𝟒 : 𝟓 है, इन ववलयनों को ककिी कंटे नर में 30 लीटर दध
ू है , क्जिमें िे 3

𝟒 : 𝟑 के अनुपाि में समलाया जािा है । पररणामी लीटर दध


ू तनकाला जािा है और उिके स्थान पर

ववलयन में अम्ल और जल का अनुपाि तया है ? पानी समलाया जािा है। यह प्रकरया िीन बार की
a) 𝟐𝟓 : 𝟐𝟏 b) 𝟐𝟑 : 𝟒𝟎 जािी है । कंटे नर में पानी और दध
ू का अंतिम
c) 𝟐𝟏 : 𝟒𝟎 d) 𝟐𝟎 : 𝟒𝟑 अनप ु ाि ज्ञाि करें |
a) 𝟕𝟐𝟗 : 𝟐𝟕𝟏 b) 𝟏𝟗 : 𝟖𝟏
33. The ratio of acid and water in the solutions in c) 𝟖𝟏 : 𝟏𝟗 d) 𝟐𝟕𝟏 : 𝟕𝟐𝟗
vessels 𝑨 and 𝑩 are 𝟒: 𝟓 and 𝟓: 𝟏, respectively.
A new solution is obtained by mixing 𝟓 liters 36. Kulbhushan stared a juice (syrup + water)
and 𝟒 liters of the solutions from 𝑨 and 𝑩, counter. Initially, he had 140 litres of juice
respectively. What is the ratio of acid and water which had 𝟒𝟎% water in it. He sold 30 litres of
in the new solution? the juice. Then he added equal amounts of
बिचन 𝑨 और 𝑩 में घोल में अम्ल और पानी का syrup and water. Now the ratio of water to
अनुपाि रमिः 𝟒: 𝟓 और 𝟓: 𝟏 है। A और B िे syrup became 𝟑 : 𝟒. What quantity of water
was added?
रमिः 𝟓 लीटर और 𝟒 लीटर घोल समलाकर एक a) 24 litres b) 28 litres
नया घोल प्राप्ि ककया जािा है। नए घोल में c) 44 litres d) 22 litres
अम्ल और पानी का अनुपाि ज्ञाि करें |
37. Babulal runs a juice corner outside a park and
a) 𝟗 : 𝟒 b) 𝟓𝟎 : 𝟑𝟏
sells giloy juice (giloy + water) in the morning.
c) 𝟐𝟓 : 𝟏𝟔 d) 𝟏𝟓 : 𝟖
Initially he had 𝟏𝟗 litres juice, which had giloy
𝟏
and water in the ratio 𝟐: 𝟗. He sold 𝟐 𝟐 litres 16. B 17. A 18. C 19. C 20. A
juice. Later, in order to dilute it, he added some 21. A 22. B 23. B 24. B 25. C
water and the ratio of giloy and water became 26. D 27. D 28. D 29. B 30. D
𝟏: 𝟓. How much water was added? 31. D 32. D 33. B 34. D 35. D
बाबूलाल, ककिी पाकच के बाहर जूि की दक
ु ान 36. D 37. A 38. D
र्लािा है और िुबह में धगलोय का जूि (धगलोय
+ पानी) बेर्िा है । िुरू में उिके पाि 𝟏𝟗 लीटर 1. For Best FREE full syllabus course of
जूि था, क्जिमें धगलोय और पानी का अनुपाि Maths & Reasoning Click here
𝟐: 𝟗 था। वह 𝟐 𝟐 लीटर जूि बेर्िा है । बाद में ,
𝟏

इिे पिला करने के सलए, वह कुछ मात्रा में पानी


समलिा है और धगलोय और पानी का अनुपाि
𝟏: 𝟓 हो जािा। ककिना पानी समलाया गया था?
𝟏 𝟏
a) 𝟏 𝟐 𝒍𝒊𝒕𝒓𝒆𝒔 b) 𝟐 𝒍𝒊𝒕𝒓𝒆𝒔
c) 𝟐 𝒍𝒊𝒕𝒓𝒆𝒔 d) 𝟏 𝒍𝒊𝒕𝒓𝒆

38. A large container has a 50 litra mixture of juice


and water in the ratio 𝟑 : 𝟐. To this, a 60 litra To join Our Telegram channel Click here
juice and water mixture is added, that has a
juice to water ratio of 𝟐 : 𝟏. After this, 11 litres
of the solution is replaced with pure juice.
What is the ratio of water to juice in the final
mixture?
ककिी बड़े कंटे नर में 𝟑 : 𝟐 अनप
ु ाि वाले जूि और
पानी का 50 लीटर समश्रण है | इिमें जि
ू और
पानी के 60 लीटर समश्रण को समलाया जािा है ,
क्जिमें जि
ू और पानी का अनप
ु ाि 𝟐 : 𝟏 हो जािा
है | इिके बाद, 11 लीटर समश्रण को िद्
ु ध जि
ू िे
बदल ददया जािा है | अंतिम समश्रण में पानी और
जिू का अनप ु ाि ज्ञाि करें |
a) 𝟑𝟕 : 𝟏𝟖 b) 𝟐𝟗 : 𝟖𝟏
c) 𝟒 : 𝟕 d) 𝟏𝟖 : 𝟑𝟕

ANSWER KEY
1. B 2. B 3. A 4. D 5. B
6. D 7. D 8. B 9. C 10. D
11. A 12. B 13. D 14. C 15. C
Time & Work
है , तो अकेले ररसाि से टं की का 𝟕𝟎% हहस्सा,
ककतने समय में (घंटों में ) िाली हो जाएगा?
a) 40 b) 35 c) 30 d) 50

28. To do a certain work, A and B work on alternate


days with B beginning the work on the first day.
A alone can complete the same work in 24
𝟏
days. If the work gets completed in 𝟏𝟏 𝟑 days,
𝟕
then B alone can complete 𝐭𝐡 part of the
𝟐𝟕
original work in:
एक तनजचचत कायि करने के शलए, A और B,
िैकजल्पक हिन पर कायि करते हैं, पहले हिन B
कायि िुरू करता है| अकेले A, उस कायि को 24
हिन में कर सकता हैं| यहि कायि 𝟏𝟏 𝟑 हिन में पूरा
𝟏

होता है , तो अकेले B, मूल कायि का भाग


𝟕
𝟐𝟕
ककतने हिन में परू ा कर सकता हैं|
𝟏
a) 𝟒 𝟐 days b) 2 days
𝟏
c) 4 days d) 𝟓 𝟐 days

29. A can complete a certain work in 30 days. He


started the work. After 4 days, B joined him and
𝟏
the whole work was completed in 10 days from
27. Two pipes A and B can fill a cistern in 𝟏𝟐 𝟐 hours the beginning. B alone can complete one-third
and 25 hours, respectively. The pipes were of the same work in:
opened simultaneously, and it was found that, A ककसी कायि को 30 हिन में पूरा कर सकता है|
due to leakage in the bottom, it took one hour
िह कायि िुरू करता है| 4 हिन बाि B उसके
40 minutes more to fill the cistern. If the cistern
is full, in how much time (in hours) will the leak साथ कायि में िाशमल होता है और पूरा कायि
alone empty 𝟕𝟎% of the cistern? िुरुआत से 10 हिन में पूरा होता है| अकेले B
िो पाइप A और B एक टं की को क्रमिः 𝟏𝟐 घंटे
𝟏
𝟐 उसके कायि के एक ततहाई भाग को ककतने हिनों
और 25 घंटे में भर सकते हैं। पाइपों को एक में पूरा कर सकता है ?
साथ िोला गया, और पाया गया कक तल में ररसाि a) 2 हिन/days b) 4 हिन/days
के कारर्, टं की को भरने में एक घंटा 40 शमनट c) 3 हिन/days d) 6 हिन/days
का समय अर्धक लगा। यहि टं की पूरी भर गई
30. Two workers A and B are engaged to do a piece है , तो टं की 6 घंटे में ही भर जाती है | यहि पाइप
of work. A working alone would take 14 hours
more to complete the work than when A and B B अकेले पररचाशलत है , तो इसके द्िारा टं की को
𝟏
work together. B working alone would take 𝟑 𝟐 भरने में लगने िाला समय (घंटे में) ज्ञात करें |
hours more than when A and B work together. a) 15 b) 18 c) 10 d) 9
The time required to finish the work together
is: 32. During rainy season, huge inflow of water takes
place into a reservoir. Measures are taken to
िो श्रशमक A और B ककसी कायि को करने के शलए
clear the reservoir while water keeps flowing
तनयुक्त ककए जाते हैं| अकेले A को इस कायि को into it at a constant rate. It has been observed
परू ा करने में , एक साथ A और B को कायि परू ा that seven and five men can clear the reservoir
in 20 and 50 days, respectively, with the initial
करने में लगने िाले समय से 14 घंटे अर्धक quantity of water in the reservoir being 24 and
लगते हैं| अकेले B को इस कायि को परू ा करने में , 36 kilolitres, respectively. What is the rate of
एक साथ A और B कायि परू ा करने में लगने िाले inflow of water into the reservoir in litres per
day?
समय से 𝟑 𝟐 घंटे अर्धक लगते हैं| उन िोनों के
𝟏
बरसात के हिनों में एक जलािय में बहुत सा
द्िारा एक साथ कायि को पूरा करने में लगने पानी आता है | जलािय को साफ करने के शलए
िाला समय ज्ञात करें |
कुछ उपाय ककये जाते हैं लेककन उसमें पानी आना
a) 7 hours b) 5 hours
c) 6 hours d) 8 hours एक तनजचचत िर से जारी रहता है | यह िे िा गया
कक सात और पांच व्यजक्त क्रमि: 20 और 50 हिनों
FOR VIDEO SOLUTIONS में जलािय को उस जस्थतत में साफ कर सकते हैं
FROM QUESTIONS 31 TO 40 CLICK HERE
OR SCAN QR CODE GIVEN BELOW जबकक उसमें पानी की आरं शभक मात्रा क्रमि: 24
और 36 ककलोमीटर हो| जलािय में प्रततहिन आने
िाले पानी की मात्रा लीटर में ककतनी है ?
a) 480 b) 540 c) 600 d) 640

33. For persons, P, Q, R. S were engaged for doing


31. When operated separately , pipe A takes 5 a task, with the condition that 𝑷; 𝑸; 𝑹; 𝑺 work,
hours less than pipe B to fill a cistern, and when respectively, on (Monday, Thursday); (Tuesday,
both pipes are operated together, the cistern Fridays); (Wednesday, Saturdays); (Sundays).
gets filled in 6 hours. In how much time (in The was began on a Monday, and got
hours) will pipe B fill the cistern, if operated completed on the 15th day, which was also a
separately? Monday. If the efficiencies of P, Q, R, S in
अलग-अलग पररचाशलत होने पर ककसी टं की को respect of doing this task were in the
proportion 𝟏 : 𝟐 : 𝟑 : 𝟒, then in how many days
भरने में पाइप A को पाइप B से 5 घंटा कम लगता could R complete the task, working alone
है और जब िोनों पाइप एक साथ पररचाशलत होते without break?
चार व्यजक्त P, Q, R. S एक काम में लगाए गए, a) 12 days b) 15 days
c) 18 days d) 10 days
िति थी कक 𝑷; 𝑸; 𝑹; 𝑺 क्रमि: (सोमिार, िीरिर);
(मंगलिार, िुक्रिार); (बुधिार, ितनिार); (रवििार) 36. Fourteen persons can do a work in 18 days.
िाले हिनों में काम करें गे| ककसी सोमिार को काम After 5 days of work, 6 workers left the work,
and joined back on the last day of the work. In
िुरू ककया गया जजसे 15िें हिन सोमिार को पूरा how many days the work got completed?
कर शलया गया| यहि इस काम को करने में P, चौिह व्यजक्त, ककसी कायि को 18 हिन में परू ा कर
Q, R. S की िक्षता 𝟏 : 𝟐 : 𝟑 : 𝟒 के अनुपात में थी, सकते हैं| 5 हिन कायि करने के बाि, 6 मजिरू
तो अकेले लगातार काम करते हुए R ककतने हिनों कायि छोड िे ते हैं और कायि के अंततम हिन िापस
में उस काम को पूरा कर सकेगा? कायि िुरू करते हैं| कायि ककतने हिनों में पूरा हुआ?
a) 13 b) 10 c) 11 d) 12 a) 27 b) 24 c) 12 d) 21

34. Five men working together can complete a 𝟐


37. A and B can do a work in 𝟐𝟔 𝟑 days. B and C
work in 8 days. Om Prakash who can complete
together can complete the same work in 48
the same work independently in 24 days joined
days, while A and C together can complete the
them after 4 days . Under the circumstances in
same work in 30 days. How long (in days) will A
how many days the work will be completed?
alone take to complete 𝟔𝟎% of the work?
पांच व्यजक्त एक साथ कायि करते हुए, ककसी कायि
A और B शमलकर एक काम को 𝟐𝟔 𝟑 हिनों में कर
𝟐

को 8 हिन में पूरा कर सकते हैं। ओम प्रकाि जो


सकते हैं। B और C शमलकर उसी काम को 48
उसी कायि को स्ितंत्र रूप से 24 हिन में पूरा कर
हिनों में पूरा कर सकते हैं, जबकक A और C शमलकर
सकता है , 4 हिन बाि उनके साथ िाशमल होता
उसी काम को 30 हिनों में पूरा कर सकते हैं. A
है। इस पररजस्थतत में ककतने हिन में कायि पूरा
अकेले काम के 𝟔𝟎% भाग को पूरा करने में
हो जाएगा?
ककतना समय (हिनों में) लेगा?
a) 5 b) 3 c) 7 d) 4
a) 24 b) 36 c) 32 d) 20
35. A and B together can complete a certain work
38. Two pipes A and B can fill a tank in 12 minutes
in 20 days whereas B and C together can
and 24 minute, respectively, while a third pipe
complete it in 24 days. If A is twice as good a
C can empty the full tank in 32 minutes. All the
workman as C, then in what time will B alone
three pipes are opened simultaneously.
do 𝟒𝟎% of the same work?
However, pipe C is closed 2 minutes before the
A और B शमलकर एक तनजचचत कायि को 20 हिन tank is filled. In how much time (in minutes) will
में परू ा कर सकते हैं, जबकक B और C शमलकर उसी the tank be full?
कायि को 24 हिन में परू ा कर सकते हैं| यहि A, C िो पाइप A और B एक टं की को क्रमिः 12 शमनट
से िोगन
ु ा कुिल श्रशमक है , तो B अकेले उस कायि और 24 शमनट में भर सकते हैं, िहीं पाइप C पूरी
के 𝟒𝟎% भाग को ककतने समय में करे गा? भरी हुई टं की को 32 शमनट में िाली कर सकती
है। तीनों पाइपों एक साथ िोल हिया गया|
हालांकक पाइप C को टं की के भरने से 2 शमनट
पहले बंि कर हिया गया| टं की ककतने समय
(शमनट में ) में पूरी तरह भर जाएगी?
a) 9 b) 10 c) 12 d) 8 41. 𝑿 can do a work in 3 days, 𝒀 does three times
the same work in 8 days, and Z does five times
39. Pipes A, B and C can fill a tank in 𝟐𝟎, 𝟑𝟎 and 60 the same work in 12 days. If they have to work
hours, respectively. Pipes A, B and C are opened together for 6 hours in a day, then in how much
at 7 a.m., 8 a.m., and 9 a.m., respectively, on time can they complete the work?
the same day. When will the tank be full?
X, ककसी कायि को 3 हिन में कर सकता है , Y उसी
पाइप A, B और C ककसी टं की को क्रमिः 𝟐𝟎, 𝟑𝟎
कायि के ततन गुने को 8 हिन में कर सकता है
और 60 घंटे में भर सकते हैं। पाइप A, B और C
और Z उसी कायि के पांच गुने को 12 हिन में कर
को एक ही हिन सब
ु ह क्रमिः 7 बजे, 8 बजे और
सकता है | यहि िे एक साथ प्रततहिन 6 घंटे कायि
9 बजे िोला गया| टं की ककतने बजे परू ी तरह से
करते है , तो उनके द्िारा कायि को पूरा करने में
भर जाएगी?
लगने िाला समय ज्ञात करें |
a) 𝟒: 𝟒𝟎 p.m. b) 𝟓: 𝟒𝟎 p.m.
c) 𝟔: 𝟐𝟎 p.m. d) 𝟕: 𝟐𝟎 p.m. a) 4 hours b) 5 hours 20 minutes
c) 4 hours 10 minutes d) 5 hours
40. 4 men and 5 women can complete a work in 15
days, whereas 9 men and 6 women can 42. Five men can complete a work in 20 days. Ten
complete it in 10 days. To complete the same women can complete the same work in 15
work in 7 days, how many women should assist days. Two men and six women started working
4 men? together. After 5 days, three women left the
work a new man joined the work. The group
4 पुरुर् और 5 महहलाएं एक कायि को 15 हिनों
continued working together till the end of the
में पूरा कर सकते हैं, जबकक 9 पुरुर् और 6 work. In how many days will they be able to do
महहलाएं इसे 10 हिनों में पूरा कर सकते हैं। उसी the remaining work?
पांच परु
ु र् ककसी कायि को 20 हिन में परू ा कर
कायि को 7 हिनों में पूरा करने के शलए, ककतनी
सकते हैं| िस महहलाएं उसी कायि को 15 हिन में
महहलाओं को 4 पुरुर्ों की सहायता करनी चाहहए?
a) 14 b) 13 c) 15 d) 11 परू ा कर सकती हैं| िो परु
ु र् और छह महहलाएं
शमलकर कायि िुरू करते हैं| 5 हिन बाि, तीन
FOR VIDEO SOLUTIONS
महहलाओं उस कायि छोड हिया और एक नया
FROM QUESTIONS 41 TO 50 CLICK HERE
OR SCAN QR CODE GIVEN BELOW पुरुर् काम करने के शलए िाशमल हुआ| यह समूह,
कायि के अंत तक कायि जारी रिता है | िेर् कायि
को िे ककतने हिन में पूरा कर पाएंगे?
𝟏 𝟐 45. A, B and C can do a work separately in 20, 35
a) 𝟏𝟖 𝟑 b) 14 c) 19 d) 𝟏𝟔 𝟑
and 60 days, respectively. They started the
𝟑𝟎 work together, but B and C left 8 days and 12
43. Pipes A, B and C can fill an empty tank in 𝟕 days, respectively, before the completion of
hours, if all the three pipes are opened the work. In how many days will the work be
simultaneously. A and B are filling pipes and C completed?
is an emptying pipe. Pipe A can fill the tank in
A, B और C एक कायि को क्रमि: 20, 35 और 60
15 hours and pipe C can empty it in 12 hours. In
how long (in hours) can pipe B alone fill the हिनों में कर सकते हैं| उन्होंने एक साथ कायि
empty tank? आरं भ ककया, लेककन B और C ने कायि पूरा होने से
तीन पाइप A, B, और C को एक साथ िोलने पर क्रमि: 8 और 12 हिन पहले काम छोड हिया| अब
ये ककसी िाली टं की को घंटे में भर सकते है |
𝟑𝟎
𝟕 िह कायि ककतने हिनों में पूरा होगा?
A और B भरने िाले पाइप है और C िाली करने a) 20 days b) 15 days
िाला पाइप है | पाइप A टैंक को 15 घंटे में भर c) 10 days d) 12 days
सकता है और पाइप C इसे 12 घंटे में िाली कर
सकता है | पाइप B अकेले उस िाली टं की को ANSWER KEY
ककतने समय में (घंटो में ) भर सकता है ?
27. B 28. B 29. C 30. A
a) 3 b) 5 c) 6 d) 4
31. A 32. A 33. C 34. C 35. A
44. To do a certain work, the ratio of the 36. A 37. A 38. B 39. B 40. B
efficiencies of A and B is 𝟓 : 𝟏𝟏. Working 41. B 42. B 43. D 44. D 45. B
𝟏
together, they can do the same work in 𝟐𝟕 𝟐
days. A and B work together for 20 days. C
alone completes the remaining work in 18 days.
C alone can do 𝟕𝟓% of the original work in:
एक तनजचचत कायि को करने के शलए, A और B
की िक्षता का अनुपात 𝟓 : 𝟏𝟏 है। िे िोनों साथ-
साथ काम करते हुए इस कायि को 𝟐𝟕 𝟐 हिनों में
𝟏

कर सकते हैं। A और B, 20 हिनों तक साथ-साथ


काम करते हैं। C बेच हुए कायि को अकेले 18
हिनों में पूरा करता है । C अकेले मल
ू कायि का
𝟕𝟓% ककतने हिनों में कर कर सकता है ?
𝟏
a) 𝟓𝟔 𝟐 days b) 35 days
𝟏
c) 33 days d) 𝟒𝟗 𝟐 days
Algebra, Equation, Surds & Indices

Instruction: - For Video Solution scan QR code 𝟏𝟏 𝟐 𝟐𝟔 𝟓


a) 𝟐𝟏 b) 𝟐𝟏 c) 𝟐𝟏 d) 𝟐𝟏
given below or you can also CLICK on Click
here button to watch the Video Solution. 𝟏
5. If 𝒙 + 𝒙 = −𝟑√𝟐, what is the value of (𝒙𝟓 +
𝟏
)?
𝒙𝟓
यदि तो (𝒙𝟓 +
𝟏
𝒙 + 𝒙 = −𝟑√𝟐,
) का माि क्या होगा?
𝟏
𝒙𝟓
1. Simplify the following expression
𝟏 𝟏 𝟏 𝟑 𝟑 𝟑 a) −𝟕𝟐𝟑√𝟐 b) −𝟕𝟐𝟎√𝟐
(𝟐 × 𝟐 × 𝟐) + (𝟐 × 𝟐 × 𝟐) + (𝟔 × 𝟔 × 𝟔) + c) −𝟕𝟏𝟓√𝟐 c) −𝟕𝟏𝟕√𝟐
𝟏 𝟑 𝟏 𝟑
𝟑 (𝟐 + 𝟐) (𝟐 + 𝟔) (𝟔 + 𝟐)
निम्िलिखित व्यजक को हि कीजजए |
𝟏 𝟏 𝟏 𝟑 𝟑 𝟑
(𝟐 × 𝟐 × 𝟐) + (𝟐 × 𝟐 × 𝟐) + (𝟔 × 𝟔 × 𝟔) +
𝟏 𝟑 𝟏 𝟑 6. If 𝒙𝟐 − 𝟗𝒙 + 𝟏 = 𝟎 what is the value 𝒙𝟖 −
𝟑 (𝟐 + 𝟐) (𝟐 + 𝟔) (𝟔 + 𝟐)
𝟔𝟐𝟑𝟗𝒙𝟒 + 𝟏?
a) 𝟔𝟒 b) 𝟓𝟐𝟏 c) 𝟐𝟓𝟔 c) 𝟓𝟏𝟐
यदि 𝒙𝟐 − 𝟗𝒙 + 𝟏 = 𝟎है , तो 𝒙𝟖 − 𝟔𝟐𝟑𝟗𝒙𝟒 + 𝟏
2. If𝑨 = का माि क्या होगा ?
(𝟎.𝟏)𝟑 +(𝟎.𝟐)𝟑 +(𝟎.𝟑)𝟑 +𝟑(𝟎.𝟎𝟎𝟓+𝟎.𝟎𝟏𝟔+𝟎.𝟎𝟐𝟕)+𝟎.𝟎𝟑𝟔 a) 𝟐 b) 𝟎 c) −𝟏 d) 𝟏
(𝟎.𝟏)𝟐 +(𝟎.𝟐)𝟐 +(𝟎.𝟑)𝟐 +𝟎.𝟎𝟒+𝟎.𝟎𝟔+𝟎.𝟏𝟐
,
Then the value of 𝟔𝟎𝑨 is: 𝟏
7. If 𝒙𝟒 + (𝒙𝟒 ) = 𝟑𝟐𝟐, then what is the value of
यदि𝑨 = 𝟏
(𝟎.𝟏)𝟑 +(𝟎.𝟐)𝟑 +(𝟎.𝟑)𝟑 +𝟑(𝟎.𝟎𝟎𝟓+𝟎.𝟎𝟏𝟔+𝟎.𝟎𝟐𝟕)+𝟎.𝟎𝟑𝟔 𝒙 𝟑 − ( 𝒙𝟑 ) ?
तो,
यदि 𝒙𝟒 + (𝒙𝟒 ) = 𝟑𝟐𝟐 है , तो 𝒙𝟑 − (𝒙𝟑 ) का
(𝟎.𝟏)𝟐 +(𝟎.𝟐)𝟐 +(𝟎.𝟑)𝟐 +𝟎.𝟎𝟒+𝟎.𝟎𝟔+𝟎.𝟏𝟐 𝟏 𝟏

𝟔𝟎𝑨 का माि है:


माि क्या है ?
a) 20 b) 60 c) 36 d) 30
a) 𝟏𝟔 b) 𝟗𝟔 c) 𝟕𝟔 d) 𝟒𝟔
3. Expand 𝒙𝟐 + 𝟐𝒙 + 𝟑 about 𝒙 = −𝟐. 𝟏 𝟏
8. If 𝒙 − 𝒙 = 𝟏𝟏, What is the value of (𝒙𝟒 + 𝒙𝟒 )?
𝒙 = −𝟐 के संिर्भ मे 𝒙𝟐 + 𝟐𝒙 + 𝟑 का प्रसार
यदि 𝒙 − 𝒙 = 𝟏𝟏 है, तो (𝒙𝟒 + 𝒙𝟒 ) का माि क्या
𝟏 𝟏
करें |
a) (𝒙 − 𝟐)𝟐 − 𝟐(𝒙 + 𝟐) + 𝟑 होगा?
b) (𝒙 + 𝟐)𝟐 + 𝟐(𝒙 + 𝟐) + 𝟑 a) 𝟏𝟒𝟏𝟔𝟑 b) 𝟏𝟒𝟏𝟓𝟗
c) (𝒙 + 𝟐)𝟐 − 𝟐(𝒙 + 𝟐) + 𝟑 c) 𝟏𝟓𝟏𝟐𝟕 d) 𝟏𝟓𝟏𝟑𝟏
d) (𝒙 − 𝟐)𝟐 − 𝟐(𝒙 − 𝟐) − 𝟑
𝟏
9. If 𝟖𝒙𝟐 + 𝟗𝒙 + 𝟖 = 𝟎, then the value of 𝒙𝟑 + 𝒙𝟑
4. If 𝟗𝒂𝟐 + 𝟒𝒃𝟐 + 𝟒𝟗𝒄𝟐 + 𝟏𝟖 = 𝟐 (𝟐𝒃 + 𝟐𝟖𝒄 − is:
𝟑𝒂), then the value of (𝒂 + 𝟐𝒃 − 𝒄) will be:
यदि 𝟖𝒙𝟐 + 𝟗𝒙 + 𝟖 = 𝟎 है , तो 𝒙𝟑 + 𝒙𝟑 का माि
𝟏

यदि 𝟗𝒂𝟐 + 𝟒𝒃𝟐 + 𝟒𝟗𝒄𝟐 + 𝟏𝟖 = 𝟐 (𝟐𝒃 +


ज्ञात कीजजए |
𝟐𝟖𝒄 − 𝟑𝒂) है, तो (𝒂 + 𝟐𝒃 − 𝒄) का माि ज्ञात 𝟏𝟗𝟗 𝟗𝟗𝟗 𝟏𝟗𝟗 𝟗𝟗𝟗
a) 𝟐𝟏𝟐 b) 𝟐𝟏𝟐 c) 𝟓𝟏𝟐 d) 𝟓𝟏𝟐
करें |
𝟏
10. If 𝒙𝟒 + 𝒙𝟒 = 𝟏𝟒𝟏𝟓𝟗, Then a Possible Value of a) −𝟏𝟕𝟖√𝟑 b) 𝟏𝟖𝟐√𝟑
𝟏 c) −𝟏𝟖𝟐√𝟑 d) −𝟏𝟖𝟎√𝟑
𝒙 + 𝒙 is:
यदि 𝒙𝟒 + 𝒙𝟒 = 𝟏𝟒𝟏𝟓𝟗 है, तो 𝒙 + 𝒙 का संर्व
𝟏 𝟏

माि क्या है ?
a) 𝟖𝟏 b) 𝟔𝟗 𝟏 𝟐
c) 𝟏𝟏 d) 𝟏𝟐𝟏 16. If (𝒂 + 𝒂 + 𝟑) = 𝟏𝟔, where 𝒂 is a non-zero
𝟏
real number, then find the value of 𝒂𝟐 + 𝒂𝟐 .
𝟐
यदि (𝒂 + 𝒂 + 𝟑) = 𝟏𝟔, जहााँ 𝒂 एक शुन्येतर
𝟏

वास्तववक संख्या है , तो 𝒂𝟐 + 𝒂𝟐 का माि ज्ञात


𝟏
𝟒 𝟏
11. If 𝒙 > 𝟎 and 𝒙 + 𝒙𝟒 = 𝟐𝟐𝟎𝟕, What is the
𝟏 कीजजए|
value of (𝒙𝟓 + 𝒙𝟓 )?
a) 𝟑 b) 𝟒𝟕 c) 𝟒𝟗 d) 𝟕
यदि 𝒙 > 𝟎 और 𝒙 + 𝒙𝟒 = 𝟐𝟐𝟎𝟕 है, तो (𝒙 +𝟒 𝟏 𝟓

𝟏
) का माि क्या होगा? 17. If 𝒂𝟐 + 𝒄𝟐 + 𝟏𝟕 = 𝟐(𝒂 − 𝟖𝒃 − 𝟐𝒃𝟐 ), then
𝒙𝟓
what is the value of (𝒂𝟑 + 𝒃𝟑 + 𝒄𝟑 )?
a) 𝟏𝟓𝟏𝟒𝟏 b) 𝟏𝟓𝟏𝟑𝟒
c)𝟏𝟓𝟏𝟑𝟎 d)𝟏𝟓𝟏𝟐𝟕 यदि 𝒂𝟐 + 𝒄𝟐 + 𝟏𝟕 = 𝟐(𝒂 − 𝟖𝒃 − 𝟐𝒃𝟐 ) है, तो
(𝒂𝟑 + 𝒃𝟑 + 𝒄𝟑 ) का माि ज्ञात करें |
𝟐 𝟖
12. If 𝒙 − 𝟖𝒙 + 𝟏 = 𝟎, what is the value of 𝒙 − a) −𝟕 b) 9 c) 10 d) −𝟒
𝟑𝟖𝟒𝟐𝒙𝟒 + 𝟏?
यदि 𝒙𝟐 − 𝟖𝒙 + 𝟏 = 𝟎 है, तो 𝒙𝟖 − 𝟑𝟖𝟒𝟐𝒙𝟒 + 𝟏 𝟔𝟒
18. If 𝒓 + 𝒓 = 𝟏𝟔, then the value of 𝒓𝟒 + 𝒓𝟑 is
𝟏

का माि ज्ञात कीजजए| _______.


यदि 𝒓 + = 𝟏𝟔 है, तो 𝒓𝟒 + 𝒓𝟑 का माि
𝟔𝟒 𝟏
a) 𝟏 b) 𝟎 c) 𝟐 d) −𝟏
𝒓

𝟏 𝟕 𝟏 _______ है।
13. If (𝒙 − 𝒙) = 𝟑, what is the value of (𝒙𝟑 − 𝒙𝟑 )? 𝟏
a) 𝟒𝟎𝟗𝟔 b) 𝟒𝟎𝟗𝟔 𝟓𝟏𝟐
यदि (𝒙 − 𝒙) = 𝟑 है , तो (𝒙 − 𝒙𝟑 ) का माि
𝟏 𝟕 𝟑 𝟏
𝟏
c) 𝟓𝟏𝟐 d) 𝟓𝟏𝟐 𝟒𝟎𝟗𝟔
ज्ञात कीजजए?
𝟐𝟎 𝟐 𝟏𝟗 𝟕
a) 𝟏𝟗 𝟐𝟕 b) 𝟏𝟗 𝟑 c)𝟏𝟗 𝟐𝟕 d) 𝟏𝟗 𝟗 𝟏 𝟐
19. If (𝒙 + 𝒙) = 𝟑, then what is the value of 𝒙𝟔 +
𝟏 𝟏 𝒙−𝟔 ?
𝟒
14. If 𝐱 + 𝒙 = −𝟏𝟑, What is the value of 𝐱 + 𝒙𝟒 ? 𝟏 𝟐
यदि (𝒙 + 𝒙) = 𝟑 है , तो 𝒙𝟔 + 𝒙−𝟔 का माि
यदि 𝐱 + 𝒙 = −𝟏𝟑 है, तो 𝐱 𝟒 + 𝒙𝟒 का माि क्या
𝟏 𝟏

क्या है ?
होगा? a) 𝟔 b) 𝟐 c) −𝟔 d) −𝟐
a) 𝟐𝟗𝟐𝟒𝟑 b) 𝟐𝟖𝟓𝟔𝟏
c) 𝟐𝟕𝟖𝟖𝟕 d) 𝟐𝟕𝟖𝟗𝟏 𝟒
20. The value of √𝟐𝟒 − 𝟏𝟔√𝟐 × √𝟒 + 𝟐√𝟐:
√𝟐𝟒 − 𝟏𝟔√𝟐 × √𝟒 + 𝟐√𝟐: का माि ककतिा
𝟒
𝟏 𝟏
15. If 𝒙 + 𝒙 = −𝟐√𝟑, What is the value of 𝒙𝟓 + 𝒙𝟓?
यदि 𝒙 + 𝒙 = −𝟐√𝟑 है , तो 𝒙𝟓 + 𝒙𝟓 का माि
𝟏 𝟏 है ?
a) 𝟒√𝟐 b) 𝟐√𝟐
ज्ञात कीजजए|
c) 𝟒 d) 𝟖 𝟏
26. If (𝟒𝒂 − 𝟑𝒃) = 𝟏, 𝒂𝒃 = 𝟐, where 𝒂 > 𝟎 and
𝒃 > 𝟎, what is the value of (𝟔𝟒𝒂𝟑 + 𝟐𝟕𝒃𝟑 )?
यदि (𝟒𝒂 − 𝟑𝒃) = 𝟏, 𝒂𝒃 = हैं, जहााँ 𝒂 > 𝟎
𝟏
𝟐
और 𝒃 > 𝟎 है , (𝟔𝟒𝒂𝟑 + 𝟐𝟕𝒃𝟑 ) का माि क्या
𝟒
21. The expression √𝟑𝟒 − 𝟐𝟒√𝟐 × √𝟒 + 𝟑√𝟐 होगा ?
simplifies to:
a) 𝟏𝟓 b) 𝟐𝟓 c) 𝟑𝟎 d) 𝟑𝟓
व्यंजक का
𝟒
√𝟑𝟒 − 𝟐𝟒√𝟐 × √𝟒 + 𝟑√𝟐
सरितम रूप ________ है| 27. If 𝒂 − 𝒃 = 𝟑 and 𝒂𝟑 − 𝒃𝟑 = 𝟗𝟗𝟗, then find the
value of 𝒂𝟐 − 𝒃𝟐 .
a) 𝟒 b) √𝟐 c) 𝟐 d) 𝟐√𝟐
यदि 𝒂 − 𝒃 = 𝟑 और 𝒂𝟑 − 𝒃𝟑 = 𝟗𝟗𝟗 है , तो
22. Find the value of √𝟐 + √𝟑 + √𝟐 − √𝟑. 𝒂𝟐 − 𝒃𝟐 का माि ज्ञात कीजजए|
√𝟐 + √𝟑 + √𝟐 − √𝟑 का माि ज्ञात कीजजए| a) 𝟔𝟎 b) 𝟔𝟒 c) 𝟔𝟑 d) 𝟔𝟐
a) √𝟔 b) 6 c) 𝟐√𝟑 d) 𝟐√𝟐
28. If 𝟐𝒂 + 𝟓𝒃 = 𝟏𝟐 and 𝒂𝒃 = 𝟑, find the value of
𝟒𝒂𝟐 + 𝟐𝟓𝒃𝟐.
√𝟑𝟖−𝟓√𝟑 𝒂+𝒃√𝟑
23. If
√𝟐𝟔+𝟕√𝟑
= 𝟐𝟑
, 𝒃 > 𝟎, then the value of यदि 𝟐𝒂 + 𝟓𝒃 = 𝟏𝟐 और 𝒂𝒃 = 𝟑 है , तो 𝟒𝒂𝟐 +
(𝒃 − 𝒂) is: 𝟐𝟓𝒃𝟐 का माि ज्ञात करे |
√𝟑𝟖−𝟓√𝟑
यदि , 𝒃 > 𝟎 हो, तो (𝒃 − 𝒂)
𝒂+𝒃√𝟑
= a) 𝟔𝟒 b) 𝟖𝟒 c) 𝟐𝟒 d) 𝟒𝟒
√𝟐𝟔+𝟕√𝟑 𝟐𝟑

का माि ककतिा होगा? 29. If 𝒂 + 𝟐𝒃 = 𝟐𝟕 and 𝒂𝟑 + 𝟖𝒃𝟑 = 𝟓𝟒𝟐𝟕, then


a) 𝟕 b) 𝟏𝟖 c) 𝟐𝟗 d) 𝟏𝟏 find the value of 𝟐𝒂𝒃.
यदि 𝒂 + 𝟐𝒃 = 𝟐𝟕 और 𝒂𝟑 + 𝟖𝒃𝟑 = 𝟓𝟒𝟐𝟕 है
√𝟐𝟔−𝟕√𝟑 𝒃+𝒂√𝟑
24. If
√𝟏𝟒+𝟓√𝟑
= 𝟏𝟏
, 𝒃 > 𝟎, then what is the तो
value of √(𝒃 − 𝒂) ? 𝟐𝒂𝒃 का माि ज्ञात करे |
√𝟐𝟔−𝟕√𝟑
यदि , 𝒃 > 𝟎 हो, तो √(𝒃 − 𝒂) a) 𝟏𝟒𝟗 b) 𝟏𝟕𝟔 c) 𝟏𝟓𝟔 d) 𝟏𝟕𝟐
𝒃+𝒂√𝟑
=
√𝟏𝟒+𝟓√𝟑 𝟏𝟏

का माि ककतिा होगा? 30. If 𝒙𝟑 + 𝒚𝟑 = 𝟒𝟏𝟔 and 𝒙 + 𝒚 = 𝟖, then find


a) 𝟓 b) 𝟐𝟓 c) 𝟏𝟐 d) 𝟗 𝒙 𝟒 + 𝒚𝟒 .
यदि 𝒙𝟑 + 𝒚𝟑 = 𝟒𝟏𝟔 और 𝒙 + 𝒚 = 𝟖 है तो
25. If 𝟑𝒙 + 𝟐𝒚 = 𝟏𝟓 and 𝒙𝒚 = 𝟔, then what is the
𝟑 𝟒 𝒙𝟒 + 𝒚𝟒 का माि ज्ञात कीजजए|
value of 𝟐 𝒙𝟑 + 𝟗 𝒚𝟑 ?
a) 𝟑𝟎𝟎𝟐 b) 𝟑𝟐𝟎𝟒 c) 𝟑𝟎𝟎𝟒 d) 𝟑𝟏𝟎𝟒
यदि 𝟑𝒙 + 𝟐𝒚 = 𝟏𝟓 और 𝒙𝒚 = 𝟔 हो, तो
𝟑
𝒙𝟑 +
𝟐
𝒚𝟑 का माि ककतिा होगा?
𝟒
𝟗
a) 𝟗𝟓. 𝟖 b) 𝟗𝟐. 𝟓 c) 𝟗𝟖. 𝟔 d) 𝟗𝟕. 𝟓
31. If 𝟖(𝒙 + 𝒚)𝟑 − 𝟐𝟕(𝒙 − 𝒚)𝟑 = (𝟓𝒚 − 𝒙)(𝑨𝒙𝟐 +
𝑩𝒚𝟐 + 𝑪𝒙𝒚), then what is the value of
(𝑨 + 𝑩 − 𝑪)?
यदि𝟖(𝒙 + 𝒚)𝟑 − 𝟐𝟕(𝒙 − 𝒚)𝟑 = (𝟓𝒚 −
𝒙)(𝑨𝒙𝟐 + 𝑩𝒚𝟐 + 𝑪𝒙𝒚) है, तो (𝑨 + 𝑩 − 𝑪) का
माि ज्ञात करें |
√𝟐
a) 36 b) 16 c) −𝟐𝟔 d) −𝟏𝟔 36. If 𝒙 = √𝟐 + 𝟏 and 𝒚 = 𝟑 , then the value of
(𝒙 − 𝟐𝒚 − 𝟏)(𝒙𝟐 + 𝟒𝒚𝟐 + 𝟐𝒙𝒚 − 𝟐𝒙 − 𝟐𝒚 +
32. Given that (𝟐𝒙 + 𝒚)𝟑 − (𝒙 + 𝟐𝒚)𝟑 = (𝒙 − 𝟏) is:
𝒚)[𝑨(𝒙𝟐 + 𝒚𝟐 ) + 𝑩𝒙𝒚],the value of (𝟐𝑨 − 𝑩)
यदि 𝒙 = √𝟐 + 𝟏 तथा 𝒚 = है, तो (𝒙 − 𝟐𝒚 −
√𝟐
is: 𝟑

दिया गया है कक (𝟐𝒙 + 𝒚)𝟑 − (𝒙 + 𝟐𝒚)𝟑 = 𝟏)(𝒙𝟐 + 𝟒𝒚𝟐 + 𝟐𝒙𝒚 − 𝟐𝒙 − 𝟐𝒚 + 𝟏) का माि


(𝒙 − 𝒚)[𝑨(𝒙𝟐 + 𝒚𝟐 ) + 𝑩𝒙𝒚] है, तो (𝟐𝑨 − 𝑩) ज्ञात कीजजए|
का माि ज्ञात करें |
𝟑𝟖√𝟐 𝟒√𝟐 𝟏𝟑√𝟐 𝟒𝟎√𝟐
a) b) c) d)
𝟐𝟕 𝟑 𝟗 𝟐𝟕
a) 7 b) 6 c) 0 d) 1
37. If (𝒙𝟑 + 𝒚𝟑 )𝟐 = (𝒙𝟐 + 𝒚𝟐 )𝟑 , then the value of
33. If (𝒙 + 𝟐𝒚)𝟑 + (𝟐𝒙 − 𝒚)𝟑 = (𝟑𝒙 + 𝒚)[𝑨𝒙𝟐 + 𝒙𝟐 +𝒚𝟐
𝒙 ≠ 𝟎, 𝒚 ≠ 𝟎, is:
𝑩𝒚𝟐 + 𝑪𝒙𝒚], then the value of (𝟑𝑨 − 𝑩 − 𝒙𝒚
𝒙𝟐 +𝒚𝟐
𝑪) : (𝑨 + 𝑩 + 𝒄) is: यदि(𝒙𝟑 + 𝒚𝟑 )𝟐 = (𝒙𝟐 + 𝒚𝟐 )𝟑 , तो 𝒙≠
𝒙𝒚
यदि (𝒙 + 𝟐𝒚)𝟑 + (𝟐𝒙 − 𝒚)𝟑 = (𝟑𝒙 + 𝟎, 𝒚 ≠ 𝟎 का माि क्या होगा:
𝟐
𝒚)[𝑨𝒙 + 𝑩𝒚 + 𝑪𝒙𝒚]𝟐
है, तो (𝟑𝑨 − 𝑩 − a) 𝒙 + 𝒚𝟐 b) 𝟑
𝟐

𝑪) : (𝑨 + 𝑩 + 𝒄) का माि क्या है ? c) 𝒙𝟐 + 𝒚 d) 𝟏
a) 𝟐 : 𝟓 b) 𝟑 : 𝟒 c) 𝟓 : 𝟕 d) 𝟏 : 𝟕
38. The factors of 𝒂𝟐 − 𝟏 − 𝟐𝒙 − 𝒙𝟐 are ______.
34. If 𝒙𝟔 − 𝟔√𝟔𝒚𝟔 = (𝒙𝟐 + 𝑨𝒚𝟐 )(𝒙𝟒 + 𝑩𝒙𝟐 𝒚𝟐 + 𝒂𝟐 − 𝟏 − 𝟐𝒙 − 𝒙𝟐 के गुणििंड _________ है |
𝑪𝒚𝟒 ), then what will be value of (𝑨𝟐 − 𝑩𝟐 + a) (𝒂 − 𝒙 − 𝟏)(𝒂 − 𝒙 − 𝟏)
𝑪𝟐 ) ? b) (𝒂 − 𝒙 + 𝟏)(𝒂 − 𝒙 − 𝟏)
यदि 𝒙𝟔 − 𝟔√𝟔𝒚𝟔 = (𝒙𝟐 + 𝑨𝒚𝟐 )(𝒙𝟒 + 𝑩𝒙𝟐 𝒚𝟐 + c) (𝒂 + 𝟏 + 𝒙)(𝒂 − 𝟏 − 𝒙)
𝑪𝒚𝟒 ) है , तो (𝑨𝟐 − 𝑩𝟐 + 𝑪𝟐 ) का माि ज्ञात d) (𝒂 − 𝒙 + 𝟏)(𝒂 − 𝒙 + 𝟏)

करें | 39. If 𝒙 = √𝟏𝟎 + √𝟏𝟏, 𝒚 = √𝟏𝟎 − √𝟏𝟏, then


a) 27 b) 42 c) 36 d) 18 value of 𝟕𝒙𝟐 − 𝟓𝟎𝒙𝒚 + 𝟕𝒚𝟐 = ______.
𝟑 यदि 𝒙 = √𝟏𝟎 + √𝟏𝟏, 𝒚 = √𝟏𝟎 − √𝟏𝟏, तो
35. If 𝒙 = √𝟐 − 𝟏 and 𝒚 = , then the value
𝟐 𝟐
√𝟐 𝟕𝒙𝟐 − 𝟓𝟎𝒙𝒚 + 𝟕𝒚𝟐 का माि ज्ञात कीजजए|
of(𝒙 − 𝟐𝒚 + 𝟏)(𝒙 + 𝟒𝒚 + 𝟐𝒚 + 𝟐𝒙𝒚 + 𝟐𝒙 +
a) 𝟑𝟖𝟔 b) 𝟏𝟑𝟔𝟎 c) 𝟑𝟒𝟒 d) 𝟕𝟎𝟒
𝟏) is:
यदि 𝒙 = √𝟐 − 𝟏 तथा 𝒚 = है, तो (𝒙 − 𝟐𝒚 +
𝟑
√𝟐 40. If 𝒂 + 𝒃 = √𝟕 and 𝒂 − 𝒃 = √𝟓, then find the
𝟏)(𝒙 + 𝟒𝒚 + 𝟐𝒚 + 𝟐𝒙𝒚 + 𝟐𝒙 + 𝟏) का माि
𝟐 𝟐 value of 𝟖𝒂𝒃(𝒂𝟐 + 𝒃𝟐 ) − (𝒂 − 𝒃)𝟐 .
ज्ञात कीजजए| यदि 𝒂 + 𝒃 = √𝟕 और 𝒂 − 𝒃 = √𝟓 है , तो
a) 𝟏𝟓√𝟐 b) 𝟑𝟎√𝟐 𝟖𝒂𝒃(𝒂𝟐 + 𝒃𝟐 ) − (𝒂 − 𝒃)𝟐 का माि ज्ञात
c) −𝟓𝟐√𝟐 d) −𝟐𝟔√𝟐 कीजजए|
a) 𝟏𝟗 b) 𝟐𝟑 c) 𝟐𝟏 d) 𝟐𝟕
41. Simplify (𝟗𝟓𝟕 + 𝟗𝟑𝟐)𝟐 − 𝟒 × 𝟗𝟓𝟕 × 𝟗𝟑𝟐. 46. If 𝒙 + 𝟐𝒚 = 𝟏𝟎 and 𝟐𝒙𝒚 = 𝟗,then one of the
(𝟗𝟓𝟕 + 𝟗𝟑𝟐)𝟐 − 𝟒 × 𝟗𝟓𝟕 × 𝟗𝟑𝟐 को सरि value of 𝒙 − 𝟐𝒚 is:
यदि If 𝒙 + 𝟐𝒚 = 𝟏𝟎 और 𝟐𝒙𝒚 = 𝟗 तो 𝒙 −
करे |
a) 𝟔𝟐𝟓 b) 𝟔𝟕𝟔 c) 𝟓𝟐𝟗 d) 𝟓𝟕𝟔 𝟐𝒚 के मािो मे से एक माि ______ है |
a) 𝟖 b) 𝟔 c) 𝟏𝟎 d) 𝟏𝟐
42. For what value(s) of K will the expression 𝒑 +
𝟏
√𝒑 + 𝒌𝟐 be a perfect square 47. If 𝒂𝟐 + 𝟒𝒃𝟐 + 𝟐𝟓𝒄𝟐 + 𝟏𝟖 = 𝟐(𝒂 − 𝟐𝒃 +
𝟗
𝟐𝟎𝒄), then what is the value of (𝒂 + 𝟐𝒃 +
K के ककस माि/ककि मािो के लिए व्यजक 𝟓𝒄)?
𝒑 + 𝟗 √𝒑 + 𝒌𝟐 एक पण
ू भ वगभ होगा?
𝟏
यदि 𝒂𝟐 + 𝟒𝒃𝟐 + 𝟐𝟓𝒄𝟐 + 𝟏𝟖 = 𝟐(𝒂 − 𝟐𝒃 +
𝟏 𝟏
a) 𝒌 = + 𝟖 b) 𝒌 = + 𝟗 𝟐𝟎𝒄) है , तो (𝒂 + 𝟐𝒃 + 𝟓𝒄) का माि ज्ञात करें |
𝟏 𝟏
c) 𝒌 = + 𝟐𝟏 d) 𝒌 = + 𝟏𝟖 a) 3 b) 4 c) 6 d) 5

(𝒂−𝒃)𝟑 +(𝒃−𝒄)𝟑 +(𝒄−𝒂)𝟑


43. If 𝒑𝟑 + 𝟑𝒑𝟐 + 𝟑𝒑 = 𝟐𝟔, then the value of 48. The value of 𝟑 𝟑 𝟑 is equal
(𝒂𝟐 −𝒃𝟐 ) +(𝒃𝟐 −𝒄𝟐 ) +(𝒄𝟐 −𝒂𝟐 )
𝒑𝟐 + 𝟐𝒑 is:
to:
यदि 𝒑𝟑 + 𝟑𝒑𝟐 + 𝟑𝒑 = 𝟐𝟔 है , तो 𝒑𝟐 + 𝟐𝒑 का (𝒂−𝒃)𝟑 +(𝒃−𝒄)𝟑 +(𝒄−𝒂)𝟑
का माि _____ के
माि ज्ञात कीजजए|
𝟑 𝟑 𝟑
(𝒂𝟐 −𝒃𝟐 ) +(𝒃𝟐 −𝒄𝟐 ) +(𝒄𝟐 −𝒂𝟐 )

a) 8 b) 12 c) 10 d)15 बराबर होगा|


a) 𝒂𝒃 + 𝒃𝒄 + 𝒄𝒂 b) 𝟎
𝟐 𝟑 (𝒙𝟑 𝟏
44. If 𝒙 = 𝒃 − 𝒂𝒙 and 𝒙 + 𝒄) = 𝒅,then c) (𝒂+𝒃)(𝒃+𝒄)(𝒄+𝒂) d) 𝟏
which of the following can be the values of 𝒄
and 𝒅 ?
49. Simplify the following expression.
यदि 𝒙𝟐 = 𝒃 − 𝒂𝒙 तथा 𝒙𝟑 (𝒙𝟑 + 𝒄) = 𝒅 हो,
निम्िलिखित व्यंजक को हि कीजजए।
निम्िलिखित में से कौि से 𝒄 और 𝒅 के माि (𝟓𝟗 × 𝟓𝟗 × 𝟓𝟗) + (𝟓𝟒 × 𝟓𝟒 × 𝟓𝟒) + (𝟓𝟕 × 𝟓𝟕 × 𝟓𝟕) − 𝟑
हो सकते हैं? (𝟓𝟗 + 𝟓𝟒 + 𝟓𝟕)
a) 𝒄 = 𝒂𝟑 + 𝟑𝒂𝒃 and 𝒅 = −𝒃𝟑 a) 𝟕𝟔 b) 𝟏𝟕𝟎
b) 𝒄 = 𝒂𝟑 − 𝟑𝒂𝒃 and 𝒅 = − 𝒃𝟑 c) 𝟑𝟖 d) 𝟏𝟗
c) 𝒄 = −𝒂𝟑 − 𝟑𝒂𝒃 and 𝒅 = 𝒃𝟑
d) 𝒄 = 𝒂𝟑 + 𝟑𝒂𝒃 and 𝒅 = 𝒃𝟑 50. If 𝒙 + 𝒚 + 𝒛 = 𝟏𝟎, 𝒙𝟐 + 𝒚𝟐 + 𝒛𝟐 = 𝟑𝟎,then
the value of 𝒙𝟑 + 𝒚𝟑 + 𝒛𝟑 − 𝟑𝒙𝒚𝒛 is ______.
45. Simplify the following expression. यदि 𝒙 + 𝒚 + 𝒛 = 𝟏𝟎, 𝒙𝟐 + 𝒚𝟐 + 𝒛𝟐 = 𝟑𝟎 है
निम्िलिखित व्यंजक को हि कीजजए। तो of 𝒙𝟑 + 𝒚𝟑 + 𝒛𝟑 − 𝟑𝒙𝒚𝒛 का
(𝟔𝟐 × 𝟔𝟐 × 𝟔𝟐) − 𝟑(𝟔𝟐 × 𝟔𝟐 × 𝟐𝟐) + 𝟑(𝟔𝟐 × 𝟐𝟐 × 𝟐𝟐) − (𝟐𝟐 × 𝟐𝟐 × 𝟐𝟐)
मािा_________है |
𝟖×𝟖×𝟖
a) 𝟐𝟐𝟓 b) 𝟏𝟐𝟓𝟎 c) 𝟏𝟐𝟓 d) 𝟐𝟓 a) −𝟑𝟎 b) −𝟕𝟎 c) −𝟓𝟎 d) −𝟏𝟎
यदि 𝟐𝒛 = 𝒙 + 𝒚 हो, तो + 𝒚−𝒛 का माि
𝒙 𝒚
𝒙−𝒛
ज्ञात करें |
a) 𝟐 b) 𝟎 c) 𝟓 d) 𝟏
51. If 𝒙 + 𝒚 + 𝒛 = 𝟏𝟗, 𝒙𝒚𝒛 = 𝟐𝟏𝟔 and 𝒙𝒚 + 𝒚𝒛 +
𝒛𝒙 = 𝟏𝟏𝟒, then the value of 𝒙𝟑 + 𝒚𝟑 + 𝒛𝟑 +
𝒙𝒚𝒛 is:
यदि 𝒙 + 𝒚 + 𝒛 = 𝟏𝟗, 𝒙𝒚𝒛 = 𝟐𝟏𝟔 और 𝒙𝒚 +
56. If 𝒎𝒙𝒎 − 𝒏𝒙𝒏 = 𝟎 then what is the value of
𝒚𝒛 + 𝒛𝒙 = 𝟏𝟏𝟒 है तो 𝒙 + 𝒚 + 𝒛 + 𝒙𝒚𝒛 का
𝟑 𝟑 𝟑
𝟏 𝟏
+ 𝐱𝐦 −𝐱𝐧 in terms of 𝒙𝒏 ?
माि ज्ञात करें | 𝐱 𝐦 +𝐱 𝐧
यदि 𝐦𝐱 𝐦 − 𝐧𝐱 𝐧 = 𝟎 है, तो 𝐱 𝐧 पिों में
a) 1225 b) 1441 c) 361 d) 577
+ 𝐱𝐦 −𝐱𝐧 का माि क्या है ?
𝟏 𝟏
𝐱 𝐦 +𝐱 𝐧
52. If 𝒙 + 𝒚 + 𝒛 = 𝟏𝟑, 𝒙𝟐 + 𝒚𝟐 + 𝒛𝟐 = 𝟗𝟏 and 𝟐𝒎 𝟐𝒎𝒏
a) 𝒙𝒏 (𝒎𝟐 −𝒏𝟐 ) b) 𝒙𝒏 (𝒎𝟐 −𝒏𝟐)
𝒙𝒛 = 𝒚𝟐 , then the difference between z and x
𝟐𝒎𝒏 𝟐𝒎𝒏
is: c) 𝒙𝒏 (𝒎𝟐+𝒏𝟐 ) d) 𝒙𝒏 (𝒏𝟐 −𝒎𝟐)
यदि 𝒙 + 𝒚 + 𝒛 = 𝟏𝟑, 𝒙𝟐 + 𝒚𝟐 + 𝒛𝟐 = 𝟗𝟏 और
𝒙𝒛 = 𝒚𝟐 है, तो z और x के बीच अंतर ज्ञात 57. The value of
𝟎.𝟖𝟗𝟔×𝟎.𝟕𝟓𝟐+𝟎.𝟏𝟏𝟐×𝟏.𝟗𝟖𝟒
+
𝟎.𝟕×𝟎.𝟎𝟑𝟒+𝟐.𝟏×(𝟎.𝟑𝟐𝟐)
करें | (𝟒.𝟐)𝟑 +(𝟐.𝟖)𝟑
(𝟒.𝟐)𝟐 −(𝟐.𝟖)𝟐
is:
a) 3 b) 8 c) 5 d) 9
(𝟒.𝟐)𝟑 +(𝟐.𝟖)𝟑
का माि
𝟎.𝟖𝟗𝟔×𝟎.𝟕𝟓𝟐+𝟎.𝟏𝟏𝟐×𝟏.𝟗𝟖𝟒
+ (𝟒.𝟐)𝟐 −(𝟐.𝟖)𝟐
𝟎.𝟕×𝟎.𝟎𝟑𝟒+𝟐.𝟏×(𝟎.𝟑𝟐𝟐)
53. If 𝟐𝒙 + 𝟑𝒚 + 𝟒𝒛 = 𝟏𝟏, 𝟖𝒙³ + 𝟐𝟕𝒚³ +
ककतिा होगा?
𝟔𝟒𝒛³ = 𝟏𝟎𝟓 and 𝒙𝒚𝒛 = 𝟏, then the value
𝟒𝒙² + 𝟗𝒚² + 𝟏𝟔𝒛² – 𝟔𝒙𝒚 – 𝟏𝟐𝒚𝒛 – 𝟖𝒙𝒛 is: a) 𝟏𝟏. 𝟎𝟖 b) 𝟏𝟎. 𝟑𝟐
c) 𝟏𝟎. 𝟗𝟐 d) 𝟗. 𝟔𝟖
यदि 𝟐𝒙 + 𝟑𝒚 + 𝟒𝒛 = 𝟏𝟏, 𝟖𝒙³ + 𝟐𝟕𝒚³ +
𝟔𝟒𝒛³ = 𝟏𝟎𝟓 और 𝒙𝒚𝒛 = 𝟏 है, तो 𝟒𝒙² + 58. If (𝒂𝒃 + 𝒃𝒄 + 𝒄𝒂) = 𝟎, then what is the value
का माि
𝟏 𝟏 𝟏
𝟗𝒚² + 𝟏𝟔𝒛² – 𝟔𝒙𝒚 – 𝟏𝟐𝒚𝒛 – 𝟖𝒙𝒛 of (𝒂𝟐 −𝒃𝒄 + 𝒃𝟐 −𝒄𝒂 + 𝒄𝟐 −𝒂𝒃)?
बताएं| यदि है , तो
𝟏
(𝒂𝒃 + 𝒃𝒄 + 𝒄𝒂) = 𝟎 (𝒂𝟐 −𝒃𝒄 +
a) 3 b) 4 c) 5 d) 6 + 𝒄𝟐 −𝒂𝒃) का माि क्या है
𝟏 𝟏
𝒃𝟐 −𝒄𝒂
a) 𝒂 + 𝒃 + 𝒄 b) 𝟎
54. If (𝟒𝒙 − 𝟓)𝟑 + (𝒙 − 𝟐)𝟑 + 𝟐𝟕 (𝟐𝒙 − 𝟓)𝟑 =
c) 𝟏 d) 𝟐
𝟗 (𝟒𝒙 − 𝟓)(𝒙 − 𝟐)(𝟐𝒙 − 𝟓), then the value of
𝟑
(𝒙 + 𝟐) will be: 59. If (𝟒𝒙 + 𝟐𝒚)𝟑 + (𝟒𝒙 − 𝟐𝒚)𝟑 = 𝟏𝟔(𝑨𝒙𝟑 +
यदि (𝟒𝒙 − 𝟓)𝟑 + (𝒙 − 𝟐)𝟑 + 𝟐𝟕 (𝟐𝒙 − 𝟓)𝟑 = 𝟏
𝑩𝒙𝒚𝟐 ), then what is the value of 𝟐 √𝑨𝟐 + 𝑩𝟐 ?
𝟗 (𝟒𝒙 − 𝟓)(𝒙 − 𝟐)(𝟐𝒙 − 𝟓) है, तो (𝒙 + 𝟐) का
𝟑
यदि (𝟒𝒙 + 𝟐𝒚)𝟑 + (𝟒𝒙 − 𝟐𝒚)𝟑 =
माि ज्ञात करें | 𝟏𝟔(𝑨𝒙𝟑 + 𝑩𝒙𝒚𝟐 ), तो √𝑨𝟐 + 𝑩𝟐 का माि क्या
𝟏
𝟐
𝟑 𝟏 𝟕 𝟓
a) 𝟐 b) 𝟐 c) 𝟐 d) 𝟐 है ?
a) 8 b) 3 c) 5 d) 7
𝒙 𝒚
55. If 𝟐𝒛 = 𝒙 + 𝒚 , then the value of 𝒙−𝒛 + 𝒚−𝒛 is :
60. What is the simplified value of निम्ि समीकरण में प्रश्ि चचह्ि (?) के स्थाि
(𝒙+𝒚+𝒛)(𝒙𝒚+𝒚𝒛+𝒛𝒙)−𝒙𝒚𝒛
(𝒙+𝒚)(𝒚+𝒛)(𝒛+𝒙)
? पर कौि-सी संख्या आ सकती है ?
का सरिीकृत माि
(𝒙+𝒚+𝒛)(𝒙𝒚+𝒚𝒛+𝒛𝒙)−𝒙𝒚𝒛
(𝟎. 𝟎𝟔𝟒)𝟏𝟐𝟑 ÷ 𝟎. 𝟏𝟔𝟒𝟕 × 𝟎. 𝟒𝟑𝟒 × 𝟎. 𝟒𝟐𝟗
(𝒙+𝒚)(𝒚+𝒛)(𝒛+𝒙)
= (𝟎. 𝟒)?
ककतिा होगा? a) 𝟑𝟓𝟎 b) 𝟑𝟐𝟎 c) 𝟑𝟑𝟖 d) 𝟑𝟒𝟏
a) 𝒚 b) 𝒛 c) 𝟏 d) 𝒙

66. Which of the following can be a rationalizing


61. If (𝟑𝒙 + 𝟐𝒚)𝟑 + (𝟑𝒙 − 𝟐𝒚)𝟑 = 𝟑𝒌𝒙 (𝟑𝒙𝟐 + factor (√𝟐 + √𝟑 + √𝟓)?
𝟒𝒚𝟐 ), then the value of k will be:
निम्िलिखित में से कौि-सा (√𝟐 + √𝟑 +
यदि (𝟑𝒙 + 𝟐𝒚)𝟑 + (𝟑𝒙 − 𝟐𝒚)𝟑 = 𝟑𝒌𝒙 (𝟑𝒙𝟐 +
√𝟓) का पररमेकरण गुणांक सकता है ?
𝟒𝒚𝟐 ), है , तो k का माि ज्ञात करें |
a) (√𝟐 − √𝟑 − √𝟓)√𝟔
a) 18 b) 9 c) 3 d) 6
b) (√𝟐 + √𝟑 + √𝟓 )√𝟔
𝟏𝟗 c) (√𝟐 − √𝟑 + √𝟓)√𝟔
62. If 𝒙𝟐 + 𝟒𝒚𝟐 + 𝟑𝒛𝟐 + 𝟒 = 𝟐 √𝟑 (𝒙 + 𝒚 + 𝒛),
d) (√𝟐 + √𝟑 − √𝟓)√𝟔
then the value of (𝒙 − 𝟒𝒚 + 𝟑𝒛) is:
यदि 𝒙𝟐 + 𝟒𝒚𝟐 + 𝟑𝒛𝟐 + = 𝟐 √𝟑 (𝒙 + 𝒚 + 𝒛)
𝟏𝟗
𝟏 𝟏 𝟏
𝟒 67. The value of − + −
है, तो (𝒙 − 𝟒𝒚 + 𝟑𝒛) का माि ज्ञात करें |
𝟒−√𝟏𝟓 √𝟏𝟓−√𝟏𝟒 √𝟏𝟒−√𝟏𝟑
𝟏 𝟏 𝟏 𝟏 𝟏
+ − + −
√𝟑 √𝟏𝟑−√𝟏𝟐 √𝟏𝟐−√𝟏𝟏 √𝟏𝟏−√𝟏𝟎 √𝟏𝟎−𝟑 𝟑−√𝟖
a) b) 𝟐√𝟑 is:
𝟑
√𝟑 𝟏 𝟏 𝟏 𝟏
c) √𝟑 d) − + − +
𝟐 𝟒−√𝟏𝟓 √𝟏𝟓−√𝟏𝟒 √𝟏𝟒−√𝟏𝟑 √𝟏𝟑−√𝟏𝟐

का माि
𝟏 𝟏 𝟏 𝟏
− + − 𝟑−√𝟖
63. Simplify (𝟓 × 𝟓 × 𝟓 × 𝟓 × 𝟓)𝟓 × (𝟓 × 𝟓 × √𝟏𝟐−√𝟏𝟏 √𝟏𝟏−√𝟏𝟎 √𝟏𝟎−𝟑

𝟓)𝟓 ÷ 𝟓 = (𝟏𝟐𝟓)? ज्ञात करें |


(𝟓 × 𝟓 × 𝟓 × 𝟓 × 𝟓)𝟓 × (𝟓 × 𝟓 × 𝟓)𝟓 ÷ 𝟓 = a) 𝟒 + 𝟐√𝟐 b) 𝟐 + 𝟐√𝟐
(𝟏𝟐𝟓)? को सरि करें | c) 𝟒 − 𝟐√𝟐 d) 𝟐 − 𝟐√𝟐
a) 𝟏𝟓 b) 𝟏𝟑
c) 𝟐𝟏 d) 𝟏𝟒 68. What is the sum of the value of 𝒑 and 𝒒 for
which the system of equations 𝟐𝒙 + 𝟑𝒚 = 𝟕;
𝟏 (𝒑 + 𝒒) 𝒙 + (𝟐𝒑 − 𝒒)𝒚 = 𝟐𝟏 has infinite
𝟗𝟑 +𝟗𝟒 +𝟗𝟓 +𝟗𝟔 +𝟗𝟕 𝟐
64. Find the value of (𝟗𝟏 +𝟗𝟐 +𝟗𝟑 +𝟗𝟒 +𝟗𝟓 ) number of solutions?
𝒑 और 𝒒 के उि मािों का योग ज्ञात कीजजए,
𝟏
𝟗𝟑 +𝟗𝟒 +𝟗𝟓 +𝟗𝟔 +𝟗𝟕 𝟐
(𝟗𝟏 +𝟗𝟐 +𝟗𝟑 +𝟗𝟒 +𝟗𝟓 ) का माि ज्ञात करें |
जजिके लिए समीकरणों 𝟐𝒙 + 𝟑𝒚 = 𝟕;
a) 𝟖𝟏 b) 𝟗 c) 𝟕𝟐𝟗 d) 𝟑
(𝒑 + 𝒒) 𝒙 + (𝟐𝒑 − 𝒒)𝒚 = 𝟐𝟏 के अपररलमत हि
65. Select the number that will come in place of हैं?
the question mark (?) in the mathematical a) 𝟓 b) 𝟕 c) 𝟔 d) 𝟒
statement.
(𝟎. 𝟎𝟔𝟒)𝟏𝟐𝟑 ÷ 𝟎. 𝟏𝟔𝟒𝟕 × 𝟎. 𝟒𝟑𝟒 × 𝟎. 𝟒𝟐𝟗 =
(𝟎. 𝟒)?
69. If 𝟖𝒌𝟔 + 𝟏𝟓𝒌𝟑 − 𝟐 = 𝟎, then the positive यदि 𝒙𝟒 + 𝒚𝟒 + 𝒙𝟐 𝒚𝟐 = 𝟏𝟕 𝟏𝟔 और 𝒙𝟐 − 𝒙𝒚 +
𝟏
𝟏
value of (𝒌 + 𝒌) is : 𝒚𝟐 = 𝟓 𝟒 है, तो (𝒙 − 𝒚) का माि बताइए|
𝟏

यदि 𝟖𝒌𝟔 + 𝟏𝟓𝒌𝟑 − 𝟐 = 𝟎 है, तो (𝒌 + 𝒌) का


𝟏
𝟓 𝟑 𝟓 𝟑
a) 𝟐 b) 𝟒 c) 𝟒 d) 𝟐
धिात्मक माि क्या है ?
𝟏
a) 𝟐 𝟐 b) 𝟐 𝟖
𝟏
c) 𝟖 𝟐
𝟏
d) 𝟖 𝟖
𝟏 74. If 𝒙𝟒 + 𝒚𝟒 + 𝒙𝟐 𝒚𝟐 = 𝟏𝟏𝟕 and 𝒙𝟐 + 𝒚𝟐 − 𝒙𝒚 =
𝟑 (𝟒 + √𝟑), then the value of (𝒙𝟐 + 𝒚𝟐 ) will
be:
70. If 𝟒√𝟑𝒙𝟐 + 𝟓𝒙 − 𝟐√𝟑 = (𝑨𝒙 + 𝟐)(𝑩𝒙 +
𝑪), then what is the value of (𝑨 + 𝑩 + यदि 𝒙𝟒 + 𝒚𝟒 + 𝒙𝟐 𝒚𝟐 = 𝟏𝟏𝟕 और 𝒙𝟐 + 𝒚𝟐 −
𝑪) ? (𝑨 > 𝟎) 𝒙𝒚 = 𝟑 (𝟒 + √𝟑) है , (𝒙𝟐 + 𝒚𝟐 ) का ज्ञात करें |
यदि 𝟒√𝟑𝒙𝟐 + 𝟓𝒙 − 𝟐√𝟑 = (𝑨𝒙 + 𝟐)(𝑩𝒙 + 𝑪) a) 𝟗 b) 𝟔√𝟑 c) 𝟏𝟐 d) 𝟏𝟑√𝟑
है, तो (𝑨 + 𝑩 + 𝑪) का माि ज्ञात करें | (𝑨 >
𝟎) 75. If 𝟏 + 𝟒𝒙² + 𝟏𝟔𝒙𝟒 = 𝟓𝟏𝟐, 𝟏– 𝟐𝒙 +
a) 𝟒 b) 𝟒 + √𝟑 𝟒𝒙² = 𝟔𝟒, then the value of 𝟏 + 𝟐𝒙 + 𝟒𝒙²
c) 𝟐√𝟑 d) 𝟒 − √𝟑 is:
यदि 𝟏 + 𝟒𝒙² + 𝟏𝟔𝒙𝟒 = 𝟓𝟏𝟐 है और
𝟏– 𝟐𝒙 + 𝟒𝒙² = 𝟔𝟒 है, तो 𝟏 + 𝟐𝒙 + 𝟒𝒙² का
माि ककतिा होगा?
71. When 𝒇(𝒙) = 𝟏𝟓𝒙𝟑 − 𝟏𝟒𝒙𝟐 − 𝟒𝒙 + 𝟏𝟎 is a) 6 b) 8 c) 12 d) 10
divided by (𝟑𝒙 + 𝟐), then the remainder is:
Answers Keys
जब 𝒇(𝒙) = 𝟏𝟓𝒙𝟑 − 𝟏𝟒𝒙𝟐 − 𝟒𝒙 + 𝟏𝟎 को
1. C 2. C 3. C 4. B 5. C
(𝟑𝒙 + 𝟐), से ववर्ाजजत ककया जाता है , तो 6. B 7. C 8. C 9. D 10. C
_______शेषफि प्राप्त होता है | 11. D 12. B 13. C 14. C 15. A
a) 𝟐 b) − 𝟏 c) −𝟐 d) 𝟏 16. B 17. A 18. B 19. D 20. B
21. B 22. A 23. C 24. A 25. D
72. If 𝒑𝒙𝟑 + 𝒙𝟐 + 𝟑𝒙 + 𝒒 is exactly divisible by 26. D 27. C 28. B 29. C 30. D
(𝒙 + 𝟐)and (𝒙 − 𝟐) , then the value of 𝒑 and 𝒒 31. A 32. D 33. C 34. C 35. C
are: 36. A 37. B 38. C 39. C 40. A
यदि 𝒑𝒙𝟑 + 𝒙𝟐 + 𝟑𝒙 + 𝒒 , (𝒙 + 𝟐) एवं (𝒙 − 𝟐) 41. A 42. D 43. A 44. D 45. C
46. A 47. B 48. C 49. D 50. C
से पूणत
भ : ववर्ाज्य है , तो 𝒑 और 𝒒 के माि
51. A 52. B 53. A 54. C 55. A
_____ है | 56. D 57. A 58. B 59. C 60. C
𝟑 𝟑 61. D 62. C 63. B 64. B 65. C
a) 𝒑 = − 𝟒 and 𝒒 = 𝟒 b) 𝒑 = 𝟒 and 𝒒 = 𝟒
𝟑 𝟑 66. D 67. C 68. C 69. A 70. A
c) 𝒑 = 𝟒 and 𝒒 = −𝟒 d) 𝒑 = − 𝟒 and 𝒒 = −𝟒
71. A 72. D 73. A 74. C 75. B
𝟏
73. If 𝒙𝟒 + 𝒚𝟒 + 𝒙𝟐 𝒚𝟐 = 𝟏𝟕 𝟏𝟔 and 𝒙𝟐 − 𝒙𝒚 +
𝟏
𝒚𝟐 = 𝟓 𝟒, then one of the values of (𝒙 − 𝒚) is:
EQUATION, SURDS & SIMPLIFICATION
FOR VIDEO SOLUTION FROM QUESTION 1 TO a) 𝟕𝑩² = 𝟑𝑪𝑨
10 : CLICK HERE OR b) 𝟕𝑩² = 𝟒𝑪𝑨
SCAN Q R CODE GIVEN BELOW c) 𝟕𝑩² = 𝟑𝟔𝑪𝑨
d) 𝟏𝟎𝑩𝟐 = 𝟒𝟗𝑪𝑨

4. If 𝛂 𝐚𝐧𝐝 𝛃 are the roots of equation 𝒙² −


𝒙 + 𝟏 = 𝟎, then which equation will have
roots 𝛂𝟑 𝒂𝒏𝒅 𝛃𝟑 ?
1. If A and B are the roots of the equation यदि 𝛂 तथा 𝛃 समीकरण 𝒙² − 𝒙 + 𝟏 = 𝟎 के मल

𝑨𝒙² − 𝑨²𝒙 + 𝑨𝑩 = 𝟎, then what is the value
है , तो ककस समीकरण के मल
ू 𝛂 तथा 𝛃 होंगे?
𝟑 𝟑
of A and B respectively?
a) 𝒙² + 𝟐𝒙 + 𝟏 = 𝟎
यदि A तथा B समीकरण 𝑨𝒙² − 𝑨²𝒙 + 𝑨𝑩 = 𝟎
b) 𝒙² − 𝟐𝒙 − 𝟏 = 𝟎
के मल
ू है तो क्रमश: A तथा B का मान क्या है ? c) 𝒙² + 𝟑𝒙 − 𝟏 = 𝟎
a)1, 0 d) 𝒙² − 𝟑𝒙 + 𝟏 = 𝟎
b)1, 1
c)0, 2 5. If the difference between the roots of the
d)0. 1 equation 𝑨𝒙² − 𝑩𝒙 + 𝑪 = 𝟎 is 4, then which
of the following is True?
2. If a and b are roots of the equation 𝒂𝒙² + यदि समीकरण 𝑨𝒙² − 𝑩𝒙 + 𝑪 = 𝟎 के मल
ु ो का
𝒃𝒙 + 𝒄 = 𝟎, then which equation will have
अंतर 4 है , तो ननम्नललखित में से कोन सा सत्य
roots ( 𝒂𝒃 + 𝒂 + 𝒃 ) 𝒂𝒏𝒅 ( 𝒂𝒃 – 𝒂 – 𝒃 )?
है ?
यदि a तथा b समीकरण 𝒂𝒙² + 𝒃𝒙 + 𝒄 = 𝟎 के
a) 𝑩² − 𝟏𝟔𝑨² = 𝟒𝑨𝑪 + 𝟒𝑩²
मूल है , तो ककस समीकरण के मूल (𝒂𝒃 + 𝒂 +
b) 𝑩² − 𝟏𝟎𝑨² = 𝟒𝑨𝑪 + 𝟔𝑨²
𝒃) तथा (𝒂𝒃 − 𝒂 − 𝒃) होगा? c) 𝑩² − 𝟖𝑨² = 𝟒𝑨𝑪 + 𝟏𝟎𝑨²
a) 𝒂²𝒙² + 𝟐𝒂𝒄𝒙 + 𝒄² + 𝒃² = 𝟎 d) 𝑩² − 𝟏𝟔𝑨² = 𝟒𝑨𝑪 + 𝟖𝑩²
b) 𝒂²𝒙² − 𝟐𝒂𝒄𝒙 + 𝒄² − 𝒃² = 𝟎
c) 𝒂²𝒙² − 𝟐𝒂𝒄𝒙 + 𝒄² + 𝒃² = 𝟎 6. A root of equation 𝒂𝒙² + 𝒃𝒙 + 𝒄 = 𝟎 (where
d) 𝒂²𝒙² + 𝟐𝒂𝒄𝒙 + 𝒄² − 𝒃² = 𝟎 a, b and c are rational numbers) is 𝟓 + 𝟑√𝟑.
(𝒂𝟐 +𝒃𝟐 +𝒄𝟐 )
what is the value of 𝒂+𝒃+𝒄
?
3. If one root of the equation 𝑨𝒙² + 𝑩𝒙 + 𝑪 =
𝟎 two and a half times the others, then समीकरण 𝒂𝒙² + 𝒃𝒙 + 𝒄 = 𝟎 (जहााँ a,b तथा c
which of the following is true? पररमय संख्या है ) का मूल 𝟓 + 𝟑√𝟑 है
यदि समीकरण 𝑨𝒙² + 𝑩𝒙 + 𝑪 = 𝟎 का एक मल
ू 𝒂𝟐 +𝒃𝟐 +𝒄𝟐
का मान क्या है ?
𝒂+𝒃+𝒄
िस
ु रे से ढाई गुणा है , तो ननम्नललखित में से कोन 𝟑𝟓 𝟑𝟕 𝟏𝟎𝟓 𝟏𝟎𝟓
a) 𝟑
b) 𝟑 c)− 𝟏𝟏
d)− 𝟏𝟑
सा सत्य है ?
7. 𝛂 𝐚𝐧𝐝 𝛃 are the roots of the quadratic 10. If 𝛂 𝐚𝐧𝐝 𝛃 are the roots of the equation
equation 𝒙² − 𝒙 − 𝟏 = 𝟎. What is the value 𝒙² + 𝒙 − 𝟏 = 𝟎, then what is the equation
of 𝜶𝟖 + 𝜷𝟖 ? whose roots are 𝜶𝟓 𝒂𝒏𝒅 𝜷𝟓 ?
𝛂 तथा 𝛃 दिघात समीकरण 𝒙² − 𝒙 − 𝟏 = 𝟎 के यदि 𝛂 तथा 𝛃 समीकरण 𝒙² + 𝒙 − 𝟏 = 𝟎 के मूल

मूल है 𝜶𝟖 + 𝜷𝟖 का मान क्या है ? है , तो वह समीकरण क्या है जजसके मूल

a)47 𝜶𝟓 तथा 𝜷𝟓 है ?

b)54 a)𝒙² + 𝟕𝒙 − 𝟏 = 𝟎
b)𝒙² − 𝟕𝒙 − 𝟏 = 𝟎
c)59
c)𝒙² − 𝟏𝟏𝒙 − 𝟏 = 𝟎
d)68 d)𝒙² + 𝟏𝟏𝒙 − 𝟏 = 𝟎

8. 𝛂 𝐚𝐧𝐝 𝛃 are the roots of quadratic equation. FOR VIDEO SOLUTION FROM QUESTION 11
TO 20 : CLICK HERE OR
If 𝛂 + 𝛃 = 𝟖 and 𝛂 − 𝛃 = 𝟐√𝟓, then which
SCAN Q R CODE GIVEN BELOW
of the following equation will have roots
𝜶𝟒 𝒂𝒏𝒅 𝜷𝟒 ?
𝛂 तथा 𝛃 दिघात समीकरण के मूल है यदि 𝛂 +
𝛃 = 𝟖 तथा 𝛂 − 𝛃 = 𝟐√𝟓 है , तो 𝜶𝟒 तथा 𝜷𝟒
ननम्नललखित में से ककस समीकरण के मल
ू है ?
11. If 𝛂 𝐚𝐧𝐝 𝛃 are the roots of equation 𝒙² −
a) 𝒙² − 𝟏𝟓𝟐𝟐𝒙 + 𝟏𝟒𝟔𝟒𝟏 = 𝟎
𝟐𝒙 + 𝟒 = 𝟎, then what is the equation whose
b) 𝒙² + 𝟏𝟗𝟐𝟏𝒙 + 𝟏𝟒𝟔𝟒𝟏 = 𝟎
𝛂𝟑 𝜷𝟑
c) 𝒙² − 𝟏𝟕𝟔𝟒 + 𝟏𝟒𝟔𝟒𝟏 = 𝟎 roots are 𝛃 𝟐 𝒂𝒏𝒅 𝜶𝟐
?
d) 𝒙² + 𝟐𝟓𝟐𝟎𝒙 + 𝟏𝟒𝟔𝟒𝟏 = 𝟎 यदि 𝛂 तथा 𝛃 समीकरण 𝒙² − 𝟐𝒙 + 𝟒 = 𝟎 के
मल
ू है तो वह समीकरण क्या है जजसके मल

9. If a and b are the roots of the equation
𝛂𝟑 𝜷𝟑
𝑷𝒙² − 𝑸𝒙 + 𝑹 = 𝟎, then what is the value of तथा है ?
𝛃𝟐 𝜶𝟐
𝟏 𝟏 𝒂 𝒃
(𝒂𝟐 ) + (𝒃𝟐 ) + (𝒃) + (𝒂) ? a) 𝒙² − 𝟒𝒙 + 𝟖 = 𝟎
यदि a तथा b समीकरण 𝑷𝒙² − 𝑸𝒙 + 𝑹 = 𝟎 की b) 𝒙² − 𝟑𝟐𝒙 + 𝟒 = 𝟎
𝒂 𝒃 c) 𝒙² − 𝟐𝒙 + 𝟒 = 𝟎
मूल है तो ( 𝟐 ) + ( 𝟐 ) + ( ) + ( ) का मान
𝟏 𝟏
𝒂 𝒃 𝒃 𝒂 d) 𝒙² − 𝟏𝟔𝒙 + 𝟒 = 𝟎
क्या है ?
(𝑸𝟐 −𝟐𝑷)(𝟐𝑹+𝑷) 12. If the roots of the equation 𝒂(𝒃 − 𝒄)𝒙² +
a) 𝑷𝑹²
𝒃(𝒄 − 𝒂)𝒙 + 𝒄(𝒂 − 𝒃) = 𝟎 are equal, then
(𝑸𝟐 −𝟐𝑷𝑹)(𝑹+𝑷)
b) 𝑷𝑹𝟐 which of the following is true?
(𝑸𝟐 −𝟐𝑹)(𝟐𝑷+𝑹)
c) 𝑷𝟐 𝑹𝟐
𝟐
(𝑸 −𝟐𝑷𝑹)(𝑹+𝑷)
d) 𝑷𝑹²
यदि समीकरण 𝒂(𝒃 − 𝒄)𝒙² + 𝒃(𝒄 − 𝒂)𝒙 + यदि 𝟑𝒙 + 𝟒𝒚 − 𝟐𝒛 + 𝟗 = 𝟏𝟕, 𝟕𝒙 + 𝟐𝒚 +
𝒄(𝒂 − 𝒃) = 𝟎 के मल
ू बराबर है , तो ननम्नललखित 𝟏𝟏𝒛 + 𝟖 = 𝟐𝟑 तथा 𝟓𝒙 + 𝟗𝒚 + 𝟔𝒛 − 𝟒 = 𝟏𝟖 है ,
में से कोन सा सही है ? तो 𝒙 + 𝒚 + 𝒛 − 𝟑𝟒 का मान क्या है ?
𝒂+𝒄
a) 𝐛 = 𝒂𝒄 a)-28
𝟐 𝟏 𝟏 b)-24
b) = ( ) + ( )
𝒃 𝒂 𝒄 c)-31
𝟏 𝟏
c) 𝟐𝒃 = (𝒂) + ( 𝒄 ) d)-45
d) 𝒂𝒃𝒄 = 𝒂𝒃 + 𝒃𝒄 + 𝒄𝒂
𝟐𝒛 𝟐
16. If 𝐱 + 𝟑𝐲 − 𝟒
= 𝟔, 𝒙 + 𝟑 (𝟐𝒚 + 𝟑𝒛) = 𝟑𝟑
13. If 𝟑𝒙 + 𝟓𝒚 + 𝟕𝒛 = 𝟒𝟗 𝒂𝒏𝒅 𝟗𝒙 + 𝟖𝒚 + 𝟏
and (𝒙 + 𝒚 + 𝒛) + 𝟐𝒛 = 𝟗, then what is
𝟐𝟏𝒛 = 𝟏𝟐𝟔, then what is the value of y? 𝟕
the value of 46x+131y?
यदि 𝟑𝒙 + 𝟓𝒚 + 𝟕𝒛 = 𝟒𝟗 तथा 𝟗𝒙 + 𝟖𝒚 +
𝟐𝒛 𝟐
यदि x+3y- 𝟒 = 𝟔, x+𝟑(2y+3z)=33 तथा
𝟐𝟏𝒛 = 𝟏𝟐𝟔 है तो y का मान क्या है ?
𝟏
a)4 𝟕
(𝒙 + 𝒚 + 𝒛) + 𝟐𝒛 = 𝟗, है , तो 46x+131y का
b)2 मान क्या है ?
c)3 a)414
d)5
b)364

14. Cost of 4 pens, 6 note books and 9 files is c)384


Rs.305. Cost or 3 pens, 4 notebooks and 2 d)464
files is Rs.145. What is the cost of 5 pens, 8
notebooks and 16 files? 17. Cost of 8 pencils, 5 pens and 3 erasers is
4 कलम, 6 नोटबुक तथा 9 फाइल का मूल्य 305 Rs111. Cost of 9 pencils, 6 pens and 5 erasers
रूपये है | 3 कलम 4 नोटबक
ु तथा 2 फाइल का is Rs130. Cost of 16 pencils, 11 pens and 3
erasers is Rs221. What is the cost of 39
मल्
ू य 145 है 5 कलम 8 नोटबक
ु तथा 16 फाइल
pencils, 26 pens and 13 erasers?
का मूल्य क्या है ?
8 पें लसल, 5 कलम तथा 3 रबड़ का मूल्य 111
a)415
रूपये है 9 पें लसल, 6 कलम तथा 5 रबड़ का मूल्य
b)465
c)440 130 है 16 पें लसल, 11 कलम तथा 3 रबड़ का
d)Cannot be determined मूल्य 221 रूपये है 39 पें लसल, 13 कलम तथा 13
रबड़ का मल्
ू य क्या है ?
15. If 𝟑𝒙 + 𝟒𝒚 − 𝟐𝒛 + 𝟗 = 𝟏𝟕, 𝟕𝒙 + 𝟐𝒚 +
a)316
𝟏𝟏𝒛 + 𝟖 = 𝟐𝟑 𝒂𝒏𝒅 𝟓𝒙 + 𝟗𝒚 + 𝟔𝒛 − 𝟒 = 𝟏𝟖,
b)546
then what is the value of 𝒙 + 𝒚 + 𝒛 − 𝟑𝟒?
c)624
d)482
18. If 𝟐𝒙 + 𝟑𝒚 − 𝟓𝒛 = 𝟏𝟖, 𝟑𝒙 + 𝟐𝒚 + 𝒛 = अगर 𝒙𝟒 + 𝒙𝟐 𝒚𝟐 + 𝒚𝟒 = 𝟐𝟕𝟑 और 𝒙𝟐 − 𝒙𝒚 +
𝟐𝟗 𝒂𝒏𝒅 𝒙 + 𝒚 + 𝟑𝒛 = 𝟏𝟕, then what is the 𝒚𝟐 = 𝟏𝟑 है तो 𝒙𝒚 का मान क्या होगा?
value of 𝒙𝒚 + 𝒚𝒛 + 𝒛𝒙?
a) 4 b) 10 c) 6 d) 8
यदि 𝟐𝒙 + 𝟑𝒚 − 𝟓𝒛 = 𝟏𝟖, 𝟑𝒙 + 𝟐𝒚 + 𝒛 =
𝟐𝟗 तथा 𝒙 + 𝒚 + 𝟑𝒛 = 𝟏𝟕 है , तो 𝒙𝒚 + 𝒚𝒛 + 22. If 𝒙𝟒 + 𝒙𝟐 𝒚𝟐 + 𝒚𝟒 = 𝟐𝟏 and 𝒙𝟐 + 𝒙𝒚 + 𝒚𝟐 =
𝒛𝒙 का मान क्या है ? 𝟕 then the value of (𝒙𝟐 + 𝒚𝟐) is:
𝟏 𝟏

a)32
अगर 𝒙𝟒 + 𝒙𝟐 𝒚𝟐 + 𝒚𝟒 = 𝟐𝟏 और 𝒙𝟐 + 𝒙𝒚 +
b)52 𝟏 𝟏
c)64 𝒚𝟐 = 𝟕 है तो ( + ) का मान बताओ.
𝒙𝟐 𝒚𝟐
d)46 𝟓 𝟕 𝟕 𝟓
a) 𝟒 b) 𝟒 c) 𝟑 d) 𝟐
𝟕 𝟑
19. If 𝒂 + 𝒃 + 𝒄 = , 𝟑𝒂 − 𝟒𝒃 + 𝟓𝒄 = and
𝟏𝟐 𝟒
𝟕
𝟕𝒂 − 𝟏𝟏𝒃 − 𝟏𝟑𝒄 = 𝟏𝟐, then what is the 23. If 𝟏𝟔𝒂𝟒 + 𝟑𝟔𝒂𝟐 𝒃𝟐 + 𝟖𝟏𝒃𝟒 = 𝟗𝟏 and 𝟒𝒂𝟐 +
value of 𝒂 + 𝒄? 𝟗𝒃𝟐 − 𝟔𝒂𝒃 = 𝟏𝟑, then what is the value of
3ab?
यदि 𝒂 + 𝒃 + 𝒄 = 𝟕/𝟏𝟐, 𝟑𝒂 − 𝟒𝒃 + 𝟓𝒄 =
अगर 𝟏𝟔𝒂𝟒 + 𝟑𝟔𝒂𝟐 𝒃𝟐 + 𝟖𝟏𝒃𝟒 = 𝟗𝟏 और
𝟑/𝟒 तथा 𝟕𝒂 − 𝟏𝟏𝒃 − 𝟏𝟑𝒄 = 𝟕/𝟏𝟐 है , तो 𝒂 +
𝟒𝒂𝟐 + 𝟗𝒃𝟐 − 𝟔𝒂𝒃 = 𝟏𝟑 है तो 𝟑𝒂𝒃 का मान
𝒄 का मान क्या है ?
बताओ.
𝟏 𝟓 𝟑 𝟏
a) b) c)𝟒 d) 𝟑 −𝟑
𝟐 𝟏𝟐 𝟒 a) −𝟑 b) 𝟓 c) d)
𝟐 𝟐

20. If 𝒂𝟒 + 𝒂𝟐 𝒃𝟐 + 𝒃𝟒 = 𝟖 and 𝒂𝟐 + 𝒂𝒃 + 𝒃𝟐 = (𝟒.𝟔)𝟒 +(𝟓.𝟒)𝟒 +(𝟐𝟒.𝟖𝟒)𝟐


𝟒, then the value of ab is 24. The value of (𝟒.𝟔)𝟐 +(𝟓.𝟒)𝟐 +𝟐𝟒.𝟖𝟒
is:

अगर 𝒂𝟒 + 𝒂𝟐 𝒃𝟐 + 𝒃𝟒 = 𝟖 और 𝒂𝟐 + 𝒂𝒃 + (𝟒.𝟔)𝟒 +(𝟓.𝟒)𝟒 +(𝟐𝟒.𝟖𝟒)𝟐


का मान क्या है ?
(𝟒.𝟔)𝟐 +(𝟓.𝟒)𝟐 +𝟐𝟒.𝟖𝟒
𝒃 = 𝟒 है तो ab का मान
𝟐
1. 24.42 2. 24.24
a) -1 b) 0 3. 25.42 4. 25.48
c) 2 d) 1
25. If 𝐱 𝟐 + 𝐱𝐲 + 𝐲 𝟐 = 𝟐𝟏 and 𝐱 + √𝒙𝒚 + 𝒚 = 𝟕,
FOR VIDEO SOLUTION FROM QUESTION 21
then the value of 𝐱 − 𝐲 is
TO 30 : CLICK HERE OR
SCAN Q R CODE GIVEN BELOW अगर 𝐱 𝟐 + 𝐱𝐲 + 𝐲 𝟐 = 𝟐𝟏 और 𝐱 + √𝒙𝒚 + 𝒚 =
𝟕 है तो 𝐱 − 𝐲 का मान बताओ.
a) 9 b) 3 c) 5 d) Can’t say

𝟏 𝟏
26. The value of 𝒂𝟐 +𝒂𝒙+𝒙𝟐
− 𝒂𝟐−𝒂𝒙+𝒙𝟐 +
𝟐𝒂𝒙
21. If 𝒙𝟒 + 𝒙𝟐 𝒚𝟐 + 𝒚𝟒 = 𝟐𝟕𝟑 and 𝒙𝟐 − 𝒙𝒚 + 𝒂𝟒 +𝒂𝟐 𝒙𝟐 +𝒙𝟒
is
𝒚𝟐 = 𝟏𝟑 then the value of 𝒙𝒚 is: a) 2 b) 1
c) -1 d) 0
27. If 𝒙 + 𝒚 + 𝒛 = 𝟏𝟏, 𝒙𝟐 + 𝒚𝟐 + 𝒛𝟐 = 𝟏𝟑𝟑 and
𝒙𝟑 + 𝒚𝟑 + 𝒛𝟑 = 𝟖𝟖𝟏, them the value of
𝟑
√𝒙𝒚𝒛 is:
यदि 𝒙 + 𝒚 + 𝒛 = 𝟏𝟏, 𝒙𝟐 + 𝒚𝟐 + 𝒛𝟐 = 𝟏𝟑𝟑
और 𝒙𝟑 + 𝒚𝟑 + 𝒛𝟑 = 𝟖𝟖𝟏 है , तो √𝒙𝒚𝒛 का
𝟑 31. x, y and z are real numbers. If 𝒙³ + 𝒚³ + 𝒛³ =
𝟏𝟑, 𝒙 + 𝒚 + 𝒛 = 𝟏 𝒂𝒏𝒅 𝒙𝒚𝒛 = 𝟏, then what is
मान क्या होगा:
the value of 𝒙𝒚 + 𝒚𝒛 + 𝒛𝒙?
1. −𝟔 2. 6 3. −𝟖 4. 8
x, y तथा z वास्तववक संख्याएाँ है यदि 𝒙³ + 𝒚³ +

28. If 𝒙 + 𝒚 + 𝒛 = 𝟏, 𝒙² + 𝒚² + 𝒛² = 𝟐 and 𝒙³ + 𝒛³ = 𝟏𝟑, 𝒙 + 𝒚 + 𝒛 = 𝟏 एंड 𝒙𝒚𝒛 = 𝟏 है , तो 𝒙𝒚 +


𝒚³ + 𝒛³ = 𝟑, then what is the value of 𝒙𝒚𝒛? 𝒚𝒛 + 𝒛𝒙 का मान क्या है ?
यदि 𝒙 + 𝒚 + 𝒛 = 𝟏, 𝒙² + 𝒚² + 𝒛² = 𝟐 तथा 𝒙³ + a)-1 b)1 c)3 d)-3
𝒚³ + 𝒛³ = 𝟑, है , तो 𝒙𝒚𝒛 का मान क्या है ?
a)1/3 b)1/6 c)1/2 d)1/4 32. 𝐩𝟑 + 𝒒𝟑 + 𝒓𝟑 − 𝟑𝒑𝒒𝒓 = 𝟒. if 𝒂 = 𝒒 + 𝒓, 𝒃 =
𝒓 + 𝒑 and 𝒄 = 𝒑 + 𝒒, then what is the value
29. If 𝒙 + 𝒚 + 𝒛 = 𝟐, 𝒙𝒚 + 𝒚𝒛 + 𝒛𝒙 = of 𝒂³ + 𝒃³ + 𝒄³ − 𝟑𝒂𝒃𝒄?
−𝟏𝟏 𝒂𝒏𝒅 𝒙𝒚𝒛 = −𝟏𝟐, then what is the 𝑷𝟑 + 𝒒𝟑 + 𝒓𝟑 − 𝟑𝒑𝒒𝒓 = 𝟒 यदि 𝒂=𝒒+

value of √𝒙𝟑 + 𝒚𝟑 + 𝒛𝟑 − 𝟐 ? 𝒓, 𝒃 = 𝒓 + 𝒑 तथा 𝒄 = 𝒑 + 𝒒 है , तो 𝒂³ + 𝒃³ +


यदि 𝒙 + 𝒚 + 𝒛 = 𝟐, 𝒙𝒚 + 𝒚𝒛 + 𝒛𝒙 = −𝟏𝟏 𝒄³ − 𝟑𝒂𝒃𝒄 का मान क्या है ?
और 𝒙𝒚𝒛 = −𝟏𝟐 है , तो √𝒙𝟑 + 𝒚𝟑 + 𝒛𝟑 − 𝟐 a)4 b)8 c)2 d)12
का मान है :
1. 6 2. 12 3. 9 4. 8 33. If 𝒙³ + 𝒚³ + 𝒛³ = 𝟑(𝟏 + 𝒙𝒚𝒛), 𝑷 = 𝒚 + 𝒛 −
𝒙, 𝑸 = 𝒛 + 𝒙 − 𝒚 𝒂𝒏𝒅 𝑹 = 𝒙 + 𝒚 − 𝒛, then
30. If 𝒙 + 𝒚 + 𝒛 = 𝟔, 𝒙𝒚𝒛 = − 𝟏𝟎 𝒂𝒏𝒅 𝒙𝟐 + what is the value 𝑷³ + 𝑸³ + 𝑹𝟑 − 𝟑𝑷𝑸𝑹?
𝒚𝟐 + 𝒛𝟐 = 𝟑𝟎, then what is the value of यदि 𝒙³ + 𝒚³ + 𝒛³ = 𝟑(𝟏 + 𝒙𝒚𝒛), 𝑷 = 𝒚 + 𝒛 −
(𝒙𝟑 + 𝒚𝟑 + 𝒛𝟑 )? 𝒙, 𝑸 = 𝒛 + 𝒙 − 𝒚 तथा 𝑹 = 𝒙 + 𝒚 − 𝒛 है , तो
यदि 𝒙 + 𝒚 + 𝒛 = 𝟔, 𝒙𝒚𝒛 = − 𝟏𝟎 और 𝒙 + 𝟐
𝑷³ + 𝑸³ + 𝑹𝟑 − 𝟑𝑷𝑸𝑹 का मान क्या है ?
𝒚𝟐 + 𝒛𝟐 = 𝟑𝟎 है , तो (𝒙𝟑 + 𝒚𝟑 + 𝒛𝟑 ) का मान a)9 b)8 c)12 d)6
क्या होगा?
1. 132 2. 135 3. 130 4. 127 34. If 𝒂 + 𝒃 + 𝒄 = 𝟗, 𝒂𝒃 + 𝒃𝒄 + 𝒄𝒂 = 𝟐𝟔, 𝒂³ +
𝒃³ = 𝟗𝟏, 𝒃³ + 𝒄³ = 𝟕𝟐 𝒂𝒏𝒅 𝒄³ + 𝒂𝟑 = 𝟑𝟓,
FOR VIDEO SOLUTION FROM QUESTION 31 then what is the value of 𝒂𝒃𝒄?
TO 40 : CLICK HERE OR
यदि 𝒂 + 𝒃 + 𝒄 = 𝟗, 𝒂𝒃 + 𝒃𝒄 + 𝒄𝒂 =
SCAN Q R CODE GIVEN BELOW
𝟐𝟔, 𝒂³ + 𝒃³ = 𝟗𝟏, 𝒃³ + 𝒄³ = 𝟕𝟐 तथा 𝒄³ + 𝒂𝟑 =
𝟑𝟓 है , तो 𝒂𝒃𝒄 का मान क्या है ?
a)48 b)24 c)36 d)42 अगर a=25, b = 15, c = -10 है तो
𝒂𝟑 +𝒃𝟑 +𝒄𝟑 −𝟑𝒂𝒃𝒄
का मान
(𝒂−𝒃)𝟐 +(𝒃−𝒄)𝟐 +(𝒄−𝒂)𝟐
35. What is the value of
(𝟏.𝟐)𝟑 +(𝟎.𝟖)𝟑 +(𝟎.𝟕)𝟑 −𝟐.𝟎𝟏𝟔
a) 30 b) 15
(𝟏.𝟑𝟓)[(𝟏.𝟐)𝟐 +(𝟎.𝟖)𝟐 +(𝟎.𝟕)𝟐 −𝟎.𝟗𝟔−𝟎.𝟖𝟒−𝟎.𝟓𝟔] c) -30 d) 15
(𝟏.𝟐)𝟑 +(𝟎.𝟖)𝟑 +(𝟎.𝟕)𝟑 −𝟐.𝟎𝟏𝟔
का मान
(𝟏.𝟑𝟓)[(𝟏.𝟐)𝟐 +(𝟎.𝟖)𝟐 +(𝟎.𝟕)𝟐 −𝟎.𝟗𝟔−𝟎.𝟖𝟒−𝟎.𝟓𝟔]
40. If 𝒙𝒂 . 𝒙𝒃 . 𝒙𝒄 = 𝟏 then the value of 𝒂𝟑 + 𝒃𝟑 +
क्या है ?
𝒄𝟑 is
a) ¼ b) ½ c) 1 d) 2 अगर 𝒙𝒂 . 𝒙𝒃 . 𝒙𝒄 = 𝟏 है तो 𝒂𝟑 + 𝒃𝟑 + 𝒄𝟑 का
मान
36. If 𝒙 = 𝒚 = 𝟑𝟑𝟑 and 𝒛 = 𝟑𝟑𝟒, then the
a) 9 b) abc
values of 𝒙𝟑 + 𝒚𝟑 + 𝒛𝟑 − 𝟑𝒙𝒚𝒛 is
c) a + b + c d) 3abc
अगर 𝒙 = 𝒚 = 𝟑𝟑𝟑 और 𝒛 = 𝟑𝟑𝟒 है तो
𝒙𝟑 + 𝒚𝟑 + 𝒛𝟑 − 𝟑𝒙𝒚𝒛 = ? FOR VIDEO SOLUTION FROM QUESTION 41
TO 50 : CLICK HERE OR
a) 0 b) 667
SCAN Q R CODE GIVEN BELOW
c) 1000 d) 2334

37. If 𝒙 + 𝒚 + 𝒛 = 𝟐𝟐 and 𝒙𝒚 + 𝒚𝒛 + 𝒛𝒙 = 𝟑𝟓,


then what is the value of (𝒙 − 𝒚)² + (𝒚 −
𝒛)² + (𝒛 − 𝒙)²?
यदि 𝒙 + 𝒚 + 𝒛 = 𝟐𝟐 तथा 𝒙𝒚 + 𝒚𝒛 + 𝒛𝒙 = 𝟑𝟓
41. Given that (𝟓𝒙 − 𝟑)𝟑 + (𝟐𝒙 + 𝟓)𝟑 +
है , तो (𝒙 − 𝒚)² + (𝒚 − 𝒛)² + (𝒛 − 𝒙)² का मान
𝟐𝟕(𝟒 − 𝟑𝒙)𝟑 = 𝟗(𝟑 − 𝟓𝒙)(𝟐𝒙 + 𝟓)(𝟑𝒙 −
क्या है ? 𝟒), then the value of (2x+1) is:
a)793 b)681 c)758 d)715 यदि (𝟓𝒙 − 𝟑)𝟑 + (𝟐𝒙 + 𝟓)𝟑 + 𝟐𝟕(𝟒 −
𝟑𝒙)𝟑 = 𝟗(𝟑 − 𝟓𝒙)(𝟐𝒙 + 𝟓)(𝟑𝒙 − 𝟒),है , तो
38. If a = 299, b = 298, c = 297 then the value of (2x+1) का मान क्या होगा?
𝟐𝒂𝟑 + 𝟐𝒃𝟑 + 𝟐𝒄𝟑 − 𝟔𝒂𝒃𝒄 is 1. -13 2. 15 3. -15 4. 13
यदि a = 299, b = 298, c = 297 हो, तो 𝟐𝒂𝟑 +
𝟐𝒃𝟑 + 𝟐𝒄𝟑 − 𝟔𝒂𝒃𝒄 का मान बताएं ? 42. Out of the given responses, one of the
𝟑 𝟑
a) 5154 b) 5267 factors of (𝒂𝟐 − 𝒃𝟐 ) + (𝒃𝟐 − 𝒄𝟐 ) +
c) 5364 d) 5456 𝟑
(𝒄𝟐 − 𝒂𝟐 ) is
𝟑 𝟑 𝟑
(𝒂𝟐 − 𝒃𝟐 ) + (𝒃𝟐 − 𝒄𝟐 ) + (𝒄𝟐 − 𝒂𝟐 ) का
39. If a=25, b = 15, c = -10, then the value of
𝒂𝟑 +𝒃𝟑 +𝒄𝟑 −𝟑𝒂𝒃𝒄 गुणनिंड है |
(𝒂−𝒃)𝟐 +(𝒃−𝒄)𝟐 +(𝒄−𝒂)𝟐
is
a) (a + b) (a – b)
b) (a + b) (a + b)
c) (a - b) (a – b)
d) (b – c) (b – c) अगर a,b और c तीन गैर शून्य वास्तववक संख्याएं
इस प्रकार हैं कक 𝒂 + 𝒃 + 𝒄 = 𝟎 और 𝒃𝟐 ≠ 𝒄𝒂 है
𝟑 𝟑 𝟑
43. If 𝒙 = 𝒚 + 𝒛 then 𝒙 − 𝒚 − 𝒛 is 𝒂𝟐 +𝒃𝟐 +𝒄𝟐
तो होगा
यदि 𝒙 = 𝒚 + 𝒛 हो तो 𝒙 − 𝒚 − 𝒛 का मान 𝟑 𝟑 𝟑 𝒃𝟐 −𝒄𝒂
a) 3 b) 2 c) 0 d) 1
बताइए l
a) 0 b) 𝟑𝒙𝒚𝒛
48. If 𝒙 + 𝒚 + 𝒛 = 𝟎, then what is the value of
c) – 𝟑𝒙𝒚𝒛 d) 1
((𝟑𝒚𝟐 +𝒙𝟐 +𝒛𝟐 ))
?
((𝟐𝒚𝟐 −𝒙𝒛))
44. If 𝒂𝟐 + 𝒃𝟐 + 𝒄𝟐 = 𝒂𝒃 + 𝒃𝒄 + 𝒂𝒄 then the ((𝟑𝒚𝟐 +𝒙𝟐 +𝒛𝟐 ))
𝒂+𝒄 यदि 𝒙 + 𝒚 + 𝒛 = 𝟎 है ,तो का मान
value of 𝒃 is ((𝟐𝒚𝟐 −𝒙𝒛))

अगर 𝒂 + 𝒃 + 𝒄𝟐 = 𝒂𝒃 + 𝒃𝒄 + 𝒂𝒄 है
𝟐 𝟐
तो क्या है ?

a)2 b)1
𝒂+𝒄 𝟑 𝟓
=? c)
𝟐
d)
𝟑
𝒃
a) 0 b) 2
c) 1 d) -1 49. If 𝒂 + 𝒃 + 𝒄 = 𝟎 then the value of
𝟏 𝟏 𝟏
(𝒂+𝒃)(𝒃+𝒄)
+ (𝒃+𝒄)(𝒄+𝒂) + (𝒄+𝒂)(𝒂+𝒃) is
𝟐 𝟐 𝟐
45. If 𝒙 + 𝒚 + 𝒛 = 𝒙𝒚 + 𝒚𝒛 + 𝒙𝒛 then the 𝟏
यदि 𝒂 + 𝒃 + 𝒄 = 𝟎 तो +
𝟑𝒙𝟒 +𝟕𝒚𝟒 +𝟓𝒛𝟒 (𝒂+𝒃)(𝒃+𝒄)
value of 𝟓𝒙𝟐𝒚𝟐+𝟕𝒚𝟐𝒛𝟐+𝟑𝒛𝟐𝒙𝟐 is 𝟏 𝟏
+ (𝒄+𝒂)(𝒂+𝒃) का मान ननकालें
अगर 𝒙𝟐 + 𝒚𝟐 + 𝒛𝟐 = 𝒙𝒚 + 𝒚𝒛 + 𝒙𝒛 है तो
(𝒃+𝒄)(𝒄+𝒂)
a) 0 b) 1 c) 3 d) 2
𝟑𝒙𝟒 +𝟕𝒚𝟒 +𝟓𝒛𝟒
= ?
𝟓𝒙 𝒚𝟐 +𝟕𝒚𝟐 𝒛𝟐 +𝟑𝒛𝟐 𝒙𝟐
𝟐

a) 2 b) 1 50. If ab + bc + ac = 0 then the value of


𝟏 𝟏 𝟏
c) 0 d) – 1 (𝒂𝟐−𝒃𝒄 + 𝒃𝟐 −𝒂𝒄 + 𝒄𝟐−𝒂𝒃) is
𝟏 𝟏
अगर ab + bc + ac = 0 है तो ( + 𝟐 +
𝟐 𝟐 𝟐
46. If 𝟑 (𝒂 + 𝒃 + 𝒄 ) = (𝒂 + 𝒃 + 𝒄) , then the 𝟐 𝒂𝟐 −𝒃𝒄 𝒃 −𝒂𝒄
𝟏
relation between a, b and c is 𝒄𝟐 −𝒂𝒃
)= ?
अगर 𝟑 (𝒂𝟐 + 𝒃𝟐 + 𝒄𝟐 ) = (𝒂 + 𝒃 + 𝒄)𝟐 है तो a) 0 b) 1
a,b और c में क्या सम्बन्ध है ? c) 3 d) a + b + c

a) 𝒂 ≠ 𝒃 ≠ 𝒄 b) 𝒂 = 𝒃 ≠ 𝒄 FOR VIDEO SOLUTION FROM QUESTION 51


c) 𝒂 ≠ 𝒃 = 𝒄 d) 𝒂 = 𝒃 = 𝒄 TO 60 : CLICK HERE OR
SCAN Q R CODE GIVEN BELOW
47. If a, b, c are three non-zero real numbers
such that 𝒂 + 𝒃 + 𝒄 = 𝟎, and 𝒃𝟐 ≠ 𝒄𝒂, then
𝒂𝟐 +𝒃𝟐 +𝒄𝟐
the value of 𝒃𝟐 −𝒄𝒂
is
𝟏 𝟏 𝟑(𝐱 𝟐 +𝟏)−𝟕𝐱
51. If 𝒙 + = 𝟏, then the value of 64𝒙𝟑 + 56. If = 𝟔, 𝐱 ≠ 𝟎, then the value of
𝟏𝟔𝒙 𝟔𝟒𝒙𝟑 𝟑𝐱
is 𝟏
√𝒙 + 𝒙 is:

𝟏 𝟏
अगर 𝒙 + = 𝟏, है तो 64𝒙𝟑 + 𝟔𝟒𝒙𝟑 का मान 𝟑(𝐱 𝟐 +𝟏)−𝟕𝐱 𝟏
यदि = 𝟔, 𝐱 ≠ 𝟎 है , तो √𝒙 + 𝒙 का
𝟏𝟔𝒙
a) 4 b) 52 𝟑𝐱 √

c) 64 d) 76 मान क्या होगा?


𝟏
52. If 𝒙 + 𝟏𝟔𝒙
= 𝟑, then the value of 𝟏𝟔𝒙𝟑 + 1. √
𝟐𝟓
2. √
𝟏𝟏
3. √
𝟑𝟓
4. √
𝟑𝟏
𝟏 𝟑 𝟑 𝟑 𝟑
𝟐𝟓𝟔𝒙𝟑
is:
𝟏 𝟏
यदि 𝒙 + = 𝟑 है , तो 𝟏𝟔𝒙𝟑 + का मान 57. If 𝒙² − 𝟏𝟐𝒙 + 𝟑𝟑 = 𝟎, then what is the
𝟏𝟔𝒙 𝟐𝟓𝟔𝒙𝟑
𝟏
होगा:
𝟐
valueof (𝒙 − 𝟒) + [( ]?
𝒙−𝟒)𝟐
1. 423 2. 441 3. 432 4. 414 यदि 𝒙² − 𝟏𝟐𝒙 + 𝟑𝟑 = 𝟎 है , तो (𝒙 − 𝟒)² +
[(𝒙−𝟒)𝟐] का मान क्या है ?
𝟏

𝟐 𝟏 𝟑
53. If 𝒙 + = 𝟗𝟖 (𝒙 > 𝟎)then the value of 𝒙
𝒙𝟐 a)16 b)14 c)18 d)20
𝟏
+ 𝒙𝟑
is
𝟏 𝟏
यदि 𝒙𝟐 + = 𝟗𝟖 (𝒙 > 𝟎) हो तो 𝒙𝟑 + का 58. If 𝒙² − 𝟏𝟔𝒙 − 𝟓𝟗 = 𝟎, then what is the value
𝒙𝟐 𝒙𝟑
𝟏
मान बताइए? of (𝒙 − 𝟔)² + [(𝒙−𝟔)𝟐] ?
a) 970 b) 1030 यदि 𝒙² − 𝟏𝟔𝒙 − 𝟓𝟗 = 𝟎 है , तो (𝒙 − 𝟔)² +
c) -970 d) -1030 ] का मान क्या है ?
𝟏
[(
𝒙−𝟔)𝟐
a)14 b)18 c)16 d)20
54. If 𝒙𝟖 − 𝟏𝟒𝟒𝟐𝒙𝟒 + 𝟏 = 𝟎, then a possible
𝟏
value of 𝒙 − 𝒙 is:
59. If 𝒙² − 𝟒𝒙 + 𝟏 = 𝟎 then what is the value of
𝟏
यदि 𝒙𝟖 − 𝟏𝟒𝟒𝟐𝒙𝟒 + 𝟏 = 𝟎, तो 𝒙 − 𝒙 का 𝒙𝟗 + 𝒙𝟕 − 𝟏𝟗𝟒𝒙𝟓 − 𝟏𝟗𝟒𝒙𝟑 ?
संभाववत मान है : यदि 𝒙² − 𝟒𝒙 + 𝟏 = 𝟎 हो, तो 𝒙𝟗 + 𝒙𝟕 −
1. 5 2. 8 3. 4 4. 6 𝟏𝟗𝟒𝒙𝟓 − 𝟏𝟗𝟒𝒙𝟑 का मान क्या है ?
a)4 b)-4 c)1 d)-1
𝟏 √𝟑+𝟏
55. If 𝒙 + (𝒙) = 𝟐
, then what is the value of
𝟒 𝟏 √𝟏𝟑+√𝟏𝟏 𝟏
𝒙 + (𝒙𝟒)? 60. Let 𝒙 = and 𝒚 = 𝒙, then the value
√𝟏𝟑−√𝟏𝟏
𝟏
यदि 𝒙 + ( ) = है , तो 𝒙𝟒 + ( 𝟒 )का मान
𝟏 √𝟑+𝟏
of 𝟑𝒙𝟐 − 𝟓𝒙𝒚 + 𝟑𝒚𝟐 is
𝒙 𝟐 𝒙
𝟏
क्या है ? अगर 𝒙 = और 𝒚 = है तो 𝟑𝒙𝟐 −
√𝟏𝟑+√𝟏𝟏
√𝟏𝟑−√𝟏𝟏 𝒙

a)
𝟒√𝟑−𝟏
b)
𝟒√𝟑+𝟏
c)
−𝟒√𝟑−𝟏
d)
−𝟒√𝟑−𝟏
𝟓𝒙𝒚 + 𝟑𝒚𝟐 = ?
𝟒 𝟐 𝟒 𝟐
a) 1717 b) 1177
c) 1771 d) 1171
FOR VIDEO SOLUTION FROM QUESTION 61 1. √𝟐 + √𝟓 + 𝟑
TO 70 : CLICK HERE OR 2. (√𝟐 + √𝟓 + √𝟑)(𝟐 − √𝟏𝟎)
SCAN Q R CODE GIVEN BELOW 3. (√𝟐 + √𝟓 + √𝟑)(𝟐 + √𝟓)
4. (√𝟐 + √𝟓 + √𝟑)(𝟐 − √𝟓)

𝟐√𝟏𝟎 √𝟓−𝟐 𝟑
66. The value of −√ − is:
√𝟓+√𝟐−√𝟕 √𝟓+𝟐 √𝟕−𝟐

𝟐√𝟏𝟎 √𝟓−𝟐 𝟑
61. If (𝒙 − 𝒂)(𝒙 − 𝒃) = 𝟏 and 𝒂 − 𝒃 + 𝟓 = 𝟎, −√ − का मान क्या है ?
√𝟓+√𝟐−√𝟕 √𝟓+𝟐 √𝟕−𝟐
𝟏
then the value of (𝒙 − 𝒂)𝟑 − (𝒙−𝒂)𝟑
is 1. 2 + √2 2. 2√5 3. √2 4. √7

अगर (𝒙 − 𝒂)(𝒙 − 𝒃) = 𝟏 और 𝒂 − 𝒃 + 𝟓 = 𝟎
𝟏 67. If √𝟏𝟎 − 𝟐√𝟐𝟏 + √𝟖 + 𝟐√𝟏𝟓 = √𝒂 + √𝒃,
है तो (𝒙 − 𝒂)𝟑 − (𝒙−𝒂)𝟑 = ?
where a and b are positive integers, then the
a) – 125 b) 1 value of √𝒂𝒃 is closet to:
c) 125 d) 140
यदि √𝟏𝟎 − 𝟐√𝟐𝟏 + √𝟖 + 𝟐√𝟏𝟓 = √𝒂 +

62. What is the value of √𝒃, में a और b धनात्मक पूणाांक है , तो √𝒃 का


(𝟐 + √𝟐) + (
𝟏
)+(
𝟏
) + (𝟐 − √𝟐)? ननकटतम मान है :
𝟐+√𝟐 𝟐−√𝟐
𝟏 𝟏 1. 4.6 2. 5.9 3. 6.8 4. 7.2
(2 +√𝟐) + ( )+( ) + (𝟐 − √𝟐) का
𝟐+√𝟐 𝟐−√𝟐
मान क्या है ?
68. The value of √𝟐𝟖 + 𝟏𝟎√𝟑 − √𝟕 − 𝟒√𝟑 is
a)2 b)4 c)8 d)6
closed to:

𝟑√𝟐 𝟒√𝟑 √𝟔
√𝟐𝟖 + 𝟏𝟎√𝟑 − √𝟕 − 𝟒√𝟑 का मान
63. The value of { − + } is
√𝟑+√𝟔 √𝟔+√𝟐 √𝟐+√𝟑 ननमंललखित में से ककसके सबसे अधधक ननकट है ?
a) √𝟐 b) 0
1. 7.2 2. 6.1 3. 6.5 4. 5.8
c) √𝟑 d) √𝟔

𝟏 𝟏 𝟏 𝟏 69. If √𝟖𝟔 − 𝟔𝟎√𝟐 = 𝒂 − 𝒃√𝟐, then what will


64. The value of 𝟏+√𝟐 + + + +
√𝟐+√𝟑 √𝟑+√𝟒 √𝟒+√𝟓 be the value of √𝒂𝟐 + 𝒃𝟐 , correct to one
𝟏 𝟏 𝟏 𝟏
+ + + decimal place?
√𝟓+√𝟔 √𝟔+√𝟕 √𝟕+√𝟖 √𝟖+√𝟗
a) 2 b) 0 यदि √𝟖𝟔 − 𝟔𝟎√𝟐 = 𝒂 − 𝒃√𝟐 है , तो
c) 4 d) 1
√𝒂𝟐 + 𝒃𝟐 का एक िशमलव स्थान तक सही मान

65. If (√𝟐 + √𝟓 − √𝟑) × 𝒌 = −𝟏𝟐, then what क्या होगा?


will be the value of k? 1. 8.4 2. 8.2 3. 7.8 4. 7.2
यदि √𝟐 + √𝟓 − √𝟑) × 𝒌 = −𝟏𝟐, तो K का
मान क्या होगा?
√𝟑 a) 2 b) 5
70. If 𝒙 = √𝟏 + − √𝟏 − √𝟑, then the value of
𝟐 𝟐 c) 0 d) 1
√𝟐−𝒙
will be closest to: यदि 𝒙 = √𝟏 +
√𝟑
𝟐

√𝟐+𝒙 74. The real value of x, that satisfies the
√𝟏 − √𝟑
है , तो
√𝟐−𝒙
का मान ककसके ननकटतम equation √𝟒𝒙 − 𝟗 + √𝟒𝒙 + 𝟗 = 𝟓 + √𝟕 is
𝟐 √𝟐+𝒙
समीकरण √𝟒𝒙 − 𝟗 + √𝟒𝒙 + 𝟗 = 𝟓 + √𝟕 को
होगा?
संतुष्ट करने वाला x का वास्तववक संख्या कौनसी
1. 0.17 2. 0.12
3. 1.4 4. 1.2 है ?
𝟑
a) b) 4
FOR VIDEO SOLUTION FROM QUESTION 71 √𝟕

TO 80 : CLICK HERE OR c) √𝟓 d) 𝟐√𝟑


SCAN Q R CODE GIVEN BELOW
𝟏 𝟏
𝒑−𝟏 𝒒𝟐 𝟐 𝒑𝟔 𝒒−𝟑 𝟐
75. If (𝒑𝟑𝒒−𝟐) ÷ (𝒑−𝟐𝒒𝟑) = 𝒑𝒂 𝒒𝒃 , then the
value of 𝒂 + 𝒃, where p and q are different
positive primes, is
𝟏 𝟏
𝒑−𝟏 𝒒𝟐 𝟐 𝒑𝟔 𝒒−𝟑 𝟐
अगर (𝒑𝟑𝒒−𝟐) ÷ (𝒑−𝟐𝒒𝟑) = 𝒑𝒂 𝒒𝒃 अदह तो
71. The expression √𝟏𝟎 + 𝟐(√𝟔 − √𝟏𝟓 − √𝟏𝟎)
𝒂 + 𝒃 का मान पता करो| 𝒑 और 𝒒 िो लभन्न
is equal to :
धनात्मक अभाज्य संख्याएं हैं|
व्यंजक √𝟏𝟎 + 𝟐(√𝟔 − √𝟏𝟓 − √𝟏𝟎) a) 1 b) 0
ननम्नललखित में से ककसके बराबर है ? c) −𝟏 d) 2

1. √𝟑 + √𝟐 − √𝟓 2. √𝟑 − √𝟐 − √𝟓
76. If 𝒂³ + 𝟑𝒂² + 𝟗𝒂 = 𝟏, then what is the value
3. √𝟑 − √𝟐 + √𝟓 4. √𝟐 − √𝟑 − √𝟓 𝟑
of 𝒂³ + (𝒂) ?
यदि 𝒂³ + 𝟑𝒂² + 𝟗𝒂 = 𝟏 हो, तो 𝒂³ + ( )का मान
𝟑
√𝟑 𝟏+𝒙 𝟏−𝒙 𝒂
72. If x = , then the value of + 𝟏−√𝟏−𝒙 is
क्या है ?
𝟐 𝟏+√𝟏+𝒙
equal to
𝟏+𝒙 𝟏−𝒙 a)31 b)26 c)28 d)24
अगर x = है तो क्या होगा?
√𝟑
+
𝟐 𝟏+√𝟏+𝒙 𝟏−√𝟏−𝒙
√𝟑 𝟏 𝟏 𝟏
a) 𝟐
b) √𝟑 77. If 𝒂 − 𝒂 = 𝒃, 𝒃 − 𝒃 = 𝒄 𝒂𝒏𝒅 𝒄 − 𝒄 = 𝒂, then
c) 0 d) 1 𝟏 𝟏 𝟏
what is the value of (𝒂𝒃) + (𝒃𝒄) + (𝒄𝒂) ?
यदि 𝒂 − (𝒂) = 𝒃, 𝒃 − (𝒃) = 𝒄 तथा 𝒄 − ( 𝒄 ) =
𝟏 𝟏 𝟏
√𝟓+𝟏 𝟐
73. If x = √ , then 𝒙 − 𝒙 − 𝟏 is equal to
𝒂 है , तो (𝒂𝒃) + (𝒃𝒄) + (𝒄𝒂) का मान क्या है ?
√𝟓−𝟏 𝟏 𝟏 𝟏

√𝟓+𝟏
अगर x = √ है तो 𝒙𝟐 − 𝒙 − 𝟏 ककतना a)-3 b)-6 c)-1 d)-9
√𝟓−𝟏

होगा?
78. If 𝒂 + 𝒃 = 𝟏, find the value of 𝒂𝟑 + 𝒃𝟑 − 81. If 𝒂 + 𝒂² + 𝒂³ − 𝟏 = 𝟎, then what is the
𝟏
𝟐 𝟐 value of 𝒂³ + (𝒂) ?
𝒂𝒃 − (𝒂𝟐 − 𝒃 )
अगर 𝒂 + 𝒃 = 𝟏 है तो यदि 𝒂 + 𝒂² + 𝒂³ − 𝟏 = 𝟎 हो, तो 𝒂³ + ( ) का
𝟏
𝒂𝟑 + 𝒃𝟑 − 𝒂𝒃 − 𝒂
𝟐 𝟐 मान क्या है ?
(𝒂 − 𝒃 ) पता करो|
𝟐

a) – 1 b) 1 a)1 b)4 c)2 d)3


c) 0 d) 2
82. If 𝒙𝒚+𝒛 = 𝟏, 𝒚𝒙+𝒛 = 𝟏𝟎𝟐𝟒 𝒂𝒏𝒅 𝒛𝒙+𝒚 = 𝟕𝟐𝟗
𝟏 𝟏
79. If [𝒂 + ( )] ²
𝒂
− 𝟐 [𝒂 − ( )]
𝒂
= 𝟏𝟐, then (x,y z are natural numbers,) then what is the
which of the following is a value of a? value of (z+1)𝒚+𝒙+𝟏 ?
यदि = 𝟏𝟐 हो, तो
𝟏 𝟏
[𝒂 + (𝒂)] ² − 𝟐 [𝒂 − (𝒂)] यदि 𝒙𝒚+𝒛 = 𝟏, 𝒚𝒙+𝒛 = 𝟏𝟎𝟐𝟒 𝒂𝒏𝒅 𝒛𝒙+𝒚 =
ननम्नललखित में से कोन सा a का मान है ? 𝟕𝟐𝟗 (x,y तथा z प्राकृनतक संख्याओं है ,)
a)-8+√3 (z+1)𝒚+𝒙+𝟏 का मान क्या है ?
b)-8-√3 a)6561 b)10000
c)-8+√5
c)4096 d)14641
d)none of these

𝟏 𝟏
√𝟑 83. If a + 𝒃 = 1 and b + 𝒄 = 1, then the value of c
80. √(𝟏 − 𝒑𝟐 )(𝟏 − 𝒒𝟐 ) = 𝟐
, then what is the
𝟏
+ is
value of √𝟐𝒑𝟐 + 𝟐𝒒𝟐 + 𝟐𝒑𝒒 + 𝒂
𝟏 𝟏 𝟏
√𝟐𝒑𝟐 + 𝟐𝒒𝟐 − 𝟐𝒑𝒒? यदि a + = 1 और b + = 1 हो, तो c + का
𝒃 𝒄 𝒂

यदि √(𝟏 − 𝒑𝟐 )(𝟏 − 𝒒𝟐 ) =


√𝟑
है मान क्या होगा?
𝟐
a) 0 b) 2
तो√𝟐𝒑𝟐 + 𝟐𝒒𝟐 + 𝟐𝒑𝒒 + √𝟐𝒑𝟐 + 𝟐𝒒𝟐 − 𝟐𝒑𝒒
c) 1 d) 3
का मान क्या है ? 𝟏 𝟏 𝟏
84. If 𝒂 + 𝒃 = 𝒃 + 𝒄 = 𝒄 + 𝒂, where 𝒂 ≠ 𝒃 ≠ 𝒄 ≠
a)2 b)√𝟐
𝟎, then the value of 𝒂𝟐 𝒃𝟐 𝒄𝟐 is
c)1 d) none of these 𝟏 𝟏 𝟏
अगर 𝒂 + = 𝒃 + 𝒄 = 𝒄 + 𝒂 है तो 𝒂𝟐 𝒃𝟐 𝒄𝟐 का
𝒃

FOR VIDEO SOLUTION FROM QUESTION 81 मान पता करो जहा 𝒂 ≠ 𝒃 ≠ 𝒄 ≠ 𝟎 है |


TO 90 : CLICK HERE OR a) – 1 b) abc
SCAN Q R CODE GIVEN BELOW c) 0 d) 1

𝟏 𝟏 𝟏
85. If a + 𝒃
= b+ 𝒄
=c+ 𝒂
(where 𝒂 ≠ 𝒃 ≠ 𝒄),
then 𝒂𝒃𝒄 is equal to
𝟏 𝟏 𝟏
यदि a + = b + 𝒄 = c + 𝒂 (जहााँ 𝒂 ≠ 𝒃 ≠ 𝒄), तो
𝒃
𝒂𝒃𝒄 ककसके बराबर है ?
𝟏 𝟏 𝟏 𝟏
a) +1 b) – 1
90. If 𝒙 = 𝒂𝟐 + 𝒂−𝟐 , y = 𝒂𝟐 − 𝒂−𝟐 then value of
c) +1 & – 1 d) None of the options
(𝒙𝟒 − 𝒙𝟐 𝒚𝟐 − 𝟏) + (𝒚𝟒 − 𝒙𝟐 𝒚𝟐 + 𝟏) is
𝟏 𝟏 𝟏 𝟏
अगर 𝒙 = 𝒂𝟐 + 𝒂 , y = 𝒂𝟐 − 𝒂−𝟐 है तो (𝒙𝟒 −

𝟐
86. 𝒂𝒃(𝒂 − 𝒃) + 𝒃𝒄(𝒃 − 𝒄) + 𝒄𝒂(𝒄 − 𝒂) is equal
to: 𝒙𝟐 𝒚𝟐 − 𝟏) + (𝒚𝟒 − 𝒙𝟐 𝒚𝟐 + 𝟏) का मान पता
𝒂𝒃(𝒂 − 𝒃) + 𝒃𝒄(𝒃 − 𝒄) + 𝒄𝒂(𝒄 − 𝒂) करो|
ननमंललखित में से ककसके बराबर है ? a) 16 b) 13
a)(𝐚 + 𝐛)(𝐛 − 𝐜)(𝐜 − 𝐚) c) 12 d) 14
b) (𝐚 − 𝐛)(𝐛 + 𝐜)(𝐜 − 𝐚)
c) (𝐚 − 𝐛)(𝐛 − 𝐜)(𝐜 − 𝐚) FOR VIDEO SOLUTION FROM QUESTION 91
d) (𝐛 − 𝐚)(𝐛 − 𝐜)(𝐜 − 𝐚) TO 100 : CLICK HERE OR
SCAN Q R CODE GIVEN BELOW
87. If 𝑿𝟏 𝑿𝟐 𝑿𝟑 = 𝟒(𝟒 + 𝒙𝟏 + 𝒙𝟐 + 𝒙𝟑 ), then
𝟏 𝟏
what is the value of((𝟐+𝑿 ) + (𝟐+𝑿 ) +
𝟏 𝟐

𝟏
(𝟐+𝑿 )) ?
𝟑

यदि 𝑿𝟏 𝑿𝟐 𝑿𝟑 = 𝟒(𝟒 + 𝒙𝟏 + 𝒙𝟐 + 𝒙𝟑 )हो, तो 91. If 𝒙𝟒 + 𝟐𝒙𝟑 + 𝒂𝒙𝟐 + 𝒃𝒙 + 𝟗 is a perfect


square, where a and b are positive real
𝟏 𝟏 𝟏
((𝟐+𝑿 ) + (𝟐+𝑿 ) + (𝟐+𝑿 )) का मान क्या है ? numbers, then the value of a and b are
𝟏 𝟐 𝟑

𝟏 यदि 𝒙𝟒 + 𝟐𝒙𝟑 + 𝒂𝒙𝟐 + 𝒃𝒙 + 𝟗 एक सम्पण


ू ण
a) 1 c) 2
𝟏
b) d) 𝟑
𝟐
वगण है जहााँ a तथा b धनात्मक वास्तववक संख्या है

88. If 𝟐𝒔 = 𝒂 + 𝒃 + 𝒄, then the value तो a और b का मान क्या होगा ?


𝒔(𝒔 – 𝒄) + (𝒔 – 𝒂) (𝒔 – 𝒃) is a) a = 5, b = 6
अगर 𝟐𝒔 = 𝒂 + 𝒃 + 𝒄 है तो b) a = 6, b = 7
c) a = 7, b = 6
𝒔(𝒔 – 𝒄) + (𝒔 – 𝒂) (𝒔 – 𝒃) =?
a) ab b) abc d) a = 7, b = 8
𝒂+𝒃+𝒄
c) 0 d) 𝟐 92. If 𝒂𝟐 + 𝒃𝟐 + 𝒄𝟐 = 𝟏𝟔, 𝒙𝟐 + 𝒚𝟐 + 𝒛𝟐 = 𝟐𝟓
and 𝒂𝒙 + 𝒃𝒚 + 𝒄𝒛 = 𝟐𝟎, then the value
89. If 𝒑 + 𝒎 = 𝟔 and 𝒑𝟑 + 𝒎𝟑 = 𝟕𝟐, then the 𝒂+𝒃+𝒄
of 𝒙+𝒚+𝒛
value of 𝒑𝒎 is
अगर 𝒑 + 𝒎 = 𝟔 और 𝒑𝟑 + 𝒎𝟑 = 𝟕𝟐 है तो यदि 𝒂𝟐 + 𝒃𝟐 + 𝒄𝟐 = 𝟏𝟔, 𝒙𝟐 + 𝒚𝟐 + 𝒛𝟐 = 𝟐𝟓
𝒂+𝒃+𝒄
𝒑𝒎 का मान क्या होगा? और 𝒂𝒙 + 𝒃𝒚 + 𝒄𝒛 = 𝟐𝟎 हो, तो का
𝒙+𝒚+𝒛
a) 6 b) 12 मान ननकालें ?
c) 9 d) 8 𝟑 𝟓 𝟒 𝟓
a) 𝟓 b) 𝟑 c) 𝟓 d) 𝟒
93. The value of 𝒙 which satisfies the equation
𝒙+𝒂𝟐 +𝟐𝒄𝟐 𝒙+𝒃𝟐 +𝟐𝒂𝟐 𝒙+𝒄𝟐 +𝟐𝒃𝟐
+ + = 0 is 97. If 3√
𝟏−𝒂
+ 𝟗 = 𝟏𝟗 − 𝟑√
𝒂
, then what is
𝒃+𝒄 𝒄+𝒂 𝒂+𝒃 𝒂 𝟏−𝒂
𝒙 का मान बताइए जो लभन्न the value of A?
𝒙+𝒂𝟐 +𝟐𝒄𝟐 𝒙+𝒃𝟐 +𝟐𝒂𝟐 𝒙+𝒄𝟐 +𝟐𝒃𝟐
+ + = 0 को 𝟏−𝒂 𝒂
𝒃+𝒄 𝒄+𝒂 𝒂+𝒃 यदि 3√ + 𝟗 = 𝟏𝟗 − 𝟑√𝟏−𝒂 है , तो A का
𝒂
संतुष्ट करता है l
मान क्या है ?
a) (𝒂𝟐 + 𝒃𝟐 + 𝒄𝟐 ) 𝟑 𝟕 𝟏 𝟗 𝟐 𝟑 𝟏 𝟒
b) −(𝒂𝟐 + 𝒃𝟐 + 𝒄𝟐 ) a)𝟏𝟎 , 𝟏𝟎 b)𝟏𝟎 , 𝟏𝟎 c)𝟓 , 𝟓 d)𝟓 , 𝟓
c) (𝒂𝟐 + 𝟐𝒃𝟐 + 𝒄𝟐 )
d) −(𝒂𝟐 + 𝒃𝟐 + 𝟐𝒄𝟐 ) 98. If a + b = 1, then 𝒂𝟒 + 𝒃𝟒 − 𝒂𝟑 − 𝒃𝟑 −
𝟐𝒂𝟐 𝒃𝟐 + 𝒂𝒃 is equal to
94. If x, y, z are the three factors of 𝒂𝟑 − 𝟕𝒂 − 𝟔, यदि a + b = 1 है , तो 𝒂𝟒 + 𝒃𝟒 − 𝒂𝟑 − 𝒃𝟑 −
then the value of x + y + z will be 𝟐𝒂𝟐 𝒃𝟐 + 𝒂𝒃 ककसके बराबर होगा ?
यदि 𝒂 − 𝟕𝒂 − 𝟔 के तीन गण
𝟑
ु निंड x, y, z हैं, तो a) 1 b) 2
x + y + z का मान होगा ? c) 4 d) 0
a) 3a b) 3
c) 6 d) a 99. If √𝐚𝟐 + 𝐛 𝟐 + 𝐚𝐛 + √𝐚𝟐 + 𝐛 𝟐 − 𝐚𝐛 = 𝟏,
then what is the value of (𝟏 − 𝒂²)(𝟏 − 𝒃²)?
95. If 𝒂 + 𝒃 + 𝒄 + 𝒅 = 𝟒 then the value of यदि √𝐚𝟐 + 𝐛 𝟐 + 𝐚𝐛 + √𝐚𝟐 + 𝐛 𝟐 − 𝐚𝐛 = 𝟏
𝟏 𝟏
+ + हो, तो (𝟏 − 𝒂²)(𝟏 − 𝒃²) का मान क्या है ?
(𝟏−𝒂)(𝟏−𝒃)(𝟏−𝒄) (𝟏−𝒃)(𝟏−𝒄)(𝟏−𝒅)
𝟏 𝟏 𝟏 𝟒 𝟓 𝟑
(𝟏−𝒂)(𝟏−𝒄)(𝟏−𝒅)
+ (𝟏−𝒂)(𝟏−𝒃)(𝟏−𝒅) is a) 𝟒 b) 𝟕 c) 𝟒 d) 𝟒
यदि 𝒂 + 𝒃 + 𝒄 + 𝒅 = 𝟒 हो तो
𝟏 𝟏 𝒙−𝒂𝟐 𝒙−𝒃𝟐 𝒙−𝒄𝟐
+ + 100. If 𝒃+𝒄
+ 𝒄+𝒂
+ 𝒂+𝒃
= 4 (a + b + c), then
(𝟏−𝒂)(𝟏−𝒃)(𝟏−𝒄) (𝟏−𝒃)(𝟏−𝒄)(𝟏−𝒅)
𝟏 𝟏
+ (𝟏−𝒂)(𝟏−𝒃)(𝟏−𝒅) का मान
x is equal to
(𝟏−𝒂)(𝟏−𝒄)(𝟏−𝒅) 𝒙−𝒂𝟐 𝒙−𝒃𝟐 𝒙−𝒄𝟐
अगर + + = 4 (a + b + c) है तो
बताइए l 𝒃+𝒄 𝒄+𝒂 𝒂+𝒃

a) 0 b) 1 x = ?
c) 4 d) 1 + 𝒂𝒃𝒄𝒅 a) (𝒂 + 𝒃 + 𝒄)𝟐
b) 𝒂𝟐 + 𝒃𝟐 + 𝒄𝟐
96. A and B are positive integers. If 𝑨 + 𝑩 + c) ab + bc + ac
𝑨𝑩 = 𝟔𝟓, then what is the difference d) 𝒂𝟐 + 𝒃𝟐 + 𝒄𝟐 − 𝒂𝒃 − 𝒃𝒄 − 𝒂𝒄
between A and 𝑩(𝑨, 𝑩 ≤ 𝟏𝟓)?
FOR VIDEO SOLUTION FROM QUESTION 101
A तथा B धनात्मक पूणाणक है यदि 𝑨+𝑩+
TO 110 : CLICK HERE OR
𝑨𝑩 = 𝟔𝟓 है , तो A तथा B के मध्य अंतर क्या है SCAN Q R CODE GIVEN BELOW
(𝑨, 𝑩 ≤ 𝟏𝟓)?
a)3 b)4 c)5 d)6
105. If 𝒙 = 𝟐 + √𝟑, 𝒚 = 𝟐 − √𝟑 𝒂𝒏𝒅 𝒛 =
𝒙 𝒚
𝟏then what is the value of ( ) + ( ) +
𝒚𝒛 𝒙𝒛
𝒛 𝟏 𝟏 𝟏
(𝒙𝒚) + 𝟐 [(𝒙) + (𝒚) + ( 𝒛)] ?
यदि 𝒙 = 𝟐 + √𝟑, 𝒚 = 𝟐 − √𝟑तथा𝒛 = 𝟏,है , तो
101. If 𝒙³ − 𝟒𝒙² + 𝟏𝟗 = 𝟔(𝒙 − 𝟏), then what is 𝒙 𝒚 𝒛 𝟏 𝟏 𝟏
𝟏
( ) + ( ) + ( ) + 𝟐 [( ) + ( ) + ( )] ?का
𝟐 𝒚𝒛 𝒙𝒛 𝒙𝒚 𝒙 𝒚 𝒛
the value of [𝒙 + ]?
मान क्या है ?
𝒙−𝟒
यदि 𝒙³ − 𝟒𝒙² + 𝟏𝟗 = 𝟔(𝒙 − 𝟏) है , तो
𝟏
a)25 b)22 c)17 d)43
[𝒙𝟐 + 𝒙−𝟒] का मान क्या है ?
a)3 b)5 c)6 d)8 106. If x = 11, then the value of 𝒙𝟓 − 𝟏𝟐𝒙𝟒 +
𝟏𝟐𝒙𝟑 − 𝟏𝟐𝒙𝟐 + 𝟏𝟐𝒙 − 𝟏 is
अगर x = 11 है तो 𝒙𝟓 − 𝟏𝟐𝒙𝟒 + 𝟏𝟐𝒙𝟑 −
𝒂 𝒃 𝒃 𝒄
102. If 𝒙 = (𝒃) + (𝒂) , 𝒚 = ( 𝒄 ) + (𝒃) and
𝒄 𝒂
𝒛 = (𝒂) + ( 𝒄 ) then what is the value of 𝟏𝟐𝒙𝟐 + 𝟏𝟐𝒙 − 𝟏 = ?
𝒙𝒚𝒛 − 𝒙² − 𝒚² − 𝒛²? a) 5 b) 10
𝒂 𝒃 𝒃 𝒄 c) 15 d) 20
यदि 𝒙 = (𝒃) + (𝒂) , 𝒚 = ( 𝒄 ) + (𝒃) तथा
𝒄 𝒂
𝒛 = (𝒂) + ( 𝒄 ) है ,तो 𝒙𝒚𝒛 − 𝒙² − 𝒚² − 𝒛² का 107. If 𝒙 = 𝟏𝟏, the value of 𝒙𝟓 − 𝟏𝟐𝒙𝟒 +
मान क्या है ? 𝟏𝟐𝒙𝟑 − 𝟏𝟐𝒙𝟐 + 𝟏𝟐𝒙 − 𝟏 is
a)-4 b)2 c)-1 d)-6 यदि 𝒙 = 𝟏𝟏, हो तो 𝒙𝟓 − 𝟏𝟐𝒙𝟒 + 𝟏𝟐𝒙𝟑 −
𝟏𝟐𝒙𝟐 + 𝟏𝟐𝒙 − 𝟏 का मान बताइए l
𝒙 𝒚 𝒙 𝒚
103. If (𝒂) + (𝒃) = 𝟑 and (𝒃) − (𝒂) = 𝟗, a) 11 b) 10
then what is the value of 𝒚?
𝒙 c) 12 d) -10

यदि ( ) + ( ) = 𝟑 तथा ( ) − ( ) = 𝟗 है , तो
𝒙 𝒚 𝒙 𝒚
𝒂 𝒃 𝒃 𝒂 108. Let 0 < x < 1. Then the correct inequality
का मान क्या है ?
𝒙
is
𝒚
𝒃+𝟑𝒂 𝒂+𝟑𝒃 𝟏+𝟑𝒂 𝒂+𝟑𝒃𝟐 यदि 0 < x < 1 हो, तो सही असमता क्या होगी ?
a)𝒂−𝟑𝒃 b)𝒃−𝟑𝒂 c)𝒂+𝟑𝒃 d)𝒃−𝟑𝒂𝟐
a) 𝒙 < √𝒙 < 𝒙𝟐
b) √𝒙 < 𝒙 < 𝒙𝟐
104. If 𝑨 = 𝟏 + 𝟐𝒑 𝒂𝒏𝒅 𝑩 = 𝟏 + 𝟐−𝒑 , then
c) 𝒙𝟐 < 𝒙 < √𝒙
what is the value of B?
d) √𝒙 < 𝒙𝟐 < 𝒙
यदि 𝑨 = 𝟏 + 𝟐 तथा 𝑩 = 𝟏 + 𝟐 है तो b का
𝒑 −𝒑

मान क्या है ? 𝟏 𝟏
109. If x + 𝐱 = c + 𝒄 then the value of x
𝑨+𝟏 𝑨+𝟐 𝑨 𝑨−𝟐
a) b) c) 𝑨−𝟏 d) 𝟏 𝟏
𝑨−𝟏 𝑨+𝟏 𝑨+𝟏 यदि x + = c + तो x का मान बताइए
𝐱 𝒄
a) C, 1/C b) C, C2
c) C, 2C d) 0, 1
𝟏 𝟏 𝟏 𝟏 𝟏
यदि 𝐀= + + + + +
𝟏×𝟐 𝟏×𝟒 𝟐×𝟑 𝟒×𝟕 𝟑×𝟒
𝒂 𝒙𝟐 +𝒙+𝒂
110. If 𝒙 + = 𝟏, then the value of is 𝟏
… …. 20 पिों तक हो, तो a का मान क्या है ?
𝒙 𝒙𝟑 −𝒙𝟐
𝟕×𝟏𝟎
𝒂 𝒙𝟐 +𝒙+𝒂
अगर 𝒙 + = 𝟏 है तो पता करो| 𝟑𝟕𝟗
a) 𝟑𝟎𝟖
𝟏𝟕𝟏
b)𝟏𝟒𝟎
𝟑𝟕𝟗
c)𝟑𝟏𝟎 d)𝟑𝟒𝟏
𝟒𝟐𝟎
𝒙 𝒙𝟑 −𝒙𝟐
𝟐 𝟐
a) 𝒂 b) −𝒂
𝒂 𝟏 𝟏
c) −2 d) – 114. What is the value of S=𝟏×𝟑×𝟓 + 𝟏×𝟒 +
𝟐
𝟏 𝟏 𝟏 𝟏
𝟑×𝟓×𝟕
+ 𝟒×𝟕 + 𝟓×𝟕×𝟗 + 𝟕×𝟏𝟎 +
FOR VIDEO SOLUTION FROM QUESTION 111
⋯ . 𝐮𝐩𝐭𝐨 𝟐𝟎 𝐭𝐞𝐫𝐦𝐬, 𝐭𝐡𝐞𝐧 𝐰𝐡𝐚𝐭 𝐢𝐬 𝐭𝐡𝐞 𝐯𝐥𝐚𝐮𝐞 𝐨𝐟 𝐒?
TO 120 : CLICK HERE OR 𝟏 𝟏 𝟏 𝟏 𝟏
SCAN Q R CODE GIVEN BELOW S=𝟏×𝟑×𝟓 + 𝟏×𝟒 + 𝟑×𝟓×𝟕 + 𝟒×𝟕 + 𝟓×𝟕×𝟗 +
𝟏
+⋯ 20 पिों तक है तो S का मान क्या है ?
𝟕×𝟏𝟎
(𝟔𝟏𝟕𝟗) (𝟔𝟎𝟕𝟎) (𝟕𝟏𝟗𝟏) (𝟓𝟏𝟖𝟑)
a) (𝟏𝟓𝟐𝟕𝟓) b) (𝟏𝟒𝟗𝟕𝟑) c)(𝟏𝟓𝟏𝟕𝟒) d) (𝟏𝟔𝟒𝟐𝟑)

115. Which of the following is correct?


111. Which of the following statement is/are ननलमजन्लखित में से कौन-सा सही है ?
true? 𝟐
a) < <
𝟑 𝟏𝟏
𝟑 𝟓 𝟏𝟓
ननम्नललखित में से कोन सा/से कथन सत्य है ? 𝟑 𝟐 𝟏𝟏
𝟏 𝟏 𝟏 𝟏 𝟏𝟐
b) < <
𝟓 𝟑 𝟏𝟓
1. + + + ⋯+ = 𝟏𝟏 𝟑 𝟐
𝟏×𝟑 𝟑×𝟓 𝟓×𝟕 𝟏𝟏×𝟏𝟑 𝟏𝟑
𝟏 𝟏 𝟏 𝟏 𝟏𝟐
c) < <
𝟏𝟓 𝟓 𝟑
2.𝟏×𝟐 + 𝟐×𝟑
+ 𝟑×𝟒
+ ⋯ + 𝟏𝟐×𝟏𝟑 = 𝟏𝟑 𝟑 𝟏𝟏 𝟐
d) 𝟓 < 𝟏𝟓 < 𝟑
a) only 1
b) only 2 116. Which of the following statement is/are
c)Both 1 and 2 true?
d)Neither 1 nor 2 ननम्नललखित में से कोन सा/ से कथन सत्य है ?
𝟑 𝟗 𝟕
1) < <
𝟏𝟏𝟎 𝟑𝟎𝟖 𝟕𝟐𝟓
𝟏 𝟏
112. What is the value of + 𝟕×𝟏𝟏 + 𝟏 𝟐 𝟑 𝟔
𝟑×𝟕 2) 99 + 𝟗𝟗 𝟕 + 𝟗𝟗 𝟕 + ......99+𝟕 = 𝟐𝟕𝟗
𝟕
𝟏 𝟏
𝟏𝟏×𝟏𝟓
+ ⋯ + 𝟖𝟗𝟗×𝟗𝟎𝟑 ? a) only 1
𝟏 𝟏 𝟏 𝟏
+ 𝟕×𝟏𝟏 + 𝟏𝟏×𝟏𝟓 + ⋯ + 𝟖𝟗𝟗×𝟗𝟎𝟑 का मान b) only 2
𝟑×𝟕
क्या है ? c) Neither 1 nor 2
d) Both 1 or 2
𝟐𝟏 𝟏𝟖 𝟐𝟓 𝟐𝟗
a)𝟓𝟎𝟗 b)𝟒𝟎𝟑 c)𝟑𝟎𝟏 d)𝟑𝟏

𝟏 𝟏 𝟏 𝟏 𝟏 117. Which of the following statement is/are


113. If 𝐀= 𝟏×𝟐
+ 𝟏×𝟒 + 𝟐×𝟑 + 𝟒×𝟕 + 𝟑×𝟒 +
𝟏 true?
… ….upto 20 terms, then what is the
𝟕×𝟏𝟎 ननम्नललखित में से कोन सा/से कथन सत्य है ?
value of A?
𝟑
1. 𝟕𝟏 < 𝟗𝟏 < 𝟗𝟗
𝟓 𝟕 c) 𝟓𝟑𝟎 d) 𝟔𝟐𝟎
𝟏𝟏 𝟏𝟐 𝟏𝟑
2. 𝟏𝟑𝟓 > 𝟏𝟓𝟕 > 𝟖𝟏 FOR VIDEO SOLUTION FROM QUESTION 121
a)only 1 TO 130 : CLICK HERE OR
b)only 2 SCAN Q R CODE GIVEN BELOW
c)Both 1 and 2
d)Neither 1nor 2

118. Which of the following statement is/are


ture?
ननम्नललखित में से कोन सा/से कथन सत्य है | 121. Which of the following statements is/are
𝟏 𝟑 𝟏 𝟏 𝟒𝟑𝟗 TRUE?
1.11𝟐 +17𝟒 −5𝟓 −2𝟏𝟎 =
ननम्नललखित में से कोन सा/से कथन सत्य है ?
𝟐𝟎
𝟗 𝟏𝟏 𝟏𝟐
2. > >
𝟏𝟎𝟕𝟖 𝟏𝟏𝟐𝟕 𝟏𝟐𝟏𝟗 1.𝟑𝟑𝟑 > 𝟑𝟑𝟑
𝟏𝟒𝟗 𝟏𝟓𝟑 𝟏𝟓𝟕
3.𝟏𝟓𝟏 > 𝟏𝟓𝟓 > 𝟏𝟓𝟗 2.𝟑𝟑𝟑 > (𝟑𝟑 )𝟑
a)only 1 a)only 1
b)only 2 b)only 2
c)only 3 c)Both 1 and 2
d)none of these d)Neither 1 nor 2

119. Which of the following statement is/are 122. Which of the following is True
true?
ननम्नललखित में से कोन सा सत्य है ?
ननम्नललखित में से कोन सा/से कथन सत्य है ? 𝟑 𝟒
a) √𝟏𝟏 > √𝟕 > √𝟒𝟓
𝟐 𝟑 𝟓
1. < < 𝟒√𝟑
𝟑
b)√𝟕 > √𝟏𝟏 > √𝟒𝟓
𝟒
𝟑√𝟓 𝟐√𝟓
𝟑 𝟐 𝟕 𝟒 𝟑
2. < < c)√𝟕 > √𝟒𝟓 > √𝟏𝟏
𝟐√𝟓 𝟑√𝟑 𝟒√𝟓
𝟒 𝟑
a)only 1 d) √𝟒𝟓 > √𝟕 > √𝟏𝟏

b)only 2
123. Which of the following statements(s)
c)Both 1 and 2 is/are true?
d)Neither 1 nor 2 ननम्नललखित में से कथन/कथनों में से कोन सा/से
सही है ?
120. The greatest number among 𝟏 𝟏 𝟏
𝟓𝟎 𝟒𝟎 𝟑𝟎 𝟐𝟎
𝟑 , 𝟒 , 𝟓 and 𝟔 is 1.(𝟔𝟓)𝟔 > (𝟏𝟕)𝟒 > (𝟏𝟐)𝟑
𝟏 𝟏 𝟏
𝟑𝟓𝟎 , 𝟒𝟒𝟎 , 𝟓𝟑𝟎 और 𝟔𝟐𝟎 में से सबसे बड़ा कौनसा 2.(𝟏𝟕)𝟒 > (𝟔𝟓)𝟔 > (𝟏𝟐)𝟑
𝟏 𝟏 𝟏
है ? 3.(𝟏𝟐)𝟑 > (𝟏𝟕)𝟒 > (𝟔𝟓)𝟔
a) 𝟑𝟓𝟎 b) 𝟒𝟒𝟎 a)only 1
b)only 2 1. √𝟓 + √𝟓 > √𝟕 + √𝟑
c)only 3 2. √𝟔 + √𝟕 > √𝟖 + √𝟓
d)none of these 3.√𝟑 + √𝟗 > √𝟔 + √𝟔
a)only 1
124. Which of the following is True? b)only 1 and 2
𝟏 𝟏 𝟏 c)only 2 and 3
a) 𝟑 > 𝟒 >
√𝟏𝟐 √𝟐𝟗 √𝟓 d)only 1 and 3
𝟏 𝟏 𝟏
b) 𝟒 > 𝟑 >
√𝟐𝟗 √𝟏𝟐 √𝟓
𝟏 𝟏 𝟏 𝟗𝟔 𝟗𝟕 𝟏
c) > 𝟑 > 𝟒
128. If 𝑷=
𝟗𝟓×𝟗𝟕
, 𝑸 = 𝟗𝟔×𝟗𝟖 𝒂𝒏𝒅 𝑹 = 𝟗𝟕 ,
√𝟓 √𝟏𝟐 √𝟐𝟗
𝟏 𝟏 𝟏 then which of the following is True?
d) > >
यदि 𝑷 = 𝑸 = 𝟗𝟔×𝟗𝟖 तथा 𝑹 = 𝟗𝟕 , है तो
√𝟓 𝟒
√𝟐𝟗
𝟑
√𝟏𝟐 𝟗𝟔 𝟗𝟕 𝟏
𝟗𝟓×𝟗𝟕
,
ननम्नललखित में से कोन सा सत्य है ?
125. Which of the following statement is/are
true? a)𝐏 < 𝐐 < 𝐑
ननम्नललखित में से कोन सा/से कथन सत्य है ?
b)𝐑 < 𝐐 < 𝐏
𝟑 𝟒
c)𝐐 < 𝐏 < 𝐑
1.√𝟏𝟐 > √𝟏𝟔 > √𝟐𝟒
𝟑 𝟒 𝟔
d)𝐑 < 𝐏 < 𝐐
2. √𝟐𝟓 > √𝟑𝟐 > √𝟒𝟖
𝟒 𝟑 𝟔
3. √𝟗 > √𝟏𝟓 > √𝟐𝟒 𝟏 𝟐
129. Find the sum of (𝟏 − 𝒏+𝟏) + (𝟏 − 𝒏+𝟏) +
a)only 1 and 2
𝟑 𝒏
b)only 1 and 3 (𝟏 − 𝒏+𝟏) + … (𝟏 − 𝒏+𝟏).
c)only 1 𝟏 𝟐 𝟑
(𝟏 − ) + (𝟏 − ) + (𝟏 − ) + … (𝟏 −
𝒏+𝟏 𝒏+𝟏 𝒏+𝟏
d)all are true. 𝒏
) का जोडफल पता करो|
𝒏+𝟏
𝟏
126. Which of the following statement is/are a) n b) n
𝟐
true? 𝟏
c) n + 1 d) (n + 1)
𝟐
ननम्नललखित में से कोन सा/से कथन सत्य है ?
√𝟏𝟏+√𝟕 < √𝟏𝟎+√𝟖. 130. The arithmetic mean of the following
√𝟏𝟕+√𝟏𝟏 > √𝟏𝟓+√𝟏𝟑 numbers

a)only 1 ननम्न संख्यायों का औसत पता करो|

b)only 2 1, 2, 2, 3, 3, 3, 4, 4, 4, 4, 5, 5, 5, 5, 5, 6, 6, 6, 6,
6, 6, 7, 7, 7, 7, 7, 7, 7
c)Both 1 and 2
a) 4 b) 5
d)Neither 1 nor 2 c) 14 d) 20

127. Which of the following statements is/are FOR VIDEO SOLUTION FROM QUESTION 131
true? TO 140 : CLICK HERE OR
SCAN Q R CODE GIVEN BELOW
ननम्नललखित में से कोन सा/से कथन सत्य है ?
ननम्नललखित में से कोन सा/से कथन सत्य है ?
1) √𝟏𝟐𝟏 + √𝟏𝟐𝟑𝟐𝟏 + √𝟏𝟐𝟑𝟒𝟑𝟐𝟏 = 𝟏𝟐𝟑𝟑
2. √𝟎. 𝟔𝟒 + √𝟔𝟒 + √𝟑𝟔 + √𝟎. 𝟑𝟔 > 𝟏𝟓
a) only 1
b) only 2
𝟑 𝟑
131. What is the value of 𝟏𝟒 + 𝟏𝟔 +
𝟏𝟖𝟑 … . . +𝟑𝟎𝟑 ? c) Neither 1 nor 2
𝟏𝟒𝟑 + 𝟏𝟔𝟑 + 𝟏𝟖𝟑 … . . +𝟑𝟎𝟑 का मान क्या है ?
d) Both 1 and 2
a)134576 b)120212
c)115624 d)111672 𝟑 𝟎.𝟎𝟎𝟎𝟕𝟐𝟗
136. √ = ?
𝟎.𝟎𝟖𝟓𝟏𝟖𝟒
𝟒𝟒 𝟐𝟕
132. If 𝑨 = 𝟐𝟑𝟐 , 𝑩 = 𝟐𝟑𝟏 + 𝟐𝟑𝟎 + 𝟐𝟐𝟗 + ⋯ 𝟐° a) b) 𝟒𝟐
𝟗
and 𝑪 = 𝟑𝟏𝟓 + 𝟑𝟏𝟒 + 𝟑𝟏𝟑 + ⋯ + 𝟑°, then 𝟐𝟕 𝟗
c) 𝟒𝟒
d) 𝟒𝟒
which of the following option is True?
यदि 𝑨 = 𝟐𝟑𝟐 , 𝑩 = 𝟐𝟑𝟏 + 𝟐𝟑𝟎 + 𝟐𝟐𝟗 + ⋯ 𝟐°
137. √𝟔𝟒𝟎𝟎𝟗 is equal to
तथा 𝑪 = 𝟑𝟏𝟓 + 𝟑𝟏𝟒 + 𝟑𝟏𝟑 + ⋯ + 𝟑° है , तो a) 352 b) 523
ननम्नललखित में से कोन सा ववकल्प सत्य है ? c) 253 d) 532
a)𝐂 > 𝐁 > 𝐀
b)𝐂 > 𝐀 > 𝐁
138. √𝟔 + √𝟔 + √𝟔+. … is equal to
c)𝐀 > 𝐁 > 𝐂
d)A> 𝐂 > 𝐁 √𝟔 + √𝟔 + √𝟔+. … ककसके सामान है
a) 2 b) 5 c) 4 d) 3
133. The value of (𝟏𝟏𝟏𝟏𝟏)𝟐 is
a) 12344321 b) 121212121
139. Find the value of
c) 123454321 d) 11344311
√𝟑𝟎 + √𝟑𝟎 + √𝟑𝟎 + ⋯
134. What is the value of
a) 5 b) 3√𝟏𝟎
√𝟏𝟐𝟏+√𝟏𝟐𝟑𝟐𝟏+√𝟏𝟐𝟑𝟒𝟑𝟐𝟏+√𝟏𝟐𝟑𝟒𝟓𝟒𝟑𝟐𝟏
c) 6 d) 7
?
√𝟏𝟐𝟏+√𝟏𝟐𝟑𝟐𝟏+√𝟏𝟐𝟑𝟒𝟑𝟐𝟏+√𝟏𝟐𝟑𝟒𝟓𝟒𝟑𝟐𝟏
140. The value of √𝟕𝟐 + √𝟕𝟐 + √𝟕𝟐+. . … .
का मान क्या है ?
a) 9 b) 8
a)12345 b)123456
c) 18 d) 12
c)12344 d)123454
FOR VIDEO SOLUTION FROM QUESTION 141
135. Which of the following statement is/are TO 150 : CLICK HERE OR
true: SCAN Q R CODE GIVEN BELOW
141. If m = √𝟓 + √𝟓 + √𝟓+. . … … … . and n =

√𝟓 − √𝟓 − √𝟓−. . … … … . then among the


following the relation between m and n
holds in
ननम्न में से कौनसा सत्य है ?
a) m – n + 1 = 0 b) m + n – 1 = 0
c) m + n + 1 = 0 d) m – n – 1 = 0

𝟑
𝟑

The value of 𝟐 √𝟒√𝟐 √𝟒√𝟐√𝟒 … . . is
𝟑
142.

a) 2 b) 𝟐𝟐
c) 𝟐𝟑 d) 𝟐𝟓

𝟏 𝟏 𝟏
143. If 𝟏 + ( ) + ( ) + ⋯ + ( ) = 𝒌, then
𝟐 𝟑 𝟐𝟎
𝟏 𝟏 𝟏
what is the value of ( ) + ( )+( )+ ⋯+
𝟒 𝟔 𝟖
𝟏
(𝟒𝟎) ?
यदि 𝟏 + (𝟐) + (𝟑) + ⋯ + (𝟐𝟎) = 𝒌 है , तो
𝟏 𝟏 𝟏

(𝟒) + (𝟔) + (𝟖) + ⋯ + (𝟒𝟎) का मान क्या है ?


𝟏 𝟏 𝟏 𝟏

𝒌 𝒌+𝟏 𝒌−𝟏
a) 𝟐 b)2k c) 𝟐
d) 𝟐

𝟏 𝟏
144. If 𝒇(𝒙) = 𝒙
− 𝒙+𝟏, then the value of
𝒇(𝟏) + 𝒇(𝟐) + 𝒇(𝟑). . . . 𝒇(𝟏𝟎)?
यदि है , तो
𝟏 𝟏
𝒇(𝒙) =
𝒙
− 𝒙+𝟏 𝒇(𝟏) + 𝒇(𝟐) +
𝒇(𝟑). . . . 𝒇(𝟏𝟎) का मान क्या है ?
𝟗 𝟏𝟎 𝟏𝟏 𝟏𝟐
a)𝟏𝟎 b)𝟏𝟏 c)𝟏𝟐 d)𝟏𝟑
Plane Geometry, Mensuration 2D, Mensuration 3D Top Latest
Pattern Difficult Questions From All Latest SSC CGL, CHSL, MTS, CPO
NTPC Exams 2020

FOR VIDEO SOLUTIONS 3. In ∆𝑨𝑩𝑪, 𝑨𝑩 = 𝟐𝟎 𝒄𝒎, 𝑩𝑪 = 𝟕𝒄𝒎 and 𝑪𝑨 =


FROM QUESTIONS 1 TO 10 CLICK HERE 𝟏𝟓 𝒄𝒎. Side BC is produced to D such that
OR SCAN QR CODE GIVEN BELOW ∆𝑫𝑨𝑩~∆𝑫𝑪𝑨. 𝑫𝑪 is equal to:
∆𝑨𝑩𝑪 में , 𝑨𝑩 = 𝟐𝟎 𝒄𝒎, 𝑩𝑪 = 𝟕𝒄𝒎और 𝑪𝑨 =
𝟏𝟓 𝒄𝒎 है| भुजा BC को त्रबंद ु D तक इस तरह
बढाया जाता है कक ∆𝑫𝑨𝑩~∆𝑫𝑪𝑨 है | 𝑫𝑪 का
माप बताइए|
a) 𝟗 𝒄𝒎 b) 𝟖 𝒄𝒎 c) 𝟏𝟎 𝒄𝒎 d) 𝟕 𝒄𝒎
1. Three sides of a triangle are √𝒂𝟐 + 𝒃𝟐 ,
4. In equilateral ∆𝑨𝑩𝑪. D and E are points on the
√(𝟐𝒂)𝟐 + 𝒃𝟐 and √𝒂𝟐 + (𝟐𝒃)𝟐 units. What is sides 𝑨𝑩 and 𝑨𝑪, respectively, such that 𝑨𝑫 =
the area (in unit squares) of the triangle?
𝑪𝑬. 𝑩𝑬 and 𝑪𝑫 intersect at 𝑭. The measure
एक त्रिभुज की तीन भुजाएँ √𝒂𝟐 + 𝒃𝟐 ,
(in degrees) of ∠𝑪𝑭𝑩 is:
√(𝟐𝒂)𝟐 + 𝒃𝟐 और √𝒂𝟐 + (𝟐𝒃)𝟐 इकाई हैं। समबाहु ∆𝑨𝑩𝑪 में , त्रबंद ु D और E क्रमर्ः 𝑨𝑩 और
त्रिभुज का क्षेिफल (वर्ग इकाई में ) ककतना 𝑨𝑪 पर इस प्रकार स्स्ित है त्रबंद ु हैं, कक 𝑨𝑫 =
होर्ा? 𝑪𝑬 है | 𝑩𝑬 और 𝑪𝑫 त्रबंद ु 𝑭 पर प्रततच्छे ददत
𝟓 𝟑
a) 𝟐 𝒂𝒃 b) 𝟑𝒂𝒃 c) 𝟒𝒂𝒃 d) 𝟐 𝒂𝒃 करती हैं। ∠𝑪𝑭𝑩 का माप (अंर् में) बताइए|
a) 𝟏𝟐𝟎° b) 𝟏𝟑𝟓° c) 𝟏𝟐𝟓° d) 𝟏𝟎𝟓°
2. AB is a chord in the minor segment of a circle
with centre O.C is a point on the minor are 5. In a square ABCD, diagonals AC and BD
(between A and B). The tangents to the circle intersect at O. The angle bisector of ∠𝑪𝑨𝑩
at A and B meet at a point P. If ∠𝑨𝑪𝑩 = 𝟏𝟎𝟖°, meets BD and BC at F and G, respectively. OF :
then ∠𝑨𝑷𝑩 is equal to: CG is equal to:
AB, केंद्र O वाले वत्त
ृ के लघु वत्त
ृ खंड में एक एक वर्ग ABCD में , ववकर्ग AC और BD, त्रबंद ु O पर
जीवा है। लघु चाप (A और B के बीच) पर एक प्रततच्छे द करते हैं। ∠𝑪𝑨𝑩 का कोर्
त्रबंद ु C है | A और B से खखची र्ई वत्त
ृ की स्पर्ग समद्ववभाजक BD और BC से क्रमर्ः त्रबंद ु F और
रे खाएँ त्रबंद ु P पर ममलती हैं। यदद ∠𝑨𝑪𝑩 = G पर ममलता है। OF : CG का मान ककतना होर्ा?
𝟏𝟎𝟖° है, तो ∠𝑨𝑷𝑩 का माप बताइए| a) 𝟏 : 𝟐 b) 𝟏 : 𝟑 c) 𝟏 : √𝟐 d) 𝟏 : √𝟑
a) 𝟑𝟔° b)𝟓𝟒° c) 𝟐𝟕° d) 𝟏𝟖°
6. 𝑶 is a point in the interior of ∆𝑨𝑩𝑪 such that
𝑶𝑨 = 𝟏𝟐 𝒄𝒎, 𝑶𝑪 = 𝟗 𝒄𝒎, ∠𝑨𝑶𝑩 =
∠𝑩𝑶𝑪 = ∠𝑪𝑶𝑨 and ∠𝑨𝑩𝑪 = 𝟔𝟎°. What is a) 𝟕√𝟐 b) 8 c) 7 d) 𝟖√𝟐
the length (in 𝒄𝒎) of 𝑶𝑩?
𝑶 एक त्रबंद ु है जो ∆𝑨𝑩𝑪 के अंतर इस प्रकार है 10. In ∆𝑨𝑩𝑪, 𝑶 is the point of intersection of the
bisector of ∠𝑩 and ∠𝑨. If ∠𝑩𝑶𝑪 = 𝟏𝟎𝟖°, then
कक 𝑶𝑨 = 𝟏𝟐 𝒄𝒎, 𝑶𝑪 = 𝟗 𝒄𝒎, ∠𝑨𝑶𝑩 =
∠𝑩𝑨𝑶 =?
∠𝑩𝑶𝑪 = ∠𝑪𝑶𝑨 और ∠𝑨𝑩𝑪 = 𝟔𝟎° है | 𝑶𝑩 ∆𝑨𝑩𝑪 में , ∠𝑩 और ∠𝑨 के समद्ववभाजकों का
की लंबाई (𝒄𝒎 में ) ककतनी है ? प्रततच्छे दन त्रबंद ु 𝑶 है। यदद ∠𝑩𝑶𝑪 = 𝟏𝟎𝟖° है ,
a) 𝟔√𝟐 b) 𝟒√𝟔 c) 𝟔√𝟑 d) 𝟒√𝟑
तो ∠𝑩𝑨𝑶 का माप क्या होर्ा?
7. In ∆𝑨𝑩𝑪, M is the midpoint of the side 𝑨𝑩. N is a) 𝟐𝟕° b) 𝟏𝟖° c) 𝟑𝟔° d) 𝟒𝟎°
a point in the interior of ∆𝑨𝑩𝑪 such that 𝑪𝑵 is FOR VIDEO SOLUTIONS
the bisector of ∠𝑪 and 𝑪𝑵 ⊥ 𝑵𝑩. What is the FROM QUESTIONS 11 TO 20 CLICK HERE
length (in 𝒄𝒎) of 𝑴𝑵, if 𝑩𝑪 = 𝟏𝟎 𝒄𝒎 and OR SCAN QR CODE GIVEN BELOW
𝑨𝑪 = 𝟏𝟓 𝒄𝒎?
∆𝑨𝑩𝑪 में, भुजा 𝑨𝑩 का मध्यत्रबंद ु M है| N एक
ऐसा त्रबंद ु है जो ∆𝑨𝑩𝑪 के अंदर इस प्रकार
स्स्ित है , कक 𝑪𝑵 ∠𝑪 का समद्ववभाजक है और
𝑪𝑵 ⊥ 𝑵𝑩 है | यदद 𝑩𝑪 = 𝟏𝟎 𝒄𝒎 और 𝑨𝑪 =
𝟏𝟓 𝒄𝒎, तो 𝑴𝑵 की लंबाई (𝒄𝒎 में) ककतनी है ? 11. G is the centroid of a triangle ABC, whose
sides 𝑨𝑩 = 𝟑𝟓 𝒄𝒎, 𝑩𝑪 = 𝟏𝟐 𝒄𝒎, and 𝑨𝑪 =
a) 2 b) 5 c) 4 d) 𝟐. 𝟓
𝟑𝟕 𝒄𝒎. The length of BG is (correct to one
decimal place):
8. In ∆𝑨𝑩𝑪, ∠𝑩 = 𝟕𝟖°, 𝑨𝑫 is a bisector of ∠𝑨
meeting 𝑩𝑪 at 𝑫, and 𝑨𝑬 ⊥ 𝑩𝑪 at 𝑬. If G एक ऐसे त्रिभज
ु ABC का केन्द्द्रक है , स्जसकी
∠𝑫𝑨𝑬 = 𝟐𝟒°, then the measure of ∠𝑨𝑪𝑩 is: भज
ु ाएँ 𝑨𝑩 = 𝟑𝟓 𝒄𝒎, 𝑩𝑪 = 𝟏𝟐 𝒄𝒎, और 𝑨𝑪 =
∆𝑨𝑩𝑪 में, 𝑩 = 𝟕𝟖° है, ∠𝑨 का समद्ववभाजक 𝟑𝟕 𝒄𝒎 हैं। BG की लंबाई (दर्मलव के एक
𝑨𝑫 है जो 𝑩𝑪 से त्रबंद ु 𝑫 पर ममलता है और स्िान तक सही) ककतनी है ?
त्रबंद ु 𝑬 पर 𝑨𝑬 ⊥ 𝑩𝑪 है| यदद ∠𝑫𝑨𝑬 = 𝟐𝟒° हैं, a) 𝟏𝟐. 𝟗 𝒄𝒎 b) 𝟏𝟐. 𝟑 𝒄𝒎
तो ∠𝑨𝑪𝑩 की माप क्या है ? c) 𝟏𝟏. 𝟕 𝒄𝒎 d) 𝟏𝟕. 𝟓 𝒄𝒎
a) 𝟑𝟐° b) 𝟑𝟖° c) 𝟑𝟎° d) 𝟒𝟐°
12. In ∆𝑷𝑸𝑹, 𝑺 is a point on the side 𝑸𝑹 such that
𝟏
9. In ∆𝑳𝑴𝑵, 𝑳𝑴 = 𝟓√𝟐 𝒄𝒎, 𝑳𝑵 = 𝟏𝟑 𝒄𝒎 and ∠𝑸𝑷𝑺 = 𝟐 ∠𝑷𝑺𝑹, ∠𝑸𝑷𝑹 = 𝟕𝟖° and ∠𝑷𝑹𝑺 =
∠𝑳𝑴𝑵 = 𝟏𝟑𝟓°. What is the length (in 𝒄𝒎) of 𝟒𝟒°. What is the measure of ∠𝑷𝑺𝑸 ?
𝑴𝑵? 𝑷𝑸𝑹 में , 𝑺 एक त्रबंद ु है , जो भुजा 𝑸𝑹 पर इस
∆𝑳𝑴𝑵 में, 𝑳𝑴 = 𝟓√𝟐 𝒄𝒎, 𝑳𝑵 = 𝟏𝟑 𝒄𝒎 और प्रकार स्स्ित है कक ∠𝑸𝑷𝑺 =
∠𝑳𝑴𝑵 = 𝟏𝟑𝟓° हैं| 𝑴𝑵 की लंबाई (𝒄𝒎 में) 𝟏
∠𝑷𝑺𝑹, ∠𝑸𝑷𝑹 = 𝟕𝟖° और 𝑷𝑹𝑺 = 𝟒𝟒° है|
𝟐
ककतनी है ? ∠𝑷𝑺𝑸 की माप क्या है ?
a) 𝟔𝟖° b) 𝟓𝟔° c) 𝟓𝟖° d) 𝟔𝟒° 𝟐𝟐
(𝑻𝒂𝒌𝒆 𝝅 = )
𝟕

13. In ∆𝑨𝑩𝑪, 𝑨𝑩 = 𝟒𝟖 𝒄𝒎, 𝑩𝑪 = 𝟓𝟓 𝒄𝒎 and 10 मीटर ब्यास वाले वत


ृ ाकार उद्द्यान के पररत:
𝑨𝑪 = 𝟕𝟑 𝒄𝒎. If 𝑶 is the centroid of the 𝟔𝟒𝒄𝒎 चौड़ा रास्ता बनाया र्या है | रास्ते का
triangle, then the length (in 𝒄𝒎) 0f 𝑩𝑶
क्षेिफल (𝒎𝟐 में) लर्भर्_____ है |
(correct to one decimal place) is:
a) 10 b) 11 c) 15 d) 9
∆𝑨𝑩𝑪 में, 𝑨𝑩 = 𝟒𝟖 𝒄𝒎, 𝑩𝑪 = 𝟓𝟓 𝒄𝒎 और
𝑨𝑪 = 𝟕𝟑 𝒄𝒎 है| यदद त्रिभुज का केंद्रक 17. In a ∆𝑨𝑩𝑪, ∠𝑩𝑨𝑪 = 𝟗𝟎°, 𝑨𝑫 is drawn
(centroid) 𝑶 है , तो 𝑩𝑶 की लंबाई (𝒄𝒎 में, perpendicular from 𝑨 on 𝑩𝑪. Which among
the following is the mean proportional
दर्मलव के एक अंक तक सही) ज्ञात करें | between 𝑩𝑫 and 𝑩𝑪?
a) 𝟐𝟓. 𝟔 b) 𝟐𝟒. 𝟑 c) 𝟐𝟎. 𝟒 d) 𝟏𝟖. 𝟑 ∆𝑨𝑩𝑪 में , ∠𝑩𝑨𝑪 = 𝟗𝟎°, 𝑩𝑪 पर 𝑨 से लंब 𝑨𝑫

14. In ∆𝑷𝑸𝑹, 𝑷𝑸 = 𝑷𝑹 and 𝑺 is a point on 𝑸𝑹 खींचा जाता है। 𝑩𝑫 से 𝑩𝑪 के बीच मध्यानुपाती


such that ∠𝑷𝑺𝑸 = 𝟗𝟔° + ∠𝑸𝑷𝑺 and ∠𝑸𝑷𝑹 = तनम्नमलखखत में से कौन सा होर्ा?
𝟏𝟑𝟐°. What is the measure of ∠𝑷𝑺𝑹? a) 𝑨𝑫 b) 𝑨𝑩 c) 𝑪𝑫 d) 𝑨𝑪
∆𝑷𝑸𝑹 में , 𝑷𝑸 = 𝑷𝑹 और 𝑺 रे खा 𝑸𝑹 पर इस
प्रकार स्स्ित एक त्रबंद ु है कक 𝑷𝑺𝑸 = 𝟗𝟔° + 18. In ∆𝑨𝑩𝑪, 𝑩𝑫 ⊥ 𝑨𝑪 at 𝑫. 𝑬 is a point on BC
such that ∠𝑩𝑬𝑨 = 𝒙°. If ∠𝑬𝑨𝑪 = 𝟒𝟔° and
∠𝑸𝑷𝑺 और ∠𝑸𝑷𝑹 = 𝟏𝟑𝟐° है| तो ∠𝑷𝑺𝑹 का ∠𝑬𝑩𝑫 = 𝟔𝟎°, then the value of 𝒙 is:
माप बताइए| ∆𝑨𝑩𝑪 में , 𝑫 पर 𝑩𝑫 ⊥ 𝑨𝑪 है | BC पर एक त्रबंद ु
a) 𝟒𝟓° b) 𝟓𝟔° c) 𝟓𝟒° d) 𝟓𝟐° E इस प्रकार है कक ∠𝑩𝑬𝑨 = 𝒙° है यदद

15. 𝑨𝟏 and 𝑨𝟐 are two regular polygons. The sum ∠𝑬𝑨𝑪 = 𝟒𝟔° और ∠𝑬𝑩𝑫 = 𝟔𝟎° है , तो 𝒙 का
of all the interior angles of 𝑨𝟏 is 𝟏𝟎𝟖𝟎°. Each मान ज्ञात करें |
interior angle of 𝑨𝟐 exceeds its exterior by a) 𝟕𝟐° b) 𝟕𝟖° c) 𝟔𝟖° d) 𝟕𝟔 °
𝟏𝟑𝟐°. The sum of the number of sides 𝑨𝟏 and
𝑨𝟐 is: 19. ABCD is a cyclic quadrilateral. AB and DC meet
𝑨𝟏 और 𝑨𝟐 दो सम बहुभज ु हैं। 𝑨𝟏 के सभी at F, when produced. AD and BC meet at E,
आंतररक कोर्ों का योर् 𝟏𝟎𝟖𝟎° है। 𝑨𝟐 का when produced. If ∠𝑩𝑨𝑫 = 𝟔𝟖° and ∠𝑨𝑬𝑩 =
𝟐𝟕°, then what is the measure of ∠𝑩𝑭𝑪?
प्रत्येक आंतररक कोर्, इसके बाहय कोर् से
ABCD चक्रीय चतुभज
ुग है | AB और DC बढाए जाने
𝟏𝟑𝟐° अधिक है। 𝑨𝟏 और 𝑨𝟐 की भज
ु ाओं की पर F पर ममलती है| AD और BC को बढाए जाने
संख्या का योर् ककतना होर्ा?
पर E पर ममलती है | यदद ∠𝑩𝑨𝑫 = 𝟔𝟖° और
a) 𝟐𝟏 b) 𝟐𝟐 c) 𝟐𝟑 d) 𝟐𝟒
∠𝑨𝑬𝑩 = 𝟐𝟕° है , तो ∠𝑩𝑭𝑪 का माप ज्ञात करें |
16. A 𝟔𝟒 𝒄𝒎 wide path is made around a circular a) 𝟐𝟐° b) 𝟐𝟕°
garden having a diameter of 10 metres. The c) 𝟏𝟓° d) 𝟏𝟕°
area (in 𝒎𝟐 ) of the path is closest to:
20. ∆𝑨𝑩𝑪 is an equilateral triangle. D is a point on त्रबंद ु P और Q क्रमर्: त्रिभज
ु ABC की भज
ु ा AB
side BC such that 𝑩𝑫 : 𝑩𝑪 = 𝟏 : 𝟑. If 𝑨𝑫 =
और BC पर स्स्ित हैं, त्रिभुज B पर समकोर् है |
𝟓√𝟕 𝒄𝒎, then the side of the triangle is :
∆𝑨𝑩𝑪, समबाहु त्रिभुज है| भुजा BC पर एक त्रबंद ु यदद 𝑨𝑸 = 𝟏𝟏 𝒄𝒎, 𝑷𝑪 = 𝟖 𝒄𝒎 और 𝑨𝑪 =

D इस प्रकार है कक 𝑩𝑫 : 𝑩𝑪 = 𝟏 : 𝟑 है| यदद 𝟏𝟑 𝒄𝒎 है, तो PQ की लम्बाई (cm में) ज्ञात

𝑨𝑫 = 𝟓√𝟕 𝒄𝒎 है, तो त्रिभुज की भुजा ज्ञात करें |


a) √𝟏𝟓 b) 𝟒. 𝟓
करें |
c) 𝟒 d) 𝟒√𝟕
a) 𝟏𝟐 𝒄𝒎 b) 𝟏𝟓 𝒄𝒎
c) 𝟐𝟎 𝒄𝒎 d) 𝟏𝟖 𝒄𝒎
23. Triangles ABC and DBC are right angled
triangles with common hypotenuse BC. BD
FOR VIDEO SOLUTIONS
and AC intersect at P when produced. If 𝑷𝑨 =
FROM QUESTIONS 21 TO 30 CLICK HERE
𝟖 𝒄𝒎, 𝑷𝑪 = 𝟒 𝒄𝒎 and 𝑷𝑫 = 𝟑. 𝟐 cm, then
OR SCAN QR CODE GIVEN BELOW
the length of BD, in cm. is:
त्रिभुज ABC और DBC, उभयतनष्ठ कर्ग BC वाले
समकोर् त्रिभुज हैं| BD और AC को बढाने पर
वे P पर प्रततच्छे ददत होती हैं| यदद 𝑷𝑨 =
𝟖 𝒄𝒎, 𝑷𝑪 = 𝟒 𝒄𝒎 और 𝑷𝑫 = 𝟑. 𝟐 cm है, तो
BD की लंबाई (cm में ) ज्ञात करें |
21. In a circle with center O, AB and CD are
parallel chords on the opposite sides of a a) 𝟔. 𝟒 b) 𝟔. 𝟖
diameter. If 𝑨𝑩 = 𝟏𝟐 𝒄𝒎, 𝑪𝑫 = 𝟏𝟖 𝒄𝒎 and c) 𝟓. 𝟔 d) 𝟕. 𝟐
the distance between the chords AB and CD is
𝟏𝟓 𝒄𝒎, then find the radius of the circle (in 24. The vertices of a ∆𝑨𝑩𝑪 lie on a circle with
cm). centre O. AO is produced to meet the circle at
the point P. D is a point on BC such that 𝑨𝑫 ⊥
केंद्र O वाले वत्त
ृ में, AB और CD व्यास के 𝑩𝑪. If ∠𝑩 = 𝟔𝟖° and ∠𝑪 = 𝟓𝟐°, then the
ववपरीत भुजाओं पर दो समानांतर जीवाएं हैं| measure of ∠𝑫𝑨𝑷 is:
यदद 𝑨𝑩 = 𝟏𝟐 𝒄𝒎, 𝑪𝑫 = 𝟏𝟖 𝒄𝒎 और जीवा ∆𝑨𝑩𝑪 के र्ीर्ग, O केंद्र वाले वत्त
ृ पर स्स्ित हैं।
AB और CD के बीच की दरू ी 𝟏𝟓 𝒄𝒎 है, तो वत्त
ृ AO को तनममगत करने पर यह वत्त
ृ से त्रबंद ु P पर
की त्रिज्या (cm में) ज्ञात करें | ममलती है | BC पर त्रबंद ु D इस प्रकार है कक
a) 𝟑√𝟏𝟑 b) 12 𝑨𝑫 ⊥ 𝑩𝑪 है| यदद ∠𝑩 = 𝟔𝟖° और ∠𝑪 = 𝟓𝟐° है ,
c) 9 d) 𝟗√𝟏𝟑 तो ∠𝑫𝑨𝑷 का माप ज्ञात करें |
a) 𝟏𝟖° b) 𝟏𝟔°
22. Points P and Q are on the sides AB and BC c) 𝟏𝟐° d) 𝟐𝟖°
respectively of a tringle ABC, right angled at B.
If 𝑨𝑸 = 𝟏𝟏 𝒄𝒎, 𝑷𝑪 = 𝟖 𝒄𝒎 and 𝑨𝑪 = 25. In a circle with centre 𝑶. 𝑨𝑩 is a chord of
𝟏𝟑 𝒄𝒎, then find the length (in cm) of PQ? length 𝟏𝟎𝒄𝒎. Tangents at points 𝑨 and 𝑩
intersect outside the circle at 𝑷.If 𝑶𝑷 = PT उस त्रबंद ु से त्रबंद ु C पर वत्त
ृ की स्पर्गरेखा है ।
𝟐 𝑶𝑨,then find the length (in cm) of 𝑨𝑷
यदद ∠𝑩𝑷𝑻 = 𝟐𝟖° है , तो ∠𝑩𝑪𝑷 का माप
केंद्र 𝑶 वाले वत्त
ृ में, 𝑨𝑩 जीवा है स्जसकी
ककतना होर्ा?
लम्बाई 𝟏𝟎𝒄𝒎 है | त्रबंद ु 𝑨 और 𝑩 पर खींची र्ई
a) 𝟔𝟐° b) 𝟒𝟓°
स्पर्गरेखाएं, वत्त
ृ के बाहर त्रबंद ु 𝑷 पर प्रततच्छे ददत c) 𝟐𝟖° d) 𝟑𝟏°
करती है | यदद 𝑶𝑷 = 𝟐 𝑶𝑨 है, तो 𝑨𝑷 की
29. In a circle with centre O. AD is a diameter and
लम्बाई (cm में) ज्ञात करें | AC is a chord. Point B is on AC such that 𝑶𝑩 =
a) 𝟏𝟐. 𝟓 b) 10 𝟕 𝒄𝒎 and ∠𝑶𝑩𝑨 = 𝟔𝟎°. If ∠𝑫𝑶𝑪 = 𝟔𝟎°, then
c) 15 d) 12 what is the length of BC (in cm)?
O केंद्र वाले वत्त
ृ में, AD व्यास है और AC जीवा
26. Point M and N are on the sides PQ and QR
respectively of a triangle PQR, right angled at है | AC पर त्रबंद ु B इस प्रकार है कक 𝑶𝑩 =
Q. If 𝑷𝑵 = 𝟗 𝒄𝒎, 𝑴𝑹 = 𝟕 𝒄𝒎 and 𝑴𝑵 = 𝟕 𝒄𝒎 और ∠𝑶𝑩𝑨 = 𝟔𝟎° है | यदद ∠𝑫𝑶𝑪 =
𝟑 𝒄𝒎, then find the length of PR (in cm).
𝟔𝟎° हो, तो BC की लम्बाई (cm में) ज्ञात करें |
त्रिभज
ु PQR की भज
ु ा PQ और QR पर क्रमर्:
a) 5 b) 7
त्रबंद ु M और N स्स्ित है , तिा यह त्रिभज
ु Q पर c) 9 d) 𝟑. 𝟓
समकोर् है | यदद 𝑷𝑵 = 𝟗 𝒄𝒎, 𝑴𝑹 = 𝟕 𝒄𝒎
30. In a triangle 𝑨𝑩𝑪 𝑨𝑩 : 𝑨𝑪 = 𝟓 : 𝟐, 𝑩𝑪 =
और 𝑴𝑵 = 𝟑 𝒄𝒎 है , तो PR की लम्बाई (cm में) 𝟗 𝒄𝒎, 𝑩𝑨 is produced to D, and the bisector
ज्ञात करें | of the Angle CAD meets BC produced at E.
a) 12 b) 11 What is the length (in cm) of CE?
c) √𝟒𝟏 d) 13 त्रिभुज 𝑨𝑩𝑪 में , 𝑨𝑩 : 𝑨𝑪 = 𝟓 : 𝟐, 𝑩𝑪 = 𝟗 𝒄𝒎

𝑨𝑩 𝑩𝑪 𝑨𝑪
हैं| 𝑩𝑨 को D तक बढाया जाता है और कोर्
27. In ∆𝑨𝑩𝑪 and ∆𝑫𝑬𝑭, we have 𝑫𝑭 = 𝑫𝑬 = 𝑬𝑭 ,
CAD का समद्ववभाजक बढाई र्ई BC से E पर
then which of the following is true?
ममलता है | CE की लम्बाई (cm में) ज्ञात करें |
∆𝑨𝑩𝑪 और ∆𝑫𝑬𝑭 में , = 𝑫𝑬 = 𝑬𝑭 है| तनम्न
𝑨𝑩 𝑩𝑪 𝑨𝑪
𝑫𝑭 a) 9 b) 10
में से कौन सा सत्य है ? c) 3 d) 6
a) ∆𝑫𝑬𝑭~∆𝑨𝑩𝑪 b) ∆𝑩𝑪𝑨~∆𝑫𝑬𝑭
c) ∆𝑪𝑨𝑩~∆𝑫𝑬𝑭 d) ∆𝑫𝑬𝑭~∆𝑩𝑨𝑪 FOR VIDEO SOLUTIONS
FROM QUESTIONS 31 TO 40 CLICK HERE
28. In a circle with centre O, a diameter AB is OR SCAN QR CODE GIVEN BELOW
produced to a point P lying outside the circle
and PT is a tangent to the circle at a point C on
it. If ∠𝑩𝑷𝑻 = 𝟐𝟖°, then what is the measure
of ∠𝑩𝑪𝑷?
O केंद्र वाले वत्त
ृ में, व्यास AB को वत्त
ृ के बाहर
स्स्ित त्रबंद ु P तक तनममगत ककया जाता है और
31. In a circle with center O. AB is a diameter and 34. In ∆𝑨𝑩𝑪, 𝑫 is the mid-point of side AC and E is
CD is a chord such that ∠𝑨𝑩𝑪 = 𝟑𝟒° and a point on side AB such that EC bisects BD at F.
𝑪𝑫 = 𝑩𝑫. What is the measure of ∠𝑫𝑩𝑪? If 𝑨𝑬 = 𝟑𝟎 𝒄𝒎, then the length of EB is:
O केंद्र वले वत्त
ृ में, व्यास AB और जीवा CD इस ∆𝑨𝑩𝑪 में , भज
ु ा AC का मध्य-त्रबंद ु 𝑫 और भज
ु ा
प्रकार हैं कक ∠𝑨𝑩𝑪 = 𝟑𝟒° और 𝑪𝑫 = 𝑩𝑫 है| AB पर एक त्रबंद ु E इस प्रकार है कक EC, BD को
∠𝑫𝑩𝑪 की माप ज्ञात करें | त्रबंद ु F पर समद्ववभास्जत करती है | यदद 𝑨𝑬 =
a) 𝟑𝟎° b) 𝟑𝟐° 𝟑𝟎 𝒄𝒎 है , तो EB की लम्बाई ज्ञात करें |
c) 𝟐𝟒° d) 𝟐𝟖° a) 𝟏𝟖 𝒄𝒎 b) 𝟏𝟎 𝒄𝒎
c) 𝟐𝟎 𝒄𝒎 d) 𝟏𝟓 𝒄𝒎
32. In a triangle ABC, point D lies on AB, and
points E and F lie on BC such that DF is parallel 35. In a triangle ABC, a point D lies on AB and
to AC and DE is parallel to AF. If 𝑩𝑬 = points E and F lie on BC such that DF is parallel
𝟒 𝒄𝒎, 𝑪𝑭 = 𝟑 𝒄𝒎, then find the length (in to AC and DE is parallel to AF. If 𝑩𝑬 =
cm) of EF. 𝟒 𝒄𝒎, 𝑬𝑭 = 𝟔 𝒄𝒎, then find the length (in
त्रिभज
ु ABC में, त्रबंद ु D, AB पर स्स्ित है तिा E cm) of BC.
त्रिभुज ABC में , त्रबंद ु D, AB पर तिा त्रबंद ु E और
और F, BC पर इस प्रकार स्स्ित हैं कक DF, AC
F इस प्रकार BC पर स्स्ित हैं कक DF, AC के
के समानांतर है और DE, AF के समानांतर है|
समानांतर हैं और DE, AF के समानांतर हैं| यदद
यदद 𝐁𝐄 = 𝟒 𝐜𝐦 और 𝐂𝐅 = 𝟑 𝐜𝐦 है, तो EF की
𝑩𝑬 = 𝟒 𝒄𝒎 और 𝑬𝑭 = 𝟔 𝒄𝒎 है , तो BC की
लम्बाई (cm में) ज्ञात करें |
लम्बाई (cm में) ज्ञात करें |
a) 5 b) 3
a) 30 b) 20
c) 2 d) 𝟏. 𝟓
c) 25 d) 15
33. AB is a diameter of a circle. C and D are points
36. Points P, Q, R, S and T lie in this order on a
on the opposite sides of the diameter AB, such
circle with centre O. If chord TS is parallel to
that ∠𝑨𝑪𝑫 = 𝟐𝟓°. E is a point on the minor
diameter PR and ∠𝑹𝑸𝑻 = 𝟓𝟖°, then find the
are BD. Find the measure of ∠𝑩𝑬𝑫 (in
measure (in degrees) of ∠𝑹𝑻𝑺.
degrees).
त्रबंद ु P, Q, R, S और T इसी क्रम में केंद्र O वाले
AB एक वत्त
ृ का एक व्यास है | त्रबंद ु C और D
वत्त
ृ पर स्स्ित हैं| यदद जीवा TS, व्यास PR के
व्यास AB के ववपरीत पक्षों पर इस प्रकार स्स्ित
समानांतर है और ∠𝑹𝑸𝑻 = 𝟓𝟖° हैं, तो ∠𝑹𝑻𝑺
हैं कक ∠𝑨𝑪𝑫 = 𝟐𝟓° है| E, लघु चाप BD पर
का माप (डडग्री में) ज्ञात करें |
स्स्ित एक त्रबंद ु है | ∠𝑩𝑬𝑫 का माप (अंर् में)
a) 45 b) 29
ज्ञात करें | c) 32 d) 58
a) 105 b) 125
c) 130 d) 115 37. In triangle 𝑨𝑩𝑪, 𝑫 is a point on 𝑩𝑪 such that
𝑩𝑫 : 𝑫𝑪 = 𝟑 : 𝟒. 𝑬 is a point on 𝑨𝑫 such that
𝑨𝑬 : 𝑬𝑫 = 𝟐 : 𝟑. Find the ratio area 40. What is the perimeter (in cm) of an equilateral
(∆𝑬𝑪𝑫) : 𝒂𝒓𝒆𝒂 (∆𝑨𝑬𝑩). triangle whose height is 𝟑. 𝟒𝟔 𝒄𝒎? Take √𝟑 =
त्रिभुज 𝐀𝐁𝐂 में, 𝐁𝐂 पर कोई त्रबंद ु 𝐃 इस प्रकार 𝟏. 𝟕𝟑
𝟑. 𝟒𝟔 𝒄𝒎 ऊँचाई वाले समबाहु त्रिभज ु का
स्स्ित है कक 𝐁𝐃 : 𝐃𝐂 = 𝟑 : 𝟒 हैं| 𝐀𝐃 पर कोई
पररमाप (in cm) ककतना होर्ा? √𝟑 = 𝟏. 𝟕𝟑 मान
त्रबंद ु 𝐄 इस प्रकार स्स्ित है कक 𝐀𝐄 : 𝐄𝐃 = 𝟐 : 𝟑.
लें|
हैं| (∆𝐄𝐂𝐃) के क्षेिफल से अनपु ात ज्ञात करें | a) 𝟏𝟎. 𝟒 b) 9
a) 𝟐 : 𝟏 b) 𝟗 : 𝟖 c) 6 d) 12
c) 𝟖 : 𝟗 d) 𝟏 : 𝟐
FOR VIDEO SOLUTIONS
38. Points 𝑨 and 𝑩 are on a circle with centre 𝑶. FROM QUESTIONS 41 TO 50 CLICK HERE
𝑷𝑨𝑴 and 𝑷𝑩𝑵 are tangents to the circle at 𝑨 OR SCAN QR CODE GIVEN BELOW
and 𝑩 respectively from a point 𝑷 outside the
circle. Point 𝑸 is on the major are 𝑨𝑩 such
that ∠𝑸𝑨𝑴 = 𝟓𝟖° and ∠𝑸𝑩𝑵 = 𝟓𝟎°, then
find the measure (in degrees) of ∠𝑨𝑷𝑩.
𝑨 और 𝑩 केंद्र 𝑶 वाले वत्त
ृ पर स्स्ित त्रबंद ु है |
वत्त
ृ के बाहर त्रबंद ु 𝑷 से 𝑷𝑨𝑴 और 𝑷𝑩𝑵 ,
क्रमर्: 𝑨 और 𝑩 पर वत्त
ृ की स्पर्ग रे खाएं है | दीघग 41. A rhombus of side 𝟐𝟖 𝒄𝒎 has one angle of
𝟔𝟎°. What is the length of the larger diagonal?
चाप 𝑨𝑩 पर त्रबंद ु 𝑸 इस प्रकार स्स्ित है कक
𝟐𝟖 𝒄𝒎 भुजा वाले समचतुभज
ुग का एक कोर्
∠𝑸𝑨𝑴 = 𝟓𝟖° और ∠𝑸𝑩𝑵 = 𝟓𝟎° है | ∠𝑨𝑷𝑩
𝟔𝟎° है | बड़े ववकर्ग की लम्बाई ज्ञात करें |
का माप (अंर् में ) ज्ञात करें |
a) 𝟐𝟖(𝟏 + √𝟑)𝒄𝒎 b) 𝟐𝟖 √𝟐𝒄𝒎
a) 32 b) 36
c) 𝟐𝟖 𝒄𝒎 d) 𝟐𝟖√𝟑𝒄𝒎
c) 30 d) 40
42. ABC is right- angled triangle, right-angled at B.
D is a point on AC such that 𝑨𝑫 = 𝟏𝟐𝒄𝒎 and
39. Chord 𝑨𝑩 of a circle of radius 𝟏𝟎 𝒄𝒎 is at a
𝑪𝑫 = 𝟏𝟔 𝒄𝒎 if BD bisects ∠𝑨𝑩𝑪 then the
distance 𝟖 𝒄𝒎 from the centre O. If tangents
perimeter of ∆𝑨𝑩𝑪 will be:
drawn at A and B intersect at P. the length of
the tangent AP (in cm) is: ABC एक समकोर् त्रिभज
ु है | स्जसमें B समकोर्
𝟏𝟎 𝒄𝒎 त्रिज्या वाले वत्त
ृ की जीवा AB, केंद्र O से है AC पर त्रबंद ु D इस प्रकार स्स्ित है कक 𝑨𝑫 =
𝟖 𝒄𝒎 की दरु ी पर स्स्ित है | यदद A और B पर 𝟏𝟐𝒄𝒎 और 𝑪𝑫 = 𝟏𝟔 𝒄𝒎 है | यदद BD, ∠𝑨𝑩𝑪
खीची र्ई स्पर्गरेखा P पर प्रततछे ददत हैं, तो AP को समद्ववभास्जत करती है तो, ∆𝑨𝑩𝑪 का
की लम्बाई (cm में ) ज्ञात करें | पररमाप ज्ञात करें |
a) 𝟕. 𝟓 b) 𝟒 a) 𝟔𝟔 𝒄𝒎 b) 𝟓𝟔. 𝟐 𝒄𝒎
c) 𝟑. 𝟕𝟓 d) 𝟏𝟓 c) 𝟔𝟔. 𝟐 𝒄𝒎 d) 𝟔𝟕. 𝟐 𝒄𝒎
43. What is the cost (in Rs.) of levelling a ∆𝑨𝑩𝑪 का पररमाप 𝟑𝟔 𝒄𝒎 है , तो BC की लंबाई
triangular park with sides 𝟔𝟎 𝒎, 𝟏𝟏𝟐 𝒎 and
𝟏𝟔𝟒 𝒎 at the rate of Rs.10.50 per 𝒎²? (सेमी. में) ज्ञात करें |
a) 5 b) 12
रु 10.50 प्रतत 𝒎² की दर से 𝟔𝟎 𝒎, 𝟏𝟏𝟐 𝒎 और
c) 10 d) 13
𝟏𝟔𝟒 𝒎 भुजा वाले त्रिभुजाकार पाकग को समतल
करने की लार्त (रु में) ज्ञात करें | 47. The in-radius and circumradius of a right-
a) 21,168 b) 21,067 angled triangle is 𝟑 𝒄𝒎 and 𝟏𝟐. 𝟓 𝒄𝒎.
c) 21,000 d) 20,160 respectively. The area of the triangle is:
ककसी समकोर् त्रिभज
ु की अन्द्त: त्रिज्या और
44. BD and CE are the medians of ∆𝑨𝑩𝑪, right
𝟓√𝟏𝟑
पररत्रिज्या क्रमर्: 𝟑 𝒄𝒎 और 𝟏𝟐. 𝟓 𝒄𝒎 हैं।
angled at A. If 𝑪𝑬 = 𝟐 𝒄𝒎, 𝑩𝑪 = 𝟏𝟎𝒄𝒎,
त्रिभज
ु का क्षेिफल ज्ञात करें |
then the length of BD is:
a) 𝟔𝟒 𝒄𝒎𝟐 b) 𝟒𝟖 𝒄𝒎𝟐
BD और CE, ∆𝑨𝑩𝑪 को मास्ध्यकाएं हैं जो कक A
c) 𝟖𝟖 𝒄𝒎𝟐 d) 𝟖𝟒 𝒄𝒎𝟐
पर समकोर् है | यदद 𝑪𝑬 =
𝟓√𝟏𝟑
𝒄𝒎, 𝑩𝑪 =
𝟐
𝟏𝟎𝒄𝒎 है, तो BD की लंबाई ज्ञात करें | 48. One side of a rectangular field is 𝟑𝟗 𝒎 and its
𝟓 diagonal is 𝟖𝟗 𝒎. What is the area of the
a) 𝟑√𝟕𝒄𝒎 b) 𝟐 √𝟕𝒄𝒎 field?
c) √𝟏𝟑𝒄𝒎 d) 𝟑√𝟓𝒄𝒎 एक आयताकार खेत की भुजा 𝟑𝟗 𝒎 है और

45. In ∆𝑿𝒀𝒁, P is the midpoint of side XZ and Q is इसका ववकर्ग 𝟖𝟗 𝒎 है | खेत का क्षेिफल ज्ञात
a point on side XY such that QZ bisects PY. If कीस्जए|
𝑿𝑸 = 𝟐𝟒 𝒄𝒎. then what is the length (in cm) a) 𝟐𝟏𝟔𝟎 𝒎² b) 𝟑𝟏𝟐𝟎 𝒎²
of QY? c) 𝟑𝟏𝟒𝟎 𝒎² d) 𝟐𝟏𝟎𝟎 𝒎²
∆𝑿𝒀𝒁 में, त्रबन्द्द ु P भुजा XZ का मध्यत्रबन्द्द ु है और
49. D, E and F are the feet of the perpendiculars
त्रबन्द्द ु Q भुजा XY पर इस प्रकार स्स्ित है कक
from the vertices A, B and C, respectively, of a
QZ, PY को समद्ववभास्जत करता है। यदद triangle ABC. If angle BED and angle BFE (in
𝑿𝑸 = 𝟐𝟒 𝒄𝒎. है , तो QY की लंबाई (cm में) degree) are 24 and 110 respectively, what is
the measure (in degree) of angle EBF?
ज्ञात करें |
एक त्रिभुज ABC में र्ीर्ग A, B और C से क्रमर्:
a) 6 b) 18
c) 8 d) 12 D, E और F लंब के पाद हैं| यदद कोर् BED और
कोर् BFE (अंर् में) क्रमर्: 24 और 110 है , तो
46. In an isosceles triangle ABC, 𝑨𝑩 = 𝑨𝑪 and
AD is perpendicular to BC. If 𝑨𝑫 = कोर् EBF का माप (अंर् में) ककतना होर्ा?
𝟏𝟐 𝒄𝒎 and the perimeter of ∆𝑨𝑩𝑪 is 𝟑𝟔 𝒄𝒎. a) 55 b) 67
Then 𝑩𝑪 =? c) 86 d) 46
समद्ववबाहु त्रिभुज ABC में , 𝑨𝑩 = 𝑨𝑪 और
50. Chord AB of a circle with radius 𝟓 𝒄𝒎 is at a
AD, BC के लंबवत है। यदद 𝑨𝑫 = 𝟏𝟐 𝒄𝒎 और distance of 𝟒 𝒄𝒎 from the centre O. If
tangents drawn at A and B intersect at P, then ∆𝑨𝑩𝑪 में , और E क्रमर्ः भुजा AB और AC पर
find the length of the tangent AP.
एसे त्रबंद ु है , कक ∠𝑨𝑬𝑫 = ∆𝑨𝑩𝑪 है | यदद
𝟓 𝒄𝒎 त्रिज्या वाले एक वत्त
ृ जीवा AB, केंद्र O से
𝑨𝑬 = 𝟔 𝒄𝒎, 𝑩𝑫 = 𝟐 𝒄𝒎, 𝑫𝑬 = 𝟑 𝒄𝒎 और
𝟒 𝒄𝒎 की दरू ी पर है| यदद त्रबन्द्दओ
ु ं A और B पर
𝑩𝑪 = 𝟓 𝒄𝒎, है तो (𝑨𝑩 + 𝑨𝑪) के----------
खींची र्ई स्पर्गरेखाएं, त्रबंद ु P पर प्रततच्छे ददत
बराबर होर्ा|
करती है , तो स्पर्गरेखा AP की लम्बाई ज्ञात करें | 𝟖𝟔 𝟒𝟗
a) b)
a) 𝟕. 𝟓 𝒄𝒎 b) 𝟑. 𝟕𝟓 𝒄𝒎 𝟑 𝟑
𝟓𝟎 𝟕𝟎
c) 𝟑 𝒄𝒎 d) 𝟐. 𝟒 𝒄𝒎 c) d)
𝟑 𝟑

FOR VIDEO SOLUTIONS 53. A square park has been divided into two
FROM QUESTIONS 51 TO 60 CLICK HERE rectangles of equal area. If the perimeter of
OR SCAN QR CODE GIVEN BELOW each of these rectangles is 39 m, then what
will be the perimeter of the square park?
एक वर्ागकार पाकग को समान क्षेिफल वाले दो
आयतों में ववभास्जत ककया र्या है। यदद इन
आयतों में से प्रत्येक का पररमाप 39 m है , तो
वर्ागकार पाकग का पररमाप ज्ञात करें |
51. A circle touches the side BC of ∆𝑨𝑩𝑪 at P and a) 104 m b) 39 m
also touches AB and AC produced at Q and R, c) 78 m d) 52 m
respectively. If the perimeter of ∆𝑨𝑩𝑪 =
𝟏𝟒. 𝟏 𝒄𝒎, then the length (in cm) of AQ will 54. If S is a point on side QR of a triangle PQR such
be: that 𝑸𝑺 = 𝟏𝟎 𝒄𝒎, 𝑸𝑹 = 𝟏𝟖 𝒄𝒎 and
एक वत्त
ृ ∆𝑨𝑩𝑪 की भुजा BC को P पर स्पर्ग ∠𝑷𝑺𝑹 = ∠𝑸𝑷𝑹, then the length of PR will
be:
करता है और आर्े बढाई र्ई AB और AC को
यदद त्रिभुज PQR की भुजा QR पर त्रबंद ु S इस
भी क्रमर्ः Q और R पर स्पर्ग करता है| यदद
प्रकार स्स्ित है कक 𝑸𝑺 = 𝟏𝟎 𝒄𝒎, 𝑸𝑹 =
∆𝑨𝑩𝑪 का पररमाप 𝟏𝟒. 𝟏 𝒄𝒎 है, तो की लंबाई
𝟏𝟖 𝒄𝒎 और ∠𝑷𝑺𝑹 = ∠𝑸𝑷𝑹 है , तो PR की
𝟏𝟒. 𝟏 (cm में)-----------होर्ी|
लम्बाई ज्ञात करें |
a) 𝟏𝟎. 𝟑 b) 𝟕. 𝟎𝟓
a) 𝟏𝟔 𝒄𝒎 b) 𝟏𝟐 𝒄𝒎
c) 𝟔. 𝟐𝟓 d) 𝟕. 𝟓
c) 𝟏𝟓 𝒄𝒎 d) 𝟏𝟒 𝒄𝒎
52. In ∆𝑨𝑩𝑪, D and E are the points on the sides
55. If the length of a chord of a circle, that makes
AB and AC, respectively such that ∠𝑨𝑬𝑫 =
an angle of 𝟔𝟎° with the tangent drawn at one
∆𝑨𝑩𝑪, if 𝑨𝑬 = 𝟔 𝒄𝒎, 𝑩𝑫 = 𝟐 𝒄𝒎, 𝑫𝑬 =
𝟑 𝒄𝒎 and 𝑩𝑪 = 𝟓 𝒄𝒎, (𝑨𝑩 + 𝑨𝑪) is equal end point of the chord, is 𝟖√𝟑 𝒄𝒎. then the
to: radius of the circle will be:
यदद ककसी वत्त
ृ की जीवा की लम्बाई 𝟖√𝟑 𝒄𝒎 ∆ 𝑨𝑩𝑪 में ∠𝑨 = 𝟏𝟑𝟓°, 𝑪𝑨 = 𝟓√𝟐𝒄𝒎 और 𝐀𝐁 =
हैं, जो कक जीवा के एक छोर पर खींची र्ई 𝟕 𝐜𝐦 है | E और F, क्रमर्: भज
ु ा AC और AB के
स्पर्गरेखा के साि 𝟔𝟎° का कोर् बनाती है , तो मध्य त्रबंद ु है | EF की लंबाई (cm में) (ज्ञात करें |
वत्त
ृ की त्रिज्या ज्ञात करें | a) 𝟔. 𝟓 b) 𝟓. 𝟓
a) 𝟖 𝒄𝒎 b) 𝟓 𝒄𝒎 c) 𝟔 d) 𝟓
c) 𝟔 𝒄𝒎 d) 𝟕 𝒄𝒎
59. Chord AB and diameter CD of a circle meet at
56. The sides AB, BC and AC, of ∆𝑨𝑩𝑪 are the point P, outside the circle when produced.
𝟏𝟐𝒄𝒎, 𝟖𝒄𝒎 and 𝟏𝟎𝒄𝒎, respectively. A circle If 𝑷𝑩 = 𝟖𝒄𝒎, 𝑨𝑩 = 𝟏𝟐 𝒄𝒎 and the
is inscribed in the triangle touching AB, BC and distance of P from the centre of the circle is
AC at D, E and F, respectively. The ratio of the 𝟏𝟖 𝒄𝒎, the radius (in cm) of the circle is
length of AD to CE is: closest to:
जब ककसी वत्त
ृ की जीवा AB और व्यास CD को
∆𝑨𝑩𝑪 की AB, BC और AC भज
ु ाएं क्रमर्:
बढाया जाता है , तो वे वत्त
ृ के बाहर त्रबंद ु P पर
𝟏𝟐𝐜𝐦, 𝟖𝐜𝐦 और 𝟏𝟎𝐜𝐦 है| त्रिभुज में एक वत्त

ममलती है | यदद 𝑷𝑩 = 𝟖𝒄𝒎, 𝑨𝑩 =
अंत:स्िावपत है , जो क्रमर्: D, E और F पर AB,
𝟏𝟐 𝒄𝒎 और वत्त
ृ के केंद्र से P की दरू ी 𝟏𝟖 𝒄𝒎
BC और AC को स्पर्ग करता है | AD और CE की
है , तो वत
ृ की त्रिज्या (cm में) ज्ञात करें
लम्बाईयों का अनुपात ज्ञात करें |
(तनकटतम)|
a) 𝟓 : 𝟕 b) 𝟑 : 𝟓 a) 𝟏𝟐. 𝟖 b) 𝟏𝟐. 𝟒
c) 𝟏𝟎 : 𝟕 d) 𝟕 : 𝟑 c) 𝟏𝟑 d) 𝟏𝟐

57. One diagonal of a rhombus is 𝟖√𝟑 𝒄𝒎. If the 60. The altitude AD of a triangle ABC is 𝟗 𝒄𝒎. If
other diagonal is equal to its side, then the 𝑨𝑩 = 𝟔 √𝟑 𝒄𝒎 and 𝑪𝑫 = 𝟑√𝟑 𝒄𝒎, then
area (𝒊𝒏 𝒄𝒎𝟐 ) of the rhombus is: what will be the measure of ∠𝑨?
ककसी समचतभ
ु ज
ुग का एक ववकर्ग 𝟖√𝟑 𝒄𝒎. है त्रिभज
ु ABC की ऊँचाई AD, 𝟗 𝒄𝒎 है| यदद 𝑨𝑩 =
यदद दस
ू रा ववकर्ग, इसकी भज
ु ा के बराबर है, तो 𝟔 √𝟑 𝒄𝒎 और 𝑪𝑫 = 𝟑√𝟑 𝒄𝒎 है , तो ∠𝑨 की
समचतभ
ु ज
ुग का क्षेिफल (𝒄𝒎 मे) मे ज्ञात करें |
𝟐
माप ज्ञात करें |
a) 𝟏𝟐√𝟑 b) 𝟐𝟒√𝟑 a) 𝟗𝟎° b) 𝟑𝟎°
c) 𝟏𝟔√𝟑 d) 𝟑𝟐√𝟑 c) 𝟒𝟓° d) 𝟔𝟎°

58. If ∆ 𝑨𝑩𝑪, ∠𝑨 = 𝟏𝟑𝟓°, 𝑪𝑨 = 𝟓√𝟐𝒄𝒎 and FOR VIDEO SOLUTIONS


𝑨𝑩 = 𝟕 𝒄𝒎. E and F are and midpoints of FROM QUESTIONS 61 TO 70 CLICK HERE
sides AC and AB, respectively. The length of EF OR SCAN QR CODE GIVEN BELOW
(in cm) is:
a) 𝟑𝒄𝒎 b) 𝟕. 𝟓𝒄𝒎
c) 𝟓 𝒄𝒎 d) 𝟒 𝒄𝒎

64. ∆𝑷𝑸𝑹 is inscribed in a circle with center O. PO


is produced to meet QR at U and the circle at
S, and 𝑷𝑻 ⊥ 𝑸𝑹, where T lies between Q and
U. If ∠𝑸 = 𝟕𝟎° and ∠𝑹 = 𝟓𝟓°, then what is
61. In ∆𝑨𝑩𝑪, AD is a median. If points E, F and G the measure (in degrees) of ∠𝑻𝑷𝑺?
are midpoints of AD, AE and DE, respectively,
∆𝑷𝑸𝑹, O केन्द्द्र वाले वत्त
ृ के भीतर उत्कीखर्गत
then what will be the area ∆𝑩𝑭𝑮?
∆𝑨𝑩𝑪 में AD एक मस्ध्यका है | यदद त्रबंद ु E, F है | PO को आर्े बढाने पर यह QR से त्रबन्द्द ु U

और क्रमर्: AD, AE और DE के मध्य त्रबंद ु हैं, तो पर ममलती है और वत्त


ृ से त्रबन्द्द ु S पर ममलती

∆𝑩𝑭𝑮 का क्षेिफल होर्ा? है और 𝑷𝑻 ⊥ 𝑸𝑹 है, जहाँ त्रबन्द्द ु Q और त्रबन्द्द ु U


𝟏
a) 𝟐 (𝑨𝒓𝒆𝒂 ∆𝑨𝑩𝑪)
𝟏
b) 𝟖 (𝑨𝒓𝒆𝒂 ∆𝑨𝑩𝑪) के बीच त्रबन्द्द ु T स्स्ित है | ∠𝑸 = 𝟕𝟎° और
𝟏
c) 𝟒 (𝑨𝒓𝒆𝒂∆𝑨𝑩𝑪)
𝟏
d) 𝟐 (𝑨𝒓𝒆𝒂 ∆𝑨𝑩𝑪) ∠𝑹 = 𝟓𝟓° है , तो ∠𝑻𝑷𝑺 की माप (डडग्री में)
ज्ञात करें |
62. If a and b are the lengths of two sides of a a) 25 b) 30
triangle such that the product 𝒂𝒃 = 𝟐𝟒, c) 15 d) 20
where a and b are integers, the how many
such triangles are possible? 65. A wire is bent to form a square of area
यदद ककसी त्रिभुज की दो भुजाओं की लंबाई a 𝟏𝟔𝟗 𝒄𝒎𝟐 , If the same wire is bent to form a
circle, then what is its area (in 𝒄𝒎𝟐 , to the
और b इस प्रकार है कक र्ुर्नफल 𝒂𝒃 = 𝟐𝟒 है, nearest whole number)?
जहां a और b पूर्ागक है , तो ऐसे संभाववत एक तार को 𝟏𝟔𝟗 𝒄𝒎𝟐 क्षेिफल का वर्ग बनाने
त्रिभज
ु ाओं की संख्या ज्ञात करें | के मलए मोड़ा जाता है | यदद उसी तार को वत्त

a) 15 b) 12
बनाने के मलए मोड़ा जाए, तो इसका क्षेिफल
c) 18 d) 16
(𝒄𝒎𝟐 में , तनकटतम पूर्ग संख्या तक) क्या होर्ा?
63. In ∆𝑨𝑩𝑪, 𝑫𝑬 ∥ 𝑨𝑩, Where D and E are points a) 215 b) 227
on sides AC and BC, respectively. F is a point c) 532 d) 531
between C and D such that 𝑬𝑭 ∥ 𝑩𝑫. If 𝐀𝐃 =
𝟏𝟓 𝒄𝒎, 𝑫𝑪 = 𝟏𝟎 𝒄𝒎, then the length of CF is: 66. How many isosceles triangles with integer
∆𝑨𝑩𝑪 में, 𝑫𝑬 ∥ 𝑨𝑩 है, जहाँ D और E क्रमर्: sides are possible such that the sum of two of
the sides is 𝟏𝟔 𝒄𝒎?
भज
ु ा AC और BC पर स्स्ित त्रबंद ु हैं| C और D के
पूर्ाांक भुजओं वाले ऐसे ककतने समद्ववबाहु
बीच त्रबन्द्द ु F इस प्रकार है कक 𝑬𝑭 ∥ 𝑩𝑫 है|
त्रिभुज संभव हैं स्जनमें दो भुजाओं का योर्
यदद 𝐀𝐃 = 𝟏𝟓 𝒄𝒎, 𝑫𝑪 = 𝟏𝟎 𝒄𝒎 है, तो CF की
लंबाई ज्ञात करें | 𝟏𝟔 𝒄𝒎 हो?
a) 18 b) 15
c) 9 d) 24 circle at A and B meet each other at a point D.
If ∠𝑨𝑫𝑩 = 𝟐𝟓°, then ∠𝑨𝑪𝑩 (in degrees) is
67. The area of a circle that is inscribed in a square equal to:
O केंद्र वाले वत्त
ृ पर दो त्रबन्द्द ु A और B स्स्ित हैं|
𝟗
of area 𝟏𝟕 𝟏𝟏 𝒄𝒎𝟐 is:
𝟏𝟕 𝟏𝟏 𝒄𝒎𝟐 क्षेिफल वाले वर्ग के भीतर तनममगत
𝟗 वत्त
ृ के लघु चाप पर त्रबन्द्दओ
ु ं A और B के बीच

अंत:वत्त एक त्रबन्द्द ु स्स्ित C है| त्रबन्द्द ु A और B पर वत्त



ृ का क्षेिफल ज्ञात करें |
a) 𝟐𝟐 𝒄𝒎² b) 𝟐𝟖 𝒄𝒎² की स्पर्ग रे खाएँ एक दस
ू रे से त्रबन्द्द ु D पर
c) 𝟏𝟒 𝒄𝒎² d) 𝟏𝟔 𝒄𝒎² ममलती हैं। यदद ∠𝑨𝑫𝑩 = 𝟐𝟓° है, तो ∠𝑨𝑪𝑩 का
मान (अंर् में) ज्ञात करें |
68. The area of a triangular field whose sides are
a) 105 b) 𝟏𝟎𝟎. 𝟓
𝟗𝟔 𝒎, 𝟏𝟏𝟎 𝒎, and 𝟏𝟒𝟔 𝒎 is equal to the area
c) 𝟏𝟎𝟐. 𝟓 d) 100
of a rectangular park whose sides are in the
ratio 𝟑 : 𝟐. What is the perimeter (in m) of the
FOR VIDEO SOLUTIONS
rectangular park?
FROM QUESTIONS 71 TO 80 CLICK HERE
𝟗𝟔 𝒎, 𝟏𝟏𝟎 𝒎 और 𝟏𝟒𝟔 𝒎 भुजा वाले OR SCAN QR CODE GIVEN BELOW
त्रिभुजाकार मैदान का क्षेिफल, उस आयताकार
पाकग के क्षेिफल के बराबर है , स्जसकी भुजाओं
का अनुपात 𝟑 : 𝟐 है। आयताकार पाकग का
पररमाप (मीटर में ) ज्ञात करें |
a) 𝟖𝟎√𝟓 b) 𝟒𝟎√𝟓𝟓
c) 𝟐𝟎√𝟓𝟓 d) 𝟒𝟎√𝟏𝟏 71. ∆𝑷𝑸𝑹 is inscribed in a circle. The bisector of
∠𝑷 cuts 𝑸𝑹 at 𝑺 and the circle at 𝑻. If 𝑷𝑹 =
69. In ∆𝑨𝑩𝑪, ∠𝑩 = 𝟗𝟎°, AD and CE are the 𝟓𝒄𝒎, 𝑷𝑺 = 𝟔𝒄𝒎 and 𝑺𝑻 = 𝟒𝒄𝒎, then the
medians drawn from A and C, respectively. If length (in cm) of 𝑷𝑸 is:
𝑨𝑪 = 𝟏𝟎 𝒄𝒎 and 𝑨𝑫 = √𝟓𝟓𝒄𝒎, then the ∆𝑷𝑸𝑹 को ककसी वत्त
ृ में अन्द्ततनममगत ककया र्या
length of CE is:
है | ∠𝑷 का समद्ववभाजक, 𝑸𝑹 को त्रबंद ु 𝑺 पर
∆𝑨𝑩𝑪 में, ∠ ∠𝑩 = 𝟗𝟎° है| AD और CE क्रमर्ः
कटता है और वत्त
ृ को त्रबंद ु 𝑻 पर काटता है |
त्रबन्द्दओ
ु ं A और C से खींची र्ई मास्ध्यकाएँ हैं।
यदद 𝑷𝑹 = 𝟓𝒄𝒎, 𝑷𝑺 = 𝟔𝒄𝒎 और 𝑺𝑻 = 𝟒𝒄𝒎
यदद 𝑨𝑪 = 𝟏𝟎 𝒄𝒎 और 𝑨𝑫 = √𝟓𝟓𝒄𝒎 है, तो
है , तो 𝑷𝑸 की लम्बाई (सेमी. में) ज्ञात करें |
CE की लंबाई ज्ञात करें |
a) 13 b) 12
a) 𝟐√𝟏𝟓 𝒄𝒎 b) √𝟕𝟎 𝒄𝒎 c) 10 d) 15
c) √𝟔𝟔 𝒄𝒎 d) 𝟓√𝟑 𝒄𝒎
72. The area of a triangular field whose sides are
70. A and B are two points on a circle with centre 𝟔𝟓𝒎, 𝟕𝟐𝒎, and 𝟗𝟕𝒎 is equal to the area of a
O, C is a point on the minor arc of the circle rectangular park whose sides are in the ratio
between points A and B. The tangents to the
of 𝟓 : 𝟏𝟑. What is the perimeter (in m) of the ∠𝑫𝑨𝑷 = 𝟑𝟔° and∠𝑨𝑷𝑪 = 𝟑𝟎°, then what
rectangular park? will be the measure∠𝑪𝑩𝑫?
𝟔𝟓𝒎, 𝟕𝟐𝒎 और 𝟗𝟕𝒎 भज
ु ा वाले त्रिभज
ु ाकार ककसी वत्त
ृ की जीवा 𝑨𝑩 और 𝑪𝑫 को बढाया
मैदान का क्षेिफल, उस आयताकार पाकग के जाता है , तो वत्त
ृ का बाहर त्रबंद ु 𝑷 पर ममलती है
क्षेिफल के बराबर है , स्जसकी भुजाओ का और 𝑨𝑫, वत्त
ृ का ब्यास है | यदद ∠𝑫𝑨𝑷 = 𝟑𝟔°
अनुपात 𝟓: 𝟏𝟑 है | आयताकार पाकग का पररमाप है , तो ∠𝑪𝑩𝑫 का माप ज्ञात करें |
(मीटर में) ज्ञात करें | a) 𝟑𝟒° b) 𝟐𝟔°
a) 108 b) 180 c) 𝟐𝟒° d) 𝟏𝟔°
c) 216 d) 144
76. The perimeter of a right triangle is 𝟔𝟎 𝒄𝒎 and
73. In ∆𝑨𝑩𝑪, 𝑫 and 𝑬 are points on sides 𝑨𝑩 and its hypotenuse is 𝟐𝟔 𝒄𝒎. What is the area (in
𝒄𝒎𝟐 ) of the triangle ?
𝑩𝑪, respectively, such that 𝑩𝑫 : 𝑫𝑨 =
𝟏 : 𝟐 and 𝑪𝑬 : 𝑬𝑩 = 𝟏 : 𝟒. If 𝑫𝑪 ककसी समकोर् त्रिभुज का पररमाप 𝟔𝟎𝒄𝒎 है
and 𝑨𝑬 intersect at 𝑭, then 𝑭𝑫 : 𝑭𝑪 is equal और इसका कर्ग 𝟐𝟔𝒄𝒎 है| त्रिभुज का क्षेिफल
to: (𝒎𝟐 में) ज्ञात करें |
∆𝑨𝑩𝑪 में, भज
ु ा 𝑨𝑩 और 𝑩𝑪 पर क्रमर्: त्रबंद ु 𝑫 a) 60 b) 96 c) 90 d) 120
और 𝑬 इस प्रकार स्स्ित है कक 𝑩𝑫 : 𝑫𝑨 =
77. The sides of a triangle are 𝟐𝟒 𝒄𝒎, 𝟐𝟔 𝒄𝒎 and
𝟏 : 𝟐 और 𝑪𝑬 : 𝑬𝑩 = 𝟏 : 𝟒 है| यदद 𝑫𝑪 और 𝟏𝟎 𝒄𝒎. At each of its vertices, circles of radius
𝑨𝑬 एक दस
ु रे को त्रबंद ु 𝑭 पर प्रततछे ददत करती 𝟒. 𝟐 𝒄𝒎 are drown. What is the area (in 𝒄𝒎𝟐 )
of the triangle, excluding the portion covered
है , तो 𝑭𝑫 : 𝑭𝑪 का मान ज्ञात करें | by the sectors of the circles?
a) 𝟑: 𝟐 b) 𝟓: 𝟐 𝟐𝟐
(𝝅 = )
𝟕
c) 𝟖: 𝟑 d) 𝟒: 𝟏
ककसी त्रिभुज की भुजाए 𝟐𝟒𝒄𝒎, 𝟐𝟔𝒄𝒎 और
74. In ∆𝑷𝑸𝑹, ∠𝑷𝑸𝑹 = 𝟏𝟑𝟓°, 𝑷𝑸 = 𝟖√𝟐𝒄𝒎 and 𝟏𝟎𝒄𝒎 है | इसके प्रतेक र्ीर्ग को छूता हुआ
𝑷𝑹 = 𝟏𝟕𝒄𝒎. What is the length (in cm) of 𝟒. 𝟐𝒄𝒎 त्रिज्या वाला वत्त
ृ खीचा जाता है | वत्त

𝑸𝑹?
के खंडो द्वारा कवर ककय र्ए भार् को छोड़कर,
∆𝑷𝑸𝑹 में, ∠𝑷𝑸𝑹 = 𝟏𝟑𝟓° है , 𝑷𝑸 = 𝟖√𝟐𝒄𝒎
त्रिभुज का क्षेिफल (𝒄𝒎𝟐 में) ज्ञात करें |
और 𝑷𝑹 = 𝟏𝟕𝒄𝒎 है | 𝑸𝑹 की लम्बाई (cm में
a) 120 b) 𝟏𝟎𝟓. 𝟖𝟔
) ज्ञात करें | c) 𝟗𝟐. 𝟐𝟖 d) 𝟐𝟕. 𝟕𝟐
a) 𝟕 b) 𝟏𝟎
c) 𝟖 d) 𝟗 78. A rectangular lawn whose length is twice of its
breadth is extended by having four semi-
75. The Chords 𝑨𝑩 and 𝑪𝑫 of a circle are circular portions on its sides. What is the total
produced to meet at the point 𝑷, outside the cost (in ₹) of levelling the entire lawn at the
circle, and 𝑨𝑫 is the diameter of the circle. If rate of ₹100 per square metre, if the smaller
side of the rectangular lawn is 𝟏𝟐 𝒎? OR SCAN QR CODE GIVEN BELOW
(𝑻𝒂𝒌𝒆 𝝅 = 𝟑. 𝟏𝟒)
कोई आयताकार लॉन, स्जसकी लम्बाई उसकी
चौड़ाई की दोर्ुनी है, स्जसे इसकी भुजाओ पर
चार अिगवत
ृ ाकार आकृततया बनाने के मलए
बढाया जाता है | पुरे लॉन को ₹100 प्रतत वर्ग
मीटर की लार्त से समतल करवाने के मलए 81. The sides of a triangular field are 𝟗𝟔 𝒎,
ककतने रुपये खचग होंर्े, यदद आयताकार लॉन की 𝟏𝟏𝟎 𝒎 and 𝟏𝟒𝟔 𝒎. The cost of levelling the
field at ₹5.60 per 𝒎𝟐 is:
छोटी भुजा की लम्बाई 12m है | (𝝅 = 𝟑. 𝟏𝟒 ले
एक त्रिभुजाकार मैदान की भुजाएँ 𝟗𝟔 𝒎, 𝟏𝟏𝟎 𝒎
)
और 𝟏𝟒𝟔 𝒎 हैं। खेत को ₹5.60 प्रतत 𝒎𝟐 पर
a) 85,320 b) 78,650
c) 86,540 d) 97,625 समतल करने की लार्त ज्ञात करें |
a) ₹30,564 b) ₹31,680
79. A rectangle of dimension 𝟏𝟎 𝒄𝒎 and 𝟓 𝒄𝒎 is c) ₹28,224 d) ₹29,568
placed adjacent to another rectangle of the
same size to draw an L shape figure. Find the 82. A path 𝟔 𝒎 wide runs around and outside of a
perimeter of the shape so formed. rectangular plot of length 𝟏𝟎 𝒎 and breadth
L आकार की आकृतत बनाने के मलए, 𝟏𝟎 𝒄𝒎 और 𝟖 𝒎. The are (𝒊𝒏 𝒎𝟐 ) of the path is:
𝟓 𝒄𝒎 भुजाओं वाले आयत को समान आकार एक 𝟏𝟎 𝒎 लंबे और 𝟖 𝒎 चौड़े आयताकार प्लॉट

वाले दस के बाहर चारों ओर एक 𝟔 𝒎 चौड़ा रस्ता है|


ू रे आयत के बर्ल में रखा जाता है ।
इस प्रकार बनी आकृतत का पररमाप ज्ञात करें । रास्ते का क्षेिफल (𝒎𝟐 में) ककतना है ?
a) 𝟏𝟎𝟎 𝒄𝒎 b) 𝟒𝟎 𝒄𝒎 a) 440 b) 600 c) 80 d) 360
c) 𝟓𝟎 𝒄𝒎 d) 𝟔𝟎 𝒄𝒎
83. A rectangular plot is 𝟒𝟐𝟎 𝒎 long and 𝟖𝟒 𝒎
80. If the width of the path around a square field wide. It is surrounded by semi-circular flower
beds along all its sides. The area of the entire
is 𝟒. 𝟓 𝒎 and the area of the path is 𝟐𝟓𝟐 𝒎𝟐 ,
plot (in hectares) is 𝒙. The value of 𝒙 (correct
then the length of the side of the field is: 𝟐𝟐
यदद एक वर्ागकार मैदान के चारों ओर पि की to one decimal place) is (𝑻𝒂𝒌𝒆 𝝅 = ):
𝟕

चौड़ाई 𝟒. 𝟓 𝒎 है और पि का क्षेिफल कोई आयताकार प्लाट 𝟒𝟐𝟎 𝒎 लम्बा और 𝟖𝟒 𝒎

𝟐𝟓𝟐 𝒎𝟐 है, तो मैदान की भुजा की लंबाई ज्ञात चौड़ा है | इसकी सभी भुजाओं के अनुददर् अद्गि-

करें | वत्त
ृ ीय फुलवाररयों बनी हुई है | पूरे प्लाट का
a) 𝟗. 𝟓 𝒎 b) 𝟗 𝒎 क्षेिफल (हे क्टे यर में) 𝒙 है| 𝒙 का मान (ठीक
c) 𝟖 𝒎 d) 𝟖. 𝟓 𝒎 एक दर्मलव स्िान तक) ज्ञात करें |
a) 𝟏𝟗. 𝟏 b) 𝟏𝟖. 𝟕
FOR VIDEO SOLUTIONS
c) 𝟏𝟕. 𝟗 d) 𝟏𝟗. 𝟒
FROM QUESTIONS 81 TO 90 CLICK HERE
भुजा का, छोटी वाली भुजा से अनुपात ज्ञात
84. A wire is in the form of a square of side
𝟑𝟑 𝒄𝒎. If the wire is molded to form a circle, कीस्जए?
then what is the radius of the circle? a) 𝟕 : 𝟐𝟒 b) 𝟏𝟏 : 𝟏𝟖
c) 𝟐𝟒 : 𝟕 d) 𝟏𝟖 : 𝟏𝟏
कोई तार 𝟑𝟑 𝒄𝒎 भुजा वाले वर्ग के आकार में
है | यदद तार को वत्त
ृ बनाने के मलए ढाला जाता 87. The length of a rectangle is three-fifth of the
है , तो वत्त
ृ की त्रिज्या ज्ञात करें | radius of a circle. The radius of the circle is
a) 𝟐𝟏 𝒄𝒎 b) 𝟑𝟑 𝒄𝒎 equal to the side of a square whose area is
c) 𝟏𝟔. 𝟓 𝒄𝒎 d) 𝟒𝟐 𝒄𝒎 𝟔𝟒𝟎𝟎 𝒎². the are (𝒊𝒏 𝒎𝟐 ) of the rectangle, if
the breadth is 𝟐𝟎 𝒎 is:
85. A typist uses a paper of size 𝟑𝟐 𝒄𝒎 × 𝟐𝟎 𝒄𝒎. एक आयत की लंबाई, एक वत्त
ृ की त्रिज्या की
है । उस वत्त
ृ की त्रिज्या एक ऐसे वर्ग की भुजा
He leaves a margin of 𝟐 𝒄𝒎 each on all the 𝟑
𝟓
sides. If he leaves a margin of 𝟏 𝒄𝒎 only on all
the sides, then what is the percentage of the के बराबर है स्जसका क्षेिफल 𝟔𝟒𝟎𝟎 𝒎² है। यदद
increase in the area available for typing आयत की चौड़ाई 𝟐𝟎 𝒎 है, तो उसका क्षेिफल
(correct to 2 decimal places)?
(𝒎𝟐 में) ज्ञात कीस्जए|
कोई टाइवपस्ट, 𝟑𝟐 𝒄𝒎 × 𝟐𝟎 𝒄𝒎 आकार वाले a) 1200 b) 480
पेपर का उपयोर् करता है | वह सभी भुजाओं पर c) 960 d) 1000
𝟐 𝒄𝒎 का ककनारा छोड़ता है | यदद वह सभी
88. A square park has area 𝟒𝟑𝟓𝟔𝒎². Taking its
भुजाओं पर केवल 𝟏 𝒄𝒎 का ककनारा छोड़े, तो one round is same as taking one round of
टाइवपंर् के मलए उपलब्ि क्षेिफल में
हुई another circular park. Find the area of the
𝟐𝟐
प्रततर्त वद्
ृ धि ज्ञात करें (दर्मलब के बाद दो circular park. (𝐮𝐬𝐞 𝝅 = 𝟕
)

स्िानों तक)| ककसी वर्ागकार पाकग का क्षेिफल 𝟒𝟑𝟓𝟔𝒎² है|


a) 𝟐𝟎. 𝟓𝟒% b) 𝟖𝟐. 𝟗𝟔% इसका एक चक्कर लर्ाना, दस
ू रे वत्त
ृ ाकार पाकग
c) 𝟏𝟕. 𝟎𝟒% d) 𝟕𝟗. 𝟒𝟔%
का एक चक्कर लर्ाने के समान है । वत्त
ृ ाकार
पाकग का क्षेिफल ज्ञात करें | (𝝅 = 𝟕 लें )
𝟐𝟐
86. A boy walked along two adjacent sides of a
rectangular field. If he had walked along the a) 𝟏𝟏𝟎𝟖𝟖 𝒎² b) 𝟓𝟓𝟒𝟒 𝒎²
diagonal, then he would have saved a distance c) 𝟐𝟐𝟏𝟕𝟔 𝒎² d) 𝟏𝟑𝟖𝟔 𝒎²
equal to one-fourth of the larger side. The
ratio of the larger to the smaller side is: 89. The area of a triangular field whose sides are
एक लड़का एक आयताकार मैदान की दो 96 m, 110 m and 146 m is equal to the area of
a rectangular park whose sides are in the ratio
सस्न्द्नकट भज
ु ाओं के समानांतर टहलता है । यदद 𝟏𝟎 : 𝟏𝟏. The length of the longer side of the
वह ववकर्ग के समानांतर टहला होता, तो उसे park is:
उस मेदान की बड़ी भुजा की एक-चौिाई दरू ी के 96 m, 110 m और 146 m भज
ु ा वाले त्रिभज
ु ाकार
बराबर दरू ी कम तय करनी पड़ती। बड़ी वाली मैदान का क्षेिफल, उस आयताकार पाकग के
क्षेिफल के बराबर है स्जसकी भज
ु ाओं का rectangular park whose sides are in the ratio
of 𝟐𝟏 : 𝟏𝟎. The perimeter (in m) of the park is:
अनुपात 𝟏𝟎 : 𝟏𝟏 हैं| पाकग की लंबी भुजा की
60 m, 175 m और 185 m भुजाओं वाले
लंबाई ज्ञात करें |
त्रिभुजाकार मैदान का क्षेिफल, उस आयताकार
a) 𝟒𝟒√𝟑 𝒎 b) 𝟑𝟑 𝒎
c) 𝟓𝟓 𝒎 d) 𝟑𝟑√𝟑 𝐦 पाकग के क्षेिफल के बराबर है स्जसकी भुजाओं
का अनुपात 𝟐𝟏 : 𝟏𝟎 हैं। पाकग की पररमाप (m
90. In the given figure, the area of the unshaded में) ज्ञात करें |
region is 𝟑𝟓% of the area of the shaded
a) 340 b) 403
region. What is the value of 𝒙?
c) 372 d) 310
दी र्ई आकृतत में , अछातयत क्षेि (unshaded
region ) का क्षेिफल, छातयत क्षेि के क्षेिफल का 93. The area of a triangular park with sides
𝟕𝟖 𝒎, 𝟏𝟔𝟎 𝒎 and 𝟏𝟕𝟖 𝒎 is equal to the area
𝟑𝟓% है। 𝒙 का मान ज्ञात करें |
of a rectangular garden whose sides are in the
a) 20 b) 15 c) 10 d) 5 ratio of 𝟏𝟑 : 𝟏𝟐. The smaller side (in m) of the
garden is:
FOR VIDEO SOLUTIONS
FROM QUESTIONS 91 TO 100 CLICK HERE 𝟕𝟖 𝒎, 𝟏𝟔𝟎 𝒎 और 𝟏𝟕𝟖 𝒎 भज
ु ा वाले
OR SCAN QR CODE GIVEN BELOW त्रिभज
ु ाकार मैदान का क्षेिफल, उस आयताकार
उद्यान के क्षेिफल के बराबर है स्जसकी भुजाओं
का अनप
ु ात 𝟏𝟑 : 𝟏𝟐 है | उद्यान की छोटी भज
ु ा
की लम्बाई (m में) ज्ञात करें |
a) 𝟐𝟔√𝟓 b) 𝟐𝟒√𝟏𝟎
c) 𝟏𝟐√𝟏𝟎 d) 𝟏𝟑√𝟓
91. A horse racecourse is in the form of an annular
ring whose outer and inner circumferences are 94. The perimeter and area of a rectangular sheet
𝟕𝟒𝟖 𝒎 and 𝟑𝟗𝟔 𝒎, respectively. The width (in are 𝟗𝟒 𝒎 and 𝟒𝟐𝟎 𝒎² respectively. The length
𝟐𝟐 of the diagonal will be:
m) of the racecourse is: (𝑻𝒂𝒌𝒆 𝝅 = )
𝟕
ककसी आयताकार र्ीट का पररमाप और क्षेिफल
कोई घुड़दौड़ का मैदान, वलयाकार ररंर् के रूप में
क्रमर्: 𝟗𝟒 𝒎 and 𝟒𝟐𝟎 𝒎² है| ववकर्ग की लम्बाई
है , स्जसकी बाहृय और आंतररक पररधि क्रमर्:
ज्ञात करें |
𝟕𝟒𝟖 𝒎 और 𝟑𝟗𝟔 𝒎 है| घुड़दौड़ के मैदान की
a) 𝟑𝟐 𝒎 b) 𝟑𝟔 𝒎
चौड़ाई (m में) ज्ञात करें | ( 𝝅 = लें)
𝟐𝟐
𝟕
c) 𝟑𝟓 𝒎 d) 𝟑𝟕 𝒎
a) 176 b) 88 c) 56 d) 28
95. The base of right pyramid is an equilateral
92. The area of a triangular field with sides 60 m, triangle, each side of which is 𝟐𝟎 𝒄𝒎. Each
175 m and 185 m is equal to that of a slant edge is 𝟑𝟎 𝒄𝒎. The vertical height (in
𝒄𝒎) of the pyramid is:
एक लंब वपराममड का आिार एक ऐसा समबाहु एक र्ोलाकार र्ुब्बारे में हवा भरकर उसकी
त्रिभुज है , स्जसकी प्रत्येक भुजा की लंबाई त्रिज्या को 𝟑. 𝟓 𝒄𝒎 से 𝟒. 𝟗 𝒄𝒎 तक बढाया जाता
𝟐𝟎 𝒄𝒎 है। प्रत्येक ततयगक कोर 𝟑𝟎 𝒄𝒎 है। है । मूल र्ुब्बारे के आयतन में होने वाली
वपराममड की ऊध्वागिर ऊंचाई (𝒄𝒎 में) ककतनी प्रततर्त वद्
ृ धि ककतनी है ?
होर्ी? a) 𝟕𝟑. 𝟔% b) 𝟏𝟕𝟑. 𝟔%
c) 𝟕𝟒. 𝟒% d) 𝟏𝟕𝟒. 𝟒%
𝟐𝟑
a) 𝟓√𝟑 b) 𝟓√ 𝟑
99. The radius of the base of a cylindrical tank is 𝟒
𝟐𝟑
c) 𝟏𝟎√ 𝟑 d) 𝟏𝟎√𝟑 m. If three times the sum of the sum of the
areas of its two circular faces is twice the area
of its curved surface, then the capacity (in
96. The surface area of a sphere is 𝟐𝟐𝟏. 𝟕𝟔 𝒄𝒎𝟐 .
kilolitres) of the tank is:
Its volume (in 𝒄𝒎𝟑 ) is (correct to one decimal
place): एक बेलनाकार टं की के आिार की त्रिज्या 𝟒 m
एक र्ोले का पष्ृ ठीय क्षेिफल 𝟐𝟐𝟏. 𝟕𝟔 𝒄𝒎𝟐 है। है । यदद इसके दो वत्त
ृ ाकार पष्ृ ठों के क्षेिफलों के
इसका आयतन (𝒄𝒎𝟑 में दर्मलव के एक स्िान योर् का तीन र्ुना, इसके वक्र पष्ृ ठीय क्षेिफल
तक सही) ककतना होर्ा? का दोर्ुना है, तो टं की की क्षमता (KL में) ककतनी
a) 𝟑𝟏𝟎. 𝟓 b) 𝟑𝟏𝟓. 𝟔 होर्ी?
c) 𝟐𝟖𝟗. 𝟖 d) 𝟐𝟖𝟎. 𝟒 a) 𝟏𝟒𝟒 𝝅 b) 𝟓𝟒 𝝅
c) 𝟏𝟎𝟖 𝝅 d) 𝟗𝟔 𝝅
97. A solid metallic cuboid of dimensions
𝟏𝟐 𝒄𝒎 × 𝟓𝟒 𝒄𝒎 × 𝟕𝟐 𝒄𝒎 is melted and 100. The areas of three adjacent faces of a
converted into 8 cubes of the same size. What cuboidal solid block of wax are
is the sum of the lateral surface areas (in 𝒄𝒎𝟐 ) 𝟐𝟏𝟔 𝒄𝒎𝟐 , 𝟗𝟔 𝒄𝒎𝟐 and 𝟏𝟒𝟒 𝒄𝒎𝟐 . It is melted
of 2 such cubes? and 8 cubes of the same size are formed it.
𝟏𝟐 𝒄𝒎 × 𝟓𝟒 𝒄𝒎 × 𝟕𝟐 𝒄𝒎 आयामों वाले िातु What is the leteral surface area (in 𝒄𝒎𝟐 ) of 3
के एक ठोस घनाभ को वपघलाया जाता है और such cubes?
मोम के एक घनाभाकार ठोस ब्लॉक के तीन
इससे समान आकार के 8 घन तनममगत ककए
आसन्द्न फलकों के क्षेिफल 𝟐𝟏𝟔 𝒄𝒎𝟐 , 𝟗𝟔 𝒄𝒎𝟐
जाते है। ऐसे 2 घनों के पार्शवग पष्ृ ठीय क्षेिफलों
और 𝟏𝟒𝟒 𝒄𝒎𝟐 है | इसे वपघलाकर समान आकार
(𝒄𝒎𝟐 में) का योर् ककतना होर्ा?
a) 2268 b) 1944 c) 2592 d) 3888 वाले 8 घन बनाए जाते हैं। ऐसे 3 घनों का
पार्शवग पष्ृ ठीय क्षेिफल (𝒄𝒎𝟐 में) ककतना होर्ा?
98. The radius of a spherical balloon is inflated a) 648 b) 432 c) 𝟓𝟕𝟔 d) 𝟐𝟖𝟖
from 𝟑. 𝟓 𝒄𝒎 to 𝟒. 𝟗 𝒄𝒎 by pushing air into it.
What is the percentage increase in the volume FOR VIDEO SOLUTIONS
of the original balloom? FROM QUESTIONS 101 TO 110 CLICK HERE
OR SCAN QR CODE GIVEN BELOW
ऊंचाई 𝟓𝟎% कम कर दी जाए, तो इसकी िाररता
(लीटर में , दर्मलव के एक स्िान तक सही)
ककतनी होर्ी?
a) 𝟐𝟕. 𝟖 b) 𝟐𝟖. 𝟐
c) 𝟐𝟔. 𝟕 d) 𝟐𝟓. 𝟗

101. A solid metallic sphere of radius 𝟒 𝒄𝒎 is 104. A room has length 15 feet, width 12 feet and
melted and recast into spheres of 𝟐 𝒄𝒎 each. height 10 feet. It has one door of dimensions 8
what is the ratio of the surface area of the ft and 𝟑. 𝟓 𝒇𝒕, and 2 windows of size
original sphere to the sum of surface areas of 𝟓 𝒇𝒕 × 𝟑 𝒇𝒕. Find the cost of painting the four
the spheres, so formed? walls and ceiling at ₹50 per 𝒇𝒕𝟐 .
𝟒 𝒄𝒎 त्रिज्या वाले िातु के एक ठोस र्ोले को ककसी कमरे की लंबाई 15 ft, चौड़ाई 12 ft और
वपघलाया उससे प्रत्येक 𝟐 𝒄𝒎 त्रिज्या वाले र्ोले ऊंचाई 10 ft है। इसमें 8 ft और 𝟑. 𝟓 ft भुजाओं
तनममगत ककए जाते है। मूल्य र्ोले के पष्ृ ठीय वाला एक दरवाजा है और 𝟓 𝒇𝒕 × 𝟑 𝒇 आकार
क्षेिफल और इस प्रकार बने सभी र्ोलों के की दो खखड़ककयां हैं। ₹50 प्रतत 𝒇𝒕𝟐 की दर से
पष्ृ ठीय क्षेिफलों के योर् का अनुपात क्या होर्ा? चारों दीवारों और छत को पेंट करने की लार्त
a) 𝟐 : 𝟏 b) 𝟐 : 𝟑 c) 𝟏 : 𝟐 d) 𝟏 : 𝟒
ज्ञात करें |
102. The base of a right pyramid is a square of a) ₹19,600 b) ₹21,200
side 𝟖√𝟐 𝒄𝒎 and each of its slant edge is of c) ₹ 24,100 d) ₹ 33,100
length 𝟏𝟎 𝒄𝒎. What is the volume (𝒊𝒏 𝒄𝒎𝟑 ) of
the pyramid? 105. A bucket in the shape of the frustum of a
cone has its top and bottom radii as
एक लंब वपराममड का आिार 𝟖√𝟐 𝒄𝒎 भज
ु ा 𝟐𝟎 𝒄𝒎 and 𝟏𝟎 𝒄𝒎, respectively. The depth of
वाला एक वर्ग है और इसकी प्रत्येक ततयगक कोर the bucket is 𝟐𝟒 𝒄𝒎. the capacity of the
𝟐𝟐
की लंबाई 𝟏𝟎 𝒄𝒎 है। वपराममड का आयतन bucket is: (𝑻𝒂𝒌𝒆 𝝅 = 𝟕
)
(𝒄𝒎𝟑 में) ककतना है ? र्ंकु तछन्द्नक के आकार वाली बाल्टी की र्ीर्ग
और तली त्रिज्याएं क्रमर्ः 𝟐𝟎 𝒄𝒎 और 𝟏𝟎 𝒄𝒎हैं।
𝟐
a) 256 b) 224 c) 𝟒𝟐𝟔 𝟑 d) 𝟗𝟔√𝟐
बाल्टी की र्हराई 𝟐𝟒 𝒄𝒎 है। बाल्टी की िाररता
103. The curved surface area of a right circular
ज्ञात करें | (𝑻𝒂𝒌𝒆 𝝅 =
𝟐𝟐
)
cone is 𝟐𝟑𝟏𝟎 𝒄𝒎𝟐 and its radius is 𝟐𝟏 𝒄𝒎. If 𝟕
its radius is increased by 𝟏𝟎𝟎% and height is a) 𝟖𝟖𝟎𝟎 𝒄𝒎𝟑 b) 𝟏𝟑𝟐𝟎𝟎 𝒄𝒎𝟑
reduced by 𝟓𝟎%, then its capacity (in litres) c) 𝟏𝟕𝟎𝟎𝟎 𝒄𝒎𝟑 d) 𝟏𝟕𝟔𝟎𝟎 𝒄𝒎𝟑
will be (correct to one decimal place):
106. The total surface area of a hemisphere is very
एक लम्ब वत्त
ृ ीय र्ंकु का वक्र पष्ृ ठीय क्षेिफल
nearly equal to that of an equilateral triangle.
𝟐𝟑𝟏𝟎 𝒄𝒎𝟐 है और इसकी त्रिज्या 𝟐𝟏 𝒄𝒎 है। The side of the triangle is how many times
यदद इसकी त्रिज्या 𝟏𝟎𝟎% बढा दी जाए और
(approximately) of the radius of the much wood is wasted in the process?
hemisphere? 𝟐𝟐
(𝒖𝒔𝒆 𝝅 = )
𝟕
एक र्ोलािग का कुल पष्ृ ठीय क्षेिफल एक
𝟏𝟎 𝒄𝒎 भुजा वाली लकड़ी के घनाकार टुकड़े से,
समबाहु त्रिभज
ु के क्षेिफल के लर्भर् बराबर है । 𝟒. 𝟐 𝒄𝒎 त्रिज्या वाला र्ोला काटकर तनकला
त्रिभज
ु की भुजा र्ोलािग की त्रिज्या की ककतना जाता है | इस प्रकक्रया में ककतनी लकड़ी बबागद
र्ुना (लर्भर्) है ?
होती है ? (𝝅 = ले)
𝟐𝟐
𝟖𝝅 𝟎.𝟓 𝟒𝝅 𝟎.𝟓 𝟕
a) ( ) b) ( 𝟑 ) a) 𝟒𝟕𝟔. 𝟐 𝒄𝒎𝟑 b) 𝟔𝟖𝟗. 𝟓𝟑𝟔 𝒄𝒎𝟑
√𝟑
𝟎.𝟓 𝟎.𝟓 c) 𝟑𝟏𝟎. 𝟒𝟔𝟒 𝒄𝒎𝟑 d) 𝟓𝟐𝟑. 𝟖 𝒄𝒎𝟑
c) (𝟐𝝅√𝟑) d) (𝟒𝝅√𝟑)
110. The internal and external radii of a hollow
107. The total surface area of a solid right circular
hemispherical bowl are 𝟐𝟎 𝒄𝒎 and 𝟐𝟑 𝒄𝒎,
cylinder of height 𝟏𝟑 𝒄𝒎, is 𝟖𝟖𝟎 𝒄𝒎𝟐 . Its
respectively. What is the total surface area (in
volume (in 𝒄𝒎𝟑 ) is 𝟏𝟏𝒌. The value of K
𝟐𝟐 𝒄𝒎𝟐 ) of the bowl?
(𝑻𝒂𝒌𝒆 𝝅 = )
𝟕 ककसी खोखले अद्गिर्ोलीय कटोरे की आन्द्तररक
𝟏𝟑𝒄𝒎 ऊंचाई के एक ठोस लम्ब वत्त
ृ ीय बेलन और बाहरी त्रिज्याएँ क्रमर्: 𝟐𝟎 𝒄𝒎 और 𝟐𝟑 𝒄𝒎
का कुल पष्ृ ठीय क्षेिफल 𝟖𝟖𝟎𝒄𝒎𝟐 है। इसका
हैं| कटोरे का कुल पष्ृ ठीय क्षेिफल (𝒄𝒎𝟐 में)
आयतन (𝒄𝒎 में) 𝟏𝟏𝒌 है। 𝑲 का मान ज्ञात करें
𝟑
ज्ञात करें |
लें )
𝟐𝟐
(𝝅 = a) 𝟐𝟎𝟖𝟐 𝝅 b) 𝟏𝟖𝟓𝟖 𝝅
𝟕
a) 208 b) 182 c) 104 d) 91 c) 𝟏𝟗𝟖𝟕 𝝅 d) 𝟐𝟏𝟗𝟒 𝝅

108. The radii of the circular ends of a frustum of FOR VIDEO SOLUTIONS
a cone are 𝟐𝟎 𝒄𝒎 and 𝟏𝟑 𝒄𝒎 and its height is FROM QUESTIONS 111 TO 120 CLICK HERE
𝟏𝟐 𝒄𝒎. What is the capacity (in litres) of the OR SCAN QR CODE GIVEN BELOW
frustum (correct to one decimal place)?
𝟐𝟐
(𝑻𝒂𝒌𝒆 𝝅 = )
𝟕
एक र्ंकु के तछन्द्नक के वत्त
ृ ीय मसरों की
त्रिज्याएँ 𝟐𝟎𝒄𝒎 और 𝟏𝟑𝒄𝒎 हैं और इसकी ऊँचाई
𝟏𝟐𝒄𝒎 है। तछन्द्नक की क्षमता (लीटर में) ज्ञात
करे (ठीक एक दर्मलव स्िान तक) 111. A hall is 𝟏𝟖 𝒎 long and 𝟏𝟐 𝒎 broad. If the
sum of the areas of the floor and the ceiling is
(𝝅 = 𝟕 लें)
𝟐𝟐
equal to the sum of the areas of the four walls,
a) 𝟏𝟎. 𝟒 b) 𝟏𝟏. 𝟐 the volume (𝒊𝒏 𝒄𝒎𝟑 ) of the hall is:
c) 𝟏𝟎. 𝟖 d) 𝟏𝟏. 𝟒
एक हॉल 𝟏𝟖 𝒎 लंबा और 𝟏𝟐 𝒎 चौड़ा है। यदद
109. From a wooden cubical block of side 𝟏𝟎 𝒄𝒎, फर्ग और छत के क्षेिफलों का योर् चारों दीवारों
a sphere of radius 𝟒. 𝟐 𝒄𝒎 is carved out. How
के क्षेिफल के योर् के बराबर है , तो हॉल का in the beg, if 1 kg grain occupies 𝟑𝟎𝟎𝟎 𝒄𝒎³
𝟐𝟐
आयतन (𝒊𝒏 𝒄𝒎 ) है:
𝟑 space? (𝑼𝒔𝒆 𝝅 = 𝟕
)
a) 𝟏𝟏𝟕𝟓. 𝟓 b) 𝟏𝟖𝟕𝟔. 𝟐 कमला के पास तीन बेलनाकार कंटे नर हैं, स्जनमें
c) 𝟏𝟓𝟓𝟓. 𝟐 d) 𝟏𝟑𝟕𝟔. 𝟒 से प्रत्येक की त्रिज्या 𝟐𝟎 𝒄𝒎 है, वह इन्द्हें अनाज

112. A wooden piece in the shape of a cuboid is से भरना चाहती है। यदद उसके अनाज वाले
divided into five equal parts by cutting it with झोले में 100 kg अनाज है , और वह तीनों कंटे नरों
four cuts perpendicular to its length, and it
को 𝟕𝟎 𝒄𝒎ऊंचाई तक भरती है। तो झोले मे
turns out that each piece is a cube of volume
𝟐𝟕 𝒄𝒎³. What is the total surface are ककतना अनाज बचेर्ा, यदद 1 kg अनाज
(𝒊𝒏 𝒄𝒎𝟐 ) of the cuboid? 𝟑𝟎𝟎𝟎 𝒄𝒎³ स्िान लेता है ? ( 𝝅 = लें )
𝟐𝟐
𝟕
घनाभ के आकार वाले लकड़ी के एक टुकड़े को a) 88 kg b) 50 kg
उसकी लम्बाई चार बार काटकर पांच बराबर c) 58 kg d) 12 kg
भार्ों में बांटा र्या, और इस प्रकार बने प्रत्येक
115. If the volume of a cube is 𝟖𝟏√𝟑𝒄𝒎³, then its
घन टुकड़े का आयतन 𝟐𝟕 𝒄𝒎³ हो र्या| घनाभ total surface area is:
का कुल पष्ृ ठीय क्षेिफल (𝒄𝒎𝟐 में) ज्ञात करें | यदद ककसी घन का आयतन 𝟖𝟏√𝟑𝒄𝒎³ है , तो
a) 150 b) 174 c) 216 d) 198 इसका कुल पष्ृ ठीय क्षेिफल ज्ञात करें |
a) 𝟏𝟔𝟔 𝒄𝒎² b) 𝟏𝟓𝟐 𝒄𝒎²
113. The material of a sphere of radius 𝒓 is melted
c) 𝟏𝟔𝟒 𝒄𝒎² d) 𝟏𝟔𝟐 𝒄𝒎²
and recast into a hollow cylindrical shell of
thickness 𝒂 and outer radius 𝒃. What is its
116. The sum of the length, breadth and height of
length assuming that no material is lost in
a cuboid is 𝟐𝟖 𝒄𝒎. If the total surface area of
recasting?
the cuboid is 𝟓𝟖𝟖 𝒄𝒎², then its diagonal is:
𝒓त्रिज्या वाले के र्ोले की सामग्री को वपघलाकर
ककसी घनाभ की लम्बाई, चौड़ाई और ऊँचाई का
ककसी बेलनाकार खोखले ढांचे में ढाला र्या है ,
योर्फल 𝟐𝟖 𝒄𝒎 है| यदद घनाभ का पष्ृ ठीय
स्जसकी मोटाई 𝒂 और बाहरी त्रिज्या 𝒃 है | यदद
क्षेिफल 𝟓𝟖𝟖 𝒄𝒎² है , तो इसका ववकर्ग ज्ञात
ढलाई में सामग्री की कोई हातन न हुई हो तो
करें |
ढांचे की अनुमातनत लम्बाई ककतनी है ? a) 𝟏𝟒 𝒄𝒎 b) 𝟏𝟐 𝒄𝒎
𝟒𝒓³ 𝟒𝒓³ c) 𝟏𝟔 𝒄𝒎 d) 𝟏𝟓 𝒄𝒎
a) 𝟑𝒂(𝟐𝒂−𝒃) b) 𝟑𝒂(𝒃+𝒂)(𝒃−𝒂)
𝟒𝒓³ 𝟒𝒓³
c) 𝟑𝒂(𝒃−𝒂) d) 𝟑𝒂(𝟐𝒃−𝒂) 117. Four solid cubes, each of volume 𝟏𝟕𝟐𝟖 𝒄𝒎³,
are kept in two rows having two cubes in each
114. Kamala has three cylindrical containers, each row. T hey form a rectangular solid with
with radius 𝟐𝟎 𝒄𝒎 that she wants to fill with square base. The total surface are ( 𝒊𝒏 𝒄𝒎𝟐 )
grain. If her bag of grain contains 100 kg of of the resulting solid is:
grain, and she fills all three containers to a
height of 𝟕𝟎 𝒄𝒎. how much grain will be left
प्रत्येक 𝟏𝟕𝟐𝟖 𝒄𝒎³ आयतन वाले चार ठोस घनों एक घनाकार कमरे के फर्ग के क्षेिफल 𝟏𝟒𝟕 𝒎𝟐
को दो पंस्क्तयों में रखा जाता है और प्रत्येक है | इस कमरे में अधिकतम ककतनी लम्बाई वाली
पंस्क्त में दो घन रखे जाते है | उनसे वर्ागकार छड रखी जा सकती है ?
आिार वाला आयताकार ठोस बनता है | a) 𝟏𝟓 𝒎 b) 𝟏𝟖 𝒎
c) 𝟐𝟏 𝒎 d) 𝟏𝟔 𝒎
पररर्ामी ठोस का कुल पष्ृ ठीय क्षेिफल (𝒄𝒎𝟐
में) ज्ञात करें | FOR VIDEO SOLUTIONS
a) 1440 b) 2304 FROM QUESTIONS 121 TO 122 CLICK HERE
c) 576 d) 1152 OR SCAN QR CODE GIVEN BELOW

118. Two similar cylindrical jugs have heights


𝟓 𝒄𝒎 and 𝟖 𝒄𝒎, respectively. If the capacity of
the smaller jut is 𝟔𝟒 𝒄𝒎³, what is the capacity
(correct to 2 decimal places, in 𝒄𝒎³) of the
larger jug?
दो समरूप बेलनाकार जार्ों की ऊँचाई क्रमर्: 121. The total surface area of a solid right circular
𝟓 𝒄𝒎 और 𝟖 𝒄𝒎 है| यदद छोटे जर् की िाररता cone is equal to that of a sphere of the same
𝟔𝟒 𝒄𝒎³ है , बड़े जर् की िाररता (𝒄𝒎³ में , radius. The height of the cone is how many
times the diameter of the sphere?
दर्मलव के बाद दो स्िानों तक र्र्ना करें )
एक ठोस लम्बवत्त
ृ ीय र्ंकु का संपूर्ग प्रष्ठीय
ज्ञात करें |
क्षेिफल समान त्रिज्या वाले र्ोले के क्षेिफल के
a) 𝟐𝟔𝟐. 𝟏𝟒 b) 𝟏𝟎𝟐. 𝟒𝟎
c) 𝟐𝟗𝟒. 𝟓𝟒 d) 𝟏𝟔𝟑. 𝟖𝟒 बराबर है | र्ंकु की ऊँचाई र्ोले के व्यास की
ककतनी र्ुनी होर्ी?
119. Three cubes each having volume 𝟏𝟕𝟐𝟖 𝒄𝒎𝟑 𝟑 √𝟐
a) √𝟐 b) √𝟐 c) 𝟐√𝟐 d)
are placed one above the other. What is the 𝟐
total surface area of the resulting solid?
122. If the area of the base of a cylinder is
प्रत्येक 𝟏𝟕𝟐𝟖 𝒄𝒎𝟑 आयतन वाले तीन घनों को
𝟑𝟒𝟔. 𝟓𝒄𝒎² and the area of the curved surface
एक दस
ू रे के ऊपर रखा र्या है । पररर्ामी ठोस 𝟐𝟐
is 𝟗𝟗𝟎𝒄𝒎², then its height is: (𝑻𝒂𝒌𝒆 𝝅 = )
का कुल पष्ृ ठीय क्षेिफल ज्ञात करें |
𝟕
यदद एक बेलन के आिार का क्षेिफल
a) 𝟐𝟑𝟎𝟒 𝒄𝒎𝟐 b) 𝟐𝟓𝟗𝟐 𝒄𝒎𝟐
c) 𝟐𝟏𝟔𝟎 𝒄𝒎𝟐 d) 𝟐𝟎𝟏𝟔 𝒄𝒎𝟐 𝟑𝟒𝟔. 𝟓𝒄𝒎² है और वक्र पष्ृ ठ का क्षेिफल
𝟗𝟗𝟎𝒄𝒎²है, तो इसकी ऊँचाई है:(𝑻𝒂𝒌𝒆 𝝅 =
120. The area of the floor of a cubical room is 𝟐𝟐
)
𝟏𝟒𝟕 𝒎𝟐 . The length of the longest rod that 𝟕
can be kept in the room is: a) 𝟏𝟎 𝒄𝒎 b) 𝟏𝟐 𝒄𝒎
c) 𝟏𝟒 𝒄𝒎 d) 𝟏𝟓 𝒄𝒎
ANSWER KEY
1. D 2. A 3. A 4. A 5. A
6. C 7. D 8. C 9. C 10. B
11. B 12. D 13. B 14. C 15. C
To join Our Telegram channel Click here
16. B 17. B 18. D 19. D 20. B
21. A 22. C 23. B 24. B 25. B
26. B 27. B 28. D 29. B 30. D
31. D 32. C 33. D 34. D 35. C
36. C 37. A 38. B 39. A 40. D
41. D 42. D 43. A 44. B 45. D
46. C 47. D 48. B 49. D 50. B
51. B 52. D 53. D 54. B 55. A
56. D 57. D 58. A 59. A 60. D
61. C 62. D 63. D 64. C 65. A
66. D 67. C 68. B 69. B 70. C
71. B 72. C 73. C 74. A 75. C
76. D 77. C 78. A 79. C 80. A
81. D 82. D 83. C 84. A 85. A
86. C 87. C 88. B 89. A 90. C
91. C 92. D 93. B 94. D 95. C
96. A 97. C 98. D 99. D 100. B
101. C 102. A 103. D 104. D 105. D
106. D 107. B 108. A 109. B 110. C
111. C 112. D 113. D 114. D 115. D
116. A 117. B 118. A 119. D 120. C
121. A 122. D

1. For Best FREE full syllabus course of


Maths & Reasoning Click here
Trigonometry And Height & Distance

Instruction: - For Video Solution scan QR code


given below or you can also CLICK on Click here
button to watch the Video Solution.

(𝒔𝒊𝒏𝜽−𝒄𝒐𝒔𝜽)(𝟏+𝒕𝒂𝒏𝜽+𝒄𝒐𝒕𝜽)(𝒄𝒐𝒔𝒆𝒄𝟐 𝜽)
6. , 𝟎° < 𝜽 < 𝟗𝟎°,
𝒔𝒆𝒄𝟑 𝜽−𝒄𝒐𝒔𝒆𝒄𝟑 𝜽
is equal to:
(𝒔𝒊𝒏𝜽−𝒄𝒐𝒔𝜽)(𝟏+𝒕𝒂𝒏𝜽+𝒄𝒐𝒕𝜽)(𝒄𝒐𝒔𝒆𝒄𝟐 𝜽)
, 𝟎° < 𝜽 < 𝟗𝟎°
𝒔𝒆𝒄𝟑 𝜽−𝒄𝒐𝒔𝒆𝒄𝟑 𝜽
𝟐
1. (𝒄𝒐𝒔𝒆𝒄𝜽 − 𝒄𝒐𝒕𝜽) =? , (𝟎° < 𝜽 < 𝟗𝟎°) ककसके बराबर होगा?
𝟏+𝒄𝒐𝒔𝜽 𝟏+𝒄𝒐𝒔𝜽
a) 𝟏−𝒔𝒊𝒏 𝜽 b) 𝟏−𝒄𝒐𝒔𝜽 a) 𝒔𝒆𝒄𝜽𝒄𝒐𝒔𝒆𝒄𝜽 b) 𝒄𝒐𝒔𝟐 𝜽
𝟏−𝒔𝒊𝒏𝜽 𝟏−𝒄𝒐𝒔𝜽
c) 𝟏+𝒄𝒐𝒔𝜽 d) 𝟏+𝒄𝒐𝒔𝜽 c) 𝒔𝒊𝒏𝟐 𝜽 d) 𝒔𝒊𝒏𝜽𝒄𝒐𝒔𝜽

2. The value of 𝒔𝒊𝒏𝟕𝟑° + 𝒄𝒐𝒔𝟏𝟑𝟕° is: 7. If 𝒔𝒆𝒄𝜽 + 𝒕𝒂𝒏𝜽 = 𝒌, 𝟎° < 𝜽 < 𝟗𝟎°, then
(𝒌−𝟏)𝟐 −𝟐
𝒔𝒊𝒏𝟕𝟑° + 𝒄𝒐𝒔𝟏𝟑𝟕° का मान क्या होगा ? (𝒌+𝟏)𝟐 −𝟐𝒌
is equal to:
a) 𝒄𝒐𝒔 𝟏𝟑° b) 𝒔𝒊𝒏 𝟏𝟑° यदि 𝒔𝒆𝒄𝜽 + 𝒕𝒂𝒏𝜽 = 𝒌, 𝟎° < 𝜽 < 𝟗𝟎° तो
c) 𝒔𝒊𝒏 𝟏𝟖° d) 𝒄𝒐𝒔 𝟏𝟖° (𝒌−𝟏)𝟐 −𝟐
(𝒌+𝟏)𝟐 −𝟐𝒌
इसके बराबर है:
3. The Value of (𝟏 + 𝒕𝒂𝒏 𝟏𝟎°)(𝟏 + 𝒕𝒂𝒏 𝟑𝟓°) is : a) 𝒔𝒊𝒏𝜽 − 𝒄𝒐𝒔𝜽 b) 𝒄𝒐𝒔𝜽 + 𝒔𝒊𝒏𝜽
(𝟏 + 𝒕𝒂𝒏 𝟏𝟎°)(𝟏 + 𝒕𝒂𝒏 𝟑𝟓°) का मान क्या c) 𝒔𝒆𝒄𝜽 𝒄𝒐𝒔𝒆𝒄𝜽 d) 𝒄𝒐𝒔𝜽 𝒔𝒊𝒏𝜽
होगा?
𝒄𝒐𝒕 𝜽+𝒄𝒐𝒔 𝜽
a) 𝟒
𝟑
b) 𝟏 c) 𝟐
𝟏
d) 𝟐 8. The value of 𝟏 + √𝒄𝒐𝒕 𝜽−𝒄𝒐𝒔 𝜽 , 𝒊𝒇 𝟎° < 𝜽 < 𝟗𝟎°,
is equal to:
4. The expression
यदि 𝟎° < 𝜽 < 𝟗𝟎° है, तो 𝟏 + √𝒄𝒐𝒕 𝜽−𝒄𝒐𝒔 𝜽 , का
𝒄𝒐𝒕 𝜽+𝒄𝒐𝒔 𝜽
(𝟏+𝒄𝒐𝒔𝜽+𝒔𝒊𝒏𝜽)÷(𝟏+𝒄𝒐𝒔𝜽−𝒔𝒊𝒏𝜽)
, 𝟎° < 𝜽 < 𝟗𝟎°, is
मान ज्ञात कीजजए|
(𝒔𝒆𝒄𝜽−𝒄𝒐𝒔𝜽)(𝒄𝒐𝒕𝜽+𝒕𝒂𝒏𝜽)(𝟏+𝒔𝒊𝒏𝜽)
equal to:
a) 𝟏 − 𝒔𝒆𝒄 𝜽 + 𝒕𝒂𝒏 𝜽 b) 𝟏 − 𝒔𝒆𝒄 𝜽 − 𝒕𝒂𝒏 𝜽
व्यंजक , जहां 𝟎° <
(𝟏+𝒄𝒐𝒔𝜽+𝒔𝒊𝒏𝜽)÷(𝟏+𝒄𝒐𝒔𝜽−𝒔𝒊𝒏𝜽)
(𝒔𝒆𝒄𝜽−𝒄𝒐𝒔𝜽)(𝒄𝒐𝒕𝜽+𝒕𝒂𝒏𝜽)(𝟏+𝒔𝒊𝒏𝜽) c) 𝟏 + 𝒔𝒆𝒄 𝜽 − 𝒕𝒂𝒏 𝜽 d) 𝟏 + 𝒔𝒆𝒄 𝜽 + 𝒕𝒂𝒏 𝜽
𝜽 < 𝟗𝟎° है, इनमें से ककसके बराबर होगा?
a) 𝒕𝒂𝒏⁡ 𝜽 b) 𝒄𝒐𝒕⁡ 𝜽 c) 𝒔𝒆𝒄⁡ 𝜽 d) 𝒔𝒊𝒏⁡ 𝜽 𝒄𝒐𝒕 𝜽+𝒄𝒐𝒔𝜽
9. The value of 𝟐 − √𝒄𝒐𝒕 𝜽−𝒄𝒐𝒔𝜽, when 𝟎° < 𝜽 <

5. If 𝒄𝒐𝒔𝟐 𝜽 + 𝟑 = 𝟑(𝒄𝒐𝒕𝟐 𝜽 + 𝒔𝒊𝒏𝟐 𝜽), 𝟎° < 𝜽 < 𝟗𝟎° is equal to :


𝟗𝟎°, then what is the value of (𝟐𝒄𝒐𝒔𝜽 + 𝟐 − √𝒄𝒐𝒕 𝜽−𝒄𝒐𝒔𝜽, का मान बताइए, जबकक 𝟎° <
𝒄𝒐𝒕 𝜽+𝒄𝒐𝒔𝜽
𝟑𝒔𝒊𝒏𝜽)?
यदि 𝒄𝒐𝒔𝟐 𝜽 + 𝟑 = 𝟑(𝒄𝒐𝒕𝟐 𝜽 + 𝒔𝒊𝒏𝟐 𝜽), 𝟎° < 𝜽 < 𝟗𝟎° है|
a) 𝟐 + 𝒔𝒆𝒄 𝜽 + 𝒕𝒂𝒏 𝜽 b) 𝟐 − 𝒔𝒆𝒄 𝜽 + 𝒕𝒂𝒏 𝜽
𝜽 < 𝟗𝟎° है, तो (𝟐𝒄𝒐𝒔𝜽 + 𝟑𝒔𝒊𝒏𝜽) का मान
c) 𝟐 − 𝒔𝒆𝒄 𝜽 − 𝒕𝒂𝒏 𝜽 d) 𝟐 + 𝒔𝒆𝒄 𝜽 + 𝒕𝒂𝒏 𝜽
ककतना होगा?
𝟏+𝟑√𝟑 𝟐√𝟑+𝟏 𝒔𝒊𝒏 𝟒𝜽
a) b) 10. The value of (𝟏−𝒄𝒐𝒔 𝟒𝜽). Is:
𝟐 𝟐
का मान क्या होगा?
√𝟑+𝟐 𝟐+𝟑√𝟑 𝒔𝒊𝒏 𝟒𝜽
c) d) (𝟏−𝒄𝒐𝒔 𝟒𝜽)
𝟐 𝟐
a) 𝒕𝒂𝒏𝜽 b) 𝒄𝒐𝒕𝜽 यदि 𝒔𝒆𝒄𝜽 − 𝒄𝒐𝒔𝜽 = 𝟏𝟒 and 𝟏𝟒𝒔𝒆𝒄 𝜽 = 𝒙 है ,
c) 𝒄𝒐𝒕 𝟐𝜽 d) 𝒕𝒂𝒏 𝟐𝜽
तो 𝒙 का मान ज्ञात कीजजए|
a) 𝒔𝒆𝒄𝟐 𝜽 b) 𝒕𝒂𝒏𝟐 𝜽
c) 𝟐𝒕𝒂𝒏 𝜽 d)𝟐𝒔𝒆𝒄 𝜽

√𝟑 𝒙−𝒙𝒄𝒐𝒔𝒆𝒄𝟐 𝟑𝟎°
11. Find 𝜽, if 𝒄𝒐𝒔𝜽 = − . 17. If = 𝒄𝒐𝒔𝟐 𝟔𝟎° + 𝟒𝒄𝒐𝒕𝟐 𝟒𝟓° −
𝟐 𝟏+𝒄𝒐𝒔𝒆𝒄𝟐 𝟑𝟎°
𝒔𝒆𝒄𝟐 𝟔𝟎°, then the value of 𝒙 is _______.
यदि 𝒄𝒐𝒔𝜽 = − है, तो 𝜽 का माना ज्ञात करें |
√𝟑
,
𝒙−𝒙𝒄𝒐𝒔𝒆𝒄𝟐 𝟑𝟎°
यदि
𝟐
𝟑𝝅 𝟐𝝅 𝟒𝝅 𝟓𝝅 = 𝒄𝒐𝒔𝟐 𝟔𝟎° + 𝟒𝒄𝒐𝒕𝟐 𝟒𝟓° −
a) b) c) d) 𝟏+𝒄𝒐𝒔𝒆𝒄𝟐 𝟑𝟎°
𝒔𝒆𝒄𝟐 𝟔𝟎° है, तो 𝒙 का मान ज्ञात कीजजए|
𝟐 𝟑 𝟑 𝟔

𝟏 𝟓 𝟓 𝟏
12. Using 𝟐𝒄𝒐𝒔𝑨 𝒄𝒐𝒔𝑩 = 𝒄𝒐𝒔 (𝑨 + 𝑩) + 𝒄𝒐𝒔(𝑨 − a) − 𝟏𝟐 b) 𝟏𝟐 c) − 𝟏𝟐 d) 𝟓
𝑩), find the value of cos 𝟕𝟓°𝒄𝒐𝒔 𝟏𝟓°.
𝟐𝒄𝒐𝒔𝑨 𝒄𝒐𝒔𝑩 = 𝒄𝒐𝒔 (𝑨 + 𝑩) + 𝒄𝒐𝒔(𝑨 − 𝑩) के 𝒕𝒂𝒏⁡𝑨 𝒄𝒐𝒕⁡𝑨
18. The expression 𝟏−𝒄𝒐𝒕⁡𝑨 + 𝟏−𝒕𝒂𝒏⁡𝑨 can be written
उपयोग से cos 𝟕𝟓°𝒄𝒐𝒔 𝟏𝟓° का मान ज्ञात as:
कीजजए| व्यंजक + 𝟏−𝒕𝒂𝒏⁡𝑨 को इस प्रकार लििा जा
𝒕𝒂𝒏⁡𝑨 𝒄𝒐𝒕⁡𝑨
𝟏−𝒄𝒐𝒕⁡𝑨

a) 𝟐
𝟏
b)
√𝟐
c)
√𝟐
d) 𝟒
𝟏
सकता है:
𝟒 𝟐
a) 𝟏 + 𝒔𝒆𝒄⁡𝑨⁡𝒄𝒐𝒔𝒆𝒄⁡𝑨 b) 𝟏 + 𝒔𝒊𝒏⁡𝑨⁡𝒄𝒐𝒔⁡𝑨
13. What is the value of (𝟒𝒔𝒊𝒏 𝒙 − 𝟑𝒔𝒊𝒏𝒙 + 𝟑 c) 𝒕𝒂𝒏⁡𝑨 + 𝒄𝒐𝒕⁡𝑨 d) 𝒔𝒆𝒄⁡𝑨 + 𝒄𝒐𝒔𝒆𝒄⁡𝑨
𝒔𝒊𝒏𝟑𝒙)?
(𝟒𝒔𝒊𝒏𝟑 𝒙 − 𝟑𝒔𝒊𝒏𝒙 + 𝒔𝒊𝒏𝟑𝒙) का मान क्या Answer Key
1. D 2. B 3. C 4. B 5. D
होगा? 6. B 7. A 8. D 9. C 10. C
a) 𝟎 b) 𝟒⁡ c) 𝟏 d) 𝟑 11. D 12. D 13. A 14. C 15. A
16. B 17. C 18. A
14. Find the value of the following.
ननम्नलिखित का मान ज्ञात कीजजए।
𝒔𝒊𝒏 𝟔𝟕° 𝒄𝒐𝒔 𝟑𝟕°−𝒔𝒊𝒏𝟑𝟕°𝒄𝒐𝒔 𝟔𝟕°
𝒄𝒐𝒔 𝟏𝟑° 𝒄𝒐𝒔 𝟏𝟕°−𝒔𝒊𝒏 𝟏𝟑° 𝒔𝒊𝒏 𝟏𝟕°
𝟐 𝟏 𝟒
a) b) 𝟕 c) d)
√𝟑 √𝟑 √𝟑

𝒔𝒊𝒏 𝟒𝜽+𝒔𝒊𝒏 𝟐𝜽
15. The Value of 𝒄𝒐𝒔 𝟒𝜽+𝒄𝒐𝒔 𝟐𝜽 is:
का माना क्या होगा?
𝒔𝒊𝒏 𝟒𝜽+𝒔𝒊𝒏 𝟐𝜽
𝒄𝒐𝒔 𝟒𝜽+𝒄𝒐𝒔 𝟐𝜽
a) 𝒕𝒂𝒏 𝟑𝜽 b) 𝒄𝒐𝒕 𝜽
c) 𝒕𝒂𝒏𝜽 c) 𝒄𝒐𝒕 𝟑𝜽

16. If 𝒔𝒆𝒄𝜽 − 𝒄𝒐𝒔𝜽 = 𝟏𝟒 and 𝟏𝟒𝒔𝒆𝒄 𝜽 = 𝒙, then


the value of 𝒙 is _________.
Trigonometry Most Difficult Questions From All Latest Exams SSC
CGL, CHSL, CPO, MTS, NTPC 2020

(𝟏−𝟐 𝒔𝒊𝒏𝟐 𝜽 𝒄𝒐𝒔𝟐 𝜽)(𝒄𝒐𝒕 𝜽+𝟏) 𝒄𝒐𝒔 𝜽


FOR VIDEO SOLUTION व्यंजक (𝒔𝒊𝒏𝟒 𝜽+𝒄𝒐𝒔𝟒 𝜽)(𝟏+𝒕𝒂𝒏 𝜽)𝒄𝒐𝒔𝒆𝒄 𝜽
−𝟏 का
FROM QUESTIONS 1 TO 10 CLICK HERE
OR SCAN QR CODE GIVEN BELOW मान बताइए, जहां 𝟎° < 𝜽 < 𝟗𝟎° है |
a) 𝒄𝒐𝒔𝟐 𝜽 b) −𝒔𝒊𝒏𝟐 𝜽
c) 𝒔𝒆𝒄𝟐 𝜽 d) −𝒔𝒆𝒄𝟐 𝜽

(𝟏+𝒔𝒆𝒄 𝜽 𝒄𝒐𝒔𝒆𝒄 𝜽)𝟐 (𝒔𝒆𝒄 𝜽−𝒕𝒂𝒏 𝜽)𝟐 (𝟏+𝒔𝒊𝒏 𝜽)


5. , 𝟎° < 𝜽 <
(𝒔𝒊𝒏 𝜽+𝒔𝒆𝒄 𝜽)𝟐 +(𝒄𝒐𝒔 𝜽+𝒄𝒐𝒔𝒆𝒄 𝜽)𝟐
𝟗𝟎°, is equal to:
(𝟏+𝒔𝒆𝒄 𝜽 𝒄𝒐𝒔𝒆𝒄 𝜽)𝟐 (𝒔𝒆𝒄 𝜽−𝒕𝒂𝒏 𝜽)𝟐 (𝟏+𝒔𝒊𝒏 𝜽)
, 𝟎° < 𝜽 <
1. If A lies between 𝟒𝟓° and 𝟓𝟒𝟎°, and (𝒔𝒊𝒏 𝜽+𝒔𝒆𝒄 𝜽)𝟐 +(𝒄𝒐𝒔 𝜽+𝒄𝒐𝒔𝒆𝒄 𝜽)𝟐
𝒔𝒊𝒏𝑨 = 𝟎. 𝟓, what is the value of 𝑨/𝟑 in 𝟗𝟎°, का मान इनमें से ककसके बराबर होगा?
degrees? a) 𝟏 − 𝒔𝒊𝒏 𝜽 b) 𝒔𝒊𝒏 𝜽
यदि A का मान 𝟒𝟓° और 𝟓𝟒𝟎° के मध्य है , और c) 𝟏 − 𝒄𝒐𝒔 𝜽 d) 𝒄𝒐𝒔 𝜽
𝒔𝒊𝒏𝑨 = 𝟎. 𝟓 है, तो 𝑨/𝟑 का मान अंश में ककतना 𝒕𝒂𝒏𝟑 𝒄𝒐𝒕𝟑 𝜽
6. (𝒔𝒆𝒄𝟐 𝜽 + 𝒄𝒐𝒔𝒆𝒄𝟐 𝜽 + 𝟐 𝒔𝒊𝒏 𝜽 𝒄𝒐𝒔 𝜽) ÷ (𝟏 +
होगा?
a) 𝟏𝟕𝟎° b) 𝟏𝟕𝟓° 𝒄𝒐𝒔𝒆𝒄𝟐 𝜽 + 𝒕𝒂𝒏𝟐 𝜽), 𝟎° < 𝜽 < 𝟗𝟎°, is equal
c) 𝟏𝟔𝟓° d) 𝟏𝟔𝟎° to:
𝒕𝒂𝒏𝟑 𝒄𝒐𝒕𝟑 𝜽
(𝒔𝒆𝒄𝟐 𝜽 + 𝒄𝒐𝒔𝒆𝒄𝟐 𝜽 + 𝟐 𝒔𝒊𝒏 𝜽 𝒄𝒐𝒔 𝜽) ÷ (𝟏 +
2. If 𝟏 + 𝟐 𝒕𝒂𝒏𝟐 𝜽 + 𝟐 𝒔𝒊𝒏 𝜽 𝒔𝒆𝒄𝟐 𝜽 = 𝒄𝒐𝒔𝒆𝒄𝟐 𝜽 + 𝒕𝒂𝒏𝟐 𝜽), 𝟎° < 𝜽 < 𝟗𝟎°, का मान
𝒂 𝒂+𝒃
, 𝟎° < 𝜽 < 𝟗𝟎𝟎 , then 𝒂−𝒃 =?
𝒃 इनमें से ककसके बराबर होगा?
यदि 𝟐
𝟏 + 𝟐 𝒕𝒂𝒏 𝜽 + 𝟐 𝒔𝒊𝒏 𝜽 𝒔𝒆𝒄 𝜽 = 𝟐
a) 𝒔𝒊𝒏 𝜽 𝒄𝒐𝒔 𝜽 b) 𝒔𝒆𝒄 𝜽
, 𝟎° < 𝜽 < 𝟗𝟎𝟎 है, तो क्या होगा?
𝒂 𝒂+𝒃
c) 𝒄𝒐𝒔𝒆𝒄 𝜽 𝒔𝒆𝒄 𝜽 d) 𝒄𝒐𝒔𝒆𝒄 𝜽
𝒃 𝒂−𝒃
a) 𝒄𝒐𝒔𝒆𝒄 𝜽 b) 𝒄𝒐𝒔 𝜽
c) 𝒔𝒆𝒄 𝜽 d) 𝒔𝒊𝒏 𝜽 7. The value of
𝟑 (𝒄𝒐𝒕𝟐 𝟒𝟕°−𝒔𝒆𝒄𝟐 𝟒𝟑°)−𝟐 (𝒕𝒂𝒏𝟐 𝟐𝟑°−𝒄𝒐𝒔𝒆𝒄𝟐 𝟔𝟕°
𝒄𝒐𝒔𝒆𝒄𝟐 (𝟔𝟖°+𝜽)−𝒕𝒂𝒏 (𝜽+𝟔𝟏°) −𝒕𝒂𝒏𝟐 (𝟐𝟐°−𝜽)+𝒄𝒐𝒕 (𝟐𝟗°−𝜽)
3. 𝟏 + 𝟐 𝒕𝒂𝒏𝟐 𝜽 + 𝟐 𝒔𝒊𝒏 𝜽 𝒔𝒆𝒄𝟐 𝜽, 𝟎° < 𝜽 < 𝟗𝟎°, is :
is equal to: 𝟑 (𝒄𝒐𝒕𝟐 𝟒𝟕°−𝒔𝒆𝒄𝟐 𝟒𝟑°)−𝟐 (𝒕𝒂𝒏𝟐 𝟐𝟑°−𝒄𝒐𝒔𝒆𝒄𝟐 𝟔𝟕°
𝟏 + 𝟐 𝒕𝒂𝒏𝟐 𝜽 + 𝟐 𝒔𝒊𝒏 𝜽 𝒔𝒆𝒄𝟐 𝜽, 𝟎° < 𝜽 < 𝟗𝟎° 𝒄𝒐𝒔𝒆𝒄𝟐 (𝟔𝟖°+𝜽)−𝒕𝒂𝒏 (𝜽+𝟔𝟏°) −𝒕𝒂𝒏𝟐 (𝟐𝟐°−𝜽)+𝒄𝒐𝒕 (𝟐𝟗°−𝜽)
का मान ज्ञात करें | का मान क्या होगा?
𝟏−𝒔𝒊𝒏 𝜽 𝟏+𝒔𝒊𝒏 𝜽 𝟏−𝒄𝒐𝒔 𝜽 𝟏+𝒄𝒐𝒔 𝜽
a) 𝟏+𝒔𝒊𝒏 𝜽 b) 𝟏−𝒔𝒊𝒏 𝜽 c) 𝟏+𝒄𝒐𝒔 𝜽 d) 𝟏−𝒄𝒐𝒔 𝜽 a) 𝟎 b) −𝟏 c) 𝟏 d) 𝟓

(𝟏−𝟐 𝒔𝒊𝒏𝟐 𝜽 𝒄𝒐𝒔𝟐 𝜽)(𝒄𝒐𝒕 𝜽+𝟏) 𝒄𝒐𝒔 𝜽 𝟏+𝒄𝒐𝒔 𝜽−𝒔𝒊𝒏𝟐 𝜽 √𝒔𝒆𝒄𝟐 𝜽+ 𝒄𝒐𝒔𝒆𝒄𝟐 𝜽
4. The expression − 8. × , 𝟎° < 𝜽 <
(𝒔𝒊𝒏𝟒 𝜽+𝒄𝒐𝒔𝟒 𝜽)(𝟏+𝒕𝒂𝒏 𝜽)𝒄𝒐𝒔𝒆𝒄 𝜽 𝒔𝒊𝒏 𝜽 (𝟏+𝒄𝒐𝒔 𝜽) 𝒕𝒂𝒏 𝜽+𝒄𝒐𝒕 𝜽

𝟏, 𝟎° < 𝜽 < 𝟗𝟎°, equals: 𝟗𝟎°, is equal to:


𝟏+𝒄𝒐𝒔 𝜽−𝒔𝒊𝒏𝟐 𝜽
×
√𝒔𝒆𝒄𝟐 𝜽+ 𝒄𝒐𝒔𝒆𝒄𝟐 𝜽
, 𝟎° < 𝜽 < यदि 𝟎° < 𝜽 < 𝟗𝟎° है, तो
𝒔𝒊𝒏 𝜽 (𝟏+𝒄𝒐𝒔 𝜽) 𝒕𝒂𝒏 𝜽+𝒄𝒐𝒕 𝜽
का मान ज्ञात
(𝟏−𝒔𝒊𝒏𝜽) (𝒔𝒆𝒄 𝜽+𝒕𝒂𝒏 𝜽) 𝒕𝒂𝒏 𝜽
𝟗𝟎, का मान इनमें से ककसके बराबर होगा? (𝒕𝒂𝒏 𝜽+𝒔𝒆𝒄 𝜽+𝟏) (𝒄𝒐𝒕 𝜽−𝒄𝒐𝒔𝒆𝒄 𝜽+𝟏)

a) 𝒕𝒂𝒏 𝜽 b) 𝒄𝒐𝒔𝒆𝒄 𝜽 करें |


c) 𝒔𝒆𝒄 𝜽 d) 𝒄𝒐𝒕 𝜽 a)
𝒔𝒊𝒏 𝜽
b) 𝒔𝒊𝒏 𝜽 c) 𝒄𝒐𝒔 𝜽 d)
𝒄𝒐𝒔 𝜽
𝟐 𝟐

9. The value of
𝟑 (𝒄𝒐𝒔𝒆𝒄𝟐 𝟐𝟔°−𝒕𝒂𝒏𝟐 𝟔𝟒°)+(𝒄𝒐𝒕𝟐 𝟒𝟐°−𝒔𝒆𝒄𝟐 𝟒𝟖°)
13. The expression
is: 𝒄𝒐𝒔𝟒 𝜽 − 𝒔𝒊𝒏𝟒 𝜽 +𝟐 𝒔𝒊𝒏𝟐 𝜽+𝟑
𝒄𝒐𝒕(𝟐𝟐°−𝜽)−𝒄𝒐𝒔𝒆𝒄𝟐 (𝟔𝟐°+𝜽)−𝒕𝒂𝒏(𝜽+𝟔𝟖°)+𝒕𝒂𝒏𝟐 (𝟐𝟖°−𝜽) , 𝟎° < 𝜽 <
𝟑 (𝒄𝒐𝒔𝒆𝒄𝟐 𝟐𝟔°−𝒕𝒂𝒏𝟐 𝟔𝟒°)+(𝒄𝒐𝒕𝟐 𝟒𝟐°−𝒔𝒆𝒄𝟐 𝟒𝟖°) (𝒄𝒐𝒔𝒆𝒄 𝜽+𝒄𝒐𝒕 𝜽+𝟏)(𝒄𝒐𝒔𝒆𝒄 𝜽−𝒄𝒐𝒕 𝜽+𝟏)−𝟐
𝒄𝒐𝒕(𝟐𝟐°−𝜽)−𝒄𝒐𝒔𝒆𝒄𝟐 (𝟔𝟐°+𝜽)−𝒕𝒂𝒏(𝜽+𝟔𝟖°)+𝒕𝒂𝒏𝟐 (𝟐𝟖°−𝜽) 𝟗𝟎°, is equal to:
𝒄𝒐𝒔𝟒 𝜽 − 𝒔𝒊𝒏𝟒 𝜽 +𝟐 𝒔𝒊𝒏𝟐 𝜽+𝟑
का मान बताइए| व्यंजक का
(𝒄𝒐𝒔𝒆𝒄 𝜽+𝒄𝒐𝒕 𝜽+𝟏)(𝒄𝒐𝒔𝒆𝒄 𝜽−𝒄𝒐𝒕 𝜽+𝟏)−𝟐
a) 𝟑 b) 𝟒 c) −𝟏 d) −𝟐 मान बताइए, जहां 𝟎° < 𝜽 < 𝟗𝟎° है |
𝟏
𝒕𝒂𝒏𝟔 𝜽−𝒔𝒆𝒄𝟔 +𝟑 𝒔𝒆𝒄𝟐 𝜽 𝒕𝒂𝒏𝟐 𝜽 a) 𝟐 𝒔𝒊𝒏 𝜽 b) 𝒔𝒆𝒄 𝜽
10. The expression , 𝟎° <
𝒕𝒂𝒏𝟐 𝜽+ 𝒄𝒐𝒕𝟐 𝜽+𝟐 c) 𝟐 𝒄𝒐𝒔𝒆𝒄 𝜽 d) 𝟐 𝒔𝒊𝒏 𝜽
𝜽 < 𝟗𝟎°, is equal to:
𝒕𝒂𝒏𝟔 𝜽−𝒔𝒆𝒄𝟔 +𝟑 𝒔𝒆𝒄𝟐 𝜽 𝒕𝒂𝒏𝟐 𝜽
व्यंजक का मान बताइए 14.
(𝟏+𝒄𝒐𝒔𝜽)(𝑪𝒐𝒔𝒆𝒄𝜽−𝑪𝒐𝒕𝜽)𝒔𝒆𝒄𝜽
का मान ज्ञात करें |
𝒕𝒂𝒏𝟐 𝜽+ 𝒄𝒐𝒕𝟐 𝜽+𝟐 𝑺𝒊𝒏𝜽(𝟏−𝒔𝒊𝒏𝜽)(𝒔𝒆𝒄𝜽+𝒕𝒂𝒏𝜽)
जहां 𝟎° < 𝜽 < 𝟗𝟎° है| 𝟐
a) 𝒔𝒆𝒄 𝜽 b) 𝒔𝒊𝒏 𝜽 𝟐
c) 𝒄𝒐𝒔𝟐 𝜽 d) 𝒄𝒐𝒔𝒆𝒄𝟐 𝜽
a) 𝒔𝒆𝒄𝟐 𝜽 𝒄𝒐𝒔𝒆𝒄𝟐 𝜽 b) −𝒔𝒆𝒄𝟐 𝜽 𝒄𝒐𝒔𝒆𝒄𝟐 𝜽
c) 𝒄𝒐𝒔𝟐 𝜽 𝒔𝒊𝒏𝟐 𝜽 d) −𝒄𝒐𝒔𝟐 𝜽 𝒔𝒊𝒏𝟐 𝜽 15. If 𝒕𝒂𝒏𝜽 + 𝒄𝒐𝒕𝜽 = 𝟒, then the ratio of
𝟑(𝒕𝒂𝒏𝟐 𝜽 + 𝒄𝒐𝒕𝟐 𝜽) to (𝟐𝒄𝒐𝒔𝒆𝒄𝟐 𝜽 𝒔𝒆𝒄𝟐 𝜽 − 𝟒)
FOR VIDEO SOLUTION will be:
FROM QUESTIONS 11 TO 20 CLICK HERE यदि 𝒕𝒂𝒏𝜽 + 𝒄𝒐𝒕𝜽 = 𝟒 है , तो 𝟑(𝒕𝒂𝒏𝟐 𝜽 +
OR SCAN QR CODE GIVEN BELOW
𝒄𝒐𝒕𝟐 𝜽) का (𝟐𝒄𝒐𝒔𝒆𝒄𝟐 𝜽 𝒔𝒆𝒄𝟐 𝜽 − 𝟒) से अनुपात
ज्ञात करें |
a) 𝟒 : 𝟑 b) 𝟑 : 𝟒 c) 𝟓 : 𝟒 d) 𝟑 : 𝟐

16. What is the average of the sixty terms given


below?
𝝅 नीचे दिए गए साठ पिों का औसत क्या होगा?
11. If 𝒔𝒊𝒏𝜶 + 𝒄𝒐𝒔𝜶 = 𝒕𝒂𝒏 𝟑 , then the value of
𝒄𝒐𝒔𝟐 𝒙, 𝒄𝒐𝒔𝟐 𝟐𝒙, 𝒄𝒐𝒔𝟐 𝟑𝒙, … , 𝒄𝒐𝒔𝟐 𝟑𝟎𝒙, 𝒔𝒊𝒏𝒙, 𝒔𝒊𝒏𝟐 𝟐𝒙, 𝒔𝒊
(𝒔𝒊𝒏𝟑 𝜶 + 𝒄𝒐𝒔𝟑 𝜶) is equal to:
a) 𝒄𝒐𝒔𝟐 𝒙 b) 𝟎. 𝟓
यदि 𝒔𝒊𝒏𝜶 + 𝒄𝒐𝒔𝜶 = 𝒕𝒂𝒏 𝟑 है, तो (𝒔𝒊𝒏𝟑 𝜶 +
𝝅
c) 𝟏 d) 𝒄𝒐𝒔𝟐 𝒙 𝒔𝒊𝒏𝟐 𝒙
𝒄𝒐𝒔𝟑 𝜶) का मान ज्ञात कीजजए|
a) ½ b) 1 c) 0 d) 3/2 17. If 𝟓𝒔𝒊𝒏𝟐 𝜽 − 𝟒 𝒄𝒐𝒔 𝜽 − 𝟒 = 𝟎, 𝟎° < 𝜽 < 𝟗𝟎°,
then the value of (𝒄𝒐𝒕 𝜽 + 𝒄𝒐𝒔𝒆𝒄 𝜽) is:
12. If 𝟎° < 𝜽 < 𝟗𝟎°, then यदि 𝟓𝒔𝒊𝒏𝟐 𝜽 − 𝟒 𝒄𝒐𝒔 𝜽 − 𝟒 = 𝟎, है , 𝟎° < 𝜽 <
(𝟏−𝒔𝒊𝒏𝜽) (𝒔𝒆𝒄 𝜽+𝒕𝒂𝒏 𝜽) 𝒕𝒂𝒏 𝜽
(𝒕𝒂𝒏 𝜽+𝒔𝒆𝒄 𝜽+𝟏) (𝒄𝒐𝒕 𝜽−𝒄𝒐𝒔𝒆𝒄 𝜽+𝟏)
=? 𝟗𝟎° है, तो (𝒄𝒐𝒕 𝜽 + 𝒄𝒐𝒔𝒆𝒄 𝜽) का मान ज्ञात
करें |
यदि 𝒄𝒐𝒔𝜽 = 𝟏+𝟒𝒙𝟐 तो 𝒔𝒊𝒏 𝜽 का मान क्या
𝟑 √𝟔 √𝟔 𝟐 𝟒𝒙
a) 𝟐 b) c) d) 𝟑
𝟐 𝟑
होगा?
𝒔𝒊𝒏𝟐 𝜽 𝟏+𝟒𝒙𝟐 𝟏+𝟒𝒙𝟐
18. If = 𝟏, 𝜽 lies in the first a) 𝟏−𝟒𝒙𝟐 b)
𝒄𝒐𝒔𝟐 𝜽 −𝟑 𝒄𝒐𝒔 𝜽+𝟐 𝟒𝒙𝟐
𝜽 𝜽
𝒕𝒂𝒏𝟐 + 𝒔𝒊𝒏𝟐 𝟏−𝟒𝒙𝟐 𝟏−𝟒𝒙𝟐
𝟐 𝟐
quadrant, then the value of is: c) 𝟏+𝟒𝒙𝟐 d)
𝒕𝒂𝒏 𝜽+𝒔𝒊𝒏 𝜽 𝟒𝒙𝟐
𝒔𝒊𝒏𝟐
यदि = 𝟏, 𝜽 प्रथम चतुथाांश में
𝜽
𝒄𝒐𝒔𝟐 𝜽 −𝟑 𝒄𝒐𝒔 𝜽+𝟐 𝟐√𝒂𝒃
𝜽
𝒕𝒂𝒏𝟐 + 𝒔𝒊𝒏𝟐
𝜽 22. If 𝒔𝒊𝒏𝜽 = , 𝒂 > 𝒃 > 𝟎, then the value of
जथथत है , तो 𝟐 𝟐
का मान बताइए| 𝒄𝒐𝒔𝜽+𝟏
𝒂+𝒃
𝒕𝒂𝒏 𝜽+𝒔𝒊𝒏 𝜽 will be:
𝟐√𝟑 𝟓√𝟑 𝟐√𝟑 𝟕√𝟑 𝒄𝒐𝒔−𝟏
a) b) c) d)
यदि 𝒔𝒊𝒏𝜽 = है , 𝒂 > 𝒃 > 𝟎 है , तो
𝟐𝟕 𝟐𝟕 𝟗 𝟓𝟒 𝟐√𝒂𝒃 𝒄𝒐𝒔𝜽+𝟏
𝒂+𝒃 𝒄𝒐𝒔𝜽−𝟏

19. For 𝜽: 𝟎° < 𝜽 < 𝟗𝟎° का मान ज्ञात करें |


𝒃 𝒂 𝒂 𝒃
𝟑 𝒔𝒆𝒄 𝜽 + 𝟒 𝒄𝒐𝒔 𝜽 = 𝟒√𝟑, find the value of a) − 𝒂 b) − 𝒃 c) 𝒃 d) 𝒂
(𝟏 − 𝒔𝒊𝒏𝜽 + 𝒄𝒐𝒔𝜽).
𝜽 के लिए: 𝟎° < 𝜽 < 𝟗𝟎° है | 23. For what value of 𝜽 (in degrees) is the following
𝟑 𝒔𝒆𝒄 𝜽 + 𝟒 𝒄𝒐𝒔 𝜽 = 𝟒√𝟑 है, तो (𝟏 − 𝒔𝒊𝒏𝜽 + equation true?
ननम्न समीकरण में 𝜽 (डिग्री में) का सही मान
𝒄𝒐𝒔𝜽) का मान ज्ञात करें |
𝟏+𝟐√𝟑 𝟏+√𝟑 𝟏−√𝟑 𝟏−𝟐√𝟑 क्या होगा?
a) b) c) d) 𝟏 𝝅
𝟐 𝟐 𝟐 𝟐
𝒔𝒊𝒏𝟑𝜽 𝒄𝒐𝒔𝜽 − 𝒄𝒐𝒔𝟑𝜽 𝒔𝒊𝒏 𝜽 = 𝟐 , 𝟎 < 𝜽 < 𝟐
20. Solve the following equation. a) 45 b) 30 c) 60 d) 15
𝜽: 𝟐𝒄𝒐𝒔𝟐 𝜽 + (𝟒 + √𝟑) 𝒔𝒊𝒏𝜽 − 𝟐(𝟏 + √𝟑) =
𝟎 where 𝜽 is an acute angle. 24. If 𝒔𝒊𝒏 𝜶 + 𝒔𝒊𝒏 𝜷 = 𝒄𝒐𝒔 𝜶 + 𝒄𝒐𝒔 𝜷 = 𝟏, then
𝒔𝒊𝒏 𝜶 + 𝒄𝒐𝒔 𝜶 =?
ननम्नलिखित समीकरण को हि करें ।
यदि 𝒔𝒊𝒏𝜶 + 𝒔𝒊𝒏 𝜷 = 𝒄𝒐𝒔 𝜶 + 𝒄𝒐𝒔 𝜷 = 𝟏 है, तो
𝜽: 𝟐𝒄𝒐𝒔𝟐 𝜽 + (𝟒 + √𝟑) 𝒔𝒊𝒏𝜽 − 𝟐(𝟏 + √𝟑) =
𝒔𝒊𝒏 𝜶 + 𝒄𝒐𝒔 𝜶 =?
𝟎 जहााँ 𝜽 एक न्यून कोण है | a) 2 b) 1 c) 0 d) −𝟏
a) 𝟑𝟎° b) 𝟒𝟓°
c) 𝟏𝟓° d) 𝟔𝟎° 𝒔𝒆𝒄 𝜽 −𝒕𝒂𝒏 𝜽 𝟏
25. If = 𝟕 , 𝜽 lies in first quadrant, then
𝒔𝒆𝒄 𝜽+𝒕𝒂𝒏 𝜽
𝒄𝒐𝒔𝒆𝒄 𝜽+𝒄𝒐𝒕𝟐 𝜽
FOR VIDEO SOLUTION the value of 𝒄𝒐𝒔𝒆𝒄 𝜽 −𝒄𝒐𝒕𝟐 𝜽 is:
यदि = 𝟕 , 𝜽 प्रथम चतुथाांश में जथथत
FROM QUESTIONS 21 TO 30 CLICK HERE 𝒔𝒆𝒄 𝜽 −𝒕𝒂𝒏 𝜽 𝟏
𝒔𝒆𝒄 𝜽+𝒕𝒂𝒏 𝜽
OR SCAN QR CODE GIVEN BELOW 𝒄𝒐𝒔𝒆𝒄 𝜽+𝒄𝒐𝒕𝟐 𝜽
है , तो का मान बताइए|
𝒄𝒐𝒔𝒆𝒄 𝜽 −𝒄𝒐𝒕𝟐 𝜽
𝟏𝟗 𝟐𝟐 𝟑𝟕 𝟑𝟕
a) b) c) 𝟏𝟐 d) 𝟏𝟗
𝟓 𝟑

26. The expression


(𝟏−𝒔𝒊𝒏 𝜽+𝒄𝒐𝒔 𝜽)𝟐 (𝟏−𝒄𝒐𝒔 𝜽)𝒔𝒆𝒄𝟑 𝜽 𝒄𝒐𝒔𝒆𝒄𝟐 𝜽
(𝒔𝒆𝒄 𝜽−𝒕𝒂𝒏 𝜽)(𝒕𝒂𝒏 𝜽+𝒄𝒐𝒕 𝜽)
, 𝟎° < 𝜽 <
𝟗𝟎, is equal to:
𝟒𝒙
21. If 𝒄𝒐𝒔𝜽 = 𝟏+𝟒𝒙𝟐 then what is the value of
𝒔𝒊𝒏 𝜽?
(𝟏−𝒔𝒊𝒏 𝜽+𝒄𝒐𝒔 𝜽)𝟐 (𝟏−𝒄𝒐𝒔 𝜽)𝒔𝒆𝒄𝟑 𝜽 𝒄𝒐𝒔𝒆𝒄𝟐 𝜽
व्यंजक (𝒔𝒆𝒄 𝜽−𝒕𝒂𝒏 𝜽)(𝒕𝒂𝒏 𝜽+𝒄𝒐𝒕 𝜽)
का
मान बताइए, जहां 𝟎° < 𝜽 < 𝟗𝟎° है|
a) 𝒔𝒊𝒏 𝜽 b) 𝟐 𝒄𝒐𝒔 𝜽 c) 𝒄𝒐𝒕 𝜽 d) 𝟐 𝒕𝒂𝒏 𝜽

𝒔𝒆𝒄 𝜽 𝒄𝒐𝒔𝒆𝒄𝜽
27. The value of 𝟐+𝒕𝒂𝒏𝟐 𝜽+𝒄𝒐𝒕𝟐 𝜽 is equal to:
का मान ज्ञात करें |
𝒔𝒆𝒄 𝜽 𝒄𝒐𝒔𝒆𝒄𝜽
𝟐+𝒕𝒂𝒏𝟐 𝜽+𝒄𝒐𝒕𝟐 𝜽
31. If 𝟕 𝒔𝒊𝒏𝟐 𝜽 + 𝟒 𝒄𝒐𝒔𝟐 𝜽 = 𝟓 and 𝜽 lies in the
a) 𝐬𝐞𝐜⁡ 𝜽𝒄𝒐𝒔𝒆𝒄𝜽 b) 𝐬𝐞𝐜⁡ 𝜽 𝐬𝐢𝐧⁡ 𝜽 first quadrant, then what is the value of
c) 𝐬𝐢𝐧⁡ 𝜽 𝐜𝐨𝐬⁡ 𝜽 d) 𝐜𝐨𝐬⁡ 𝜽𝒄𝒐𝒔𝒆𝒄𝜽 √𝟑 𝒔𝒆𝒄 𝜽+𝒕𝒂𝒏 𝜽
?
√𝟐 𝒄𝒐𝒕 𝜽−√𝟑 𝒄𝒐𝒔 𝜽

28. If 𝒄𝒐𝒕𝟐 𝜽 + 𝒄𝒐𝒕𝟒 𝜽 = 𝟐, then the value of यदि 𝒔𝒊𝒏 𝜽 + 𝟒 𝒄𝒐𝒔𝟐 𝜽 = 𝟓 और


𝟐
𝜽 पहिे
𝟐 𝒔𝒊𝒏𝟒 𝜽 + 𝒔𝒊𝒏𝟐 𝜽 is: चतुथााश में जथथत है , तो का मान
√𝟑 𝒔𝒆𝒄 𝜽+𝒕𝒂𝒏 𝜽
√𝟐 𝒄𝒐𝒕 𝜽−√𝟑 𝒄𝒐𝒔 𝜽
यदि 𝒄𝒐𝒕 𝜽 + 𝒄𝒐𝒕 𝜽 = 𝟐 है , तो 𝟐 𝒔𝒊𝒏 𝜽 +
𝟐 𝟒 𝟒
क्या होगा?
𝒔𝒊𝒏𝟐 𝜽 का मान ज्ञात करें | a) 𝟒√𝟐 b) 𝟐(𝟏 + √𝟐 )
a) 3 b) 5 c) 1 d) 2 c) 𝟑√𝟐 d) 𝟐(√𝟐 − 𝟏)
𝟓
29. If 𝒔𝒊𝒏 𝑨 = 𝟏𝟑 and 𝟕 𝒄𝒐𝒕 𝑩 = 𝟐𝟒, then the value 32. The value of
(𝒄𝒐𝒔 𝟗°+𝒔𝒊𝒏 𝟖𝟏°)(𝒔𝒆𝒄 𝟗°+𝒄𝒐𝒔𝒆𝒄 𝟖𝟏°)
𝒄𝒐𝒔𝒆𝒄𝟐 𝟕𝟏°+𝒄𝒐𝒔𝟐 𝟏𝟓°−𝒕𝒂𝒏𝟐 𝟏𝟗°+ 𝒄𝒐𝒔𝟐 𝟕𝟓°
of (𝒔𝒆𝒄 𝑨 𝒄𝒐𝒔 𝑩)(𝒄𝒐𝒔𝒆𝒄 𝑩 𝒕𝒂𝒏 𝑨) is: is:
यदि 𝒔𝒊𝒏 𝑨 = 𝟏𝟑 और है , तो
𝟓
𝟕 𝒄𝒐𝒕 𝑩 = 𝟐𝟒 का मान
(𝒄𝒐𝒔 𝟗°+𝒔𝒊𝒏 𝟖𝟏°)(𝒔𝒆𝒄 𝟗°+𝒄𝒐𝒔𝒆𝒄 𝟖𝟏°)
𝒄𝒐𝒔𝒆𝒄𝟐 𝟕𝟏°+𝒄𝒐𝒔𝟐 𝟏𝟓°−𝒕𝒂𝒏𝟐 𝟏𝟗°+ 𝒄𝒐𝒔𝟐 𝟕𝟓°
(𝒔𝒆𝒄 𝑨 𝒄𝒐𝒔 𝑩)(𝒄𝒐𝒔𝒆𝒄 𝑩 𝒕𝒂𝒏 𝑨) का मान क्या बताइए|
होगा? a) 𝟏 b) 𝟒 c) −𝟑 d) 𝟐
𝟏𝟑 𝟏𝟓 𝟏𝟑 𝟔𝟓
a) 𝟏𝟒 b) 𝟏𝟑 c) d) 𝟒𝟐
𝟕 𝟓𝒄𝒐𝒕 𝜽+√𝟑𝒄𝒐𝒔𝒆𝒄 𝜽
33. If = 𝟏, 𝜽 < 𝜽 < 𝟗𝟎°, then the
𝟐√𝟑𝒄𝒐𝒔𝒆𝒄𝜽+𝟑𝒄𝒐𝒕𝜽
𝟐 𝟐
30. Let 𝟎° < 𝜽 < 𝟗𝟎°. (𝟏 + 𝒄𝒐𝒕 𝜽)(𝟏 + 𝒕𝒂𝒏 𝜽) × 𝟕 𝟑
𝒄𝒐𝒕𝟐 𝜽− 𝒄𝒐𝒔𝒆𝒄𝟐 𝜽
(𝒔𝒊𝒏𝜽 − 𝒄𝒐𝒔𝒆𝒄𝜽)(𝒄𝒐𝒔𝜽 − 𝒔𝒆𝒄𝜽) is equal to: value of 𝟐 𝟒
𝟑 will be:
𝟒𝒔𝒊𝒏𝟐 𝜽+ 𝒕𝒂𝒏𝟐 𝜽
𝟐
मान िें कक 𝟎° < 𝜽 < 𝟗𝟎° है| तो (𝟏 +
यदि = 𝟏, 𝜽 < 𝜽 < 𝟗𝟎° है , तो
𝟓𝒄𝒐𝒕 𝜽+√𝟑𝒄𝒐𝒔𝒆𝒄 𝜽

𝒄𝒐𝒕𝟐 𝜽)(𝟏 + 𝒕𝒂𝒏𝟐 𝜽) × (𝒔𝒊𝒏𝜽 − 𝟕


𝟐√𝟑𝒄𝒐𝒔𝒆𝒄𝜽+𝟑𝒄𝒐𝒕𝜽
𝟑
𝒄𝒐𝒕𝟐 𝜽− 𝒄𝒐𝒔𝒆𝒄𝟐 𝜽
𝒄𝒐𝒔𝒆𝒄𝜽)(𝒄𝒐𝒔𝜽 − 𝒔𝒆𝒄𝜽) का मान इनमें से 𝟐 𝟒
𝟑 का मान ज्ञात करें |
𝟒𝒔𝒊𝒏𝟐 𝜽+ 𝒕𝒂𝒏𝟐 𝜽
ककसके बराबर होगा?
𝟐
a) 7 b) 2 c) 3 d) 5
a) 𝒔𝒆𝒄 𝜽 𝒄𝒐𝒔𝒆𝒄 𝜽 b) 𝒔𝒊𝒏 𝜽 + 𝒄𝒐𝒔 𝜽
c) 𝒔𝒆𝒄 𝜽 + 𝒄𝒐𝒔𝒆𝒄 𝜽 d) 𝒔𝒊𝒏 𝜽 𝒄𝒐𝒔 𝜽 34. Two pillars A and B of the same height are on
opposite sides of a road which is 𝟒𝟎 𝒎 wide.
FOR VIDEO SOLUTION The angles of elevation of the tops of the pillars
FROM QUESTIONS 31 TO 37 CLICK HERE A and B are 𝟑𝟎° are 𝟒𝟓°, respectively, at a point
OR SCAN QR CODE GIVEN BELOW on the road between the pillars. What is the
distance (in m) of the point from the foot of
pillar A?
समान ऊाँचाई वािे िो थतंभ A और B, एक ऐसी
37. Let A and B be two towers with same base.
सड़क के िोनों और जथथत हैं। जजसकी चौड़ाई From the midpoint of the line joining their feet,
𝟒𝟎 𝒎 है | िोनों थतंभों के बीच सड़क पर जथथत the angles of elevation of the tops of A and B
एक बबंि ु से थतंभों A और B के शीर्षों के उन्नयन are 𝟑𝟎° and 𝟔𝟎°, respectively. The ratio of the
heights of B and A is:
कोण क्रमशः 𝟑𝟎° और 𝟒𝟓° हैं। थतंभ A के पाि
माना A और B सामान आधार वािी िो मीनारे है |
से उस बबंि ु की िरू ी (m में) ककतनी है ?
और िोनों मीनारों की आधारों को जोड़ने वािी
a) 𝟒𝟎(√𝟑 − 𝟏) b) 𝟐𝟎(𝟐 − √𝟑)
रे िा के मध्यबबंि ु से, A और B के शीर्षा का उन्नयन
c) 𝟐𝟎(𝟑 − √𝟑 ) d) 𝟑𝟗√𝟑
कोण क्रमशः 𝟑𝟎° और 𝟔𝟎° है | B और A की ऊंचाई
35. A person was standing on a road near a mall. का अनुपात ज्ञात करें |
He was 1425 m away from the mall and able to
a) 𝟏 : 𝟑 b) 𝟑 : 𝟏 c) 𝟏 : 𝟐 d) 𝟏 : √𝟑
see the top of the mall from the road in such a
way that the top of a tree, which is in between
him and the mall, was exactly in line of sight ANSWER KEY
with the top of the mall. The tree height is 10 m 1. A 2. A 3. B 4. B 5. A
and it is 30 m away from him. How tall (in m) is 6. A 7. B 8. D 9. A 10. D
the mall? 11. C 12. A 13. D 14. A 15. D
एक व्यजक्त मॉि के ननकट सड़क पर िड़ा है | 16. B 17. B 18. D 19. B 20. D
वह मॉि से 1425m की िरु ी पर है और सड़क से 21. C 22. B 23. D 24. B 25. A
मॉि के शीर्षा को इस प्रकार िे िने में सक्षम है 26. D 27. C 28. C 29. D 30. A
की उसके और मॉि के बीच जथथत पेड़ को शीर्षा 31. B 32. D 33. D 34. C 35. A
36. A 37. A
मॉि के शीर्षा के साथ दिथटी रे िा में है | पेड़ की
उचाई 10m है और यद् उस व्यजक्त से 30m की
िरु ी पर जथथत है | मॉि की ऊंचाई (m में) ज्ञात 1. For Best FREE full syllabus course of
करें | Maths & Reasoning Click here
a) 475 b) 300 c) 425 d) 525

36. The length of the shadow of a vertical pole on


the ground is 18 m. If the angle of elevation of
𝟏𝟐
the sun at that time is 𝜽, such that 𝒄𝒐𝒔𝜽 = 𝟏𝟑,
then what is the height (in m) of the pole?
भू-ति पर ककसी अधोिंब िम्बे की परछाई की
िम्बाई 𝟏𝟖𝒎 है | यदि उस समय सूया का
उन्नयन कोण 𝜽 इस प्रकार है कक 𝒄𝒐𝒔𝜽 = 𝟏𝟑 है ,
𝟏𝟐
To join Our Telegram channel Click here
तो िम्बे की ऊंचाई (m में) ज्ञात करें |
a) 𝟕. 𝟓 b) 9 c) 18 d) 12
Permutation and combination, AP, Co ordinate Geometry, Statistics

Instruction: - For Video Solution scan QR code 5. A fruit seller sells 𝟒𝟓% of the oranges that he
given below or you can also CLICK on Click here has along with one more orange to a customer.
button to watch the Video Solution. He then sells 𝟐𝟎% of the remaining oranges
and 𝟐 more oranges to a second customer. He
then sells 𝟗𝟎% of the now remaining oranges
to a third customer and is still left with 𝟓
oranges. How many oranges did the fruit seller
have initially?
1. What is the sum of all the common terms एक फल ववक्रेता ककसी ग्राहक को अपिे पास
between the given series S1 and S2?
𝑺𝟏 = 𝟐, 𝟗, 𝟏𝟔, … … … , 𝟔𝟑𝟐 मौिूद कुल सृंतरों में से 𝟒𝟓% और एक सृंतरा
𝑺𝟐 = 𝟕, 𝟏𝟏, 𝟏𝟓, … … . , 𝟕𝟒𝟑 बेचता है | कफर वह शेर्ष सृंतरों के 𝟐𝟎% और 𝟐
दी गयी श्ृंख
र लाओृं S1 तथा S2 के मध्य सभी और सृंतरे दस
ू रे ग्राहक को बेचता है | कफर वह बचे
उभयनिष्ठ पदों का योग क्या है ? हुए सृंतरों के 𝟗𝟎% सृंतरे तीसरे ग्राहक को बेचता
𝑺𝟏 = 𝟐, 𝟗, 𝟏𝟔, … … … , 𝟔𝟑𝟐
𝑺𝟐 = 𝟕, 𝟏𝟏, 𝟏𝟓, … … . , 𝟕𝟒𝟑 है और उसके पास अभी भी 𝟓 सृंतरे बच िाते हैं|
a) 𝟔𝟗𝟕𝟒 b) 𝟔𝟕𝟓𝟎 फल ववक्रेता के पास आरृं भ में ककतिे सृंतरे थे?
c) 𝟕𝟏𝟒𝟎 d) 𝟔𝟖𝟔𝟎 a) 𝟏𝟎𝟎 b) 𝟏𝟏𝟏 c) 𝟏𝟐𝟏 d) 𝟏𝟐𝟎

2. How many numbers are there from 𝟒𝟎𝟎 to 𝟕𝟎𝟎


in which the digit 𝟔 occurs exactly twice?
𝟒𝟎𝟎 से 𝟕𝟎𝟎 तक ऐसी ककतिी सृंख्याएृं है , जििमे
6. The given histogram represents the marks of
अृंक 𝟔 ठीक दो बार आता है ?
students in Mathematics test of a certain class.
a) 𝟏𝟗 b) 𝟏𝟖 c) 𝟐𝟏 d) 𝟐𝟎 The total number of students is 𝟑𝟓𝟎 and the
maximum marks of the test are 𝟐𝟎𝟎. What is
3. Fine the radius of the circle 𝒙𝟐 + 𝒚𝟐 − 𝟒𝒙 + the class average (correct up to one place of
𝟔𝒚 = 𝟏𝟐. decimal) of mathematics test?
वत्त
र 𝒙𝟐 + 𝒚𝟐 − 𝟒𝒙 + 𝟔𝒚 = 𝟏𝟐 की त्रिज्या ज्ञात ददया गया दहस्टोग्राम ककसी कक्षा के गणणत की
करें | परीक्षा में छािों के अृंकों का प्रदर्शुत करता है।
a) 𝟐 b) 𝟓 c) 𝟒 d) 𝟑
छािों की कुल सृंख्या 𝟑𝟓𝟎 है और परीक्षा का
4. Calculate the area of the quadrilateral formed पूणाुक 𝟐𝟎𝟎 हैं। गणणत की परीक्षा का कक्षा औसत
with the vertices (−𝟑, 𝟐), (𝟓, 𝟒), (𝟕, −𝟔) and (दशमलव के एक स्थाि तक सही) क्या है ?
(−𝟓, −𝟒).
शीर्षों (−𝟑, 𝟐), (𝟓, 𝟒), (𝟕, −𝟔) और (−𝟓, −𝟒) से
बिे चतभ ु ि
ु के क्षेिफल की गणिा कीजिए ।
a) 𝟎 Square units b) 𝟏𝟓𝟎 square units
c) 𝟏𝟔𝟎 square units d) 𝟖𝟎 square units
a) 𝟏𝟏𝟗. 𝟑 b) 𝟏𝟐𝟑. 𝟕 d) an array
c) 𝟏𝟏𝟓. 𝟖 d) 𝟏𝟐𝟕. 𝟑
Answer Key
7. Based on the date given in the following table, 1. A 2. D 3. B 4. D 5. D
find the arithmetic mean of the number of 6. A 7. D 8. B
major meals that include a potato-based dish,
which the 𝟑𝟗 students surveyed, had eaten in a
week.
निम्िर्लणखत तार्लका में ददए गए डेटा के आधार
पर उि प्रमख
ु भोिि की सृंख्या का समान्तर
माध्य ज्ञात कीजिए जििमे एक आलू आधाररत
व्यृंिि को शार्मल था जिसे सवेक्षण में शार्मल
𝟑𝟗 छािों द्वारा खाया गया था।

a) 𝟕 b) 𝟓
c) 𝟖 d) 𝟔

8. A way of punching data that exhibits the values


of the variables and corresponding frequencies
is called:
आँकड़ों की प्रववजष्ट का ऐसा तरीका िो चरों
केववर्भन्ि मािों तथा प्रत्येक चर की सृंगत
बारम्बारताओृं प्रदर्शुत करे , वह क्या कहलाता है ?
a) Frequency of observation
b) Frequency of distribution
c) Raw data
Data Interpretation for SSC CGL CHSL
Exercise 1

FOR VIDEO SOLUTION FROM QUESTION 1 TO


10: CLICK HERE OR
SCAN Q R CODE GIVEN BELOW

1. The given table represents the number of


computers sold by four dealers 𝑨, 𝑩, 𝑪 and 3. What is the ratio of the total number of
D during the first six months of 2016. Study computers sold by dealer A in February,
the table and answer the questions that April and May to the total number of
follows: computers sold by dealer D in March, May
and June?
दी गई तालिका में 2016 के प्रारम्भिक छह
महीनों के दौरान चार डीिरों A,B,C और D द्वारा
बेचे गए कंप्यूटरों की संख्या दर्ााई गई है | डीिर A द्वारा फरवरी, अप्रैि और मई में बेचे गए
तालिका का अध्धयन करें और ननभन प्रश्न का कुि कंप्युटरों की संख्या का, डीिर D द्वारा माचा,
उत्तर दें | मई और जून में बेचे गए कुि कंप्युटरों की संख्या
से अनुपात क्या है ?
a) 15:13 b) 10:9 c) 20:27 d) 6:5

2. The number of months, in which the number


of computers sold by dealer B was less than
average number of computers sold by
dealer C over six months was:
4. The total number of computers sold by dealer
डीिरों B द्वारा बेचे गए कंप्युटरों की संख्या A during February to June is what
ककतने महीनों में, डीिर C द्वारा छह महीने में बेचे percentage more than the total number of
गए कंप्युटरों की औसत संख्या से कम थी? computers sold by all the dealers in June?
a) 5 b) 4 c) 2 d) 3 डीिर A द्वारा फरवरी से जून के दौरान बेचे गए
कंप्युटरों की कुि संख्या, जून में सिी डीिरों
द्वारा बेचें गए कंप्युटरों की कुि संख्या से ककतने
प्रनतर्त अधधक है ?
a) 17.5 b) 25.3 c) 24.4 d) 21.2 ककस महीने में कंपनी II द्वारा उवारक उत्पादन
उसके पांच महीनों में हुए कुि उत्पादन के िगिग
20% के बराबर है ?
a) April b) May c) March d) January

5. The total number of computers sold by dealer


B in April, May and June is what percentage
of the total number of computers sold by all 8. There is a continuous decrease in production
the dealers in February and April? over the months in:
डीिर B द्वारा अप्रैि, मई और जून में बेचे गए ददए गए महीनों में , ककस कंपनी के उवारक
कंप्युटरों की कुि संख्या फ़रवरी और अप्रैि में उत्पादन में िगातार कमी हुई है |
सिी डीिरों द्वारा बेचे गए कंप्युटरों की कुि a) Company III b) Company II
संख्या का ककतने प्रनतर्त है ? c) Company I d) Company IV
𝟕 𝟔 𝟓 𝟑
a) 50 b) 43 c) 48 d) 38
𝟖 𝟕 𝟕 𝟖

9. In a particular year, the number of students


enrolled in different streams in a college is
as following:
6. The following table shows the production of
एक ववर्ेष वषा में, एक कॉिेज में ववलिन्न
fertilizers (in lakh tone) by six companies for
five months (Jan to may). ववद्याियों में नामांककत छात्रों की संख्या इस
ननभनलिखित तालिका 5 महीनों के लिए छह प्रकार है :
कंपनीयों द्वारा उवारकों के उत्पादन को दर्ााती है | The percentage of girl students is:
छात्राओं का प्रनतर्त है :
a) 𝟓𝟒% b) 𝟒𝟔% c) 𝟏𝟑𝟓% d) 𝟓𝟎%

10. From the following table, how many


patients were in the age group 40 – 60?
7. In which month does company II have a ननभनलिखित तालिका से, 40 – 60 आयु वगा में
contribution of approximately 20% of its
ककतने रोगी थे?
total fertilizer production in five months?
a) 29 b) 45 c) 16 d) 6 a) 41% b) 54% c) 58% d) 42%

14. The following table shows the number of


employees working in various departments
FOR VIDEO SOLUTION FROM QUESTION 11 TO
of an organization from 2016 to 2019.
20: CLICK HERE OR
SCAN Q R CODE GIVEN BELOW ननभन तालिका ककसी संगठन के ववलिन्न वविागों
में 2016 से 2019 में कायारत कमाचाररयों की
संख्याको दर्ााती है I
In which year were the maximum number
of employees working in the organization:
ककस वषा में उस संगठनमें कायारत कमाचाररयों की
11. From the given table. What is the
percentage of students scoring 40 or more, संख्या अधधकतम थी?
but less than 70? a) 2016 b) 2017 c) 2019 d) 2018
दी गई तालिका से, 40 या अधधक िेककन 70 से
कम स्कोर करने वािे छात्रों का प्रनतर्त क्या है ।
a) 48% b) 96% c) 8% d) 56%

15. The table below shows income (in rupee) for


a particular month, together with their
sources in respect of employees A, B, C, D
12. In a school, the distribution of teachers is as and E.
follows: नीचे दी गई तालिका एक ववर्ेष महीने की आय
एक स्कूि में, लर्क्षकों का ववतरण इस प्रकार है : (रुपये में) दर्ााती है , क्रमर्ः कमाचाररओं A ,B ,C
The total number of teachers of age less
,D और E के सिी आय के स्त्रोतों के साथ |
than 40 years is:
How many employees have their salary
40 वषा से कम आयु के लर्क्षक की कुि संख्या
more than four times their other incomes?
है : ककतने कमाचाररयों का वेतन उनकी अन्य आयों
a) 12 b) 18 c) 10 d) 39
के. चार गुना से अधधक है ?
a) 3 b) 2 c) 1 d) 4

13. As per data in the table, what is the


percentage of students who got 20 or more
marks?
तालिका में ददए गए आंकड़ों के अनुसार, 20 या
उससे अधधक अंक पाने वािे छात्रों का प्रनतर्त
16. The given table represents the exports (in
क्या है ? crores) of four items A, B, C and D over a
period of six years. Study the table and
answer the question that follows:
नीचे दी गई तालिका छह वषा की अवधी में चार exports of all the four items in 2011 and
वस्तुओं A , B , C और D के ननयाात को ननरुवपत
2012?
2010,2012 और 2014 में वस्तु D का कुि
करती है | तालिका का अध्धयन करे ओर प्रश्नों के
ननयाात 2011 और 2012 में सिी चार वस्तओ
ु ं के
उत्तर दें |
कुि ननयाात का ककतना प्रनतर्त है ?
a) 44% b) 44.8% c) 45% d) 46.2%

17. What is the ratio of the total exports of


items A in 2014 and 2015 to the total 20. In which year, the export of items D were
exports of item C in 2011 and 2015? 1.4 times the average of the item B during
2014 और 2015 में वस्तु A के कुि ननयाात का, six years?
2011 और 2015 में वस्तु C के कुि ननयाात से ककस वषा में, वस्तु D का ननयाात छह के दौरान
अनुपात क्या है ? वस्तु B के औसत ननयाात का 1.4 गुणा था?
a) 3:2 b) 7:5 c) 5:4 d) 4:3 a) 2013 b) 2012 c) 2014 d) 2011

FOR VIDEO SOLUTION FROM QUESTION 21 TO


18. The total exports of items A from 2012 to 30: CLICK HERE OR
2014 is what percentage less than the total SCAN Q R CODE GIVEN BELOW
exports of all the four items in 2015?
2012 से 2014 तक वस्तु A का कुि ननयाात 2015
में सिी चार वस्तुओं के कुि ननयाात से ककतना
कम है ?
a) 15.2% b) 13.8% c) 16.7% d)14.3% 21. The following table represents the number
of candidates that appeared and qualified in
a competitive examination from different
states over five years. Study the table and
answer the question follow.

19. The total exports of items D in 2010,2012


and 2014 is what percentage of total
ननभन तालिका उन उभमीदवारों की संख्या का 1998 में सिी राज्यों के उत्तीणा प्रत्यार्ीयों की
प्रनतननधधत्व करती है जो पांच वषों में ववलिन्न कुि संख्या, 2001 में सिी राज्यों के उत्तीणा
राज्यों से एक प्रनतयोगी परीक्षा में उपम्स्थत हुए प्रत्यार्ीयों की कुि संख्या की िगिग ककतनी
और योग्य थे। तालिका का अध्ययन करें और प्रनतर्त है ?
प्रश्न का उत्तर दें । a) 96.70% b) 90.72%
c) 94.7% d) 97.72%

22. What is the difference between the number


of candidates qualified in the year 1998, in 25. What is the average of candidates that
the states M and P? appeared from State Q during the given
राज्यों M और P में वषा 1998 में योग्य years?
उभमीदवारों की संख्या के बीच क्या अंतर है ? ददए गए वषो के दौरान राज्य Q से िाग िेने वािे
a) 60 b) 50 c) 44 d) 40 प्रत्यार्ीयों की औसत संख्या क्या होगी?
a) 8890 b) 9000 c) 8980 d) 8880

23. What was the total number of candidates 26. The following table shows the imports and
appearing from all the states in the year exports (in crore) of a country over 4 year
1997? (2016 to 2019).
वषा 1997 में सिी राज्यों से र्ालमि होने वािे ननभन तालिका 4 वषो (2016 से 2019) के दौरान
उभमीदवारों की कुि संख्या ककतनी थी? एक दे र् के आयात और ननयाात को दर्ााती है |
a) 22,700 b) 27,200 The average trade balance (in crore) is:
c) 27,000 d) 72,200 औसत व्यापार संतिु न ककतना है ?
a) 8 b) 6 c) 4 d) 12

24. The total number of candidates that 27. Expenditures of a company (in lakh rupees)
qualified from all the states together in per annum over the given years.
1998 is approximately what percentage of Total expenditure on all the items in 2014
the total number of candidates that was approximately what percent of the
qualified from all the states together in total expenditure in 2018?
2001? (Correct to two decimal points)
वषा 2014 में कंपनी का सिी मदों पर होने वािा four products to the total revenue from the
व्यय, वषा 2018 के सिी मदों पर होने वािे व्यय
sale of product C in 2014 to 2017?
2014 में कंपनी के उन सिी चार उत्पादों की
का ककतना प्रनतर्त है ?
बबक्री से प्राप्त कुि राजस्व का, उत्पाद C की
a) 60.32% b) 68.32%
c) 66.32% d) 65.32% 2014 से 2017 तक की बबक्री से प्राप्त कुि
राजस्व से क्या अनुपात है ?
a) 14:23 b) 18:19 c) 7:10 d) 7:9

28. Study the given table carefully and answer FOR VIDEO SOLUTION FROM QUESTION 31 TO
the question that follows: 40: CLICK HERE OR
दी गई तालिका का ध्यानपूवक
ा अध्ययन करें और SCAN Q R CODE GIVEN BELOW
उस प्रश्न का उत्तर दें :
The percentage of students who have
passed with distinction in the year 2012, is:
वषा 2012 में उत्तीणा होने वािे छात्रों का प्रनतर्त
31. By what percentage is the total revenue of
है : the company from the sale of products A,B
a) 20% b) 27% c) 25% d) 22% and D in 2012 and 2013 more than the total
revenue from the sale of product B in 2013
to 2016?
2012 और 2013 में उत्पाद A, B और D की बबक्री
से प्राप्त कंपनी का कुि राजस्व, 2013 से 2016 के
बीच उत्पाद B की बबक्री से प्राप्त कुि राजस्व से
29. The given table represents the revenue (in ककतने प्रनतर्त अधधक है ?
crore) of a company from the sale of four a) 44.5 b) 31.2 c) 43.6 d) 45.4
products A, B, C and D in 6 years. Study the
table and answer the question that follows.
दी गई तालिका 6 वषो में चार उत्पादों A, B, C
और D की बबक्री से ककसी कंपनी का राजस्व
ददिती है | तालिक का अध्ययन करें और ददए गये
और ददए गए प्रश्न का उत्तर दें | 32. The number of years in which the revenue
of the company from the sale of product D
is more than the average revenue from the
sale of product A over six years, is:
ऐसे ककतने वषा है , म्जनमे उत्पाद D की बबक्री से
प्राप्त कंपनी का राजस्व, छह साि में उत्पाद A की
30. What is the ratio of the total revenue of the बबक्री प्राप्त होने वािे औसत राजस्व से अधधक है
company in 2014 from the sale of all the ?
a) 4 b) 1 c) 3 d) 2
वपछिे वषा की तुिना में िाि में अधधकतम
प्रनतर्त वद्
ृ धध ककस अवधध में हुई?
a) 2015 -2016 b) 2018 – 2019
c) 2016 – 2017 d) 2017 – 2018

33. The total revenue of the company from the


sale of products B, C and D in 2014 is what
percentage of the total revenue from the
sale of product C and D in 6 years?
2014 में उत्पादों B, C और D की बबक्री से प्राप्त 36. The following table shows the percentage of
marks obtained by seven students in six
कंपनी का कुि राजस्व, 6 वषों में उत्पादों C और
different subjects in an examination:
D की बबक्री से प्राप्त कुि राजस्व का ककतना ननभन तालिका ककसी परीक्षा में सात ववद्याधथायों
प्रनतर्त है ?
द्वारा छह अिग-अिग ववषयों में प्राप्त अंक
a) 18 b) 25 c) 28 d) 20
प्रनतर्त को दर्ााती है I
What are the average marks obtained by all
the seven students in Physics?
िौनतक ववज्ञान में सिी सात ववद्याधथायों द्वारा
प्राप्त ककये गये औसत अंक ज्ञात करें I
a) 89.14% b) 80.15%
34. The following table shows the age – wise c) 75.90 d) 85.12
brand ownership of mobile phone handsets.
ननभन तालिका मोबइि फोन हैंडसेट की ननमााण-
अवधी के अनस
ु ार ब्ांड स्वालमत्व को दर्ााती है ?
If a total of 5000 ‘C’ mobile phone sets are
sold till date, then how many are more than
one year old?
यदद आजतक कुि 5000 ‘C’ मोबाइि फोन सेट
बेचे गए तो उनमे से एक वषा से अधधक परु ाने 37. The number of students enroll in different
सेट ककतने थे? faculties in a school is as follows:
a) 4000 b) 4200 c) 4500 d) 4350 एक स्कूि में ववलिन्न संकायों में दाखििा िेने
वािे छात्रों की संख्या इस प्रकार है :
The percentage of students studying in
science or vocational subjects is:
ववज्ञान या व्यावसानयक ववषयों में अध्ययन करने
वािे छात्रों का प्रनतर्त है :
a) 37.2% b) 25% c) 50% d) 93%
35. The following table shows the annual profit
of a company (in lakh)
ननभन तालिका ककसी कंपनी का वावषाक िाि
दर्ााती है ?
The period which has the maximum 38. The following table gives the details of Five
percentage increase in profit over the commodities A, B, C, D and E with quantity
previous year is:
required and their costs for a family in a
month.
Study the table and answer the question
that follows.
ननभनलिखित तालिका में 5 वस्तुओं A, B, C, D
41. The amount spent extra on commodities B
और E की आवश्यक मात्रा और एक माह में , एक
and C in the year 2019 as compared to that
पररवार के लिए नके क्रय मूल्य का वववरण ददया in the year 2016, is:
गया है I तालिका का अध्ययन करें और ददए गये वषा 2016 की तुिना में वषा 2019 में वस्तुओं B
प्रश्न का उत्तर दें : और C पर अनतररक्त रालर् िचा की गई, जो है :
a) 260 b) 192 c) 248 d) 110

39. The ratio of total amount spent on A and D


commodities in the year 2019 is: 42. The total amount spent on the Five
commodities by the family in the year 2019
वषा 2019 में, A और D वस्तओ
ु ं पर व्यय की गयी
is:
कुि रार्ीयों का अनुपात _________ है I
वषा 2019 में पररवार द्वारा पांच वस्तुओं पर िचा
a) 3:8 b) 15:17 c) 6:17 d) 1:1
की गई कुि रालर् है :
a) 6,856 b) 7,248 c) 8,122 d) 5,400

40. The percentage of increase (per kg) in the


rate of commodity D from 2016 to 2019 is:
2016 से 2019 तक वस्तु D के मल्ू य में वद्
ृ धध- 43. The given table represents the number of
प्रनतर्त (प्रनत kg) ________ है I engineers recruited by four companies A, B,
a) 38.24% b) 5% c) 13.33% d) 22.17% C and D over the years. Study the table and
answer the question that follows:
दी गई तालिका चार कंपननयों A, B, C और D
द्वारा िती ककये गए इंम्जननयरों की संख्या को

FOR VIDEO SOLUTION FROM QUESTION 41 TO


50: CLICK HERE OR
SCAN Q R CODE GIVEN BELOW
वषों में दर्ााती है | तालिका का अध्ययन करें और ऐसे वषो की संख्या ककतनी है म्जनके दौरान
ददए गए प्रश्नों का उत्तर दें | कंपनी D द्वारा िती ककए गए इंम्जननयरों की
संख्या का औसत, कंपनी B द्वारा ददए गए छह
वषो में िती ककए जाने वािे इंम्जननयरों की
औसत संख्या से कम है |
44. The total number of engineers recruited by
company A in 2014 to 2017 is what a) 1 b) 3 c) 4 d) 2
percentage more than the total number of
engineers recruited by all four companies in
2019.
2014 से 2017 तक कंपनी A द्वारा िती ककए गए
इंम्जननयरों की कुि संख्या, 2019 में उन सिी
चार कंपननयो द्वारा िती ककये गए इंम्जननयरों की
कुि संख्या से ककतना प्रनतर्त अधधक है ?
47. The ratio of the total number of engineers
a) 3.5 b) 2.5 c) 3 d) 4
recruited by companies A and B in 2015 and
2018 to the total number of engineers
recruited by C and D in 2014 and 2018, is :
कंपननयों A और B द्वारा 2015 और 2018 में िती
ककए गए इंम्जननयरों की कुि संख्या का, कंपननयों
C और D द्वारा 2014 और 2018 में िती ककए
गए इंम्जननयरों की कुि संख्या से अनुपात क्या

45. The total number of engineers recruited by होगा?


company B in 2014 and 2017 is what a) 17:14 b) 13:21 c) 28:19 d) 9:14
percentage of the total number of engineers
recruited by C during 2015 to 2019.
2014 से 2017 में कंपनी B द्वारा िती ककए गए
इंम्जननयरों की कुि संख्या 2015 से 2019 तक
कंपनी C द्वारा िती ककए गए इंम्जननयरों की कुि
संख्या का ककतने प्रनतर्त है ?
a) 38.2 b) 38.4 c) 39.2 d) 37.8
48. The given table represents the production of
different type of motorcycles (in thousands)
over a period of six years. Study the table
and answer the question that follows.
दी गई तालिका छह वषों की अवधी में ववलिन्न
टाइप की मोटरसाइककिों के कुि उत्पादन को
दर्ााती है | तालिका का अध्धयन करें और ददए
46. The number of the year in which the
number of engineers recruited by company गये प्रश्नों का उत्तर दें |
D is less than the average number of
engineers recruited by B in the given six
years, is:
51. What is the percentage increase in the total
production of all type of motorcycles from
2014 to 2018?
2014 से 2018 तक हुए सिी टाइप की
मोटरसाईककिों के कुि उत्पादन में प्रनतर्त –
वद्
ृ धध क्या है ?
49. During 2015, the production of which type 𝟐 𝟐 𝟑 𝟏
a) 16𝟑 b) 14𝟕 c) 14𝟕 d) 17𝟑
of motorcycle was more than 25% of the
total production of all types of motorcycles
in 2017?
2015 के दौरान, ककस टाइप की मोटरसाईककिों का
उत्पादन 2017 में हुए सिी टाइप की
मोटरसाईककिों के कुि उत्पादन से 25% अधधक
था? 52. By what percentage is the total production
a) D b) B c) A d) C of type A motorcycles over six years, less
than the total production of all types of
motorcycles in 2013 and 2016?
छह वषों में टाईप A वािी मोटरसाईकािों का
उत्पादन, 2013 और 2016 में हुए सिी प्रकार की
मोटरसाईककिों के उत्पादन से ककतने प्रनतर्त कम
है ?
50. What is the ratio of the total number of
a) 32.2 b) 31.6 c) 32.8 d) 30.5
motorcycles of type B produced in 2016 and
2018 to the total number of motorcycles of
type D produced in 2013, 2015 and 2016?
2016 और 2018 में बनाई गई टाइप B वािी
मोटरसाईककिों की कुि संख्या का अनप
ु ात 2013,
2015 और 2016 में उत्पाददत टाइप D वािी
मोटरसाईककिों की कुि संख्या से क्या है ? 53. In the following table, the production of
a) 3:5 b) 3:4 c) 2:3 d) 1:2 various crops (in tonnes) is given from 2015
to 2019. Study the table and answer the
question that follow:
ननभनलिखित तालिका में , ववलिन्न फसिों का
2015 से 2019 तक उत्पादन (टनों में) ददया गया
है I तालिका का अध्ययन करें और ददए गये प्रश्न
का उत्तर दें :
FOR VIDEO SOLUTION FROM QUESTION 51 TO
60: CLICK HERE OR
SCAN Q R CODE GIVEN BELOW
54. The average production of wheat (in tonnes) c) 71.43% d) 77.77%
in the period given in the table is:
तालिका में दी गयी अवधी में गें हू का औसत
उत्पादन (टनों में) ककतना है ?
a) 3600 b) 3300 c) 3482 d) 4218

58. The following table shows the numbers of


students enrolled in different faculties in a
college:
ननभनलिखित तालिका एक महाववद्यािय के
55. The highest growth of sugarcane production ववलिन्न संकायों में दाखिि हुए छात्रों की संख्या
over its previous year is recorded in the दर्ााती है I
year: The percentage of students studying in
गत वषा की तुिना में गन्ने के उत्पादन में सबसे faculties other than science is:
अधधक वद्
ृ धध वषा ______ में दजा की गयीI ववज्ञान को छोड़कर अन्य संकायों में अध्ययन
a) 2018 b) 2017 c) 2019 d) 2016
करने वािे छात्रों का प्रनतर्त ककतना है ?
a) 80% b) 60% c) 20% d) 75%

59. The ratio of the number of girls studying


56. The difference (in tonnes) between the Arts to the number of girls studying in all
average production of barley and average other streams is:
production of rice is: किा का अध्ययन करने वािी िड़ककयों की संख्या
जौ के औसत उत्पादन और चावि औसत के का अनुपात अन्य सिी ववषयों में पढ़ने वािी
उत्पादन में अंतर ________ (टनों में) है I िड़ककयों की संख्या का अनुपात क्या है :
a) 549 b) 780 c) 231 d) 471 a) 1:3 b) 3:1 c) 2:1 d) 1:2

60. The heights of some girls in a school were


noted and the data obtained are as show in
57. The percentage growth of maize in the year the table:
2019 over the year 2015 is: एक ववद्यािय में कुछ िड़ककयों की ऊंचाई मापी
वषा 2015 की तुिना में, 2019 में मक्का के गयी और प्राप्त आंकड़ों को तालिका में दर्ााया
उत्पादन में ककतने प्रनतर्त की वद्
ृ धध हुई है ? गया है I
a) 65.12% b) 60.28% How many girls have a height of 135 cm or
more but less than 150 cm?
ककतनी िड़ककयों की ऊंचाई 135 cm या उससे
अधधक िेककन 150 cm से कम है ?
a) 35 b) 88 c) 64 d) 80 is what percentage of the total numbers of
cars sold by all four companies in 2013 and
2016? (Correct to one decimal place)
कंपनी A द्वारा 2017 में और C द्वारा 2013 में
बेची गयी कारों की कुि संख्या, सिी चारों
FOR VIDEO SOLUTION FROM QUESTION 61 TO
कंपननयों द्वारा 2013 और 2016 में बेचीं गयी
70: CLICK HERE OR
SCAN Q R CODE GIVEN BELOW कारों की कुि संख्याका ककतना प्रनतर्त है ?
a) 23.8 b) 24.2
c) 25.6 d) 23.3

61. The following table shows the monthly


salaries (in ₹) of 50 families.
ननभनलिखित तालिका 50 पररवारों का मालसक
वेतन (₹ में) दर्ााती है :
How many families have a monthly salary
less than ₹40,000?
ककतने पररवारों का मालसक वेतन ₹ 40,000 से 64. The total number of cars sold by company C
in 2018 exceeds the average number of cars
कम है ?
sold by company A during 2014 to 2018 by:
a) 46 b) 9 c) 18 d) 27
2018 में कंपनी C द्वारा बेची गयी कारों की कुि
संख्या, 2014 से 2018 के दौरान कंपनी A द्वारा
बेचीगयी कारों की औसत संख्या से ककतनी
अधधक है ?
a) 15,000 b) 14,000
62. The given table represents the sale (in
c) 16,000 d) 12,000
thousands) of cars by four companies A, B, C
and D in six years. Study the table and
answer the question that follows:
नीचे दी गयी तालिका चार कंपननयों A, B, C और
D द्वारा छ: वषों में कारों की बबक्री (हजारों में)
दर्ााती है I तालिका का अध्ययन करें और ददए गये
प्रश्न का उत्तर दें :

65. The total number of cars sold by company B


during 2015,2017 and 2018 is what
percentage less than the total number of
cars sold by company C in 2013,2015,2017
and 2018?
2015, 2017 और 2018 के दौरान कंपनी B
द्वारा बेची गयी कारों की कुि संख्या, 2013,
63. The total number of cars sold by companies
2015, 2017 और 2018 में कॉभपे C द्वारा बेचीं
A in 2017 and C in 2013
गयी कारों की कुि संख्या से ककतने प्रनतर्त कम
है ?
𝟐 𝟏
a) 𝟏𝟔 b) 𝟑𝟑 c) 𝟓𝟎 d) 𝟒𝟎
𝟑 𝟑 68. As per data shown in the following table,
what is the percentage of students who got
less than 50 marks?
ननभनलिखित तालिका में ददिाए गए आंकड़ों के
अनुसार, 50 प्रनतर्त से कम अंक पाने वािे छात्रों
का प्रनतर्त क्या है ?
a) 48% b) 72% c) 38% d) 74%

66. What is the ratio of the total number of cars


69. The following table shows the percentage
sold by companies A, B and D in 2017 to the
distribution of the population of five states
total number of cars sold by all four
A, B, C, D and E on the basis of the poverty
companies in 2018?
line and also on the basis of sex:
2017 में A, B और D कंपननयों द्वारा बेची गयी
ननभन तालिका गरीबी रे िा के आधार पर और
कारों की कुि संख्या का, 2018 में सिी चार
लिंग के आधार पर पांच राज्यों A, B, C, D और E
कंपननयों द्वारा बेची गयी कारों की कुि संख्या से
की जनसंख्या के प्रनतर्त ववतरण को दर्ााती है
अनुपात क्या है ?
a) 3:4 b) 6:13 c) 9:14 d) 18:23

70. If the male population above the poverty


line for state B is 2.5 million, then what is
the total population of state B?
यदद राज्य B के लिए गरीबी रे िा से ऊपर की

67. The following table gives the frequency of परु


ु ष जनसंख्या 2.5 लमलियन है , तो राज्य B की
vowels used in the page of a book: कुि जनसंख्या ककतनी है ?
ननभनलिखित तालिका ककसी पुस्तक के पष्ृ ठ में 𝟐
a) 𝟖 𝟑 𝒎𝒊𝒍𝒍𝒊𝒐𝒏
𝟏
b) 𝟖 𝟑 𝒎𝒊𝒍𝒍𝒊𝒐𝒏
प्रयक्
ु त स्वरों की बारभबारता दर्ााती है I 𝟐 𝟓
c) 𝟕 𝟑 𝒎𝒊𝒍𝒍𝒊𝒐𝒏 d) 𝟏𝟎 𝟑 𝒎𝒊𝒍𝒍𝒊𝒐𝒏
As per the given data, which vowels occur
less than 80 times?
ददए गये आंकड़ों के अनुसार, कौन सा स्वर 80
बार से कम प्रयुक्त हुआ है ?
a) a, o, u b) a, e c) a, e, i d) a, i, o, u
FOR VIDEO SOLUTION FROM QUESTION 71 TO 73. The following table showing the percentage
80: CLICK HERE OR of the total population of a state in different
SCAN Q R CODE GIVEN BELOW age groups:
ननभनलिखित तालिका में ववलिन्न आयु समूहों में
एक राज्य की कुि जनसंख्या का प्रनतर्त ददिाया
गया है :
Out of every 50,000 people, find the
71. If the population of males below the approximate number of persons below the
poverty line for state C is 3 million, then age of 35.
what is the total population of state C? प्रत्येक 50,000 िोगों में से, 35 वषा से कम आयु
यदद राज्य C के लिए गरीबी रे िा से नीचे वािे के व्यम्क्तयों की अनुमाननत संख्या का पता
परु
ु षों की जनसंख्या 3 लमलियन है , तो राज्य C की िगाएं।
कुि जनसंख्या ककतनी है ? a) 26,260 b) 26,250 c) 25,230 d) 25,250
a) 23.361 million b) 24.486 million
c) 26.316 million d) 25.617 million

74. Years 2012-2017. Study the table and


72. The following table shows the items of answer the question that follow:
expenditure of a company (in ₹ lakh per दी गयी तालिका वषा 2012-2017 के दौरान
annum), from 2015 to 2019.
ववलिन्न मॉडि A, B, C, D तथा E की उत्पाददत
ननभन तालिका 2015 से 2019 तक एक कंपनी
कारों की कुि संख्या (हज़ारों में) को दर्ााती है I
के व्यय के ववषय को दर्ााती है ।
तालिका का अध्ययन करें और ददए गये प्रश्न का
What is the average amount of interest per
year which the company had to pay during उत्तर दें :
the period?
प्रनत वषा ब्याज की औसत रालर् क्या है जो कंपनी
को अवधध के दौरान चक
ु ानी थी?
a) ₹ 35 lakh b) ₹ 30.7 lakh
c) ₹ 34.7 lakh d) ₹ 25 lakh

75. In the year 2015, which type of car


constitutes exactly 20% of the total number
of cars produced that year?
वषा 2015 में उत्पाददत ककस प्रकार की कारों की
संख्या, उस वषा में उत्पाददत कुि कारों की ठीक
20% है ?
a) E b) A c) D d) B a) C b) E c) A d) D

76. If 2013 and 2014 are put together, which


79. The following table shows the daily earnings
type of cars constitute exactly 25% of the
of 45 skilled workers:
total number of cars produced in those 2
years? ननभन तालिका 45 कुर्ि श्रलमकों की दै ननक आय

2013 और 2014 में लमिाकर ककस प्रकार की को दर्ााती है :

कारों की संख्या, उन दो वषों में उत्पाददत कुि How many workers earn less than 1100
rupee in a day?
कारों की संख्या का ठीक 25% है ?
ककतने श्रलमक प्रनतददन 1100 से कम कमाते हैं?
a) B b) E c) C d) D
a) 39 b) 29 c) 10 d) 43

80. The numbers of students enrolled in


different streams in a college is shown in
the following table:
77. The percentage increase in the total cars in
एक ववलर्ष्ट वषा में, एक कॉिेज में ववलिन्न
2016 over 2012 is:
2012 की ति
ु ना में 2016 में कारों के उत्पादन संकायों में नामांककत छात्रों की संख्या ननभन

में ककतने प्रनतर्त की वद्


ृ धध हुई? तालिका में दर्ााया गयी है

a) 33.33% b) 45% c) 50% d) 62.33% The ratio of the total number of boys to that
of girls in the college is:
कॉिेज में िड़कों की कुि संख्या का, िड़ककयों की
कुि संख्या से अनप
ु ात क्या है ?
a) 13:12 b) 1:1 c) 23:27 d) 27:23

78. The percentage decrease in the production


of which type of car in 2017 with reference FOR VIDEO SOLUTION FROM QUESTION 81 TO
to 2016 was the maximum: 86: CLICK HERE OR
2016 के सन्दिा में, 2017 में ककस तरह की कार SCAN Q R CODE GIVEN BELOW
के उत्पादन प्रनतर्त में सवााधधक कमी दजा की
गयी थी?
81. During a medical check-up, the heights of 40 a) 2 b) 3 c) 1 d) 4
students in a class were recorded as shown
in the following table:
एक धचककत्सा जांच के दौरान, एक कक्षा के 40
छात्रों की ऊंचाई दजा की गयी म्जसे ननभनलिखित
तालिका में ददिाया गया है I
How many students have a height of 165 cm
or more?
ककतने छात्रों की ऊंचाई 165 cm या अधधक है ? 85. For employee D, the income from overtime
a) 25 b) 15 c) 16 d) 10 is what percentage of his total income?

82. The following table shows the income (in ₹)


for a particular month together with their
source in respect of 5 employees (A, B, C, D
and E)
कमाचारी D के लिए, ओवरटाइम से प्राप्त आय,
ननभन तालिका 5 कमाचाररयों (A, B, C, D और E)
उसकी कुि आय का ककतने प्रनतर्त है ?
के सभबन्ध में एक ववर्ेष महीने के लिए आय (₹
a) 22.5% b) 6.25% c) 7.5% d) 5%
में) को उनके स्रोत दर्ााती है I

86. How many employees have their salary


83. How many employees got more than a total more than four times their other incomes?
of ₹ 10,000 as income other than salary? ककतने कमाचाररयों का वेतन उनकी अन्य आयों
ककतने कमाचाररयों को वेतन के अिावा अन्य आय
के. चार गन
ु ा से अधधक है ?
के रूप में कुि ₹ 10,000 से अधधक रालर् प्राप्त a) 3 b) 2 c) 1 d) 4
हुई ?
a) 3 b) 1 c) 2 d) 4

84. How many employees got more arrears than


the average arrears received by all the
employees?
ककतने कमाचाररयों को, सिी कमाचाररयों द्वारा
प्राप्त की गयी औसत बकाया रार्ी की तुिना में
अधधक बकाया रार्ी लमिी?
STATISTICS
1. The mode & median of the data 8, 6, 8, 7, संख्िाओं की ननम्न सूची की माध्यिका
8, 6, 8, 7, 6, is
क्िा है ? 5, 3, 6, 9, 11, 19 और 1
8, 6, 8, 7, 8, 6, 8, 7, 6, का बहुलक और
a) 5 b) 6 c) 9 d) 11
माध्यिका है:
a) 7 and 8 b) 6 and 7 7. What is the mode of the following list of
c) 8 and 7 d) 6 and 8 numbers: 2, 4, 5, 6, 5, 4, 3, 5, 3, 1, and 7?
संख्िाओं की ननम्नललखित सूची का
2. Find the standard deviation of {11, 7, 10,
13, 9} बहुलक क्िा है
{11, 7, 10, 13, 9} का मानक विचलन ज्ञात 2, 4, 5, 6, 5, 4, 3, 5, 3, 1 और 7 ?
a) 3 b) 4 c) 5 d) 2
करो |
a) 1 b) 2 c) 4 d) 5 8. The mean of a distribution is 21 and the
standard deviation is 7. What is the value
3. Find the median, mode and mean of 9, 5, of the coefficient variation?
8, 9, 9, 7, 8, 9, 8,
वितरण का मायि 21 है और मानक
9, 5, 8, 9, 9, 7, 8, 9, 8 की माध्यिका, बहुलक
विचलन 7 है | विचरण गण ु ांक क्िा होगा?
और मायि क्िा होंगें ?
a) 16.66% b) 66.66%
a) 9, 9, 9, b) 9, 8, 9, c) 33.33% d) 100%
c) 8, 9, 8 d) 8, 9, 9
9. If the standard deviation of a population
4. Find the range of the data 2, 1, 2, 3, 5, 4, is 6.55, what would be the population
7, 3, 5, 2, 4. variance?
2, 1, 2, 3, 5, 4, 7, 3, 5, 2, 4 का पररसर िदि एक जनसंख्िा का मानक विचलन
ज्ञात करो | 6.55 है , तो जनसंख्िा का विचरण क्िा
a) 5 b) 4 c) 3 d) 6
होगा ?
5. The standard deviation of the set {10, 11, a) 40.25 b) 42.25
12, 9, 8} is c) 18.25 d) 13
{10, 11, 12, 9, 8} का मानक विचलन है :
10. The median of the following numbers
a) 1 b) √𝟐 c) 2 d) 𝟐√𝟐 arranged in ascending order is 2.5, if so
find x?
6. What is the median of the following list
आरोही क्रम में व्ििध्थित ननम्न संख्िाओं
of numbers: 5, 3, 6, 9, 11, 19 and 1?
की माध्यिका 2.5 है, तो x ज्ञात करो :
0, 0, 1, 1, 2, 2, x, 3, 3, 4, 5, 7 16. If the standard deviation of a population
a) 2 b) 3 c) 4 d) 0 is 7, what would be the population
variance?
11. Find the mean of the values: 1, 9, 7, 3, 5, िदि एक जनसंख्िा का मानक विचलन 7
6, 4, 2, 8
है तो जनसंख्िा का विचरण क्िा होगा ?
1, 9, 7, 3, 5, 6, 4, 2, 8 का मायि ज्ञात करो:
a) 14 b) 21 c) 49 d) 28
a) 3 b) 4 c) 5 d) 6
17. The mean of a distribution is 11 and the
12. The variance of a set of data is 121. Then standard deviation is 5. What is the value
the standard deviation of the data is: of the coefficient variation?
आंकड़े के एक सेट का विचरण 121 है। एक वितरण का मायि 11 है और मानक
तो आंकड़े का मानक विचलन है: विचलन 5 है | विचरण गुणांक का मान
a) ± 11 b) 11 c) 21 d) 60.5
क्िा होगा ?
13. The median of the data -3, 4, 0, 4, -2, -5, a) 45.45% b) 35.35%
1, 7, 10, 5 is: c) 25.25% d) 55.55%
-3, 4, 0, 4, -2, -5, 1, 7, 10, 5 की माध्यिका
18. Find the median of the data set: 1.9, 8.4,
है: 3.6, 5.8
a) 2 b) 2.5 c) 2.75 d) 3 1.9, 8.4, 3.6, 5.8 की माध्यिका ज्ञात करें
a) 5.1 b) 4.8 c) 5.2 d) 5.6
14. If the standard deviation of a population
is 3, What would be the population 19. The mean deviation of the data 3, 10, 10,
variance? 4, 7, 10, 5, is
िदि एक जनसंख्िा का मानक विचलन 3 3, 10, 10, 4, 7, 10, 5, का मायि विचलन है:
है , तो जनसंख्िा का विचरण क्िा होगा ? a)
𝟒𝟗
b)
𝟏𝟗
𝟕 𝟕
a) 9 b) 6 c) 8 d) 15 𝟓𝟎 𝟏𝟖
c) d)
𝟕 𝟕
15. The mean of a distribution is 15 and the
standard deviation is 5. what is the value 20. The mode of the data 12, 1, 10, 1, 9, 3, 4,
of the coefficient variation? 9, 7, 9 is:
एक वितरण का मायि 15 है और मानक 12, 1, 10, 1, 9, 3, 4, 9, 7, 9 का बहुलक है :
a) 9 b) 12 c) 1 d) 7
विचलन 5 है | विचरण गुणांक का मान
क्िा है ? 21. The median of the data 7, 21, 2, 17, 3, 13,
a) 16.66% b) 66.66% 7, 4, 9, 7, 9 is:
c) 33.33% d) 100% 7, 21, 2, 17, 3, 13, 7, 4, 9, 7, 9 की
माध्यिका है:
a) 4 b) 17 c) 7 d) 9
28. The variance of a set of data is 144. Then
22. If the standard deviation of a distribution the standard deviation of the data is:
is 6, What is the value of its variance? आंकड़ों के एक सेट का विचरण 144 है।
िदि ककसी वितरण का मानक विचलन 6 किर डेटा का मानक विचलन है:
है, तो उसके विचरण का मूल्ि क्िा है? a) ±12 b) 12 c) 44 d) 72
a) 8 b) 24 c) 36 d) 12
29. The mode of the data – 3, 4, 0, 4, -2, -5, 1,
23. What is the median of the following 7, 10, 5 is:
distribution? – 3, 4, 0, 4, -2, -5, 1, 7, 10, 5 का बहुलक
ननम्नललखित वितरण की माध्यिका क्िा है:
है? a) 0 b) 4 c) -2 d) 7
67, 31, 53, 22, 66, 98, 43, 54, 77, 13, 43,
97, 36, 12, 90, 53 30. The mean deviation of the data 2, 9, 9, 3,
a) 53 b) 56.5 c) 54 d) 54.5 6, 9, 4 from the mean is:
मायि से आंकड़े 2, 9, 9, 3, 6, 9, 4 का
24. The mode of the data 3, 12, 4, 6, 8, 5, 4
is: विचलन होगा :
𝟒𝟐 𝟏𝟖
3, 12, 4, 6, 8, 5, 4 का बहुलक है : a) 𝟕
b) 𝟕
𝟓𝟎
a) 5 b) 8 c) 4 d) 3 c) 2.5 d) 𝟕

25. The mean of a distribution is 13 and the 31. If the standard deviation of a distribution
standard deviation is 7. What is the value is 9, what is the value of variance?
of the coefficient variation?
िदि ककसी वितरण का मानक विचलन 9
वितरण का मायि 13 है और मानक
है , तो विचरण का मान क्िा होगा ?
विचलन 7 | विचरण गण
ु ांक का मान क्िा a) 18 b) 27 c) 81 d) 36
है?
a) 50 % b) 76.77 % 32. What is the median of the following
c) 53.85 % d) 38.88 % distribution?
ननम्नललखित वितरण की माध्यिका क्िा
26. If the standard deviation of a population
है ?
is 9.5, what would be the population
42, 65, 18, 69, 29, 42, 48, 79, 25, 24, 98,
variance?
87, 63, 27, 17
27. िदि जनसंख्िा का मानक विचलन 9.5 है , a) 43.5 b) 65
तो जनसंख्िा का विचरण क्िा होगा? c) 42 d) 69
a) 19 b) 90.25
c) 81.25 d) 93.25
33. What is the mean of the following
distribution?
ननम्नललखित वितरण का मायि क्िा है ?
54, 23, 66, 44, 79, 21, 54, 67, 29, 59
a) 51.4 b) 49.6
c) 48.7 d) 45.3

34. What is the mean deviation about


median of the following distribution?
ननम्नललखित वितरण का मध्यिका के
साक्षेप मायि विचलन क्िा है ?
38, 70, 48, 34, 63, 42, 55, 44, 53, 47

You might also like